You are on page 1of 173

INDEX

SN Name of the Chapter Page No. No. of Questions

1 Basic Concepts of Income Tax 1–7 33

2 Residential Status & Scope of Total Income 8 – 19 35

3 Exempt Incomes & Agricultural Income 20 – 26 19

4 HEADS OF INCOME 235

4A Salaries 27 – 55 56

4B House Property 56 – 69 26

4C Profits & Gains of Business & Profession 70 – 98 55

4D Capital Gains 99 – 116 52

4E Income from other Sources 117 – 123 18

5 Clubbing of Income 124 – 132 24

6 Set off & Carry Forward of Losses 133 – 143 21

7 Chapter VI-A Deductions 144 – 154 20

8 TDS & TCS 155 – 164 35

9 Advance Tax & Interest u/s 234 165 – 168 14

10 Return of Income 169 – 172 28

11 Total Computation Of Income 173 – 210 24

CONNECT WITH US ON
YouTube Channel: @ CA PRANAV CHANDAK.

Telegram Channel: t.me/pranavchandak

WhatsApp: 8888111134

Instagram: Pranav Chandak classes

Facebook Page: Pranav Chandak Classes

INCOME TAX MCQs BY CA PRANAV CHANDAK 8888111134


1. BASIC CONCEPT OF INCOME TAX
Q1. Explain the concept of “Marginal Relief ”.
Answer:
▪ Marginal Relief is a reduction from the Tax Payable by the Assessee.
▪ Relief from Tax Payable shall be given, where the Tax Payable together with Surcharge exceeds the Income
earned by an Assessee in excess of Rs. 50 Lacs or Rs. 1 Crore. Such Relief is known as Marginal Relief.
▪ The principle in Marginal Relief is that the Additional Amount of Income Tax Payable with Surcharge in excess
of Income over Rs. 50 Lacs or Rs. 1 Crore should not be more than the amount in excess of Rs. 50 Lacs/Rs. 1 Cr.
▪ Marginal Relief = Tax on Total Income including Surcharge - (Total Income – Rs. 50 Lacs or Rs. 1 Cr) + (Tax on
Rs. 50 Lacs or Rs. 1 Cr excluding Surcharge).
▪ Tax Payable = Tax on Total Income including Surcharge - Marginal Relief as computed above.

Q2. Compute tax liability & Marginal Relief for Resident Assessees in the following situations for PY 2018-19.
Name of Individual Mr. X Mr. Y Mr. Z
Age of Assessee 45 years 65 years 84 years
Total Income Rs. 1.01 Crores Rs. 1.01 Crores Rs. 1.01 Crores
Solution:
Particulars Mr. X Mr. Y Mr. Z
1(a). Tax on Total Income 1,12,500 + (91 L x 30% = 1,10,000 + (91 L x 30% = 1,00,000 + 91 L x 30%
28,42,500 28,40,000 = 28,30,000
1(b). SC @ 15% 4,26,375 4,26,000 4,24,500
1. Total Tax = 1(a) + 1(b) 32,68,875 32,66,000 32,54,500
2(a). Tax if Income = Rs. 1 Cr 1,12,500 + 90 L x 30% = 1,10,000 + 90 L x 30% = 1,00,000+ 90 L x 30% =
28,12,500 28,10,000 28,00,000
2(b). SC @ 10% on 2(a) 2,81,250 2,81,000 2,80,000
Total Tax = 2(a) + 2(b) 30,93,750 30,91,000 30,80,000
3. Excess Tax payable 1,75,125 1,75,000 1,74,500
4. Excess Income (i.e > 1 Cr) 1,00,000 1,00,000 1,00,000
5. Marginal Relief (3 – 4) 75,125 75,000 74,500
6. Tax Payable (1 - 5) 31,93,750 31,91,000 31,80,000
7. HEC at 4 % on (6) 1,27,750 1,27,640 1,27,200
8. Tax Liability 33,21,500 33,18,640 33,07,200

Q3. Compute the tax liability of Mr. PC (Age 24), having total income of Rs. 51 Lacs for AY 2019-20.
Solution: Computation of tax liability of Mr. PC for AY 2019-20
1. Tax payable including surcharge on Total Income of Rs. 51,00,000 Rs 14,76,750.
2. Tax Payable on total income of Rs. 50 Lacs Rs. 13,12,500.
3. Excess tax payable = [(1)-(2)] = Rs. 14,76,750 – Rs. 13,12,500 Rs. 1,64,250.
4. Marginal Relief = [(Rs. 1,64,250- Rs. 1,00,000) Income in excess of Rs. 50,00,000] Rs. 64,250.
Tax Payable = Rs. 14,76,750 – Rs. 64,250 = Rs. 14,12,500 + 4% HEC = 14,69,000.

Q4. Compute the tax liability of Mr. A (aged 42), having total income of Rs. 1,01,00,000 for AY 2019-20.
Solution: Computation of tax liability of Mr. A for AY 2019-20.
1. Tax payable including surcharge on total income of Rs. 1,01,00,000 Rs. 32,68,875.
2. Tax Payable on total income of Rs. 1 crore Rs. 30,93,750.
3. Excess tax payable = [(1)-(2)] = Rs. 32,68,875 - Rs. 30,93,750 Rs. 1,75,125.
4. Marginal Relief = [(Rs. 1,75,125 - Rs. 1,00,000) Income in excess of Rs. 1,00,00,000] Rs. 75,125.
Tax Payable = Rs. 32,68,875 – Rs. 75,125 = Rs. 31,93,750 + 4% HEC = 33,21,500.

DT QUESTION BANK BY CA PRANAV CHANDAK Subscribe us on YouTube 1


Q5. Differentiate between Diversion of Income & Application of Income:
Answer:
Diversion of Income Application of Income
(1) Income never reaches to the assessee as his own Income reaches to the assessee as his own
income. By virtue of an obligation the income is income & is subsequently applied to discharge
diverted at source before it reaches the assessee. an obligation.
(2) The obligation is on the source of income. The obligation is on the receipt of income i.e.
after income reaches to the assessee.
(3) There is an overriding title by virtue of which There is no overriding title in this case.
diversion of income takes place.
(4) In case of diversion, the income is not included in the In case of application, income is included in the
income of the assessee. income of the assessee.
(5) Since the assessee do not have title, Income cannot be Income is said to have accrued or arisen &
said to have accrued or arisen. therefore is taxable in the hands of assessee.

Q6. Anand & Sons is a partnership firm consisting of father & three sons. The partnership deed provided that after
the death of father, the business shall be continued by his sons, subject to the condition that the firm shall pay 30%
of the profits to their mother. Father died on 31st March, 2018. In the previous year 2018-19, the reconstituted
firm paid Rs. 1 Lac (equivalent to 30% of the profit) to their mother & claimed the amount as deduction from its
income. Examine the correctness of claim of the firm. [CS Exam]
Answer:
▪ The issue raised in the problem is based on the concept of diversion of income by overriding title which is well
recognized in the income tax law.
▪ In the instant case, Rs. 1 lac being 30% of profits of the firm, paid to the mother gets diverted at source by the
charge created in her favour as per the terms of the partnership deed. Such income does not reach the assessee.
▪ Rather, such income stands diverted to other person as such other person has a better title over the assessee.
▪ Firm might have received the said amount but it so received for & on behalf of mother, who possesses the
overriding title.
▪ Therefore, the amount paid to the mother should be excluded from income of the firm. The view has been
confirmed in CIT v. Nariman B. Bharucha & Sons [1981] 130 ITR 863 (Bom.)

Q7. An employee instructs his employer to pay a certain portion of his salary to a charity & claims it as exempt as
it is diverted by overriding charge / title. Comment.
Answer:
▪ In the instant case, it is an application of income & in the nature of foregoing of salary.
▪ According to the Supreme Court judgment in CIT v. L.W. Russel (1964) 52 ITR 91, the salary which has been
foregone after its accrual in the hands of the employee is taxable.
▪ Thus, amount paid by the employer to a charity as per the employee’s directions is taxable to the employee.

Q8. Explain the purpose for which ‘Proviso’ & ‘Explanation’ are incorporated under various sections of the Act?
Answer:
1. Proviso to a section/sub-section/clause gives the exceptions to the provision contained in the respective
section, sub-section/clause. (Proviso gives the cases where the provision contained in the respective
section/sub- section/clause would not apply or where the provision would apply with certain modification).
2. The Explanation to a section/sub-section/clause gives a clarification relating to the provision contained in the
respective section/subsection/clause.

Q9. Explain the purpose for which "Notifications" & "Circulars" are issued.
Answer:
1. Notification:
▪ Notifications are subordinate legislation issued by CG to give effect to the provisions of the Act.
▪ The CBDT is also empowered to make & amend rules by issuing notifications.
▪ They are binding on everyone. [Assessee + Income Tax department].
2. Circulars:
▪ Circulars are issued by the CBDT to deal with certain specific problems & to clarify the doubts regarding
the scope & meaning of the provisions of the law.
▪ Circulars provide guidance to the Income Tax officers & Assessees.
▪ These circulars are binding on the department but not on the assessee.
▪ However assessee can take advantage of beneficial circulars.

DT QUESTION BANK BY CA PRANAV CHANDAK Subscribe us on YouTube 2


Q10. Define the term ‘India” for the purpose of Income Tax Act, 1961.
Answer: The term 'India' means –
▪ Territory of India as per Article 1 of the Constitution,
▪ Territorial Waters of India (TWI), seabed & subsoil underlying such waters,
▪ Continental Shelf,
▪ Exclusive Economic Zone;
▪ Any other specified maritime zone & air space above its territory & TWI.
▪ Specified maritime zone means the maritime zone as referred to in Territorial Waters, Continental Shelf,
Exclusive Economic Zone & other Maritime Zones Act, 1976.

Q11. Who is an “Assessee”? Explain with suitable examples.


Answer: Any person by whom any tax or any other sum of money is payable under this Act. It includes –
(a) Tax Payable: Every Person by whom any tax or any other sum of money is payable under this Act whether
or not any proceeding under this act has started against him.
(b) Proceeding started: Any Person in respect of whom any proceeding under the act has been taken whether
or not any tax, interest or penalty is payable by him under this act. Proceeding may be taken for/of:
▪ Assessment of his income (or loss) sustained by him;
▪ Income (or loss) of any other person in respect of whom he is assessable;
▪ Refund due to him or to such other person.
▪ Assessment of Fringe Benefits.
(c) Deemed Assessee: Sometimes, a person becomes assessable in respect of the income of some other persons.
In such a case, he may be deemed as an assessee.
(d) Representative Assessee: Sometimes a person may be assessed for Income of another person. Such person
is known as representative assessee. Ex: Legal Heir is assessable for the income of deceased person.
(e) Assessee in default: Any person who does not deduct tax at source or after deducting tax, fails to pay
deducted tax to the government or who fails to pay advance tax is deemed to be assessee in default u/s
[201(1)]/218.

Q12. Who is a “Deemed Assessee”? Explain with suitable examples.


Answer: “Deemed Assessee” means a person who is treated as an Assessee under the IT Act.
This would include -
1. Trustee of a Trust,
2. Legal Representative of a Deceased Person u/s 159,
3. Representative Assessee of a Non-Resident u/s 160(2) (Agent of a Non-Resident),
4. Legal Guardian or Manager entitled to receive the Income on behalf of a Minor, Lunatic or Idiot,
5. Court of Wards/Official Trustee/Receiver entitled to receive Income on behalf of any other person.

Q13. Who is an “Assessee in Default”? Explain with suitable examples.


Answer: Assessee in Default includes persons who -
1. fail to deduct & remit TDS (Sec. 191).
2. fail to pay tax & any other sum demanded (Sec. 220).

*Q14. A single letter of enquiry was issued by the income tax department to Mr. PC of Pune. In this letter there was
no specific mention of any provision of the Income-tax Act, 1961. Can Mr. PC be treated as 'assessee'? [Nov. 1998]
Answer:
▪ If AO issues a letter to a Person, to conduct an enquiry, without mentioning any of the provisions of the Act, it
shall not be treated as a proceeding under the Income Tax Act. Such Person shall not be treated as an “Assessee”.
▪ A person is treated as 'assessee' only when any proceeding is initiated in his respect under the Act.
▪ In the given case, the letter was issued to Mr. PC only for an enquiry & not for any assessment.
▪ Such issuance of letter cannot be regarded as proceeding without any specific reference to the provisions of the Act.
▪ Therefore, Mr. PC is not an 'assessee' under the Act.

Q15. Explain the term “Person” under the Income Tax Act, 1961.
Answer: Refer Page No. 6 of “Concept Book”.

Q16. What are the different opinions given by “Dayabaga school of Hindu law” & “Mitakshara school of Hindu law”?
Answer: Refer ‘HUF’ on Page No. 6 of “Concept Book”.

Q17. Explain the term “Different Classes of Companies” under the Income Tax Act, 1961.
Answer: Refer Page No. 8 of “Concept Book”.

DT QUESTION BANK BY CA PRANAV CHANDAK Subscribe us on YouTube 3


Q18. When is an individual said to have a substantial interest in a concern?
Answer: As per section 2(32), an individual is said to have substantial interest in a concern if he or she, along
with his or her relatives, is, at any time during the previous year, -
(a) beneficial owner of equity shares carrying 20% or more of the voting power in a company; or
(b) entitled to 20% or more of the profits of such concern.

Q19. Describe ‘Average rate of tax’ & ‘Maximum marginal rate’ u/s 2(10) & 2(29C) of the IT Act, 1961.
Answer:
Amount of income tax calculated on the total income × 100
1. Average Rate of tax =
Total Income

2. Maximum Marginal Rate is the rate of income-tax (including surcharge, if any) applicable in relation to the
highest slab of income, specified in the Finance Act for the relevant year, in the case of the following persons an
individual; or an AOP or Body of Individuals.
Note: MMR for AY 2019-20 is 35.88% (being tax @ 30% + 15% SC + 4% HEC on Income-tax).

*Q20. State whether various heads of income are mutually exclusive?


Answer:
▪ The heads of income are mutually exclusive i.e. an income, which falls under one head cannot be brought to tax
under any other head of income.
▪ The Income-tax Act contains provisions describing the incomes to be included in each of the heads of income
listed above.
▪ Correct classification of income under the proper head is mandatory.
▪ Incorrect classification to obtain undue incentives in the law may lead to penalties & prosecution.

Q21. List the differences between Capital Receipts & Revenue Receipts.
Answer:
Capital Receipts Revenue Receipts
Receipts relating to fixed capital are capital receipts. Receipts relating to circulating capital are revenue
Ex: Receipt on sale of asset is a capital receipt. receipts. Ex: Receipt on sale of SIT is revenue receipt.
Compensation received for extinction of a profit Compensation received for loss of profits or earnings is
earning source (in whole/part) is a capital receipt. a revenue receipt.
Receipt in substitution of source of income A receipt in substitution of income is revenue receipt.
Ex: Compensation for loss of employment.
Profits from sale of shares or securities, which were Profits from sale of shares acquired in ordinary course
purchased as an investor, are capital receipts. of business by dealer in shares, are revenue receipts.
Capital receipts are exempt from tax unless expressly Revenue receipts are taxable unless expressly exempt.
taxable. Ex: Capital gains. Ex: Income exempt u/s 10 to 13A.
Compensation received for relinquishing interest (in Compensation received for relinquishing interest in
whole/part) in a capital asset is a capital receipt. stock-in-trade of the business is a revenue receipt.
Profits from transactions outside the purview of Profits from transactions entered into the business
regular trading activities of the assessee are capital regularly carried on by the assessee or are incidental
receipts. to/associated with business, are revenue receipts.
Subsidy is treated as capital receipt if it is given to set- Subsidy is treated as revenue receipt if it is given for
up a new business or to complete a project or to existing business or to meet any specific revenue
acquire an asset. expenditure or reimbursement of such expenditure.

Q22. X starts a new business on 29th March 2018. He closes down first set of books of A/c on 31st March 2019. He
wants that income generated during this period should be chargeable to tax for AY 2019-20. Is he legally correct?
Solution: Previous year ends on March 31 immediately before the commencement of AY.
Ex: For AY 2018-19, previous year is the period which ends on March 31,2018.
In this case, the period which commences on March 29, 2018 & ends on March 31,2018, is the previous year for
the AY 2018-19.
In other words, income generated by X during March 29, 2018 & March 31, 2018 is taxable for AY 2018-19.
X does not have any option to include this income in the income of the AY 2019-20.
Previous year & AY, in this case, will be determined as follows:
First PY: March 29, 2018 - March 31, 2018 Income of PY 2017-18 will be taxable in AY 2018-19
2nd PY: April 1, 2018 to March 31, 2019 Income of PY 2018-19 will be taxable in AY 2019-20

DT QUESTION BANK BY CA PRANAV CHANDAK Subscribe us on YouTube 4


Q23. State the instances where the income of the PY is assessable in the PY itself instead of the AY.
Answer: Income earned during any PY is assessed to tax in immediately succeeding AY.
However, in following circumstances, income is taxed in PY in which it is earned. Thus AY & PY in these exceptional
circumstances will be the same. These exceptions have been made to protect the interests of revenue.

1. Shipping Business of a Non-Residents [section 172]


▪ If a ship belonging to or chartered by NR carries passengers/livestock/mail/goods shipped at a port in India,
▪ Such Ship is allowed to leave the port only when tax has been paid or satisfactory arrangement has been
made for payment thereof.
▪ Income = 7.5% of freight paid/payable to the owner/his agent whether in India or o/s India for such carriage.
▪ Such income is charged to tax in the same year in which it is earned.

2. Persons leaving India [section 174]


▪ Where it appears to AO that any individual may leave India during current AY or shortly after its expiry &
▪ He has no present intention of returning to India, then
▪ Total Income of such individual for the period from the expiry of the respective PY up to the probable date
of his departure from India is chargeable to tax in that AY.

3. AOP/BOI/AJP formed for a particular event or purpose [Sec 174A]


▪ If AOP/BOI etc. is formed or established for a particular event or purpose &
▪ AO apprehends that AOP/BOI is likely to be dissolved in the same year or in next year,
▪ he can make assessment of the income upto date of dissolution as income of relevant AY.

4. Persons likely to transfer property to avoid tax [Sec 175]


▪ If during current AY, it appears to AO that a person is likely to charge, sell, transfer, dispose any of his assets
▪ to avoid payment of any liability under this Act,
▪ Total income of such person for the period from the expiry of PY to the date when AO commences
proceedings is chargeable to tax in that AY.

5. Discontinued Business [Section 176]


▪ If any business or profession is discontinued in any AY,
▪ Income of the period from the expiry of the PY up to the date of such discontinuance may,
▪ at the discretion of AO, may be charged to tax in that AY.
Section 176 is a Discretionary power. The Assessing Officer has the discretion of applying it. AO may choose not
to apply it & wait till the end of the AY.

Q24. What will be the previous year in case of undisclosed sources of income? [4 Marks, PCC June, 2009]
Answer: Following undisclosed incomes shall be deemed to be the income of FY in which they are found:
1. Unexplained Cash Credits [Section 68]: Where the assessee offers no explanation to the Assessing Officer for
any sum credited in the books of accounts or the explanation so offered is not satisfactory, then the sum shall
be charged as income in the previous year in which it is credited in the books of accounts.
2. Unexplained investments [Section 69]: Where the assessee fails to record the investments made by him &
thereafter offers no explanation or the explanation so offered is not satisfactory, then the value of such
investments are taxed as income in the financial year in which such investment is made.
3. Unexplained Money [Section 69A]: Where the assessee owns any money, bullion, jewellery, etc. & does not
record the same in his books of accounts, then the same shall be regarded as his income of the financial year in
which it is found, if the assessee fails to offer any explanation regarding such money or the explanation offered
by him seems to be unsatisfactory to the Assessing Officer.
4. Investment not fully disclosed [Section 69B]: In case, it is found that the real value of any investments,
money, bullion, etc. is more than the amount recorded in the books & no explanation is offered by the assessee
or the explanation so offered is not satisfactory, then such excess amount shall be treated as his income in the
financial year in which such investment, etc. is made.
5. Unexplained Expenditure [Section 69C]: When the assessee incurs any expenditure for which he offers no
explanation about its source or offers an explanation which is not satisfactory, then the same shall be
considered as his income in the financial year in which such expenditure is incurred.
6. Amount borrowed or repaid on hundi [Section 69D]: If any amount is borrowed or repaid on hundi through
any mode other than the account-payee cheque, then the same shall be regarded as the income in the financial
year in which such amount is so borrowed or repaid.

Rate of Tax for Deemed Income u/s 68 & 69 [Section 115BBE]


▪ (60% + SC @ 25 % of tax) + 4 % HEC. Thus, Effective rate of tax is 78 % [Section 115BBE].
▪ Neither BEL nor any allowance nor set off of any loss shall be allowable against such income.

DT QUESTION BANK BY CA PRANAV CHANDAK Subscribe us on YouTube 5


*Q25. State the incomes which are chargeable at flat rate of Income tax.
Answer:
Section Income Rate
112 Long term capital gains (other than LTCG u/s 112A) 20%
112A LTCG on transfer of (i) Equity share (ii) Units of EOMF/Business trust. 10% if >
Note: LTCG upto Rs. 1 lac is exempt. LTCG > Rs. 1 lac is taxable @10%. 1 Lacs
111A STCG u/s 111A [STT Paid] 15%
115BB (i) Winning from Lotteries (ii) Crossword puzzles (iii) Race including horse races 30%
(iv) Card game & other game of any sort Gambling or betting
115BBDA Income by way of dividend exceeding Rs. 10 Lacs in aggregate 10%
115BBE Income u/s 68/69/69A/69B/69C/69D. 60%

Q26. Write a short note on Surcharge.


Answer:
▪ Surcharge is an additional tax payable over & above the income-tax.
▪ Surcharge is levied as a percentage of income-tax.
▪ Rate of Surcharge is given below in the table:
Rate of Surcharge if Total Income ↓
Assessee
> 50 Lacs but ≤ 1 Cr >1 Cr but ≤ 10 Cr > 10 Cr
1. Individual/HUF/AOP/BOI/AJP 10 % of IT 15 % of IT 15 % of IT
2. Firms/LLP/Co-operative society/LA - 12 % of IT 12 % of IT
3. Domestic Companies - 7 % of IT 12 % of IT
4. Foreign Companies - 2 % of IT 5 % of IT

*Q27. Explain the provisions relating to Rebate u/s 87A.


Answer:
▪ Rebate u/s 87A is applicable for a Resident Individual whose Total Income does not exceed Rs. 3.5 Lacs.
▪ Rebate = Rs. 2,500 or 100% of Tax Payable, whichever is lower.
▪ Rebate u/s 87A shall be before adding 4% of Health & Education Cess.
▪ Rebate u/s 87A is not available in respect of tax payable @ 10% on LTCG u/s 112A.

Q28. Total income of Mr. X aged 35 years resident in India is Rs. 3,35,000. Compute tax liability for AY 2019-20.
Solution: Since Mr. X is a resident having Total Income < Rs. 3,50,000, rebate u/s 87A is available.
First Rs. 2,50,000 Nil
Next Rs. 85,000 @ 5% Rs. 4,250
Total Rs. 4,250
Rebate u/s 87A = Lower of (i) Tax payable or (ii) Rs. 2,500 (Rs. 2,500)
Tax after rebate Rs. 1,750
Add: 4% HEC Rs. 70
Tax rounded off Rs. 1,820

Q29. Total income of Mr. Jon aged 35 years (NR) in India is Rs. 3,35,000. Compute tax liability for AY 2019-20.
Solution: Since Mr. X is a Non- Resident, rebate u/s 87A is Not available.
First Rs. 2,50,000 Nil
Next Rs. 85,000 @ 5% Rs. 4,250
Total Rs. 4,250
Add: 4% HEC Rs. 170
Tax rounded off Rs. 4,420

DT QUESTION BANK BY CA PRANAV CHANDAK Subscribe us on YouTube 6


Q30. Compute tax liability of Mr. X aged 44 years NR in India, whose total income for AY 2019-20 is Rs. 10 lacs.
Solution:
First Rs. 2,50,000 Nil
Next Rs. 2,50,000 @ 5% Rs. 12,500
Balance Rs. 5,00,000 @ 20% Rs. 1,00,000
Total Rs. 1,12,500
Add: 4% HEC Rs. 4,500
Tax rounded off Rs. 1,17,000

Q31. Total income of Mrs. X aged 44 years resident in India, for AY 2019-20 is Rs. 10 lacs. Compute tax liability.
Solution: BEL is same for male & female Assessee. Thus Mrs. X will get the same BEL of Rs. 2,50,000.
First Rs. 2,50,000 Nil
Next Rs. 2,50,000 @ 5% Rs. 12,500
Balance Rs. 5,00,000 @ 20% Rs. 1,00,000
Total Rs. 1,12,500
Add: 4% HEC Rs. 4,500
Tax rounded off Rs. 1,17,000

Q32. Total income of Mr. Joe aged 70 Non-resident in India for PY 2018-19 is Rs. 10 lacs. Compute his tax
liability.
Solution:
▪ Since increased BEL of Rs. 3 lacs is available only to resident person, Mr. Joe is not eligible.
▪ Thus he will be eligible for BEL of Rs. 2,50,000 only.
First Rs. 2,50,000 Nil
Next Rs. 2,50,000 @ 5% Rs. 12,500
Balance Rs. 5,00,000 @ 20% Rs. 1,00,000
Total Rs. 1,12,500
Add: 4% HEC Rs. 4,500
Tax rounded off Rs. 1,17,000

Q33. Total income of Mr. X aged 83 is Rs. 15,00,0000. Compute his tax liability for AY 2019-20.
Solution: Since Mr. X is a super-senior citizen, he will get the BEL of Rs. 5 lacs & remaining slabs will be same.
First Rs. 500,000 Nil
Next Rs. 5,00,000 @ 20% Rs. 1,00,000
Balance Rs. 5,00,000 @ 30% Rs. 1,50,000
Total Rs. 2,50,000
Add: 4% HEC Rs. 10,000
Tax rounded off Rs. 2,60,000

DT QUESTION BANK BY CA PRANAV CHANDAK Subscribe us on YouTube 7


2. RESIDENTIAL STATUS
Q1. Mr. Atif Aslam, a Non-Resident, residing in Pakistan since 2002 came to India on 19.02.2017 for permanent
settlement in India. Explain his Residential Status for the AY 2019-20. [May 2015]
Solution:
Basic Conditions:
(a) Stay in India in relevant PY should be ≥ 182 days Since Assessee has come back to India in India on
19.2.2017 & has not gone back from India, his stay
during PY 2018-19 will be 365 days.
Thus he satisfy first basic condition for PY 2018-19.
Hence, Assessee is a Resident.
(b) (i) Stay in India in relevant PY should be ≥ 60 days & No need to check since the assessee has satisfied 1st
(ii) Stay in India in Last 4 PYs should be ≥ 365 days basic condition. We will directly check additional
conditions.

Additional Conditions
(a) Total Stay in India in Last 7 PYs ≥ PY 2017- 2018: 365 days;
730 days PY 2016-2017: Feb (11 days) + March (31 days) = 42 days;
PY 2015-16 : Nil
PY 2014-15: Nil.
Total 42+365 = 407 days.
So, 1st Condition is Not satisfied. Thus he is a RNOR.
(b) Resident in any 2 PYs during Last 10 No Need to check because to be ROR, both conditions are to be
PYs satisfied.

Q2. Mr. X is a foreign national. During PY 2018-19, he comes to India for 91 days. Determine his residential status
for AY 2019-20 if during PY 2005-2006 to PY 2017-18, he was present in India as follows:
2005-06 315 days 2008-09 72 days 2011-12 22 days 2014-15 307 days 2017-18 134 days
2006-07 16 days 2009-10 179 days 2012-13 359 days 2015-16 67 days
2007-08 40 days 2010-11 362 days 2013-14 180 days 2016-17 12 days
Solution: During PY 2018-19, Mr. X is in India for 91 days & during Last 4 PYs, he is in India for 520 days (134 +
12 + 67 + 307 days). Thus, he satisfies 2nd basic condition & thus he is Resident.
Additional conditions:
Stay in India
2017-18 134 (566 days in last 4 PYs) Resident
2016-17 12 (866 days in last 4 PYs) Non-resident
2015-16 67 (868 days in last 4 PYs) Resident (for 2nd time)
2014-15 307
2013-14 180 Not necessary to determine
2012-13 359 further as resident for 2 years
2011-12 22
Total stay in 7 preceding PY is 1081 days. Thus R satisfies both the additional conditions.
Thus, he is a ROR in India for AY 2019-20.

Q3. During PY 2018-19, X, a foreign citizen, stayed in India for just 69 days. Determine his residential status for AY
2019-20 on the basis of the following information:
(i) During PY 2015-16, X was present in India for 365 days.
(ii) During PY 2012-13 & 2011-12, X was in Japan for 359 & 348 days respectively.
Solution:
Basic Condition:
(a) Stay in India in Relevant PY: 69 days. Thus 1st basic condition is not satisfied.
(b) (i) Stay in India in Relevant PY: 69 days; &
(ii) Stay in Last 4 PYs (PY 2017-18, 2016-17, 2015-16, 2014-15) for > 365 days.
Mr. X is a resident in India for PY 2018-19.

DT QUESTION BANK BY CA PRANAV CHANDAK Subscribe us on YouTube 8


Additional Conditions:
PY Stay in India Whether Resident or Non-Resident
2017-18 Nil Non-resident
2016-17 Nil Non-resident
2015-16 365 days Resident
2014-15 Nil Non-resident
2013-14 Nil Non-resident
2012-13 7 days Non-resident
2011-12 17 days Non-resident
Total 389 days

He was in India for < 730 days in Last 7 PYs. Hence, he is RNOR. [No need to check 2nd condition]

Q4. Mr. C, Japanese citizen left India after a stay of 10 years on 1.6.2016. During PY 2017-18, he comes to India for
46 days. Later, he returns to India for 1 year on 10.10.2018. Determine his residential status for AY 2019-20.
Solution: Mr. C leave India on 1.6.2016 (i.e in PY 2016-17). Before that he was in India for 10 years.
Basic Conditions:
(a) Stay in India in relevant PY Stay in India during PY 2018-19 = 173 days (22+30+31+31+28+31 days). So
should be ≥ 182 days 1st basic condition is not satisfied.
(b) (i) Stay in India during PY 2018-19 = 173 days
(i) Stay in India in relevant PY (ii)
should be ≥ 60 days & PY 2017-18 46 Given in the question
(ii) Stay in India in Last 4 PYs
should be ≥ 365 days PY 2016-17 62 (April: 30 days + May: 31 days + June: 1 day)
PY 2015-16 365 (since he left India on 1.6.2016 after 10 years)
PY 2014-15 365 (since he left India on 1.6.2016 after 10 years)
Total stay 838

Mr. C satisfies 2nd Basic Condition since his stay in India in PY 2018-19 is 173
days which is more than 60 days & in Last 4 PYs > 365 days.
Thus Mr. C is Resident in India for AY 2019-20.

Additional Conditions
(a) Stay in India in Last 4 PYs should be ≥ 730 Since Mr. C left India after 10 years in PY 2016-17, his stay
days in India during Last 7 PYs > 730 days. Thus he satisfies
first additional condition.
(b) Resident in any 2 PYs during Last 10 PYs Since Mr. C left India after 10 years in PY 2016-17, he must
be resident before that PY.
Thus he satisfies both the additional conditions.
Therefore, Mr. C is ROR for AY 2019-20.

Q5. Mr. Brett Lee, an Australian cricket player visits India for 100 days in every FY. This has been his practice for
the past 10 FYs. Find out his residential status for the AY 2019-20. [ICAI SM]
Solution: Determination of Residential Status of Mr. Brett Lee for AY 2019-20
Basic Conditions:
(a) Period of stay during PY 2018-19 = 100 days which is less than 182 days. Thus he does not satisfy 1 st basic
condition.
(b) (i) Period of stay during PY 2018-19 = 100 days which is more than 60 days.
(ii) Period of stay in India during 4 preceding PY (100 x 4 = 400 days) which is more than 365 days.
Thus he satisfy 2nd basic condition. Thus Mr. Brett Lee is a Resident in India.

Additional Conditions:
(a) Period of stay during 7 preceding PYs = 100 x 7 = 700 days. Since his period of stay in India during the past 7
PY is less than 730 days, he does not satisfy 1st additional condition.
We know that in order to be ROR, Individual must satisfy both additional conditions.
Since Mr. Brett Lee does not satisfy 2nd additional condition, we do not need to check for 1 st additional condition.
Thus Mr. Brett Lee is a RNOR for AY 2019-20.

DT QUESTION BANK BY CA PRANAV CHANDAK Subscribe us on YouTube 9


Q6. During the last 4 PYs, Mr. X, a Citizen of India, was present in India for 430 days. During last 7 PYs, he was
present in India for 830 days. Mr. X is a Member of Crew of a Dubai bound Indian Ship, carrying passengers in the
international waters, which left Kochi Port on 12 th August 2018. Following details are given for PY 2018-2019:
Particulars Date
Date entered into Continuous Discharge Certificate in respect of joining the ship by Mr. X 12th Aug, 2018
Date entered into Continuous Discharge Certificate in respect of signing of the ship by Mr. X 21st Jan 2019
In May 2018, he had gone out of India to Singapore & Malaysia on a private tour for a continuous period of 29 days.
You are required to determine the Residential Status of Mr. X for PY 2018-2019.
Solution:
▪ The given voyage is Eligible Voyage since Origin Port is Kochi (India) & Destination Port is Dubai (outside India).
▪ Indian Citizen being Crew Member of Indian Ship, leaving India during Relevant PY is considered as Resident
only if he stayed in India for a period of 182 days or more during the Relevant PY. [2nd Basic condition - NA]
▪ In computing stay in India, the period entered in CDC (i.e. from joining to signing off) shall be excluded.
Stay in India during PY 2018-19 = April (30 days) + May (2 days) + June (30days) + July (31 days) + Aug (11
days) + Jan (10 days) + Feb (28 days) + March (31 days) = 173 days which is less than 182 days.
Thus Mr. X does not satisfy 1st basic condition.
2nd basic condition is not applicable in his case since he is an Indian Citizen & has left India as member of Indian
Crew. Thus Mr. X is a Non-resident for PY 2018-19.

*Q7. 'M' an Indian citizen left India for the first time on 24.9.2017 for employment in USA. During PY 2018-19, he
comes to India on 5.6.2018 for 165 days. Determine the residential status of 'M' for the PY 2017-18 & PY 2018-19.
Solution:
(a) He is a citizen of India & has left India during PY 2017-18 for employment outside India. Thus 2nd basic
condition is not applicable in his case.
During PY 2017-18, his stay in India is of 177 days (i.e. 30 + 31 + 30 + 31 + 31 + 24). Thus he does not satisfy the
first basic condition ( 182 days or more) & therefore, he is NR in India during PY 2017-18.
(b) During PY 2018-19, his stay in India is for 165 days. Since he is a citizen of India & comes on a visit to India,
2nd basic condition is not applicable in his case.
During PY 2018-19, his stay in India is of 165 days. Thus he does not satisfy the first basic condition ( 182 days or
more) & therefore, he is NR in India during PY 2018-19.

*Q8. Mr. Raj, a citizen of India, is an export manager of XYZ Ltd, an Indian Company, since 1.5.2014. He has been
regularly going to USA for export promotion. He spent following days in USA for last 5 yrs:
PY PY 2018-19 PY 2017-18 PY 2016-17 PY 2015-16 PY 2014-15
Days spent in USA 294 311 271 150 317
Determine his residential status for AY 2019-20 assuming that prior to 1.5.2014 he had never travelled abroad.
Solution: This case does not fall in the exceptions since he has not gone for employment outside India but has
gone out of India during the employment in India. Thus Both Basic Conditions are applicable.
Basic Conditions:
(a) Stay during PY: 71 Days (365-294) & thus 1st basic condition is not satisfied.
(b) (i) Stay in India during PY 2018-19 = 71 days &
(ii) Stay in 4 preceding PYs [48 + 215 + 94 + 55] = 412 days.
PY PY 2018-19 PY 2017 - 18 PY 2016 - 17 PY 2015 - 16 PY 2014 - 15
Days in USA 294 311 271 150 317
Days in India 71 55 94 215 48
Thus, Mr. Raj satisfies 2nd basic condition. Thus he is a resident in India.
Additional conditions:
(a) Stay in India in Last 7 PYs = 55+94+215 +48+365+365+365 = 1507. Thus he satisfy 1st additional condition.
PY 2017-18 2016-17 2015-16 2014-15 2013-14 2012-13 2011-12
Days 55 94 215 48 365 365 365
(b)
PY Stay in India Residential Status
2017-18 55 Days NR
2016-17 94 Days NR
2015-16 215 Days Resident
2014-15 48 Days NR
Prior to 2013-14 Resident
He satisfies 2nd additional condition of being resident in at least 2 out of 10 PY prior to relevant PY.
Since Mr. Raj satisfy both the additional conditions, he is a ROR.

DT QUESTION BANK BY CA PRANAV CHANDAK Subscribe us on YouTube 10


*Q9. Mr. PC, UK citizen has come to India for first time on 1.7.2015 as an executive of a MNC. His employer has
allowed him to visit UK every year & for this purpose he will be leaving India every year on 1 st Nov & shall come
back on 31st Dec. Besides that he has visited China on several occasions in connection with the official work. Details
of the same are given below:
Date of leaving India Date of arriving in India
10.09.2015 30.09.2015
07.02.2016 08.05.2016
11.07.2016 21.10.2016
10.02.2017 23.07.2017
11.02.2018 12.06.2018
01.02.2019 10.04.2019
Determine his residential status for PY 2015-16 to 2018-19. [Modified May 1998]
Solution:
PY 2015-16: No. of days in India during PY 2015-16 = 144. Thus, he is Non – resident for PY 2015-16.
[July -31, Aug -31, Sep - 11, Oct - 31, Nov - 1, Dec - 1, Jan - 31, Feb – 7]

PY 2016-17: No. of days in India during PY 2016 - 17 = 119. Thus, he is Non – resident for PY 2016-17.
[May - 24, June - 30, July - 11, Oct - 11, Nov - 1, Dec – 1; Jan - 31, Feb – 10]

PY 2017-18: No. of days in India during PY 2017 - 18 = 145. Thus, he is Non – resident for PY 2017-18.
[July - 9, Aug - 31, Sep - 30, Oct - 31, Nov - 1, Dec - 1, Jan - 31, Feb -11].

PY 2018-19: (i) No. of days in India during PY 2017 - 18 = 176.


[June - 19, July - 31, Aug - 31, Sep - 30, Oct - 31, Nov - 1, Dec - 1, Jan- 31, Feb – 1]
(ii) Stay in India in Last 4 PYs = 365 days or more so he is resident.
Additional: His stay during 7 years is 729 days or less, hence he is a resident but not ordinarily resident.

Q10. X was born in UK in 1980. His parents were born in Pakistan on 2.2.1947. He has been staying in USA since
1980. He comes to India for 200 days on 10th Oct 2018. Determine residential status of X for AY 2019-20.
Solution:
▪ X is a person of Indian origin as his parents were born in undivided India.
▪ Thus 2nd basic condition is not applicable in his case.
▪ Thus, he can become resident in India only if he visits India for at least 182 days during PY.
▪ In this case, he comes to India on 10th October 2018.
▪ His total stay in India during the previous year ending March 31, 2019 is of 173 days.
▪ He is, therefore, non-resident in India for AY 2019-20.

Q11. Mr. X was born in 1977 in India. His parents were also born in India in 1950. However, his grandparents were
born in England. Mr. X was residing in India till 16.3.2015. Thereafter, he migrated to England & took the
citizenship of that country on 15.3.2016. He visits India during PY 2018-19 for 90 days. Determine the residential
status of Mr. X for AY 2019-20.
Solution:
Mr. X is neither a citizen of India nor a person of Indian origin, because neither he nor his parents nor his
grandparents were born in undivided India.
Thus 2nd basic condition is applicable.
(a) 1st Basic Condition: Stay in India during PY 2018-19: 90 days. Thus he does not satisfy 1st basic condition.
(b) 2nd Basic Condition: (i) Stay in India during PY 2018-19: 90 days;
(ii) Stay in India during Last 4 PY: 350 days
2017-18 Nil 2016-17 Nil
2015-16 Nil 2014-15 350 days
Since he does not satisfy Any Basic Condition, he is a Non- Resident in India.

*Q12. X is a businessman. His parents & grandparents were born in USA. He was born in UK but later on, he
migrated to Karachi & took Indian citizenship on 1st June 1946. After division of India, he stayed in Pakistan & took
Pakistani citizenship in December 1948. Find out the residential status of X for AY 2019-20. His stay in India is:
PY 2018-19 70 days PY 2014-15 260 days PY 2010-11 55 days
PY 2017-18 60 days PY 2013-14 46 days PY 2009-10 25 days
PY 2016-17 40 days PY 2012-13 182 days PY 2008-09 24 days
PY 2015-16 5 days PY 2011-12 55 days

DT QUESTION BANK BY CA PRANAV CHANDAK Subscribe us on YouTube 11


Solution:
▪ X is a foreign citizen. X, his parents & grandparents were not born in undivided India.
▪ He is not a person of Indian origin, even if he was Indian citizen between June 1946 & December 1948.
▪ He is in India for 70 days during PY 2018-19 & 365 days during last 4 PYs. Thus, he is resident in India.

Additional Conditions:
PY Presence in India Residential status
2017 - 18 60 days Non-resident
2016 - 17 40 days Non-resident
2015 - 16 5 days Non-resident
2014 - 15 260 days Resident
2013 - 14 46 days Non-resident
2012 - 13 182 days Resident
2011 - 12 55 days Non-resident
2010 - 11 59 days Non-resident
2009 - 10 25 days Non-resident
2008 - 09 24 days Non-resident
Out of preceding 10 years, X is resident in India for 2 years. However, out of preceding 7 years, X is in India for
648 days. Consequently, X is resident but not ordinarily resident in India for AY 2019-20.

*Q13. Head Office of RG, a Hindu undivided family, is situated in Singapore. Since 1980, the family is managed by
R who is non-resident in India in only 7 out of 10 years preceding the PY 2018-19. Determine the residential status
of the family for the AY 2019-20 if the affairs of the family's business are (a) wholly controlled from Singapore, (b)
partly controlled from India.
Solution:
(a) Since the affairs of the Hindu undivided family are wholly controlled from Singapore i.e. outside India, the
HUF will be non-resident in India.
(b) Since affairs of family's business are partly controlled from India during PY 2018-19, HUF is resident in India.
It is given that Karta is non-resident in 7 out of last 10 PY, this means he is resident in 3 PYs out of Last 10 PYs.
The details regarding second condition are not given.
▪ If Karta’s stay in India during last 7 PYs is 729 days or less, HUF will be RNOR.
▪ If Karta’s stay in India during last 7 PYs is 730 days or more, HUF will be ROR.

Q14. The business of a HUF is transacted from Australia & all the policy decisions are taken there. Mr. E, the karta
of the HUF, who was born in Kolkata, visits India during PY 2018-19 after 15 years. He comes to India on 1.4.2018
& leaves for Australia on 1.12.2018. Determine the residential status of Mr. E & HUF for AY 2019-20.
Solution: Determination of Residential Status of Mr. ‘E’
Basic Condition:
▪ During PY 2018-19, Mr. E has stayed in India for 245 days (30+31+30+31+31+30+31+30+1 days).
▪ Thus he satisfies 1st basic condition & Therefore, he is a Resident in India.
Additional Conditions
▪ Since it is given in the question that he has come to India after 15 years, his total stay in India during Last 7 PYs
is Zero days & thus he does not satisfy 2nd additional condition.
▪ We know that in order to be ROR, Individual must satisfy both additional conditions.
▪ Since Mr. E does not satisfy 2nd additional condition, we do not need to check for 1st additional condition.
▪ Thus Mr. E is a RNOR.
Residential Status of HUF
▪ Since the business of the HUF is transacted from Australia & nothing is mentioned regarding its control &
management, it is assumed that C & M is also wholly outside India. Therefore, HUF is a NR for PY 2018-2019.

Q15. A Ltd. is an Indian Company. It carries on business in New Delhi & London. Place of effective management of
A Limited is situated outside India. 80% of the total income of the company is from the business in London. What
is the residential status of A Ltd.?
Solution:
▪ As A Ltd. is an Indian Company, it is always resident in India even if its POEM is outside India.
▪ It is irrelevant that 80% of the total income of the company is from the business outside India.

DT QUESTION BANK BY CA PRANAV CHANDAK Subscribe us on YouTube 12


Q16. XYZ Ltd is registered in China. All the meetings of BODs of XYZ Ltd were held in India during PY 2018-19.
Determine the residential status of XYZ Ltd. for AY 2019-20.
Solution:
▪ XYZ Ltd is not Indian Company.
▪ Thus to determine the residential status of XYZ Ltd, we need to check where is the POEM of XYZ Ltd.
▪ It is given in the question that All the board meetings are held in India.
▪ Thus POEM is in India. Thus XYZ is Resident in India.

Q17. ABC Ltd is registered in India. All the meetings of BODs of ABC Ltd were held in China during PY 2018-19.
Determine the residential status of ABC Ltd. for AY 2019-20.
Solution:
▪ As ABC Ltd. is an Indian Company, it is always resident in India even if its POEM is outside India.
▪ It is irrelevant that all the board meetings are held in China.

*Q18. ABC Ltd., a Swedish company headquartered at Stockholm, not having a permanent establishment in India,
has set up a liaison office in Mumbai in April, 2018 in compliance with RBI guidelines to look after its day to day
business operations in India, spread awareness about the company’s products & explore further opportunities.
The liaison office takes decisions relating to day to day routine operations & performs support functions that are
preparatory & auxiliary in nature. The significant management & commercial decisions are, however, in substance
made by the Board of Directors at Sweden. Determine the residential status of ABC Ltd. for AY 2019-20.
Answer:
▪ As per Section 6(3), a company would be resident in India in any previous year, if
(i) it is an Indian company; or
(ii) its place of effective management, in that year, is in India.
▪ In this case, ABC Ltd. is a foreign company. Therefore, it would be resident in India for PY 2018-19 only if its
place of effective management, in that year, is in India.
▪ “Place of effective management” means a place where key management & commercial decisions that are
necessary for the conduct of the business of an entity as a whole are, in substance made.
▪ In case of ABC Ltd., its place of effective management for PY 2018-19 is not in India, since the significant
management & commercial decisions are, in substance, made by the Board of Directors outside India in Sweden.
▪ ABC Ltd. has only a liaison office in India through which it looks after its routine day to day business operations
in India.
▪ It cannot be said to have POEM in India just because of this liaison office in India since having a place where
decisions relating to day to day routine operations are taken & support functions that are preparatory or
auxiliary in nature are performed are not relevant in determining the place of effective management.
▪ Hence, ABC Ltd. being a foreign company is a non-resident for AY 2019-20, since its place of effective
management is outside India in the PY 2018-19.

Q19. In a partnership firm, there are 3 partners namely A, B & C. A & B reside in India while C lives in Germany.
The firm is fully controlled by C. During PY 2018-19, Mr. C stayed for 6 months in India. Determine the residential
status of the partnership firm for AY 2019-20.
Solution:
▪ A partnership firm is said to be resident in India if C & M of its affairs is wholly/partly situated in India.
▪ It is given in the question that the Firm is controlled by Mr. C. & Mr. C was in India for 6 months in this PY,
Control & Management can be said to have been in India during this PY & thus Firm is a Resident in India.

Q20. Determine the residential status in the following cases for AY 2019-20:
(i) The control & management of a HUF is situated in India. The manager of the HUF visited England with his wife
from 14.8.2018 to 30.6.2019. Earlier to that he was always in India.
(ii) Company, whose registered office is in America, has a place of its effective management during PY in India.
(iii) A VIP Club is in India, whose director Mr. X belongs to China. The Club is controlled fully by Mr. X. In PY 2018-
19, Mr. X did not come for a single day to India.
Solution:
(i) HUF is a resident in India, as it is partly controlled from India. Further, the Karta of the HUF satisfies both the
additional conditions. He was resident in at least 2 out of 10 PY prior to relevant PY & was in India for 730 days
or more in the 7 preceding PYs. Hence, the Karta is "ROR in India".
(ii) Company is resident in India as is place of effective management in the PY is in India.
(iii) VIP Club is a Non-Resident as no part of the control & management was in India.

DT QUESTION BANK BY CA PRANAV CHANDAK Subscribe us on YouTube 13


Q21. Mr. David, a Government employee serving in the Ministry of External Affairs, left India for the first time on
31.03.2018 due to his transfer to High Commission of Canada. He did not visit India any time during PY 2018-19. He
has received the following income for PY 2018-19. Discuss tax-treatment of the following receipts.
Particulars Rs.
(i) Salary 5,00,000
(ii) Foreign Allowance 4,00,000
(iii) Interest on fixed deposit from bank in India 1,00,000
(iv) Income from Agriculture in Pakistan 2,00,000
(v) Income from House property in Pakistan 2,50,000
Solution: Mr. David is a NR for AY 2019-20, since he was not present in India at any time during PY 2017-18.
Thus Only Indian Income (being taxable to everyone) will be taxable in his hands.
▪ Income from Agriculture in Pakistan & Income from house property in Pakistan → Not taxable.
▪ Income from ‘Salaries’ payable by GOI to a citizen of India for services rendered o/s India is deemed to accrue
or arise in India as per section 9(1)(iii). Hence, such income is taxable in the hands of Mr. David, even though
he is a non- resident. It has been assumed that Mr. David is a citizen of India.
▪ Allowances/perquisites paid or allowed as such outside India by GOI to citizen of India for rendering service
o/s India is exempt u/s 10(7). Hence, foreign allowance of Rs. 4,00,000 is exempt.

Q22. X is a citizen of Pakistan. His maternal grandfather was born in a village near Karachi in 1932. He comes to
India on April 7, 2017 to complete a foreign assignment of his employer-company. He goes back on 4.10.2018. Mrs.
X is resident & ordinarily resident in India for PY 2018-19. Prior to April 7, 2018, X was in India as follows –
PY 2017-18 2016-17 2015-16 2014-15 2013-14 2012-13
Stay in India 179 182 180 72 175 Nil
Income of X for the previous year 2018-19 is as follows –
Salary Income from Pakistani company (received in Pakistan) (for rendering service in Pakistan) 14,34,000
Salary Income from Pakistani company (received in Pakistan) (for rendering service in India) 6,63,000
Business income (Business is situated in UAE & income is received in UAE) (Business is partly 9,95,000
controlled from India from Pakistan & partly from India)
Rental income from a property situated in Mumbai (income is received in Pakistan) [Computed] 2,10,000
Business income (business is situated in UK & it is partly controlled from UK & partly from 10,72,000
Pakistan) (income is received in UK & later on Rs. 50,000 is remitted to India)
Birthday gift (he celebrated his birthday on June 10, 2018, received cash gifts in India from his 74,000
friends, some of them have sent gift cheques from Pakistan)
Find out residential status & taxable income of X for the AY 2019-20.
Solution: X is a foreign citizen. One of his grandfathers was bom in undivided India in 1932. Consequently, he is a
person of Indian origin.
▪ He comes to India on a visit from April 7,2018 to October 4,2018. His total stay in India during PY 2018-19 is
181 days (i.e., 24 days + 31 days + 30 days + 31 days + 31 days + 30 days + 4 days).
▪ He can become resident in India only if he is in India for at least 182 days.
▪ He is, therefore, non-resident in India for AY 2019-20.
Income of X for AY 2019-20 shall be calculated as follows:
Particulars Nature of income Rs.
Salary of Rs. 14,34,000 for rendering service in Pakistan Foreign income Nil
Salary of Rs. 6,63,000 for rendering service in India Indian income 6,63,000
Business income of Rs. 9,95,000 Foreign income Nil
Rental income from Mumbai property Indian income 2,10,000
Business income of Rs. 10,72,000 Foreign income Nil
Birthday gift Indian income 74,000
Net income 9,47,000

Q23. Compute the total income for the AY 2019-20 of Mr. A if he is: (i) ROR (ii) RNOR (iii) NR [RTP N-14]
Particulars Rs.
(a) STCG on sale of shares in Indian Company received in Germany 15,000
(b) Dividend from a Japanese Company received in Japan 10,000
(c) Rent from property in London deposited in a bank in London, later on remitted to India 75,000
(d) Dividend from RP Ltd., an Indian Company 6,000
(e) Agricultural income from lands in Gujarat 25,000

DT QUESTION BANK BY CA PRANAV CHANDAK Subscribe us on YouTube 14


Solution: Computation of total income of Mr. A for AY 2019-20
Particulars ROR RNOR NR
(a) STCG on sale of shares of an Indian co., received in Germany 15,000 15,000 15,000
(b) Dividend from a Japanese company, received in Japan 10,000
(c) Rent from property in London deposited in a bank in London [Note (i)] 52,500
(d) Dividend from RP Ltd., an Indian Company [Note (ii)]
(e) Agricultural income from land in Gujarat [Note (iii)]
Total Income 77,500 15,000 15,000
Notes:
(i) GAV = Rent Received. Standard deduction u/s 24 @ 30% has been provided & NAV is taken into account for
determining the total income of a ROR. Income from house property = Rs. 75,000 – 30% = Rs. 52,500.
(ii) Dividend from Indian company is Exempt u/s 10(34) upto 10 lacs.
(iii) Agricultural income is exempt u/s 10(1).

Q24. Compute the taxable income of DJ for AY 2019-20 assuming his to be (1) ROR (2) RNOR (3) NR.
Interest received in London on money lent in London to Resident in India, but money used in India 46000
Remuneration for consultancy service in Japan but received in India 100000
FTS payable by G (Resident) related to a business carried on in India 50000
Royalty received from Government of India 20000
Royalty from S (resident) for technical services provided to a business o/s India but received in India 8000
FTS paid by a Non-Resident for a business outside India 20000
Solution: Computation of Taxable Income of MR. DJ for AY 2019-20
ROR RNOR NR
46,000 46,000 46,000
1,00,000 1,00,000 1,00,000
50,000 50,000 50,000
20,000 20,000 20,000
8,000 8,000 8,000
20,000 - -

Q25. Mr. Ramesh & Mr. Suresh are brothers & they earned the following income during PY 2018-19. Mr. Ramesh
settled in Canada in the year 2001 & Mr. Suresh settled in Delhi. Compute the Total Income for the AY 2019-20.
SN Particulars Ramesh Suresh
1 Interest on Canada Development Bond (50% of Interest is received in India) 35,000 40,000
2 Dividend from British Company received in London 28,000 20,000
3 STCG on sale of Shares of an Indian Company received in India 60,000 90,000
4 Fees for Technical Services rendered in India, but received in Canada 1,00,000 -
5 Interest on Savings Bank Deposit in UCO Bank, Delhi 7,000 12,000
6 Agricultural Income from a land situated in Andhra Pradesh 55,000 45,000
7 Life Insurance Premium paid - 30,000
Solution:
(a) Ramesh stayed in India for only 20 days every year. Hence, he is a Non-Resident (NR).
(b) Suresh stayed in India for 365 days & hence he is a resident for all 10 preceding PYs. Hence, he is a ROR
Taxability of Income:
NR: Incomes which are deemed to accrue in India (or) Received or deemed to be received is in India, are taxable.
ROR: All Incomes are taxable in India.
Computation of Total Income for the AY 2019-20
SN Particulars Ramesh (NR) Suresh (ROR)
1 Interest on Canada Development Bond (50% is received in India) 17,500 40,000
2 Dividend from British Company received in London - 20,000
3 STCG on sale of Shares of an Indian Company received in India 60,000 90,000
4 Fees for Technical Services rendered in India, but received in 1,00,000 -
Canada

DT QUESTION BANK BY CA PRANAV CHANDAK Subscribe us on YouTube 15


5 Interest on Savings Bank Deposit in UCO Bank, Delhi 7,000 12,000
6 Agricultural Income from a land situated in Andhra Pradesh - -
Gross Total Income 1,84,500 1,62,000
Less: Deductions u/s 80TTA: Interest on Savings Bank (Max: 10,000) - (10,000)
Total Taxable Income 1,84,500 1,52,000

Q26. Discuss the tax treatment of the following receipts in the hands of PC, who is a Non-Resident for AY 2019-20:
(i) Interest paid by Government of India.
(ii) Interest paid by Mr. R, resident for money borrowed for earning capital gains outside India.
(iii) Amount paid by Government of India for use of a patent developed by Mr. PC.
(iv) Royalty paid by Mr. S, a non-resident. Royalty is payable in respect of an information used for profession
carried on by Mr. S in India.
(v) Income from operations confined to the purchase of goods in India for Export.
(vi) Income from operations confined to shooting of a cinematography film in India. Mr. PC is Indian citizen.
Solution: For Non-Resident, only Indian Income is taxable. Accordingly -
(i) Interest paid by GOI shall be taxable for Mr. PC as it is deemed to accrue in India [Section 9(i)(v)(a)].
(ii) Interest paid by R is not a deemed to accrue in India since the money borrowed is used by NR for earning
Capital gains outside India. Thus it is not taxable in the hands of Mr. PC unless it is received in India.
(iii) Amount paid for use of a patent is Royalty & since it is paid by Government of India, it is deemed to accrue in
India & hence it is an Indian Income. Thus it is Taxable to Mr. PC.
(iv) Royalty paid by S (Non-Resident) is deemed to accrue in India since it is given for the profession in India. Thus
it is an Indian income & Hence it is taxable in India in the hands of Mr. PC.
(v) Income from purchase of goods in India for Export is not treated as Business Connection in India. Thus it not
taxable u/s 9(1)(i)(ii) in the hands of Mr. PC.
(vi) Income from shooting of a cinematograph film in India is Taxable in India as PC is an Indian Citizen.

Q27. A Chinese Company entered in to the following transactions during PY 2018-19. Explain briefly, whether,
these receipts are chargeable to Tax in India.
(i) Received Rs. 20 Lacs from a Non-Resident for use of Patent for a business in India.
(ii) Received Rs. 15 Lacs from a Non-Resident Indian for use of Know-How for a business in Sri Lanka & this amount
was received in Japan.
(iii) Received Rs. 7 Lacs from RR Co. Ltd, an Indian Company, for providing Technical Know-How in India.
(iv) Received Rs. 5 Lacs from R & Co. Mumbai for conducting the Feasibility Study for a new project in Nepal, & the
payment was made in Nepal.

Solution:
(i) Taxable. Payer, being a Non-Resident, Income received for Patent used for business in India, is deemed to accrue
or arise in India.
(ii) Not Taxable. Payer, being a Non-Resident, Income Received for use of Know-How for a Business outside India,
is not deemed to accrue or arise in India.
(iii) Taxable. Payer is an Indian Company, i.e. Resident in India, & also Technical Know-How is used for business
in India. Hence, deemed to accrue or arise in India.
(iv) Not Taxable. Payer is assumed as a Resident in India. Fees for Feasibility Study conducted for business outside
India, is not deemed to accrue or arise in India.

Q28. Discuss the tax treatment in India in the hands of US-based Company having its POEM outside India on the
following transactions & also discuss the rate of tax applicable to such company.
(a) Rs. 5 Lacs received from an Indian Domestic Company for providing Technical Know-how in India.
(b) Rs. 6 Lacs from an Indian Firm for conducting the feasibility study for the new project in Finland.
(c) Rs. 4 Lacs from a Non-Resident for the use of Patent for a Business in India.
(d) Rs. 8 Lacs from a Non-Resident Indian for use of know-how for a Business in Singapore.
(e) Rs. 10 Lacs for supply of manuals & designs for the Business to be established in Singapore.

Solution: In this case, US-based Company has its Place of Effective management outside India. So it is a NR.
(a) Rs. 5,00,000 will be Taxable since the payer [Indian Company (Resident in India)] has used the technical know-
how for business in India. Hence, such income is deemed to accrue or arise in India.

DT QUESTION BANK BY CA PRANAV CHANDAK Subscribe us on YouTube 16


(b) Not Taxable since Payer is an Indian Firm, & a Resident in India. Fees for Feasibility Study conducted for
business outside India is not deemed to accrue or arise outside India.
(c) Rs. 4,00,000 will be Taxable since Payer, being a Non-Resident, Income received for Patent used for business
in India, is deemed to accrue or arise in India.
(d) Not Taxable since Payer, being Non-Resident, income received for use of know-how for a business outside
India, is not deemed to accrue or arise in India.
(e) Income received for supply of manuals & designs for business outside India, is not deemed to accrue or arise
in India. Thus it is not taxable to NR.

Taxable Income = Rs. 9 Lacs; Rate of Tax + HEC = 40 % + 4 % of 40% = 3,60,000 + 14,400 = Rs. 3,74,400.

*Q29. State with reasons whether the following attract Income Tax in India in the hands of Recipients -
1. Salary of Rs. 7,00,000 paid by CG to Mr. John, a Citizen of India, for the services rendered outside India.
2. Interest on moneys borrowed from outside India Rs. 5,00,000 by a Non-Resident for the business within India.
3. Post Office Savings Bank Interest of Rs. 12,000 received by a Resident Assessee.
4. Royalty paid by a Resident to a Non-Resident in respect of business carried on outside India.
5. Legal Charges of Rs. 5,00,000 paid to a Lawyer of UK who visited India to represent a case at the Delhi High
Court.
6. X Ltd. is a foreign company. It has 10 shareholders, all of them are non-resident & foreign citizens. It has shot a
feature film on location situated in Kerala. The film will be exhibited in UK. However, Kerala Government has
power to telecast it in South India free of charge.
7. Mr. X is a foreign citizen. He is a person of Indian origin. During the previous year 2018-19, X is in India for a
visit of 150 days. He holds 1,000 equity shares in Reliance Industries. However, share certificates are in the office
of X situated in Paris where X normally resides. These shares are transferred by X to a foreign company in the UK
by signing the agreement in the UK. Certificates are handed over to the purchaser in the UK & consideration is
received by X in foreign currency in the UK. On this transaction, X has generated a long-term capital gain of Rs.
95,00,000.
8. Y is a citizen of the UK. He is non-resident in India. He is employed by the Indian High Commissioner in the UK.
Salary is paid to him in the UK in pound sterling. (b) Does it make any difference if Y is an Indian citizen?
Answer:
1. Payer is CG. Salary paid for rendering services outside India, deemed to accrue or arise in India. However,
exemption is available u/s 10(7) for allowances or perquisites.
2. Taxable. U/s 9(1 )(v), Interest paid by Non-Resident for the purpose of carrying on Business or Profession in
India is deemed to accrue or arise in India.
3. Taxable. Any Income received by Resident is taxable in India.
4. Not Taxable. Payment relating to a Business/Profession carried on o/s India, not deemed to accrue or arise in
India.
5. Taxable. Legal Charges paid to a Non-Resident for earning any source of Income in India is deemed to accrue or
arise in India.
6. In the case of a non-resident no income shall be deemed to accrue or arise in India through or from operations
which are confined to the shooting of any cinematograph film in India. This rule is applicable only if such non-
resident is a foreign citizen or if such resident is a company, the company should not have any shareholder who is
a citizen of India or is resident in India. In this case, nothing is taxable in the hands of X Ltd.
7. Since X is in India only for 150 days, he is non-resident in India. In the case of non-resident, Indian income is
chargeable to tax. Shares in a company incorporated in India are always situated in India. On transfer of such
shares, capital profit is deemed to accrue or arise in India [sec. 9(1)(i)]. This rule is applicable even if share
certificates are handed over to the purchaser outside India under an agreement made outside India for which
consideration is also payable outside India in foreign currency. Capital gain of Rs. 95,00,000 is taxable as income
of X in India.
8. If Services are rendered outside India, Income is deemed to accrue or arise outside India. It is received outside
India & the employee is non-resident in India. It is a foreign income & not taxable in India in the hands of non-
resident employee.
(b) However, Section 9(1)(m) provides an exception to this rule. If the employee is a citizen of India & service is
rendered to the Government of India o/s India, salary paid to such Citizen is deemed to be accrued in India & thus,
it is an Indian Income. If Y is a citizen of India, then income is deemed to accrue or arise in India & chargeable to
tax in India.

DT QUESTION BANK BY CA PRANAV CHANDAK Subscribe us on YouTube 17


Q30. Ms. Vivitha paid a sum of $ 5,000 to Mr. Kulasekhara, a Management Consultant practicing in Colombo,
specializing in Project Financing. The payment was made in Colombo. Mr. Kulasekara is Non-Resident. The
Consultancy is related to a project in India with possible Ceylonese collaboration. Is this payment chargeable to
tax in India in the hands of Mr. Kulasekhara, since the services were used in India?

Solution:
▪ Fees for Technical Services paid by Resident or Non-Resident, for the purpose of carrying on Business or
Profession in India or to earn any Income from any Source in India is deemed to accrue or arise in India.
▪ Hence, it is taxable for all Assessees, irrespective of their Residential Status.
▪ The above payment of $ 5,000 made by Ms. Vivitha to Mr. Kulasekhara, a Non-Resident, related to a project in
India, is taxable in the hands of Mr. Kulasekhara, in India.

Q31. A firm of solicitors in Calcutta engaged a barrister of London for arguing a case before the Supreme Court of
India. A payment of 10,000 Pounds was made to the barrister in London as per the terms of the professional
engagement. It is claimed that since the payment is made outside India, no tax is payable on the fee paid. Is the
claim correct.

Solution: The Problem is based on a case decided by the Supreme Court in ITO v Barendra Prasad Ray & Others
(1981) 129 ITR 295 in which the court held that business connection includes professional connections also. In
view of the aforesaid case, the fee paid to the barrister shall be deemed to accrue or arise in India if the barrister
firm has any business connection in India.

Q32. Mr. Federer, a Non-Resident residing in Sweden, has received rent from Mr. Nadal, another NR residing in
France in respect of a property taken on lease at Mumbai. Since this income is received outside India from a NR,
Federer claims that his income is not chargeable to tax in India. [Nov 2016]
Answer: As per Sec. 9(1 )(i), if the Source of Income, directly or indirectly, through or from a Property, Asset in
India, it shall be deemed to accrue or arise in India. Hence, the Rent is taxable in India.

Q33. Mr. A, a Citizen of India, left for USA for the purposes of employment on 01.05.2018. He has not visited India
thereafter. Mr. A borrows money from his friend Mr. B who left India one week before Mr. A's departure, to the
extent of Rs. 10 Lacs & buys Shares in X Ltd, an Indian Company. Discuss the taxability of the interest charged at
10% in B's hands where the same has been received in New York.
Answer:
▪ The Receiver “B” is a Non-Resident. U/s 9(1 )(v), Interest paid by Non-Resident (Mr. A) to any person (Mr. B)
(other than for carrying on business or profession in India) is not deemed to accrue or arise in India.
▪ Interest Received by Mr. B from Mr. A is not taxable in India.

*Q34. Discuss the correctness or otherwise of the statement - “Income deemed to accrue or arise in India to a NR
by way of interest, royalty & fees for technical services is to be taxed irrespective of territorial nexus”.
Answer:
▪ As per section 9, if any non-resident has provided any patent right or any managerial, technical services & such
patent right etc was used in India, in such cases any royalty or fee received by nonresident shall be considered
to be income accruing/arising in India & shall be taxable & it do not matter that the non-resident do not have
residence or place of business or business connection in India i.e. there is no territorial nexus or non-resident
has not rendered services in India. E.g. If Suzuki Incorporation of Japan a non-resident company has provided
technical know-how in Japan to Maruti Udyog Limited for use in India & has received Rs. 300,00,000 in this
case, such income is deemed to be accruing/arising in India & is taxable in India even if Suzuki Incorporation
do not have any Territorial Nexus with India i.e. the company do not have place of residence or place of business
in India. Similarly if any loan was given by a nonresident to some other non-resident & such other non-resident
has utilized loan amount in India in business/profession, interest received by the non-resident shall be
considered to be his income accruing/arising in India even if such non-resident do not have any territorial
nexus with India.

DT QUESTION BANK BY CA PRANAV CHANDAK Subscribe us on YouTube 18


MASTER QUESTION ON RESIDENTIAL STATUS
MQ35. Compute total income of Mr. PC assuming him (i) ROR (ii) RNOR (iii) NR for AY 2019-20.
Particulars Amt
1. Interest on UK Development Bonds, (50% of interest received in India) 10,000
2. Interest for debentures in an Indian company (received in London) 10,000
3. Income from a business in Chennai managed from London (50% is received in India) 20,000
4. Profits on sale of shares of Indian company (received in London) 20,000
5. Dividend from British Company (received in London) 5,000
6. Profits on Sale of Plant at Germany (50% of profits are received in India) 40,000
7. Business income in Germany which is controlled from Delhi (40,000 is received in India) 70,000
8. Profits from a business in Delhi but managed entirely from London 15,000
9. Income from House in London deposited in Indian Bank at London, brought to India (computed) 50,000
10. Royalty/Fees for technical services rendered in India (received in London) 8,000
11. Pension for services rendered in India (received in Burma) 4,000
12. Income from property situated in Pakistan received there 16,000
13. Past foreign untaxed Income brought to India during the PY 5,000
14. Income from agricultural land in Nepal received there & then brought to India 18,000
15. Income from profession in Kenya which was set up in India, received there but spent in India 5,000
16. Gift received on the occasion of his wedding 20,000
17. Income from a business in Russia, controlled from Russia 20,000
18. Dividend from Reliance Petroleum Limited, an Indian company 5,000
19. Honorarium received from Government of India (Rs. 15,000 was paid for travelling expenses) 20,000
20. Income from Business connection in India, received in London 10,000
21. Speculation profit earned & received outside India on 15.4.2018 20,000
22. Salary drawn for 2 months for working in Indian Embassy's Office in Australia & received there 80,000
Solution: Computation of Total Income for AY 2019-20
Particular ROR RNOR NR
1. Interest on UK Development Bonds. 10,000 5,000 5,000
It is foreign Income. But 50% of interest received in India is Indian Income.
2. Interest for debentures in an Indian company 10,000 10,000 10,000
Since Interest is paid on debentures by Indian company, it is an Indian Income. Thus, taxable to Everyone.
3. Income from a business in Chennai (50% is received in India) 20,000 20,000 20,000
Since business is situated in India, 100% is Indian Income irrespective of the place where it is Managed from.
4. Profits on sale of shares of Indian company received in London 20,000 20,000 20,000
5. Dividend from British company received in London [Foreign Income] 5,000 - -
6. Profits on sale of plant at Germany [50% Foreign & 50% Indian Income] 40,000 20,000 20,000
7. Income earned from business in Germany which is controlled from Delhi 70,000 70,000 40,000
Since the business has been controlled from India, such foreign income is taxable to RNOR also.
8. Profits from business in Delhi [Indian Income & thus taxable to everyone] 15,000 15,000 15,000
9. Income from property in London deposited in London, later on remitted 50,000 - -
to India [Foreign Income & thus taxable to ROR]; No tax on Remittance
10. Royalty/FTS rendered in India; Indian Income & thus taxable to everyone 8,000 8,000 8,000
11. Pension for services rendered in India & [Thus Indian Income] 4,000 4,000 4,000
12. Income from property situated in Pakistan [Foreign Income; tax – ROR] 16,000 - -
13. Past foreign untaxed income brought to India during the PY - - -
Since income relates to past year, it is assumed that it must have been taxed in that PY. Thus, it is not taxable in this PY.
14. Income from Agricultural land in Nepal received there [Foreign Income] 18,000 - -
15. Income from profession in Kenya which was set up in India [Refer Pt. 7] 5,000 5,000 -
16. Gift received on the occasion of his wedding [not taxable] - - -
17. Income from business in Russia, controlled from Russia [Foreign Income] 20,000 - -
18. Dividend from Reliance Limited, Indian Company [Exempt u/s 10(34)] - - -
19. Honorarium received from Government of India [Indian Income]. But 5,000 5,000 5,000
Allowance are exempt u/s 10(7). Thus, travelling expenses are not taxable.
20. Income from Business connection in India, received in London 10,000 10,000 10,000
21. Speculation profit earned & received outside India [Foreign Income] 20,000 - -
22. Salary for working in Indian Embassy's Office in Australia & received 80,000 80,000 80,000
there. [Deemed to accrue in India & thus Indian Income]

DT QUESTION BANK BY CA PRANAV CHANDAK Subscribe us on YouTube 19


3. EXEMPT INCOMES
Q1. Briefly discuss about the provisions relating to deductibility of expenditure incurred in relation to income not
includible in assessee's total income. [PCC May 2007]
Answer: As per Section 14A, while computing the total income of an assessee -
(1) No deduction shall be allowed to the assessee in respect of expenditure incurred in relation to income which
does not form the part of the total income under the Act.
(2) The Assessing Officer shall determine the amount of expenditure incurred in relation to such income, if having
regard to the accounts of the assessee, he is not satisfied with the correctness of the claim of the assessee in respect
of such expenditure made by the assessee.
(3) The Assessing Officer has the power to determine the amount of such expenditure even if where an assessee
claims that no expenditure has been incurred by him in relation to income which does not form part of the total
income under this Act.

Q2. State with reasons whether the receipt is taxable under the Income Tax Act, 1961 for AY 2019-20?
(i) Mr. Suri received a sum of Rs. 5,00,000 as compensation, from 'Yatra Foundation', towards the loss of property
on account of Flood Disaster. [Nov 16]
Answer:
▪ Disaster Compensation received/receivable from CG/SG/Local Authority is exempt u/s 10(1OBC).
▪ In this case, since Yatra Foundation is not covered in the above, the receipt is taxable in Suri’s hands.
(ii) Any amount received by an individual or his legal heir as compensation for natural disaster from the
Government, is taxable. Comment. [May 16]
Answer:
▪ According to Section 10(10BC), any amount received or receivable as compensation by an individual or his
legal heir on account of any disaster is exempt from tax if Such compensation should be granted by CG/SG/LA.
▪ However, such exemption would not be available in respect of compensation for alleviating any damage or loss,
which has already been allowed as deduction under the Act.
(iii) Rent Received for letting out Agricultural Land for a movie shooting – Rs. 7,00,000. [Nov 2013]
Answer: Rent Received for letting out Agricultural Land for a movie shooting is fully taxable, as it is not considered
as Agricultural Income.

Q3. The Government of India pays a salary of US $ 80,000 to a Non-Resident for services rendered by him in USA.
Such NR is Indian Citizen. In addition, he gets allowances & Perquisites of US$ 20,000. Discuss the tax consequence.
Solution:
▪ As per section 9(1)(iii), salary income for service rendered outside India shall be deemed to accrue or arise in
India if all the following conditions are satisfied:
(a) Such salary is paid to an individual who is a citizen of India.
(b) Salary must be paid by Government.
(c) It should be paid for services rendered outside India.
▪ However any allowances/perquisites paid outside India by the Government shall be exempt u/s 10(7).
▪ Thus salary of US $ 80,000 is taxable in India. But Allowances & perquisites of US $ 20,000 is exempt u/s 10(7).

Q4. Ms. J, a Sikkimese woman married Mr. K, a Non-Sikkimese on 1st Jan 2009. During PY 2018-19, she received
rent of Rs. 12 Lacs from letting out house properties situated in Sikkim. Is she liable to Income Tax for AY 2019-
20? (b) Will your solution be different, if she married Mr. K on 16th April, 2013. [May 09]
Answer:
U/s 10(26AAA),in case of an Individual, being a Sikkimese, any income which accrues or arises to him from the
following sources are exempted (i) from any Source in Sikkim & (ii) Dividend or Interest on Securities.
Exception: This provision shall not apply to a Sikkimese woman who on/after 1.4.2008 marries an individual who
is not Sikkimese.
Here, the Assessee marries a Non-Sikkimese individual before 1.4.2008. So, the exemption u/s 10(26AAA) shall be
allowed in her case. Hence, Ms. J is eligible for exemption in respect of House Property Income situated in Sikkim.
(b) Marriage occurred on 16th April 2013. i.e. after 1.4.2008. So, Ms. J is not eligible for exemption u/s 10(26AAA).

Q5. Calculate the amount of tax liability of Mrs. Mahima who is 50 years old & resident in India for AY 2019-20:
Share of profit from partnership firm 40,000
Income from salary (Computed) 2,80,000
Receipt of accumulated balance in PPF 80,000
Lottery income (Gross) 7,000

DT QUESTION BANK BY CA PRANAV CHANDAK Subscribe us on YouTube 20


Solution: Computation of taxable income of Mrs. Mahima
(i) Income from salary 2,80,000
(ii) Income from other sources: Lottery Income 7,000
Gross Total Income 2,87,000
Tax on Rs. 7,000 of lottery income @ 30% 2,100
Tax on Rs. 2,80,000 1,500 3,600
Less: Rebate u/s 87A: (Not available from Lottery Income) (1,500)
Total Tax 2,100
Add: HEC @ 4% 84
Total tax payable (rounded off) 2,180
Notes:
1. Share of profit from firm is exempt u/s 10(2A).
2. Receipt of accumulated balance in PPF is exempt u/s 10(11).

Q6. Calculate the amount of taxable income of Mr. Raju for AY 2019-20:
Income from Horse Race 20,000
Income from Lottery (Gross) 30,000
Salary Income (Computed) 3,60,000
Income of his minor child 11,500
Income of his wife 2,00,000
Solution: Computation of taxable income of Mr. Raju
(i) Income from salary 3,60,000
(ii) Income from other sources
Lottery Income 30,000
Horse Race 20,000 50,000
Minor Child Income 11,500
Less: Exemption u/s 10(32) (1,500) 10,000 60,000
Gross Total Income 4,20,000
Note: Minor's income will be clubbed in the hands of that parent whose income from all other sources is higher.

Q7. Determine the taxability of the following sums received by Mr. Tularam from LIC on account of the Life
Insurance Policies taken by him –
Date of Policy Person insured Sum Assured Annual Bonus received
Premium during PY 2018-19
20.10.2011 Minor Son 5,00,000 75,000 60,000
10.06.2015 Spouse 2,00,000 30,000 15,000
11.04.2018 Handicapped daughter 8,00,000 1,00,000 18,000
Answer: Taxability of Sums received from Insurance Policies
Date of Policy Taxability
20.10.2011 Sum received is Exempt u/s 10(10D), as the policy is issued on or before 31.3.2012 & the
Annual Premium does not exceed 20% of the Actual Capital Sum Assured.
10.06.2015 Sum received is taxable, as the policy is issued after 1.4.2012, & the annual premium
exceeds 10% of the Actual Capital Sum Assured.
11.04.2018 Sum received is exempt u/s 10(10D), as policy is issued after 1.4.2013 for insuring the Life
of handicapped daughter & Annual Premium ≤ 15% of the Actual Capital Sum Assured.

*Q8. X Ltd. is a manufacturing & trading company. It owns 3 units. Unit A manufactures goods in a special economic
zone since 2016 for export purposes & qualified for exemption u/s 1OAA. Unit Bis a manufacturing unit & goods
are sold in domestic market. It is not qualified for any tax holiday. Unit C owns retail outlets in different parts of
the country. From the information given below, find out net income of X Ltd. for AY 2019-20:
Particulars Unit A Unit B Unit C
Net profit as per profit & loss account 90 (-) 40 10
Total Turnover 1200 400 150
Export Turnover included in Total Turnover 1180 10 -
Amount remitted in convertible foreign exchange upto 30 Sep 2019 1002 2 -
Freight & insurance (charged over & above sale price from importers & included 10 - -
in amount remitted in convertible foreign exchange & turnover above)

DT QUESTION BANK BY CA PRANAV CHANDAK Subscribe us on YouTube 21


Solution: Exemption u/s 10AA shall be calculated as follows:
Net profit of Unit A (a) 90
Export turnover of Unit A (Rs. 1002 Lac - freight & insurance : Rs. 10 Lac) (b) 992
Total turnover of Unit A (Rs. 1200 Lac - freight & insurance : Rs. 10 Lac) (c) 1190
Dedution u/s 10AA = Profits from unit in SEZ ×
Export Turnover of unit in SEZ
= 90 ×
992 75.03
Total turnover of Unit in SEZ 1190

Computation of Income of X Ltd:


Income of Unit A (Rs. 90 Lac - deduction u/s 10AA : Rs. 75.03 Lac) 14.97
Income of Unit B (40)
Income of Unit C 10
Loss to be carried forward (15.03)

Q9. Mr. Pranay is running two Industrial Undertakings, one in a SEZ (Unit A) & another in a DTA (Unit B). The brief
details for the year ended 31.03.2018 are as under: (amounts Rs. in Lacs) [M 11 (Mod), N 13]
Particulars Unit A Unit B
Domestic Turnover 10 100
Export Turnover 120 Nil
Gross Profit 20 10
Less: Expenses & Depreciation 07 06
Profits derived from the Units 13 04
Brought Forward Business Loss pertaining to AY 2016-2017 for Unit B is Rs. 3.2 Lacs. Briefly compute the Business
Income of the Assessee.
Solution: Computation of Business Income
Particulars A B
Profits derived from the Units 13 4
Less: Exemption u/s 10AA [Profit of Business of the Undertaking x Export Turnover/ (12)
Total Turnover = 13.00 × 120/130]
Taxable Profits 1.00 4.00
Less: Brought Forward Business Loss - (3.20)
Business Income 1.00 0.80
Note: It is assumed that the above Financial Year falls within the first 5-year period commencing from the year
of manufacture or production of articles / things, or provision of services, by Unit A.

Q10. Rudra Ltd has one unit at Special Economic Zone (SEZ) & other unit at Domestic Tariff Area [May 2015]
Particulars Rudra Ltd ( Rs. ) Unit in DTA ( Rs. )
Total Sales 6,00,00,000 2,00,00,000
Export Sales 4,60,00,000 1,60,00,000
Net Profit 80,00,000 20,00,000
Calculate the eligible deduction u/s 10AA of Income-Tax Act, for the AY 2019- 2020, in the following situations:
(i) If both the units were set up & start manufacturing from 22-05-2012.
(ii) If both the units were set up & start manufacturing from 14-05-2016.
Answer: Profits derived from Unit in SEZ = Total Profit – Profit in DTA = Rs. 80 Lacs – Rs. 20 Lacs = Rs. 60 Lacs.
Export Turnover 460 − 160 300
Export Profit = Profit of Business x = 60 x = 60 x = Rs. 45 Lacs.
Total Turnover 600 − 200 400

Particulars (a) Commencement in PY 2012-13 (b) Commencement in PY 2016-17


Exemption u/s 10AA
100% for first 5 Years PY 2012-2013 to 2016-2017 PY 2016-2017 to 2020-2021
50% for next 5 Years PY 2017-2018 to 2021-2022 PY 2021-2022 to 2025-2026
Exemption for PY 2018-19 50% of Profits = 50% of 45 = 22.5 100 % of Profits=100% × 45 = 45
Note:
1. No deduction is available for Unit in Domestic Tariff Area (DTA) as it is not covered u/s 10AA.
2. Figures relating to SEZ are computed as Total (for Rudra Ltd) less amount for Unit in DTA.
3. For first 5 consecutive AYs commencing from relevant PY in which unit has begun to manufacture or produce
articles or things, Deduction shall be 100% of Profits from Export. For the next 5 AYs, deduction shall be 50%.

DT QUESTION BANK BY CA PRANAV CHANDAK Subscribe us on YouTube 22


Q11. X Ltd., a unit in SEZ, provides you the following particulars relating to its first year of operation (Rs. In lacs)
Total receipts from providing services 64
Receipts from export of services 50
Profits of business 7
Out of the total exports, Rs. 5 Lacs could not be realized on account of death of the Foreign Service recipient.
Plant & machinery used in the business had been depreciated @ 20% on SLM basis & depreciation of Rs. 4 Lacs
was charged in the P & L A/c. Compute the taxable income of the company.
Solution: Computation of taxable income of X Ltd:
Net Profit as per P&L A/ c 7,00,000
Add: Depreciation charged in books 4,00,000
Less: Depreciation as per IT Rules (15% of Rs. 20 Lacs) [WN-1] 3,00,000
Profits & Gains of Business 8,00,000
Less: Deduction u/s 10AA [WN-2] 5,62,500
Taxable Income 2,37,500
Working Note:
(1) Cost of machinery = Rs. 4 lacs/20% = Rs. 20 Lacs.
(2) Deduction allowable u/s 10AA = 100% of [(Profits of Business ÷ Total turnover) × Export Turnover realised
(net of bad debts)] = (Rs. 8,00,000 ÷ Rs. 64,00,000) × Rs. 45,00,000 = Rs. 5,62,500.

Q12. Mr. X, a resident, has provided the following particulars of his income for PY 2018-19.
Income from salary (computed) Rs. 1,80,000
Income from house property (computed) Rs. 2,00,000
Agricultural income from a land in Jaipur Rs. 2,80,000
Expenses incurred for earning agricultural income Rs. 1,70,000
Compute his tax liability assuming his age is (a) 45 years; (b) 70 years.
Solution: (a) Computation of tax liability (Age 45 years)
Income from salary Rs. 1,80,000
Income from house property Rs. 2,00,000
Net agricultural income [ Rs. 2,80,000 – Rs. 1,70,000 ] 1,10,000
Less: Exempt u/s 10(1) (1,10,000) Nil
Gross Total Income Rs. 3,80,000
Step 1 Tax on Rs. 4,90,000 (Rs. 3,80,000 + Rs. 1,10,000) = Rs. 12,000.
Step 2 Tax on Rs. 3,60,000 (Rs. 1,10,000 + Rs. 2,50,000) = Rs. 5,500.
Step 3 Rs. 12,000 – Rs. 5,500 = Rs. 6,500
Step 4 & 5 Rs. 6,500 + 4% = Rs. 6,760.
(b) Computation of tax liability (Age 70 years)
Step 1 Tax on Rs. 4,90,000 (Rs. 3,80,000 + Rs. 1,10,000) = Rs. 12,000.
Step 2 Tax on Rs. 4,10,000 (Rs. 1,10,000 + Rs. 3,00,000) = Rs. 8,000.
Step 3 Rs. 12,000 – Rs. 8,000 = Rs. 4,000
Step 4 & 5 Rs. 4,000 + 4% = Rs. 4,160.

*Q13. The broad break-up of tax & allied details of Mrs. X, born on 31st March, 1959 are as under:
Long-term capital gains on sale of house 2,00,000
STCG on sale of shares in B Ltd. (STT paid) 30,000
Prize winning from a T.V. show 20,000
Business income 2,90,000
Net agricultural income 4,40,000
Deduction allowed u/s 80C to 80U 60,000
Compute the tax payable by Mrs. X for the AY 2019-20. [MAY – 2007]
(b) What if Income from business is Rs. 5,00,000.
Answer: Computation of Total Income
Business Income 2,90,000
Long term capital gain on sale of house 2,00,000
STCG on sale of shares in B Ltd. (STT paid) 30,000
Casual Income (Prize winning from a TV show) 20,000
Gross Total Income 5,40,000
Less: Deduction u/s 80C to 80U (60,000)
Total Income 4,80,000

DT QUESTION BANK BY CA PRANAV CHANDAK Subscribe us on YouTube 23


Computation of tax payable by Mrs. X for AY 2019-20
Tax on long-term capital gain of Rs. 1,30,000 (2,00,000 - 70,000) @ 20% 26,000
Tax on short term capital gain of Rs. 30,000 @ 15% 4,500
Tax on winnings of Rs. 20,000 from a T.V. show @ 30% 6,000
Tax on balance income of Rs. 3,00,000 at slab rate Nil
Tax on Total Income 36,500
Add: HEC @ 4% 1,460
Tax payable by Mrs. X 37,960

Note:
1. Unexhausted BEL of Rs. 3,00,000 – (Rs. 2,90,000 – Rs. 60,000) = Rs. 70,000 has been allowed from LTCG.
2. Mrs. X has completed 60 years of age on 31st March, 2019 i.e. she has completed the age of 60 years on the last
day of the previous year. Therefore, she is entitled to the higher basic exemption limit of Rs. 3,00,000.
3. Partial integration is not applicable because her Normal non-agricultural income is < BEL of Rs. 3,00,000.

(b) Presume income from business is Rs. 5,00,000.


Computation of Gross Total Income
Business Income 5,00,000
Long term capital gain on sale of house 2,00,000
Short-term capital gains on sale of shares in B Ltd. (STT paid) 30,000
Casual Income (Prize winning from a T.V. show) 20,000
Gross Total Income 7,50,000
Less: Deduction u/s 80C to 80U (60,000)
Total taxable (Non- Agricultural) Income 6,90,000
Add: Agricultural Income 4,40,000
Total Agricultural + Non – Agricultural Income 11,30,000

Computation of tax payable by Mrs. X for AY 2019-20


1. Tax on Agricultural + Non – Agricultural Income = Tax on Rs. 11,30,000.
Tax on long-term capital gain of Rs. 2,00,000 @ 20% 40,000
Tax on short term capital gain of Rs. 30,000 @ 15% 4,500
Tax on winnings of Rs. 20,000 from a T.V. show @ 30% 6,000
Tax on balance income of Rs. 8,80,000 86,000
Total Tax 1,36,500

2. Tax on Agricultural Income + BEL = Tax on Rs. 7,40,000 = Rs. 58,000.


3. Tax payable = (Tax on Agricultural + Non-Agricultural Income) – (Tax on Agricultural Income + BEL)
= Rs. 1,36,500 – Rs. 58,000 = Rs. 78,500.
4. Add 4 % HEC on (3) above = 3140.
5. Tax payable by Mrs. X = Rs. 81,640

Q14. Whether Rent Received for letting out Agricultural land for a Movie shooting & Amount Received from Sale
of seedlings in Nursery adjacent to Agricultural Land owned by Assessee can be regarded as Agricultural Income.
Answer: Rent for Movie shooting: It is not an Agricultural Income, since it is not Income derived ‘through
Agriculture’. This constitutes Rental Income for ‘non – agricultural purposes’.
Sales of seedlings in Nursery: Income from Sale of Plants & Seedlings grown in Posts in Nursery constitutes
Agricultural Income. However, in this case, such income is derived not from agricultural land, but from a Nursery
‘adjacent’ to it. Hence, it does not constitute Agricultural Income.

Q15. Mr. B grows sugarcane & uses the same for the purpose of manufacturing sugar in his factory. 30% of
sugarcane produce is sold for Rs. 10 lacs & cost of cultivation of such sugarcane is Rs. 5 lacs. Cost of cultivation of
70% is Rs. 14 lacs & market value of the same is Rs. 22 lacs. After incurring Rs. 1.5 lacs in manufacturing process
on the balance sugarcane, the sugar was sold for Rs. 25 lacs. Compute B’s business income & agricultural income.
Solution: Income from sale of sugarcane is agricultural income & Income from sale of sugar is business income.
(i) Business income = Sale proceeds - MV of 70% of sugarcane (used in manufacture of sugar) - Manufacturing
expenses = Rs. 25 lacs - Rs. 22 lacs - Rs. 1.5 lacs = Rs. 1.5 lacs.
(ii) Agricultural income = Market value of sugarcane produce - Cost of cultivation.
= [Rs. 10 lacs + Rs. 22 lacs] – [Rs. 5 lacs + Rs. 14 lacs] = Rs. 13 lacs.

DT QUESTION BANK BY CA PRANAV CHANDAK Subscribe us on YouTube 24


Q16. X Ltd. grows sugarcane to manufacture sugar. The data for the PY 2018-19 is as follow-
1. Cost of cultivation of sugarcane 6,00,000
2. Market value of Sugarcane when transferred to factory 10,00,000
3. other manufacturing cost 6,00,000
4. Sale of sugar 25,00,000
5. Salary of Managing Director who looks after all operation of the company 3,00,000
Determine the Income of the company.
Solution:
Particulars Rs. Rs.
1. Profit & Gain of Business or Profession:
Sales of sugar 25,00,000
Less: Average market Value of Sugarcane 10,00,000
Salary to managing Director 3,00,000
Manufacturing cost 6,00,000 (19,00,000)
Business Income 6,00,000
2. Computation of Agricultural Income:
Market Value of Sugarcane 10,0000
Less: Cost of Cultivation (6,00,000)
Agricultural Income 4,00,000

Q17. Mr. Avani, a resident aged 25 years, manufactures tea leaves from the tea plants grown by him in India. These
are then sold in the Indian market for Rs. 40 Lacs. The cost of growing tea plants was Rs. 15 Lacs & the cost of
manufacturing tea leaves was Rs. 10 Lacs. Compute her tax liability for AY 2019-20.
Solution: As per Rule 8, if any person is engaged in growing & manufacturing of tea, income shall be computed
combined for agriculture & business. 40% of such income shall be business income & balance shall be agricultural.
Income = Sale - Cost of growing tea plant - cost of manufacturing tea leaves = 40 lacs - 15 lacs - 10 lacs = 15 lacs.
Business Income = 15,00,000 x 40% = 6,00,000; Agriculture Income = 15,00,000 x 60% = 9,00,000.
Computation of Tax Liability
Tax on (6,00,000 + 9,00,000) at slab rate 2,62,500
Tax on (2,50,000 + 9,00,000) at slab rate (1,57,500)
Tax on normal income (2,62,500 - 1,57,500) 1,05,000
Add: HEC @ 4% 4,200
Tax Liability 1,09,200

Q18. Calculate the amount of tax liability of Mr. PC, a businessman for the AY 2019-20. Total income (inclusive of
share from HUF Rs. 2,50,000 & interest on Government Securities Rs. 25,000) is Rs. 5,75,000.
Solution: Computation of Total Income
(a) PGBP: Business Profit 3,00,000
(b) Income from other sources : Interest on Government Securities 25,000
Gross Total Income 3,25,000
Deduction u/c VIA Nil
Total Income 3,25,000
Tax on Rs. 3,25,000 3,750
Less: Tax Rebate u/s 87A (2,500)
Total Tax 1,250
Add: HEC @ 4% 50
Total Tax Payable (rounded off) 1,300
Note: Share from HUF is exempt from tax u/s 10(2).

DT QUESTION BANK BY CA PRANAV CHANDAK Subscribe us on YouTube 25


MASTER QUESTION ON ‘EXEMPT INCOMES’
*MQ19. Find out the net income & tax liability of Mr. X (35 years) for the AY 2019-20. [Modified RTP]
Income from the activity of owning & maintaining race horses 7,00,000
Winnings from camel races in Dubai 3,00,000
Winnings from Government lottery 1,00,000
Cost of purchase of Lottery ticket 11,000
Salary from A Ltd. (includes entertainment allowance of Rs. 5,000) engaged in cultivation & 5,40,000
manufacture of coffee in India
Share of profit from the LLP firm for PY 2018-19 (X is a sleeping partner in the firm & share of 10,70,000
profit includes LTCG on transfer of a plot of land of Rs. 1,50,000)
Share of profit from the family business 2,65,000
Interest income of minor child (of deposit made out of gift received by the child from brother 21,500
of Mrs. X)
Income of Mrs. X [It is interest on company deposit (50% deposit is made out of gift received 1,00,000
from X & 50% is made out of gift received form her father).
PPF contribution 2,05,000
Receipt of accumulated balance of PPF A/c (it includes interest of Rs. 1,10,000) 6,10,000

Solution: Computation of Total Income & tax liability of Mr. X


(i) Salary (after standard deduction of Rs. 40,000) [Note 1] 5,00,000
(ii) Business income (owning & maintaining race horses) 7,00,000
(iii) Income from other sources -
- Interest income of minor child [Rs. 21,500 - Rs. 1,500 exemption u/s 10(32)] 20,000
- Winnings from camel races (expenditure is not deductible) 3,00,000
- Winnings from lottery (expenditure is not deductible) 1,00,000
- Interest income of Mrs. X on deposit made by her out of money gifted by X 50,000
Gross total income 16,70,000
Less: Deduction u/s 80C (1,50,000)
Total Income 15,20,000
Tax on Total Income
Income-tax (30% of Rs. 4,00,000 + Normal tax @ Slab Rate = 12,70,000) 3,13,500
Add: Health & education cess 12,540
Tax liability (rounded off) 3,26,040

Note:
1. Salary shall be taxable even if it is paid out of agricultural income by the employer.
2. Accumulated balance of PPF & Interest on such balance is exempt u/s 10(11).
3. Share of profit from a firm (LLP) is Exempt u/s 10(2A) even if X is a sleeping partner & profit includes LTCG.
4. Share of profit from HUF Business received by the member of HUF is Exempt u/s 10(2).

DT QUESTION BANK BY CA PRANAV CHANDAK Subscribe us on YouTube 26


4A. INCOME FROM SALARY
Q1. Write short note on “Salary paid tax-free” & “Loan or advance against salary”
Answer:
(a) Salary paid tax-free
▪ This does not mean that tax is not levied on such salary.
▪ It simply means that Employer bears the burden of the tax on the salary of the employee.
▪ It does not matter whether employer pays the tax under terms of contract or voluntarily.
▪ Thus, Income from salaries = Salary Income + Tax on employee’s Salary paid by the employer.
▪ However, as per section 10(10CC), Income-tax paid by the employer on Non-Monetary Perquisites on behalf
of the employee would be exempt in the hands of the employee.
(b) Loan or advance against salary
▪ Loan is different from salary. It cannot be taxed as Salary.
▪ Similarly, advance against salary is different from advance salary. It is an advance taken by the employee
from his employer. This advance is generally adjusted with his salary over a specified time period.
▪ When an employee takes a loan from his employer, which is repayable in certain specified instalments, the
loan amount cannot be brought to tax as salary of the employee.

Q2. Allowance received by an employee working in a transport system at Rs. 10,000 per month to meet his
personal expenditure while in duty. He is not receiving any daily allowance.
Answer:
▪ U/s 10(14), any allowance granted to an employee working in a transport system to meet his personal
expenditure during his duty is exempt provided he is not in receipt of daily allowance.
▪ Exemption is 70% of such allowance (Rs. 7,000 p.m) or Rs. 10,000 p.m whichever is less.
▪ Hence, Rs. 84,000 (i.e., 7,000 x 12) is allowable as deduction u/s 10(14).

Q3. AB Co. Ltd., alloted 1000 sweat equity shares to Mr. X in June 2018. The shares were allotted at Rs. 200 per
share as against the fair market value of Rs. 300 per share on the date of exercise of option by the allottee viz. Mr.
X. The fair market value was computed in accordance with the method prescribed under the Act.
(a) What is the perquisite value of sweat equity shares allotted to Mr. X?
(b) In the case of subsequent sale of those shares by Mr. X, what would be the cost of acquisition of those sweat
equity shares? [Nov 2010]
Answer:
(a) Value of Perquisite = FMV of Sweat Equity Shares as on the date of Exercise of Option - Amount recovered from
the Employee. Therefore, Value of perquisite of sweat equity shares allotted to Mr. X = 1,000 Shares x (300 – 200)
= Rs. 1,00,000.
(b) As per section 49(2AA), where capital gain arises from transfer of sweat equity shares, the cost of acquisition
of such shares = FMV which has been considered for perquisite valuation u/s 17(2)(vi).
Therefore, in case of subsequent sale of sweat equity shares by Mr. X, the cost of acquisition would be Rs. 3,00,000.

Q4. Telephone provided at the residence of employee & the bill aggregating to Rs. 25,000 paid by the employer.
Determine the perquisite value taxable in the hands of employee.
Answer: Telephone provided at the residence of the employee & payment of bill by the employer is a tax-free
perquisite as per section 17(2)(viii) Rule 3(7)(ix).

Q5. Find out the Taxable value of perquisite from the following particulars in case of an Employee to whom the
following assets held by the Company were sold on 13.06.2018 –
Amount Car Laptop Furniture
Cost of Purchase (May 2017) 8,72,000 1,22,500 35,000
Sale Price 5,15,000 25,000 10,000
The Assets were put to use by the Company from the day these were purchased.
Solution: Computation of Taxable value of perquisites
Amount Car Laptop Furniture
Actual Cost of the Assets 8,72,000 1,22,500 35,000
Less: Depreciation for every completed year (1,74,400) (61,250) (3,500 )
(8,72,000 x 20%) (1,22,500 x 50%) (35,000 x 10%)
Value of the Asset Transferred 6,97,600 61,250 31,500
Less: Amt recovered from the Employee (5,15,000) (25,000) (10,000)
Taxable Value of the Perquisite 1,82,600 36,250 21,500

DT QUESTION BANK BY CA PRANAV CHANDAK Subscribe us on YouTube 27


Q6. Following benefits have been granted by ABC Software Ltd. to one of its employees Mr. X:
(i) Housing loan @ 6% p.a. Amount outstanding is Rs. 6 lacs. Mr. X pays Rs. 12,000 p.m, on 5th of each month.
(ii) AC purchased 4 years back for Rs. 2,00,000 have been given to Mr. X for Rs. 90,000.
Compute the chargeable perquisite in the hands of Mr. X for the AY 2019-20.
Lending rate of State Bank of India as on 01.04.2018 for housing loan may be taken as 10.50%. [M 10/M 08]
Answer: Perquisite value for housing loan
(i) Housing Loan: Since Interest charged by Employer (6%) is lower than SBI Rate (10%), the Taxable Value of
Perquisite shall be = SBI Interest Rate on 1st day of relevant PY – Max. Outstanding Monthly Balance - Interest paid
by Employee on that loan.
“Maximum outstanding monthly balance” means the aggregate outstanding balance for loan as on the last day of
each month. The perquisite value for computation is 10.5 % - 6 % = 4.5 %
Month Maximum outstanding balance as on last Perquisite value at 4.5% for the
date of month month
April, 2018 5,88,000 5,88,000 x 4.5% x 1/12 = 2,205
May, 2018 5,76,000 5,76,000 x 4.5% x 1/12 = 2,160
June, 2018 5,64,000 5,64,000 x 4.5% x 1/12 = 2,115
July, 2018 5,52,000 5,52,000 x 4.5% x 1/12 = 2,070
August, 2018 5,40,000 5,40,000 x 4.5% x 1/12 = 2,025
September, 2018 5,28,000 5,28,000 x 4.5% x 1/12 = 1,980
October, 2018 5,16,000 5,16,000 x 4.5% x 1/12= 1,935
November, 2018 5,04,000 5,04,000 x 4.5% x 1/12 = 1,890
December, 2018 4,92,000 4,92,000 x 4.5% x 1/12 = 1,845
January, 2019 4,80,000 4,80,000 x 4.5% x 1/12 = 1,800
February, 2019 4,68,000 4,68,000 x 4.5% x 1/12 = 1,755
March, 2019 4,56,000 4,56,000 x 4.5% x 1/12 =1,710
Total value of this perquisite 23,490
Note: It is assumed that payment of Rs. 12,000 is excluding interest.
(ii) Perquisite Value of Air Conditioners:
Original cost 2,00,000
Less: Depreciation on SLM basis for 4 years @ 10 % [Rs. 2,00,000 x 10% x 4] (80,000)
Written down value 1,20,000
Amount recovered from the employee 90,000
Perquisite value 30,000

Q7. An Indian Company pays Gross Salary including Allowances & Monetary Perquisites amounting to Rs. 5,80,000
to its General Manager on which the tax liability works out to Rs. 46,000 for PY 2018-19. Besides, Company
provides Non-Monetary Perquisites to him for the same period whose value is estimated at Rs. 1,20,000. Discuss
the liability for tax deduction at source in the above case.
Solution: Computation of Taxable Salary
Gross Salary before inclusion of Non-Monetary Perquisites 5,80,000
Add: Non-Monetary Perquisites 1,20,000
Gross Salary after inclusion of Non-Monetary Perquisites [A] 6,60,000
Less: Deduction u/s 16 (ia) Standard Deduction (40,000)
Income under the Head Salary 6,20,000
Tax on above [12,500 + (6,60,000 – 5,00,000) x 20%] 34,500
Add: HEC at 4% 1,380
Net Tax Payable [B] 35,880
Average Rate of Tax [B/A] 5.43%
Notes:
▪ Tax on Non-Monetary perquisites: Perquisites x Average Rate of Tax = Rs. 1,20,000 x 5.43% = Rs. 6,516.
▪ Tax payable by Employer on Non-Monetary Perquisites = 6,520; Balance tax of Rs. 29,360 (35,880 – 6,520)
should be deducted from the Salary payable to the Employee.

Q8. Mr. Garg received retrenchment compensation of Rs. 10,00,000 after 30 years 4 months of service. At the time
of retrenchment, he was drawing basic salary Rs. 20,000 p.m.; dearness allowance Rs. 6,000 p.m. Compute his
taxable retrenchment compensation.
Solution: Taxable Retrenchment compensation = Rs. 10 lacs – Rs. 4,50,000 = Rs. 5,50,000
Calculation of exempt Retrenchment compensation = Lower of
(a) Actual Amount Received = Rs. 10 lacs

DT QUESTION BANK BY CA PRANAV CHANDAK Subscribe us on YouTube 28


(b) Rs. 5,00,000.
(c) 15 days Average Pay × Length of service (More than half year shall be treated as full year).
= Rs. 26000 × 15/26 × 30 years = Rs. 4,50,000.

Q9. Mr. Dutta received voluntary retirement compensation of Rs. 7,00,000 after 30 years 4 months of service. He
still has 6 years of service left. At the time of voluntary retirement, he was drawing basic salary Rs. 20,000 p.m.;
Dearness allowance (which forms part of pay) Rs. 5,000 p.m. Compute his taxable VRS, assuming that he does not
claim any relief u/s 89.
Solution:
▪ Exemption of Rs. 5,00,000 is available if Amount payable for VRS does not exceed Higher of
(a) 3 Month Salary for each completed year of service = 3 × Rs. 25,000 × 30 years = Rs. 22,50,000;
(b) Salary @ time of retirement × Balance months of service left before retirement = 25,000 × 72 = 18,00,000;
▪ Thus, If VRS Compensation received does not exceed Rs. 22,50,000, Exemption of Rs. 5,00,000 will be available.
▪ Thus Rs. 5,00,000 will be Exempt;
▪ Amount of VRS Received = Rs. 7,00,000; Taxable VRS = Rs. 2,0,000.

Q10. Mr. X retired on 15.6.2018 after completion of 26 years 8 months of service & received gratuity of Rs.
6,00,000. At the time of retirement his salary was:
Basic Salary Rs. 5,000 p.m.
Dearness Allowance Rs. 3,000 p.m. (60% of which is for retirement benefits)
Commission 1% of turnover (turnover in the last 12 months was Rs. 12,00,000)
Bonus Rs. 12,000 p.a.
Compute his taxable gratuity assuming:
(a) He is non-government employee & covered by the Payment of Gratuity Act 1972.
(b) He is non-government employee & not covered by Payment of Gratuity Act 1972.
(c) He is a Government employee.
Solution:
(a) He is non-government employee & covered by the Payment of Gratuity Act 1972:
Least of the following shall be exempt:
1. Gratuity Actually Received = Rs. 6,00,000.
2. Rs. 20 Lacs
3. Length of Service (LOS) x 15 days Salary = 27 x [(Rs. 5,000 + Rs. 3,000) x 15/26] = Rs. 1,24,615.
Taxable Gratuity (6,00,000 - 1,24,615) = Rs. 4,75,385.

Note:
1. 15 days salary shall be determined on the basis of Salary Last drawn.
2. LOS: More than half year = Full year [Ignore less than half part].

(b) He is non-government employee & not covered by Payment of Gratuity Act 1972
Least of the following shall be exempt:
1. Gratuity Actually Received = Rs. 6,00,000.
2. Rs. 10 Lacs
3. ½ Month’s Average Salary x Completed YOS = ½ x 7800 x 26 = Rs. 101,400
Taxable Gratuity = (6,00,000 - 1,01,400) = Rs. 4,98,600.

Note:
1. Average Monthly Salary: Avg. salary for 10 months immediately preceding Retirement Month (& not date).
2. Salary = Basic salary + DA (Forming part of retirement benefits) + Commission (% of turnover) = 5,000 +
(3,000 × 60%) + (12,000/12) = Rs. 7,800.

(c) He is a government employee Entire amount of gratuity is exempt

Q11. Mr. X retired w.e.f 1.10.2018 receiving Rs. 5,000 p.m. as pension. On 1.2.2019, he commuted 60% of his
pension & received Rs. 3,00,000 as commuted pension. You are required to compute his taxable pension
assuming:
(a) He is a government employee.
(b) He is a non-government employee, receiving gratuity of Rs. 5,00,000 at the time of retirement.
(с) He is a non-government employee & is in receipt of no gratuity at the time of retirement.
𝐶𝑜𝑚𝑚𝑢𝑡𝑒𝑑 𝑃𝑒𝑛𝑠𝑖𝑜𝑛 3,00,000
Solution: Total Pension = = = Rs. 5 lacs.
% 𝑜𝑓 𝑐𝑜𝑚𝑚𝑢𝑎𝑡𝑖𝑜𝑛 60%

DT QUESTION BANK BY CA PRANAV CHANDAK Subscribe us on YouTube 29


(a) He is a government employee
Uncommuted pension received (October - March) 24,000
[( Rs. 5,000 x 4 months) + (40% of Rs. 5,000 x 2 months)]
Commuted pension received 3,00,000
Less: Exempt u/s 10(10A) (3,00,000) Nil
Taxable pension 24,000

(b) He is a non-government employee, receiving gratuity Rs. 5,00,000 at the time of retirement
Uncommuted pension received (October - March) 24,000
[( Rs. 5,000 x 4 months) + (40% of Rs. 5,000 x 2 months)]
Commuted pension received Rs. 3,00,000
Less: Exempt u/s 10( 10A) = 1/3 × 5 lacs (Rs. 1,66,667) 1,33,333
Taxable Pension 1,57,333

(c) He is a non-government employee & not in receipt of gratuity at the time of retirement
Uncommuted pension received (October - March) 24,000
[( Rs. 5,000 x 4 months) + (40% of Rs. 5,000 x 2 months)]
Commuted pension received Rs. 3,00,000
Less: Exempt u/s 10( 10A) = 1/2 × 5 lacs (Rs. 2,50,000) 50,000
Taxable Pension 74,000

Q12. X was employed by PQR Ltd. upto March 15, 2004. At the time of leaving PQR Ltd, he was paid Rs. 3,50,000
as leave salary out of which Rs. 57,000 was exempt from tax u/s 10(AA)(ii). Thereafter he joined ABC (P.) Ltd. &
received Rs. 4,12,200 as leave salary at the time of his retirement on 31.12.2018. Determine taxable leave salary:
Particulars Rs.
Salary at the time of retirement (per month) 22,900
Average salary received during 10 months ending on December 31,2018
- From March 1, 2018 to July 31, 2018 (per month) 22,600
- From August 1, 2018 to December 31, 2018 (per month) 22,900
Duration of service (a) 14.75 years
Leave entitlement for every year of service (b) 45 days
Leave availed while in service (c) 90 days
Leave at the credit of employee at the time of retirement [(14 x 45- 90)/30] 18 months
Leave salary paid at the time of retirement @ Rs. 22,900 per month (i.e., Rs. 22,900 x 18) Rs. 4,12,200
Solution:
(i) Rs. 4,12,200.
(ii) Rs. 3,00,000 – 57000 (already exempted from previous employer)= Rs. 2,43,000.
(iii) 10× 22,750 = Rs. 2,27,500. [Note 1]
(iv) 11 × 22750 = Rs. 2,50,250. [Note 2]
▪ Leave salary exempt = Rs. 2,27,500;
▪ Taxable leave salary = Rs. 4,12,200 - Rs. 2,27,500 = Rs. 1,84,700.

Note:
(1) AMS = [ (22600×5) + (22900×5) ] = Rs. 22750.
(2) Leaves Earned = {Completed years of service × No. of leaves credited/month (Maximum 30 days allowed in a
year)] – Leaves actually taken.}/30 days. = [{14 × 30} – 90]/30 = 11 months.

Q13. Mr. Hari, who turned 66 years on 28.3.2019, salary for PY 2018-19 is Rs. 10,20,000 & Arrears of salary
received is Rs. 3,45,000. Further, you are given the following details relating to the earlier years to which the
arrears of salary received is attributable to:
Previous year Taxable Salary Arrears now received
2010 – 2011 7,10,000 1,03,000
2011 – 2012 8,25,000 1,17,000
2012 – 2013 9,50,000 1,25,000
Compute the relief available u/s 89 & the tax payable for the AY 2019-20.
Note: Rates of Taxes:
AY Slab rates of income-tax
For resident individuals of the age of 60 years or For other resident individuals
more at any time during the previous year
Slabs Rate Slabs Rate

DT QUESTION BANK BY CA PRANAV CHANDAK Subscribe us on YouTube 30


2011–12 Upto 2,40,000 Nil Upto Rs. 1,60,000 Nil
2,40,001 - 5,00,000 10% Rs. 1,60,001 - Rs. 5,00,000 10%
5,00,001 - 8,00,000 20% Rs. 5,00,001 - Rs. 8,00,000 20%
Above 8,00,000 30% Above Rs. 8,00,000 30%
2012–13 Upto 2,50,000 Nil Upto Rs. 1,80,000 Nil
2,50,001 - 5,00,000 10% Rs. 1,80,001 - Rs. 5,00,000 10%
5,00,001 - Rs. 8,00,000 20% Rs. 5,00,001 - Rs. 8,00,000 20%
Above Rs. 8,00,000 30% Above Rs. 8,00,000 30%
2013–14 Upto Rs. 2,50,000 Nil Upto Rs. 2,00,000 Nil
2,50,001 - 5,00,000 10% Rs. 2,00,001 - Rs. 5,00,000 10%
5,00,001 - 10,00,000 20% Rs. 5,00,001 - Rs. 10,00,000 20%
Above 10,00,000 30% Above Rs. 10,00,000 30%
Note: Education cess @ 2% & SHEC @ 1% was attracted on the income-tax for all above preceding years.
Solution: Computation of tax payable by Mr. Hari for the AY 2019-20
Particulars Incl. arrears of salary Excl. arrears of salary
Current year salary 10,20,000 10,20,000
Add: Arrears of salary 3,45,000 -
Taxable Salary 13,65,000 10,20,000
Income-tax thereon 2,19,500 1,16,000
Add : HEC @ 4% 8,780 4,640
Total payable 2,28,280 1,20,640

Computation of tax payable on arrears of salary if charged to tax in respective AYs


Particulars AY 2011-12 AY 2012-13 AY 2013-14
Incl. arrears Excl. arrears Incl. arrears Excl. arrears Incl. arrears Excl. arrears
Taxable salary 7,10,000 7,10,000 8,25,000 8,25,000 9,50,000 9,50,000
Add: Arrears of salary 1,03,000 - 1,17,000 - 1,25,000 -
Taxable salary 8,13,000 7,10,000 9,42,000 8,25,000 10,75,000 9,50,000
Tax 97,900 76,000 1,34,600 99,500 1,47,500 1,15,000
Add: Cess@3% 2,937 2,280 4,038 2,985 4,425 3,450
Tax payable 1,00,837 78,280 1,38,638 1,02,485 1,51,925 1,18,450

Computation of Relief u/s 89


Particulars Rs. Rs.
(i) Tax payable in AY 2019-20 on arrears:
Tax on income including arrears 2,28,280
Less: Tax on income excluding arrears (1,20,640) 1,07,640
(ii) Tax payable in respective years on arrears :
Tax on income including arrears (Rs. 1,00,837 + Rs. 1,38,638 + Rs. 1,51,925) 3,91,400
Less: Tax on income excluding arrears (Rs. 78,280 + Rs. 1,02,485 + Rs. 1,18,450) (2,99,215) 92,185
Relief u/s 89 [Diffn b/w tax on arrears in AY 2019-20 & tax on arrears in respective years] (i – ii) 15,455

Q14. Salary in lieu of notice period.


Answer:
▪ Meaning of notice period: Normally, if any employer wants to terminate the services of an employee, he gives
notice of his intention to do so.
▪ Ex: As per the contract of service, he may have to give three months notice in advance to the employee. This is
known as notice period.
▪ Sometime employer instead of giving him a notice gives him salary for the notice period & terminates him
immediately.
▪ This amount paid by employer is known as salary in lieu of notice period & is fully taxable to employee in PY in
which it is received.

DT QUESTION BANK BY CA PRANAV CHANDAK Subscribe us on YouTube 31


Q15. What is annuity. Discuss its tax implication.
Answer:
▪ As per the definition, annuity is treated as salary. Annuity is a sum payable in respect of a particular year. It is a
yearly grant. If a person invests some money entitling him to series of equal annual sums, such annual sums are
annuities in the hands of the investor.
▪ Annuity received by a present employer is to be taxed as salary. It does not matter whether it is paid in
pursuance of a contractual obligation or voluntarily.
▪ Annuity received from a past employer is taxable as profit in lieu of salary.
▪ Annuity received from person other than employer is taxable as u/h IFOS.

Q16. Profit in lieu of salary. [Section 17(3)]


Answer: These payments are received by employee in lieu of or in addition to salary. They are:
(i) Terminal Compensation:
▪ Compensation received by employee from his present/former employer in connection with termination
(retirement, premature termination, resignation or otherwise) of his employment or the modification of
terms & conditions of the employment.
(ii) Payment from URPF/URSF:
▪ Accumulated balance of URPF/URSF consists of employee’s contribution plus interest on employee’s
contribution & employer’s contribution plus interest on employer’s contribution.
▪ Employer’s contribution & interest on the employee’s contribution as well as employer’s contribution are
not taxed during the period of employment.
▪ When the accumulated balance is paid to the employee either on retirement or on termination of service,
the untaxed portion, i.e. the employer’s contribution & interest thereon is taxed
▪ Interest on employee’s contribution is taxed as ‘Income from other sources’.
Note: Since employee is not eligible for deduction u/s 80C for contribution to URPF, employee’s share received
from the URPF is not taxable at the time of withdrawal.
(iii) Payment under Keyman Insurance Policy:
▪ Any payment received by an employee, under a Keyman Insurance Policy including bonus on such policy
will be regarded as profit in lieu of salary.
(iv) Any amount due or received before joining or after cessation of employment.
(v) Any other sum received by the employee from the employer:
▪ This is a comprehensive provision by virtue of which all payments made by employer to employee whether
made in pursuance of a legal obligation or voluntarily are brought to tax under “profit in lieu of salary”.
Following receipts are not termed as ‘profits in lieu of salary’ to the extent they are exempt u/s 10.
Death-cum-retirement gratuity – Sec. 10(10) Commuted Pension – Sec. 10(10A)
Payment received from SPF - Sec 10(11) Payment received from RPF – Sec. 10(12)
Any payment from an Approved Superannuation fund as per Sec. 10(13)
HRA exempt u/s 10(13A)
Retrenchment compensation received by a workman Sec. 10(10B)

Q17. Mr. X retires from service on December 31, 2018, after 25 years of service. Following are the particulars of
his income/investments for the previous year 2018-19:
Basic pay @ Rs. 16,000 per month for 9 months 1,44,000
Dearness pay (50% forms part of the retirement benefits) Rs. 8,000 per month for 9 months 72,000
Lumpsum payment received from the Unrecognised Provident Fund 6,00,000
Deposits in the PPF account 40,000
Out of the amount received from the provident fund, the employer’s share was Rs. 2,20,000 & the interest thereon
Rs. 50,000. The employee’s share was Rs. 2,70,000 & the interest thereon Rs. 60,000. What is the taxable portion
of the amount received from the unrecognized provident fund in the hands of Mr. A for AY 2019-20?
Solution: Taxable portion of the amount received from URPF in hands of Mr. A for AY 2019-20 is computed as:
Amount taxable under the head “Salaries”:
Employer’s share in the payment received from the URPF 2,20,000
Interest on the employer’s share 50,000
Total 2,70,000
Amount taxable under the head “Income from Other Sources”:
Interest on the employee’s share 60,000
Total amount taxable from the amount received from the fund 3,30,000

DT QUESTION BANK BY CA PRANAV CHANDAK Subscribe us on YouTube 32


Q18. The question whether a particular income “Income from Salary” or “Income from Business” depends upon
whether the contracts is a ‘Contract of Service’ or is a ‘Contract for Service’. Discuss.
Answer:
▪ Income is taxable under the head salary, if there is a ‘contract of service’ i.e. the relationship is that of employer-
employee.
▪ In other words, the employee does the work for his master. Control & supervision vests in the master.
▪ Contract for service
▪ A ‘contract for service’, on the other hand, is one, in which a person offers his services to any person who
is willing to pay the prescribed charges.
▪ He has discretion to do the work in his own way. He is entitled to the fruits of his labour & liable for its
losses.
▪ Such receipts constitute income from business in his hands.

Q19. Raghav requests you to compute his Taxable Income for PY 2018-19 from the following data: [M 02]
(a) Joined service on 01.10.2018, on a consolidated salary of Rs.60,000 per month.
(b) He was paid Rs.30,000 in September 2018, so that he should not join elsewhere.
(c) He contributed towards – (i) Life Insurance Premium – Rs.20,000, (ii) PPF – Rs.1,50,000.
Solution: Computation of Total Income
Particulars Amount Amount
Income under the head Salaries
Consolidated Salary (Rs,60,000 x 6 Months) 3,60,000
Amount received for not to join elsewhere – Profits in Lieu of Salary u/s 17(3)(iii) 30,000
Gross Salary 3,90,000
Less: Deduction u/s 16 (ia) Standard Deduction (40,000)
Income under the head Salary / Gross Total Income (Since no other Heads of 3,50,000
Income)
Less: Deduction Under Chapter VI-A
U/s 80C – LIC Premium paid 20,000
-PPF 1,50,000 (1,50,000)
Total Income 2,00,000
Note: Maximum deduction u/s 80C is restricted to Rs.1,50,000.

Q20. Mrs. Jaya is the marketing manager in XYZ limited. She gives you the following particulars:
▪ Basic Salary: Rs. 65,000 p.m
▪ Dearness Allowance: Rs. 22,000 p.m. (30% forms part of all retirement benefits)
▪ Bonus: Rs. 17,000 p.m.
▪ Her employer has provided her with an accommodation on 1.4.2018 at a concessional rent. House was
taken on lease by XYZ Ltd. for Rs. 12,000 p.m. Mrs. Jaya occupied the house from 1st Nov 2018. Rs. 4,800
p.m. is recovered from the salary of Mrs. Jaya.
▪ The employer gave her a gift voucher of Rs. 8,000 on her birthday.
▪ She contributes 18% of salary (BS+30% of DA) towards RPF & matching contribution is made by company.
▪ The company pays medical insurance premium to effect insurance on the health of Mrs. Jaya Rs. 18,000.
▪ Motor car owned by the employer (CC : 1.4 litres) provided to Mrs. Jaya from 1 st Nov 2018 which is used
for both official & personal purposes. Repair & running expenses of Rs. 50,000 were fully met by the
company. The motor car was self- driven by the employee.
Compute the income chargeable to tax under the head “Salaries” in the hands of Mrs. Jaya for the AY 2019-20.
Solution: Computation of Salary chargeable to tax of Mrs. Jaya for AY 2019-20
Basic Salary (65,000 x 12) 7,80,000
Dearness Allowance (22,000 x 12) 2,64,000
Bonus (17,000 x 12) 2,04,000
Health Insurance by Employer proviso to Section 17(2) Nil
Gift Voucher (Rule 3(7)(iv)) (8,000 - 5,000) 3,000
Perquisite value of accommodation at concessional rent {Sec 17(2)(ii) Rule 3(1)} 36,000
Employer’s contribution to provident fund [In excess of 12% is taxable] [(8,59,200 x 6%)] 51,552
Perquisite value of use of motor car [section 17(2)(iii)/Rule 3(2)] [(1,800x5)] 9,000
Gross Salary 13,47,552
Less: Standard Deduction u/s 16(ia) (40,000)
Gross Total Income 13,07,552
Less: Deduction u/s 80C (Rs. 8,59,200 x 18%) but subject to maximum of Rs. 1,50,000 (1,50,000)
Total Income 11,57,552

DT QUESTION BANK BY CA PRANAV CHANDAK Subscribe us on YouTube 33


Working Note:
1. Value of Unfurnished House: In the given question accommodation is given on 1.4.2018 but occupied on
1.11.2018. In such cases it will be taxable from 1.11.2018 & taxable value shall be as given below:
Lower of (i) Rent paid or (ii) 15% of rent-free accommodation salary.
(i) Rent paid = Rs. 12,000 x 5 = Rs. 60,000; (ii) 15% of salary = 15 % of Rs. 4,43,000 = Rs. 66,450
Thus, Value of Unfurnished House = Rs. 60,000 – Rs. 24,000 (Amount recovered) = Rs. 36,000.
2. Salary for the purpose of RFA for the period of 5 months = Basic salary + DA (forming part of Retirement
benefits) + Bonus + Commission (any) + All taxable allowances excluding perquisites = Rs. 7,80,000 + 30% ×
2,64,000 + 2,04,000 = Rs. 10,63,200 × 5/12 = Rs. 4,43,000.
3. Salary for the purpose of Employer’s contribution = Basic Salary + DA (forming part of retirement benefits +
commission (paid as % of Turnover) = [Rs. 65,000 + (30% of Rs. 22,000)] x 12 = Rs. 8,59,200.

Q21. Mr. Nambi, a salaried employee, furnishes the following details for the financial year 2018-19:
▪ Basic salary: Rs. 6,00,000; Dearness allowance: Rs. 3,20,000; Commission: Rs. 50,000.
▪ Entertainment allowance: Rs. 7,500; Profession Tax (of this, 50% paid by employer): Rs. 7,000.
▪ Health insurance premium paid by employer: 9,000; Life insurance premium Paid by employer: 34,000.
▪ Gift voucher given by employer on his birthday: Rs. 12,000.
▪ Laptop provided for use at home . Actual cost of Laptop to employer: Rs. 30,000.[Children of the assessee
are also using the Laptop at home]
▪ Employer - Company owns a Tata Nano car, which was provided to the assessee, Both for official &
personal use. No driver was provided. (Engine CC: Less than 1.6 litres)
▪ Annual credit card fees paid by employer [not exclusively used for Official purposes;] : Rs. 2,000. [Details
of usage are not available]
You are required to compute the income chargeable under the head "Salaries" for the AY 2019-20. [MAY-17]
Solution: Computation of Salary Income of Mr. Nambi for AY 2019-20
Basic Salary 6,00,000
Dearness Allowance 3,20,000
Commission 50’0OO
Entertainment allowance 7,500
Professional Tax paid by the employer Section 17 (2)(iv) 3,500
Facility of Laptop/computer (Rule 3(7)(vii)) Nil
Life Insurance paid by Employer (section 17 (2) (v)) 34,000
Perquisite value of use of motor car (section 17 (2) (iii)/Rule 3(2)) [(1,800 x 12)] 21,600
Annual Credit card fees (Rule 3 (7)(v)) 2,000
Gross Salary 10,38,600
Less: Standard Deduction u/s 16(ia) (40,000)
Less: Deduction of professional tax u/s 16(iii) (7,000)
Income under the head Salary 9,91,600
Working Notes:
▪ As per section 16(ii) Deduction of entertainment allowance is allowed only to Government employees & not to
other employees. Thus, entertainment allowance is fully taxable in the hands of Mr. Nambi.
▪ Professional tax paid by employer shall be added to the gross salary of the employee then deduction u/s 16
(iii) shall be allowed for professional tax.
▪ As per section 17 (2) (v), Life Insurance premium of employee paid by employer shall be included in his income
as it is a perquisite for an employee.
▪ Credit card facility is exempt only if it is exclusively used for official purpose & employer has maintained
complete records.

Q22. Mr. X is a regular employee of ABC Ltd. in Mumbai. He was appointed on 1.3.2018 [25,000 - 2,500 - 35,000].
▪ He is paid dearness allowance (which forms part of salary for retirement benefits) @ 15% of basic pay &
bonus equivalent to one & a half month’s basic pay as at the end of the year.
▪ He contributes 18% of (BS + DA) towards RPF & Company contributes the same amount.
▪ He is provided free housing facility which has been taken on rent by the Company at Rs. 15,000 per month.
▪ The monthly salary of Rs. 2,000 of a house keeper is reimbursed by the Company.
▪ He is getting telephone allowance @ Rs. 1,000 per month.
▪ The Company pays medical insurance premium to effect an insurance on the health of Mr. X Rs. 12,000.
▪ Motor car running & maintenance charges fully paid by employer of Rs. 36,600. (Car is owned & driven
by Mr. X. CC is below 1.60 litres. Motor car is used for both official & personal purpose by the employee.)
▪ Value of free lunch provided during office hours is Rs. 2,200. [NOV - 2013]
From the following details, find out the salary chargeable to tax of Mr. X for the AY 2019-20:

DT QUESTION BANK BY CA PRANAV CHANDAK Subscribe us on YouTube 34


Solution: Computation of taxable salary of Mr. X for AY 2019-20
Particulars Rs.
Basic pay [( Rs. 25,000x 11) + ( Rs. 27,500x 1)] = Rs. 2,75,000 + Rs. 27,500 3,02,500
Dearness allowance [15% of basic pay] 45,375
Bonus [ Rs. 27,500 x 1.5] 41,250
Employer’s contribution to RPF in excess of 12% (18% - 12% = 6% of Rs. 3,47,875) 20,873
Telephone allowance 12,000
Rent-free accommodation Sec 17(2)(i)/Rule 3(1)
15% of salary (3,02,500 + 45,375 + 41,250 +12,000) or Rs. 1,80,000 whichever is less. 60,169
Reimbursement of salary of housekeeper [ Rs. 2,000 x 12] Sec 17(2)(iv) 24,000
Motor car owned & driven by employee [36,600 - 21,600 (Rs. 1,800 x 12)] 15,000
Value of free lunch facility (Presuming that value per lunch was upto Rs. 50) -
Gross Salary 5,21,167
Less: Standard Deduction u/s 16(ia) (40,000)
Income under the head Salary 4,81,167
▪ Medical insurance premium paid by employer to effect an insurance on health of the employee is fully exempt.

Q23. Mr. X employed in ABC Co. Ltd. as Finance Manager gives you the list of perquisites provided by the company
to him for the entire financial year 2018-19: [MAY-2011]
(i) Domestic servant was provided at the residence of Mr. X. Salary of domestic servant is Rs. 1,500 per month.
The servant was engaged by him & the salary is reimbursed by the company (employer).
(b) In case, the company has employed the domestic servant, what is the value of perquisite?
(ii) Free education was provided to his two children Mr. S & Mr. A in a school maintained & owned by the company.
The cost of such education for Mr. S is computed at Rs. 900 per month & for Mr. A at Rs. 1,200 per month. No
amount was recovered by the company for such education facility from Mr. X.
(iii) The employer has provided movable assets such as television refrigerator & air conditioner at the residence
of Mr. X. The actual cost of such assets provided to the employee is Rs. 1,10,000.
(iv) A gift voucher worth Rs. 10,000 was given on the occasion of his marriage anniversary. It is given by the
company to all employee above certain grade.
(v) Telephone provided at the residence of Shri Bala & the bill aggregating to Rs. 25,000 paid by the employer.
State the taxability or otherwise of the above said perquisites.
Answer: Taxability of perquisites provided by ABC Co. Ltd. to Mr. X
(i) Domestic servant was employed by the employee & salary of such domestic servant was paid/reimbursed by
the employer. It is taxable as perquisite for all categories of employees. Taxable perquisite = 1,500 x 12 = 18,000.
(b) If the company had employed the domestic servant & the facility of such servant is given to the employee, then
the perquisite is taxable only in the case of specified employees. Taxable perquisite = 1,500 x 12 = 18,000.
(ii) If educational institution is owned by the employer, value of perquisite in respect of free education facility
shall be determined with reference to the reasonable cost of such education in a similar institution in near locality.
However, there would be no perquisite if the cost of such education per child ≤ Rs. 1,000 per month.
Thus, there is no taxable perquisite in the hands of Mr. S since the cost < Rs. 1,000 p.m.
However, cost of free education provided to his child Mr. A would be taxable, since the cost exceeds Rs. 1,000 per
month. Only the sum in excess of Rs. 1,000 per month is taxable. The value of perquisite would be Rs. 2,400.
(iii) If employer has provided movable assets to the employee/his household member, 10% p.a of actual cost of
such asset owned or amount of hire charges incurred by employer shall be the value of perquisite. However, this
will not apply to laptops & computers. Value of Perquisite = 10% of Actual cost = 10% of Rs. 1,10,000 = Rs. 11,000.
(iv) Only the sum in excess of Rs. 5,000 is taxable in view of the language of Circular No. 15/2001 dated 12.12.2001
that such gifts upto Rs. 5,000 in the aggregate p.a would be exempt, beyond which it would be taxed as a perquisite.
(v) Telephone provided at the residence of the employee & payment of bill by the employer is a tax-free perquisite.

Q24. Write a short note on “Foregoing of salary”.


Answer:
▪ Salary is chargeable to tax on due or receipt basis (whichever is earlier).
▪ If employee foregoes his salary, it does not mean that salary so foregone is not taxable.
▪ Once salary accrues, subsequent waiver by employee does not make it exempt from tax.
▪ Such waiver is only an application & hence, is taxable.
▪ Surrender of Salary to Central Government: If an employee surrenders his salary to u/s 2 of Voluntary
Surrender of Salaries (Exemption from Taxation) Act, 1961 → Such Salary is Exempt.

DT QUESTION BANK BY CA PRANAV CHANDAK Subscribe us on YouTube 35


Q25. Mr. Lakshman informs you the particulars of salary for previous year ending 31.03.2019:
Basic pay: Rs. 36,000; DA: Rs. 4,800 (not forming part of salary); Bonus: Rs. 6,000; Commission: Rs. 4,000; City
Compensatory Allowance: Rs. 3,600. Calculate the value of perquisite in respect of rent-free furnished house if Mr.
Lakshman stays in a city with a population (a) more than 25 Lacs, (b) less than 10 Lacs, (c) b/w 10 Lacs & 25 Lacs.
Cost of furniture provided is Rs. 16,000. Sofa was taken on rent for Rs. 300 per month.
Solution: Salary for this purpose = BS + Bonus + Commission + City compensatory allowance = Rs. 36,000 + Rs.
6,000 + Rs. 4,000 + Rs. 3,600 = Rs. 49,600.
Value of Rent-free unfurnished Accomodation
(a) Population > 25 Lacs → 15% of salary = 15% of Rs. 49,600 = Rs. 7,440
(b) Population 10 lac – 25 lacs → 10% of salary = 10% of Rs. 49,600 = Rs. 4960
(c) Population < 10 lacs → 7.5% of salary = 7.5% of Rs. 49,600 = Rs. 3,720
Value of Furnished Accommodation
Particulars Population > 25L 10 Lacs – 25 Lacs Population < 10L
Value of unfurnished accommodation 7,440 4,960 3,720
Add: Perquisites for value of furniture 5,200 5,200 5,200
[(10% of Rs. 16,000) + (300 x 12)]
Value of furnished accommodation 12,640 10,160 8,920
Note: Dearness allowance since not forming part of salary is not considered for the computation of Salary.

Q26. Compute taxable perquisite on medical facilities received by Mr. G from his employer during PY 2018-19:
Medical premium paid for insuring health of Mr. G Rs. 7,000
Treatment of Mr. G by his family doctor Rs. 5,000
Treatment of Mrs. G in a Government hospital Rs. 25,000
Treatment of Mr. G’s grandfather in a private clinic Rs. 12,000
Treatment of Mr. G’s mother (68 years & dependant) by family doctor Rs. 8,000
Treatment of Mr. G’s sister (dependant) in a nursing home Rs. 3,000
Treatment of Mr. G’s brother (independent) Rs. 6,000
Treatment of Mr. G’s father (75 years & dependant) abroad Rs. 50,000
Expenses of staying abroad of the patient Rs. 30,000
Limit specified by RBI Rs. 75,000
Solution: Medical Facilities outside India
Total Expenditure on Treatment + Expense of Stay = Rs. 50,000 + Rs. 30,000 Rs. 80,000
Less: Exempt to the extent permitted by RBI (Limit specified by RBI) Rs. 75,000
Value of Taxable Perquisite Rs. 5,000
Medical Facilites outside India
Medical premium paid for insuring health of Mr. G Exempt
Treatment of Mr. G by his family doctor Rs. 5,000
Treatment of Mrs. G in a Government hospital Exempt
Treatment of Mr. G’s grandfather in a private clinic Rs. 12,000
Treatment of Mr. G’s mother (dependant) by family doctor Rs. 8,000
Treatment of Mr. G’s sister (dependant) in a nursing home Rs. 3,000
Treatment of Mr. G’s brother (independent) Rs. 6,000
Perquisite of Medical facilities in India Rs. 34,000
Perquisite of Medical facilities outside India Rs. 5,000
Total Perquisite Rs. 39,000
Note: Grandfather & Independent brother are not included within the meaning of family of Mr. G.

Q27. Find out the taxable value of the perquisite in the following cases for the AY 2019-20:
1. Mr. X is given a laptop by his employer for using it for private purpose. Cost of the laptop is Rs. 96,000.
2. On 18.10.2018, the company gives its music system to Mr. X for domestic use. Ownership is not transferred. Cost
of music system (in 2010) to the employer is Rs. 30,000.
3. The employer sells the following assets to the employees on 1.1.2019
Name of employee W X Y
Asset sold Car Computer Fridge
Cost of the asset to employer Rs. 8,50,000 Rs. 95,000 Rs. 30,000
Date of purchase [put to use on the same day) 14.5.2016 14.5.2016 14.5.2016
Sale price Rs. 3,00,000 Rs. 19,000 Rs.10,000
Before sale on 1.1.2019, these assets were used for business purpose by the employer.

DT QUESTION BANK BY CA PRANAV CHANDAK Subscribe us on YouTube 36


Solution:
1. Free use of laptop is not a taxable perquisite.
2. S is provided a music system by the employer. Taxable perquisite is determined @ 10% p.a. of cost for the
165
period of use (From 18.10.2018 – 31.3.2019). Thus Taxable perquisite = Rs. 1,356 [Rs. 30,000 x 10% x ].
365
3. The taxable value of the perquisite in the hands of W, X & Y shall be determined as follows-
Particulars Car Computer Fridge
Cost of the asset on 14.5.2016 8,50,000 95,000 30,000
Less: Normal wear & tear for first year ending 13.5.2017
1,70,000 47,500 3,000
(20% of Rs. 8,50,000; 50% of Rs. 95,000; 10% of Rs.30,000)
Balance on 14.5.2017 6,80,000 47,500 27,000
Less: Normal wear & tear for second year ending 13.5.2018
1,36,000 23,750 3,000
(20% of Rs. 6,80,000; 50% of Rs.47,500; 10% of Rs. 30,000)
Balance on 14.5.2018 5,44,000 23,750 24,000
Less: Sale consideration 3,00,000 19,000 10,000
Taxable value of the perquisite 2,44,000 4,750 14,000
Note: Normal wear & tear for a part of the year is not taken into consideration.

Q28. Mr. X is employed with ABC Ltd. His son is allowed to use a motor cycle belonging to the company. The
company had purchased this motor cycle for Rs. 60,000 on 01.05.2015 & given him on the same date. The motor
cycle was finally sold to him on 01.08.2018 for Rs. 30,000. Compute the taxable perquisite in the hands of Mr. X.
Solution:
(i) Perquisite for Use of motor cycle = 60,000 x 10% p.a. for 4 months [1.4.2018 – 31.7.2018] = Rs. 2,000.
Note:
▪ Only the period of use in this previous year shall be considered for valuation of perquisite.
▪ Because we are determining the taxability for this PY. Students generally make mistake on this point.
(ii) Perquisite in respect of Transfer of motor cycle:
▪ Depreciated value of the motor cycle = Original cost - Depreciation @ 10% p.a. for 3 completed years
= Rs. 60,000 - ( Rs. 60,000 x 10% p.a. x 3 years) = Rs. 42,000.
▪ Taxable Perquisite = Rs. 42,000 - Rs. 30,000 = Rs. 12,000.

Q29. Mrs. Roma, an employee of XYZ Ltd., submits the following information for the AY 2019-20:
Salary: 1,86,000; City compensatory allowance: 8,000; Bonus: 10,200; Education allowance: 4,000 (for her
grandchildren); Income tax penalty paid by the employer: 2,000: Medical expenses reimbursed by the employer:
12,000; Leave travel concession: 1,000 (expenditure incurred by the employee nil); Free residential telephone:
4,000; Free refreshment during office hours 4,000; reimbursement of electricity bill by the employer: 1,060;
reimbursement of gas bills: 1,000; Professional tax paid by the employer: 300 on behalf of Mrs. Roma; Professional
tax paid by Mrs. Roma: 150. Determine the Total Income of Mrs. Roma for the AY 2019-20.
Solution: Computation of Salary
Basic Salary 1,86,000
City Compensatory Allowance 8,000
Bonus 10,200
Education Allowance [Fully taxable since given for grandchildren] 4,000
Income tax Penalty paid by employer [Income tax paid by employer on Non-monetary 2,000
perquisites is exempt. In this case, penalty is paid. Thus, it is a taxable perquisite]
Medical Reimbursement [other medical facilities are fully taxable] 12,000
Leave Travel Concession [taxable since actual expenditure is not incurred] 1,000
Refreshment [Since during office hours] Nil
Residential Telephone Nil
Payment of electricity bills by employer [It is a taxable perquisite] 1060
Reimbursement of gas bills [It is a taxable perquisite] 1000
Professional tax paid by employer [First included in salary & then allowed as deduction from 300
gross salary u/s 16(iii)]
Gross Salary 2,25,560
Less: Professional Tax paid by employee as well as employee [300 + 150] (450)
Taxable salary 2,25,110

DT QUESTION BANK BY CA PRANAV CHANDAK Subscribe us on YouTube 37


Q30. State the perquisites which will not be taxable in the hands of non-specified employees only?
Answer: Non- Monetary perquisites are taxable in the hands of specified employees only.
▪ Following perquisites will be taxable in the hands of specified employees.
▪ Provision of sweeper, gardener, watchman or personal attendant
▪ Facility of use of gas, electricity or water supplied by employer
▪ Free or concessional tickets/ Free or concessional educational facilities.
▪ Use of motor car
Note: Monetary perquisites are taxable in the hands of all employees [Specified + Non-Specified].
▪ Specified employee means any employee who has complied with atleast one the conditions given below:
(a) Any employee who is a director in a company whether full time or part time
(b) Any employee who has substantial interest in the company. [20% or more of voting power of the company].
(c) Any employee whose monetary income under the head ‘Salaries’ exceeds Rs. 50,000.
▪ If any employee is not a specified employee, he will be considered to be non-specified employee & the above
perquisites shall be exempt from income tax.
▪ Monetary income means income u/h salary but it will not include perquisite value of non-monetary perquisites
Thus it will include: (i) Basic Pay (ii) Dearness Allowance/Pay (iii) Bonus/commission/fees etc.
(iv) Taxable portion of all allowances (v) Monetary perquisites (vi) Any other payment in cash like
gratuity, pension, leave salary. but it will not include employee’s/employer’s contribution to PF & it’s interest
▪ Any arrears of salary or advance salary shall also be taken into consideration.
▪ Where salary is received from more than one employer during the relevant previous year, the aggregate salary
from these employers is to be taken into account in determining the above ceiling limit of Rs. 50,000.
▪ Deduction u/s 16(ia), 16(ii) & 16(iii) shall also be allowed.

Q31Mr. X is employed in ABC Ltd. getting basic pay Rs. 22,000 p.m. Employer has paid professional tax of Rs. 75
p.m. on behalf of employee & employee himself has paid professional tax of Rs. 25 p.m. Employer has provided
him rent free accommodation which is owned by employer himself (population of 5,00,000).
Employer has provided him three motor cars for official as well as personal use with particulars as given below:
Particulars I II III
Actual cost 4,00,000 3,00,000 2,50,000
Engine capacity 1.8 litres 1.6 litres 1.4 litres
Petrol expenses 3,000 10,000 15,000
Repairs 5,000 4,000 3,000
Driver 4,000 p.m. 3,000 p.m. no driver
All the expenses met by the employer. Compute his Tax Liability for the Assessment Year 2019-20.
Solution:
Basic Pay (22,000 x 12) 2,64,000
Professional Tax paid by employer (75 x 12) 900
Rent free accommodation (Sec 17(2)(i) Rule 3(1)} [7.5 % of Rs. 2,64,000] 19,800
Motor Car {Sec 17(2)(iii) Rule 3(2)} [WN 1] 1,62,600
Gross Salary 4,47,300
Less: Standard Deduction u/s 16(ia) (40,000)
Less: 16(iii) Professional Tax (1,200)
Income under the head Salary 4,06,100
Working Note for Motor Car
Option I: Car I is for official & personal purposes; Car II & III for personal purposes; perquisite value shall be:
Car I = (Rs. 2,400 + Rs. 900) x 12 Rs. 39,600
Car II = Rs. 30,000 + Rs. 10,000 + Rs. 4,000 + Rs. 36,000 Rs. 80,000
Car III = Rs. 25,000 + Rs. 15,000 + Rs. 3,000 Rs. 43,000
Perquisite value = Rs. 1,62,600
Option II: Car II is for official & personal purpose; Car I & Car III is for personal use; perquisite value shall be:
Car I = Rs. 40,000 + Rs. 3,000 + Rs. 5,000 + Rs. 48,000 Rs. 96,000
Car II = (Rs. 1,800 + Rs. 900) x 12 Rs. 32,400
Car III = Rs. 25,000 + Rs. 15,000 + Rs. 3,000 Rs. 43,000
Perquisite Value = Rs. 1,71,400
Option III: Car III is for official & personal purpose; Car I & Car II is for personal use; perquisite value shall be:
Car I = Rs. 40,000 + Rs. 3,000 + Rs. 5,000 + Rs. 48,000 Rs. 96,000
Car II = Rs. 30,000 + Rs. 10,000 + Rs. 4,000 + Rs. 36,000 Rs. 80,000
Car III = Rs. 1,800 x 12 Rs. 21,600
Perquisite Value = Rs. 1,97,600
So, 1st option is better.

DT QUESTION BANK BY CA PRANAV CHANDAK Subscribe us on YouTube 38


Q32. R, aged 50 years who was employed with X Ltd. up to 31.10.2018, got the following salary & benefits-
Basic pay - Rs. 20,000 p.m; bonus & incentive- Rs. 6,000 p.m; free club facility (employer's expenditure - Rs.1000
p.m.), free services of personnel attendant (salary paid by employer- Rs.800 p.m.), free services of watchman
(salary paid by the employer- Rs. 1000 p.m.), free education facility to R's son at a school maintained by the
employer (cost of education- Rs. 980 p.m.), employer's contribution towards recognised provident fund- Rs. 3,000
p.m. (R also makes a matching contribution).
On 1.11.2018, R joins Y Ltd. on monthly salary of Rs. 24,000, Besides, he gets motor car of 1.6 ltr. capacity for
private & official use- Rs. 36,000 being expenditure of the company, house rent allowance- Rs. 8,000 p.m., club
facility (expenditure of employer- Rs. 1,500 p.m.), & employer's contribution towards unrecognized provident
fund- Rs. 1000 p.m. (R also makes a matching contribution).
On 5.1.2019, Y Ltd. pays Rs. 6,000, being accident insurance premium on the life of R to cover is liabilities under
labour laws.
On 10.2.2019, Y Ltd. purchases a laptop computer for Rs. 1,00,000 & the some is given to R for office & private use.
On 1.3.2019, the company purchases a fridge for Rs. 30,000 for the kitchen of R. In the two cases ownership of
assets is not transferred to R.
Income of R from other sources is Rs. 2,46,000.
Determine the tax liability of R for the AY 2019-20 on the following assumptions-
1. Salary, in both cases, falls due on first of next month & is paid on seventh of next month.
2. R is neither a director nor a shareholder in X Ltd., & Y Ltd.
3. He resides in Delhi & pays rent of Rs. 10,000 p.m. throughout the PY.
4. Conveyance provided by X Ltd. can be used not only by R but also by his family members.
Solution:
During the PY 2018-19, X is employed with X Ltd. up to 31.10.2018 & with Y Ltd. from 1.11.2018. As salary falls
due on first of next month, 8 months' salary of X Ltd. (i.e., salary due on 1.4.2018, 1.5.2018, 1.6.2018, 1.7.2018,
1.8.2018, 1.9.2018, 1.10.2018 & 1.11.2018) & 4 months' salary of PQR (P) Ltd. (i.e., salary due on 1.12.2018,
1.1.2019, 1.2.2019 & 1.3.2019) is chargeable to tax for AY 2019-20.
Salary from X Ltd.
Basic salary (Rs.20,000 x 8) 1,60,000
Bonus (Rs.6,000 x 8) 48,000
Club facility (1,000 x 8) 8,000
Personnel attendant (Rs. 800 x 8) (being specified employee) 6,400
Watchman (Rs.1000 x 8) 8,000
Education facility (as amount does not exceed Rs. 1,000 p.m., it is not chargeable) Nil
Employer's contribution towards RPF in excess of 12% of salary (i.e. Rs. 3,000 x 8 - 12% of 4,800
Rs.1,60,000)
Salary from Y Ltd.
Basic salary ( Rs. 24,000 x 4) 96,000
Motor car facility (Rs.1800 x 4) 7,200
House rent allowance (see Note) (Rs.32,000 - 30,400) 1,600
Club facility (1,500 x 4) 6,000
Employer's contribution towards unrecognized PF(Not taxable in yr. of contribution) Nil
Computer (use of computer is not taxable) Nil
Fridge (10% of Rs.30,000 x 1/12) 250
Income from salary 3,46,250
Income from other sources 2,46,000
GTI 5,92,250
Less- Deduction u/s 80C (Contribution towards recognised provident fund) (24,000)
Net income (rounded off) 5,68,250
Note: House rent allowance is exempt from tax to the extent of the minimum of the following-
(a) Rs. 32,000 (being the amount of house rent allowance); or
(b) Rs. 30,400 (being the excess of rent paid over 10% of salary, i.e., Rs.40,000 - 10% of Rs. 96,000).
(c) Rs. 48,000 being 50% of salary, i.e., Rs. 96,000). Rs. 30,400 (being the least) is exempt from tax.

Q33. Mr. X is an area manager of M/s N. Steels Co. Ltd. During the financial year 2018-19, he gets the following
emoluments from his employer: [NOV – 08]
Basic Salary: Up to 31.08.2018 Rs. 20,000 p.m.
From 01.09.2018 Rs. 25,000 p.m.
Transport allowance Rs. 2,000 p.m.
Contribution to RPF by employer & employee 15% of basic salary (each)
Children education allowance Rs. 500 p.m. for two children

DT QUESTION BANK BY CA PRANAV CHANDAK Subscribe us on YouTube 39


City compensatory allowance Rs. 300 p.m.
Hostel expenses allowance Rs. 380 p.m. for two children
Tiffin allowance (actual expenses Rs. 3,700) Rs. 5,000 p.a.
Tax paid on employment Rs. 2,500
Compute Total Income of Mr. X for the AY 2019-20.
Solution: Computation of Taxable Salary of Mr. X for AY 2019-20
Particulars Amount Amount
Basic Salary (Rs. 20,000 x 5) + (Rs. 25,000 x 7) 2,75,000
Transport allowance (Rs. 2,000 x 12) 24,000
Children education allowance (Rs. 500 x 12) 6,000
Less: Exempt u/s 10(14) (Rs. 100 x 2 x 12) (2,400) 3,600
City Compensatory Allowance (Rs. 300 x 12) 3,600
Hostel Expenses Allowance (Rs. 380 x 12) 4,560
Less: Exempt u/s 10(14) (300 x 2 x 12 = 7,200 but restricted to 4,560) 4,560 Nil
Tiffin allowance (fully taxable) 5,000
Tax paid on employment (Professional Tax) 2,500
Employer’s contribution to RPF in excess of 12% of salary (3% of 2,75,000) 8,250
Gross Salary 3,21,950
Less: Standard deduction u/s 16 (ia) (40,000)
Less : Tax on employment u/s 16(iii) 12,500
Income u/h Salary 2,79,450
Gross Total Income 2,79,450
Less: Deduction u/s 80C: Employee’s contribution in RPF (41,250)
Total Income 2,38,200
Note: Professional tax paid by employer should be included in the salary of Mr. X as a perquisite since it is
discharge of monetary obligation of the employee by the employer. Thereafter, deduction of professional tax paid
is allowed to the employee from his gross salary.

Q34. Mr. X is Production Manager of XYZ Ltd. From the following details, compute Total Income for AY 2019-20.
Basic salary Rs. 50,000 p.m
Dearness allowance 40% of Basic Salary
Transport allowance(for commuting between Place of residence & office) Rs. 3,000 per month
Motor car running & maintenance charges fully paid by employer Rs. 60,000
The motor car is owned by the company & driven by the employee. The engine cubic capacity is above 1.60
litres. The motor car is used for both official & personal purpose by the employee.
Expenditure on accommodation in hotels while touring on official duties met by the Rs. 80,000
employer
Loan from recognized provident fund (maintained by the employer) Rs. 60,000
Lunch provided by the employer during office hours: Cost to the employer Rs. 24,000
Computer kept by the employer in the residence of Mr. X from 1.6.2018 Rs. 35,000
Medical insurance premium paid by Mr. Y: Paid in cash Rs. 4,800
Medical insurance premium paid by account payee crossed cheque Rs. 15,200
Solution:
Basic Pay [(50,000 x 12) 6,00,000.00
Dearness Allowance (50,000 x 40% x 12) 2,40,000.00
Transport Allowance 36,000.00
Motor Car Facility {Section 17(2)(iii) Rule 3(2)} [2,400 x 12] 28,800.00
Gross Salary 9,04,800.00
Less: Standard Deduction u/s 16(ia) (40,000.00)
Income under the head salary 8,64,800.00
Gross Total Income 8,64,800.00
Less: Deduction u/s 80D-Medical Insurance Premium (15,200.00)
Total Income 8,49,600.00
Note:
1. Expenditure on accommodation in hotels while touring on official duties met by the employer is not taxable.
2. Lunch provided by the employer during office hours is not taxable as per Section 17(2)(viii) Rule 3(7)(iii)
assuming that the expenditure per meal is upto Rs. 50.
3. Computer provided at the residence of Mr. X is not taxable as per section 17(2)(viii)Rule 3(7)(vii).

DT QUESTION BANK BY CA PRANAV CHANDAK Subscribe us on YouTube 40


Q35. R (age 45) employee of X Ltd receives the following salary & perquisites from his employer in PY 2018-19:
▪ Basic pay- Rs. 3.60,000, profit bonus- Rs. 36,000
▪ Commission on sales @ 1% of turnover (turnover of PY 2018-19 achieved by R- Rs. 12,00,000);
▪ Advance salary of April, May & June 2019 - Rs. 90,000;
▪ Employer's contribution towards RPF: Rs. 54,000, interest credited on 16.10.2018 in RPF A/c @ 12% - Rs.
36,000;
▪ Conveyance Allowance: 6,000 which is fully used for official purpose
▪ Rent-free furnished house in Mumbai (lease rent of unfurnished house paid by employer - Rs. 96,000, rent
of furniture- Rs. 36,000);
▪ Free services of a gardener (salary- Rs. 9,000); free services of personnel attendant (salary- Rs. 12,000);
free services of watchman (salary - Rs. 250 p.m.)
▪ R takes on interest-free loan of Rs. 2,00,000 on 1.12.2018 from the employer repayable after 3 years for
higher education of his daughter (SBI lending rate on 1.4.2018- 12%).
▪ He owns a house at Mumbai. During the PY, he earns taxable income of Rs. 2,43,000 from the house.
▪ R holds 20% equity shares in the employer-company.
▪ During the PY, R makes the following payments & investments:
1. Own contribution towards recognised provident fund- Rs. 60,000.
2. Fixed deposit of Rs. 24,000 in a bank u/s 80C notified for the purpose of section 80C.
Determine the amount of the liability for the AY 2019-20.
Solution: Taxable income of R for AY 2017-18
Basic pay 3,60,000
Profit bonus 36,000
Commission on sales (1% of Rs. 12,00,000) 12,000
Advance salary 90,000
Employer's contribution in excess of 12% of salary (Rs. 54,000 - 12% of Rs. 3,72,000) 9,360
Interest credited to PF account, being the amount in excess of 9.5% p.a. (36,000 x 2.5%/12) 7,500
Conveyance allowance (Exemption as used for official purposes) -
Rent-free house (see Note 1) 97,200
Services of gardener 9,000
Services of personnel attendant 12,000
Services of watchman 3,000
Interest free loan (interest on Rs.2,00,000 @ 12% p.a. from 1.12.2016 to 31.3.2017) 8,000
Gross salary 6,44,060
Less- Deduction u/s 16 -
Income from salary 6,44,060
Income from house property 2,43,000
Gross Total Income 8,87,060
Less: Deduction u/s 80C: Contribution towards RPF 60,000
Notified Fixed Deposit in Bank 24,000 (84,000)
Taxable Income (rounded off) 8,03,060
Note:
1. Salary for the computation of taxable value of perquisite in respect of rent-free house works out to Rs. 4,08,000
(i.e., Rs. 3,60,000 + Rs. 36,000 + Rs. 12,000). Advance salary for a period other than the PY is not taken into account.
Lease rent of unfurnished house is Rs. 96,000.
As lease rent > 15% of salary, Rs. 61,200, being 15% of salary, is taxable value of unfurnished house.
Value of furnished house will, therefore, be Rs.97,200 i.e., Rs.61,200 + rent of furniture- Rs.36,000).

Q36. Mr. Srivastava (age 40 years) a salaried employee of Niija Ltd. was contributing to National Pension Scheme
Rs. 50,000 every year since 2016 & was claiming deduction u/s 80CCD. In December 2018, he opted out of the
pension scheme & withdrew a lump sum amount of Rs. 2,00,000. Is the amount so withdrawn taxable? If yes, how
much is the taxable amount? [NOV - 2017]
Answer:
▪ As per section 80CCD, if assessee has received any amount from accumulated balance under NPS, amount so
received is taxable but w.e.f. AY 2018-19 some exemption has been granted u/s 10(12A) & is as given below:
▪ Any payment from the National Pension System Trust to an employee on closure of his account or on his opting
out of the pension scheme referred to in section 80CCD shall be exempt to the extent of 40% of the total amount
payable to him at the time of such closure or his opting out of the scheme.
▪ As per section 80CCD, Lumpsum amount received by the nominee on the death of the assessee is fully exempt.
▪ Thus, Taxable Value = Rs. 2,00,000 - (40% x 2,00,000) [Exempt u/s 10(12A)] = Rs. 1,20,000.

DT QUESTION BANK BY CA PRANAV CHANDAK Subscribe us on YouTube 41


Q37. Compute the amount of LTC Exemption in the following cases: [NOV 16] /[MAY 13]
(a) Mr. S went on a holiday on 9.9.2018 to Mysore with his wife & 3 children - one daughter bom on 2.2.2012 &
twin sons born on 5.5.2014. The total cost of travel was Rs. 80,000. The ticket cost for Mr. X & his wife was Rs.
50,000 & for all three children was Rs. 30,000.The Employer reimbursed total ticket cost Rs. 80,000.
(b) In the above case (a), if among his 3 children the twin sons bom on 02.02.2012 & the daughter was bom on
05.05.2014, what shall be the exemption?
Solution:
Case (a):
▪ Maximum Exemption of LTC is for Cost of Travel on the shortest route for
(i) Spouse & Assessee
(ii) Children – when first child is there, & second children are twins, then the Twin Children are considered as
one child, & hence exemption is available for all 3 Children in this case.
▪ Total Amount Exempt = 50,000 + 30,000 = Rs. 80,000.
Case (b):
▪ If the First Child is Twin sons, & Second Child is a Daughter, the exemption is not available to Daughter, as the
twin sons are considered as two children.
▪ So, the exempted amount is [(30,000/3) x 2)] = Rs. 20,000.
▪ Total Amount exempt = Rs. 70,000 (Rs. 50,000 for spouse & Rs. 20,000 for children)
▪ Taxable Amount shall be Rs. 80,000 - Rs. 70,000 = Rs. 10,000.

Q38. R, a manager of X Ltd., which is engaged in manufacturing chemicals gives the details for AY 2019-20:
▪ Basic salary: 36,000 p.a; Bonus: 30,000 p.a; Commission: Rs. 45,000 p.a
▪ Entertainment allowance: Rs. 12,000 p.a.
▪ A rent-free unfurnished house has been provided in Kolkata; lease rent of the house- Rs. 2,40,000 p.a;
▪ Employer has provided free use of Ford IKON 1300CC car for official as well as personal purposes; expenses
for the employer- Rs. 84,000 (out of which 30% is attributable towards personal purposes). Car can also be
used by the family members of R. Rs. 12,000 is recovered from R for using the car for private purpose.
▪ The employer pays a sum of Rs. 1,200 on account of professional tax on behalf of R.
▪ On 12.7.2018, X Ltd. gifts a computer, manufactured by it, on its founder day to R (cost of the computer to
employer being Rs. 34,000).
▪ Employer's contribution towards RPF: Rs. 54,000 p.a; Contribution of R towards RPF: Rs.75,000;
▪ Payment of insurance premium by R on his wife's insurance policy- Rs. 65,000;
▪ Payment of insurance premium by Mrs. R on R's insurance policy- Rs. 16,000;
▪ Contribution to PPF - Rs. 50,000. Interest received on company deposit- Rs.3,52,000 (gross).
Determine the taxable income & tax liability for the AY 2019-20 on the assumption that R is neither a director in
the employer-company nor a shareholder.
Solution:
Basic salary (Rs. 36,000 x 12) 4,32,000
Bonus 30,000
Commission 45,000
Entertainment allowance 12,000
Rent-free house (see Note 1 below) 77,850
Car (1800 x 12) 21,600
Employer's contribution towards RPF (i.e., Rs.54,000 -12% of Rs.4,32,000) 2,160
Professional tax paid by employer 1,200
Gift by employer of computer (34,000 - 5,000) 29,000
Gross salary 6,50,810
Less- Deductions u/s 16
Professional tax 1,200
Income from salary 6,49,610
Income from other sources (interest on company deposit) 3,52,000
GTI 10,01,610
Less- Deduction u/s 80C (see Note 2) 1,50,000
Net income (rounded off) 8,51,610
Income-tax on Rs.8,51,610 95,322
Add- Education cess& SHEC @ 3% 2,859.66
Tax payable (rounded off) 98,180
Notes:
1. Salary for the purpose of valuation of the perquisite in respect of rent-free house comes to Rs.5,19,000 (i.e.,
basic salary- Rs.4,32,000 + bonus- Rs.30,000 + commission Rs.45,000 + entertainment allowance- Rs. 12,000).

DT QUESTION BANK BY CA PRANAV CHANDAK Subscribe us on YouTube 42


2. Lease rent of the house is Rs.1,50,000. As lease rent of the house exceeds 15% of salary, Rs. 77,850 (being
15% of salary) is taxable value of the perquisite.
3. Deduction u/s 80C
Contribution towards recognised provident fund 75,000
Insurance premium on wife's insurance policy 65,000
Insurance premium paid by Mrs. R (not eligible) Nil
Contribution to PPF 50,000
Gross qualifying amount 1,90,000

Q39. From the following information submitted by A compute his taxable salary for the AY 2019-20.
▪ Basic salary Rs. 10,000 p.m; D.A. Rs. 1,000 p.m. (60% of which forms part of retirement benefits).
▪ City Compensatory Allowance: Rs. 100 p.m;
▪ House Rent Allowance: Rs. 2,000 p.m. He pays Rs. 3,000 p.m. as rent for a house situated in Delhi.
▪ Children Education Allowance: Rs. 120 p.m. for 3 children; Hostel Allowance: Rs. 700 p.m. for 2 children.
▪ Medical allowances: Rs. 500 p.m. He spends Rs. 3,000 during the year on his own medical treatment & Rs.
2,000 on the medical treatment of his brother not dependent on him.
▪ He was also provided with the facility of a watchman, sweeper & gardener. X Ltd., the employer, pays wages
@ Rs. 1,000 p.m. each to said persons.
▪ He has also been provided with a motor car of 1400 cc engine cubic capacity rating with driver which he
uses partly for his official purposes & partly for his personal purposes.
▪ His professional tax of Rs. 300 is paid by employer.
▪ He contributes Rs. 600 p.m. to his RPF to which his employer contributes equal amount.
▪ Interest @11% p.a. amounting to Rs. 22,000 was credited to his provident fund for year ending 31.3.2019.
Solution:
Salary 1,20,000
DA 12,000
City Compensatory Allowance 1,200
HRA [WN1] 720
Children Education Allowance [(120 × 1 children + 20 × 2 children) × 12 months] 1,920
Hostel Expenditure Allowance [Exempt = Rs. 300 × 12 months × 2 child] = 7200 [8400-7200] 1,200
Medical Allowance – Fixed medical allowance did always taxable [500 × 12] 6,000
Salary to Watchman (1,000 x 12) 12,000
Sweeper (1,000 x 12) 12,000
Gardener (1,000 x 12) 12,000
Value of car [(1800 + 900) x 12] 32,400
Professional Tax 300
Interest on RPF (Taxable in excess of 9.5%) [(
22,000
) × 1.5 %] 3,000
11%
Gross Salary 2,13,300
Less: Professional Tax u/s 16 (300)
Net Salary 2,13,000
Working Notes
1. HRA
(1) Actual HRA 2,000 p.m.
(2) 3,000 - 1,060 (Rent Paid - 10% of Salary) 1,940 p.m.
(3) 50% of salary (10,000 + 600) 5,300 p.m.
Exempt HRA 1,940 p.m
Taxable HRA [2000 – 1940] 60 p.m.

Q40. Mr. X, employed as Production Manager in B Ltd., furnishes you the following information for PY 2018-19:
▪ Basic salary upto 31.10.2018: 50,000 p.m. [From 1.11.2018: 60,000 p.m] Salary is due & paid on last day.
▪ Dearness allowance @ 40% of basic salary.
▪ Bonus = one-month salary paid in Oct 2018 on Basic salary + DA applicable for that month.
▪ Contribution of employer to RPF A/c of the employee @ 16% of basic salary.
▪ Professional tax paid Rs. 3,000 of which Rs. 2,000 was paid by the employer.
▪ Facility of laptop & computer was provided to Mr. X for both official & personal use. Cost of laptop: 45,000
& computer: 35,000 were acquired by the company on 01.12.2018.
▪ Motor car owned by the employer (cubic capacity of engine exceeds 1.60 litres) provided to the employee
from 01.11.2018 meant for both official & personal use. Repair & running expenses of Rs. 45,000 from
01.11.2018 to 31.03.2019 were fully met by the employer. The motor car was self-driven by the employee.
▪ Leave travel concession given to employee, wife & 3 children (one daughter: age 7 & twin sons - age 3).

DT QUESTION BANK BY CA PRANAV CHANDAK Subscribe us on YouTube 43


▪ Cost of air tickets (economy class) reimbursed by the employer Rs. 30,000 for adults & Rs. 45,000 for
three children. Mr. X is eligible for availing exemption this year to the extent it is permissible in law.
Compute the salary income & also tax liability of Mr. X for the AY 2019-20. [NOV-2011]
Answer: Computation of Salary chargeable to tax of Mr. X for AY 2019-20
Basic Salary (50,000 x 7)+ (60,000 x 5) 6,50,000.00
Dearness Allowance (40% x 6,50,000) 2,60,000.00
Bonus (50,000 + 20,000) 70,000.00
Contribution to recognized provident fund (6,50,000 x 4%) 26,000.00
Professional Tax paid by the employer 2,000.00
Facility of Laptop/computer NIL
Perquisite value of use of motor car (2,400 x 5) 12,000.00
Leave Travel concession NIL
Gross Salary 10,20,000.00
Less: Standard Deduction u/s 16(ia) (40,000.00)
Less: Deduction of professional tax u/s 16(iii) (3,000.00)
Income under the head Salary 9,77,000.00
Gross Total Income 9,77,000.00
Less: Deduction u/s 80C to 80U Nil
Total Income 9,77,000.00

Q41. R (age 55), RNOR, is employed by Indian company. For the PY 2018-19, he submits the following details:
▪ Salary of 3 months of service in New York (paid by the foreign branch in Germany): Rs. 40,000 p.m.
▪ Salary of 9 months of service in Delhi: Rs. 50,000 p.m.
▪ Bonus: Rs. 16,000.
▪ Employer's contribution towards RPF @ Rs.6,000 p.m. for the entire PY (PF is maintained in India when
R was posted abroad; employer's contribution was transferred to a separate account in Germany & later
on , it is remitted to India with employee contribution): Rs. 48,000.
▪ Free car facility of 1500 cc in Delhi for office & private purpose of R & his family members - Expenditure
of the employer: Rs. 36,000
▪ Car allowance in Germany @ Rs. 10,000 p.m. (one-third of which is utilised for private purpose): 30,000
▪ Besides, the employer-company provides a rent-free furnished flat both in Delhi & Germany. While lease
rent of the flat provided of Delhi is Rs. 20,000 p.m. (rent of furniture- Rs. 7,000 p.m.), lease rent of the flat
provided at Germany is Rs. 40,000 p.m. (rent of furniture- Rs.9,000 p.m.).
▪ During PY 2018-19, R is paid a special allowance of $ 10,000 by UNO in appreciation of his services
rendered in Germany.
▪ His income from other sources is Rs. 3,60,000. On 29.3.2019, the employer sells a computer to R for Rs.
24,000 (it was purchased by the company for Rs. 90,000 on 16.6.2015, & up till its transfer to R, it was
used by the employer for business purposes).
R makes the following payments- (a) his contribution to RPF- Rs. 56,000 (contribution was remitted from Germany
when he was posted abroad); (b) contribution to PPF - Rs. 39,000; (c) life insurance premium to German insurance
company- Rs. 48,000 (sum assured - Rs.4,00,000);& (d) tuition fees of R son- Rs. 2,000 p.m.
Determine taxable income & tax liability for AY 2019-20 on assumption that he holds 20% equity share capital in
employer-company & on 1.4.2019, he pays Rs. 2000, being professional tax pertaining to PY 2017-18.
Solution: R is a RNOR in India for the AY 2018-19. Income earned & received out of India is not taxable in India.
Therefore, salary & perquisites received o/s India for rendering service in Germany is not taxable in India.
Salary of 3 months of service in Germany -
Salary of 9 months of service in Delhi (Rs.50,000 x 9) 4,50,000
Bonus 16,000
Employer's contribution towards RPF in excess of 12% of salary (6,000 x 12 - 12% of (1,20,000 +
4,50,000); salary of the entire PY will be taken, as the provident fund is maintained in Delhi) 3,600
Free car facility in Delhi (1800 x 9) 16,200
Car allowance in Germany not taxable as R is a RNOR -
Rent-free flat in Delhi (see Note 1) 1,46,700
Rent-free flat in Germany is not taxable as R is a RNOR -
Sale of computer (see Note 4) Nil
Pension from Delhi Government (Rs.12,000 x 12) 1,44,000
Income from salary 7,76,500
Income from other sources 3,60,000
Gross Total Income 11,36,500
Less: Deduction u/s 80C (see Note 5) 1,50,000
Net income (rounded off) 9,86,500

DT QUESTION BANK BY CA PRANAV CHANDAK Subscribe us on YouTube 44


Note:
1. Salary for the purpose of valuation of rent-free flat at Delhi comes to Rs.5,58,000 (i.e., Rs.4,50,000 + pension of
9 months- Rs.1,08,000). Lease rent of the flat is Rs. 1,80,000 (i.e., Rs.20,000 x 9). As lease rent of the flat exceeds
15% of salary, Rs. 83,700 (being 15% of salary) is taxable value of unfurnished flat. Rent of furniture is Rs. 63,000.
Therefore, Rs. 1,46,700 (i.e., Rs. 83,700 + Rs.63,000) is value of rent-free furnished flat provident in Delhi.
2. Salary, allowance or perquisite from UNO is not chargeable to tax.
3. Professional tax is deductible on payment basis. As professional tax is paid after the end of PY 2018-19, it is not
deductible.
4. Perquisite in respect of sale of computer
Cost of computer to the employer 90,000
Less: Normal wear & tear for first year ending 15.6.2016 (50%of Rs.90,000) 45,000
Balance as on 16.6.2016 45,000
Less: Normal wear & tear for second year ending 15.6.2017 (50% of Rs.45,000) 22,500
Balance as on 16.6.2017 22,500
Less: Sale price paid by R 24,000
Balance Nil

5. Deduction u/s 80C


Contribution to RPF 56,000
Contribution to PPF 39,000
Payment of insurance premium to German insurance company (Max of 10% of sum assured) 40,000
Tuition fees of R's son (Rs. 2,000 x 12) 24,000
Total 1,59,000
Deduction u/s 80C – Maximum 1,50,000

Q42. Mr. X retired from the services of M/s Y Ltd. on 31.01.2019 after completing service of 30 years & one
month. He had joined the company in 1988 at the age of 30 years & received the following on his retirement:
▪ Gratuity 76,00,000. He was covered under the Payment of Gratuity Act, 1972.
▪ Leave encashment of 73,30,000 for 330 days leave balance in his account. He was credited 30 days leave for
each completed year of service.
▪ He purchased one motor car from the company on 31.03.2019. This car was purchased on 01.07.2015 by the
company for 75,00,000. It was put use by the company on the same date. The car was sold by the company to
Mr. X for 72,00,000. Company depreciates the vehicles at the rate of 15% on Straight Line Method.
▪ An amount of Rs. 3,00,000 as commutation of 2/3 of his pension.
▪ Company presented him a gift voucher worth Rs. 6,000 on his retirement.
▪ His colleagues also gifted him a Television (LCD) worth Rs. 1,50,000 from their own contribution.
▪ He has drawn a Basic Salary of Rs. 20,000 & 50% DA p.m from 1.4.2018 to 31.1.2019.
▪ Received pension of Rs. 5,000 p.m for the period 1.2.2019 to 31.3.2019 after commutation of pension.
Compute his total income & tax liability from the above for AY 2019-20.
Solution: Computation of Total Income
Basic Salary (20,000 x 10) 2,00,000
DA (2,00,000 x 50%) 1,00,000
Gift voucher (6,000 - 5,000) 1,000
Motor Car [Sec 17(2)(viii)] [WDV – Amount recovered] = 2,56,000 – 2,00,000 [WN1] 56,000
Uncommuted Pension {Sec 17(1)} (5,000 x 2) 10,000
Commuted pension {Sec 1O(1OA)} [Exempt = 3,00,000 x 3/2 x 1/3 = 1,50,000] 1,50,000
Gratuity {Sec 10(10)} [WN 2] 80,769
Leave Salary {Sec 10(10A)} [WN 3] 1,30,000
Gross Salary 7,27,769
Less: Standard Deduction u/s 16(ia) (40,000)
Income under the head Salary 6,87,769
Income from Other Sources (Since LCD is not covered u/s u/s 56) Nil
Gross Total Income 6,87,769

Working Note:
1. Motor Car
Actual Cost of car
Rs. 5,00,000
Less: Depreciation @ 20%: 1.7.2015 - 30.6.2016: (1,00,000
1.7.2016 - 30.6.2017: (80,000)

DT QUESTION BANK BY CA PRANAV CHANDAK Subscribe us on YouTube 45


1.7.2017 - 30.6.2018: (64,000) (Rs. 2,44,000)
WDV of car Rs. 2,56,000

2. Gratuity: 3. Leave salary


Least of the following is exempt = Rs. 5,19,231; Least of the following is Exempt: Rs. 2,00,000
(a) Gratuity received Rs. 6,00,000 (a) Rs. 3,30,000
(b) Rs. 10,00,000 (b) Rs. 10 x 20,000 = Rs. 2,00,000
(c) 15/26 x 30,000 x 30 = 5,19,231 (c) Rs. 3,00,000
Taxable gratuity = 6,00,000 – 5,19,231 = 80,769. (d) 330/30 x 20,000 = 2,20,000;
Taxable Leavy salary = 3,30,000 – 2,00,000 = 1,30,000.

Q43. Mr. Devan is the Director of a tyre company at Bangalore since 1.1.2000.
▪ Salary grade is 14,000 – 400 - 18,000 – 600 - 27,000.
▪ Dearness Allowance @ 20% basic pay (50% forms part of retirement benefits.
▪ He contributes 12% of (salary & 50% of DA) to a RPF to which his employer contributes an equal amount.
▪ He has been provided with rent-free house owned by the company, the fair rent of which is Rs.72,000 p.a.
▪ He is getting a Transport allowance of Rs.1,800 p.m. for travelling from residence to office & back.
▪ Medical Allowance of Rs. 1,000 p.m. & Servant Allowance of Rs. 500 p.m.
▪ His club bills worth Rs. 5,000 were also reimbursed by the company.
▪ He has been provided with the facility of gardener & cook who are each paid Rs. 150 p.m. by the employer.
▪ He is also provided with a car of 2000 cc by the employer for official use only.
▪ Two children of Mr. Devan are studying in an institute run by the employer for which no fees is paid.
Normal expenditure per student in such institution is Rs. 800 p.m.
Compute Mr. Devan's taxable salary for AY 2019-20.
Solution:
Basic Salary (22,800 x 9 + 23,400 x 3) 2,75,400
Dearness Allowance 55,080
Transport Allowance 21,600
Medical Allowance 12,000
Servant Allowance 6,000
Club Bills 5,000
Cook 1,800
Gardner 1,800
Motor car given for official purpose Exempt
Children Education facility [Nothing is taxable as less than Rs. 1000 p.m. per child] Nil
*Rent Free Accommodation 51,381
Gross Salary 4,30,061
Less: Standard Deduction u/s 16(ia) (40,000)
Income under the head salary 3,90,061
Working Note: Rent free Accommodation: Salary for the purpose of rent-free accommodation = 2,75,400 +
27,540 + 21,600 + 12,000 + 6,000 = Rs. 3,42,540 × 15% = 51,381.

Q44. Mr. X, Marketing Manager of KL Ltd. based at Mumbai furnishes following information for PY 2018-19:
▪ Basic salary: Rs. 1,00,000 per month
▪ Dearness allowance - Rs. 50,000 per month (forming part for retirement benefit salary)
▪ Bonus - 2 Months basic salary
▪ Contribution of employer to RPF @ 15% of basic salary + Dearness allowance
▪ Rent free unfurnished accommodation provided by company at Mumbai (owned by company).
▪ RPF contribution made by Mr. X: Rs. 1,50,000
▪ Health insurance premium for his family paid by cheque: Rs. 20,000
▪ Health insurance premium in respect of parents (senior citizens) paid by cheque: Rs. 28,000
▪ Medical expenses of dependent brother with severe disability (covered): 60,000
▪ Interest on loan taken for education of his son studying B.com (full-time) in a recognized college: 24,000
▪ Interest on loan taken for education of a student for whom Mr. X is the legal guardian for pursuing B.Sc.
(Physics) (full-time) in a recognized university: Rs. 20,000.
Compute the Total Income of Mr. X for the AY 2019-20. [Nov 2010]
Answer: Computation of Total Income of Mr. X for AY 2019-20
Particulars Rs.
Basic salary 12,00,000
Dearness allowance 6,00,000
Bonus 2,00,000

DT QUESTION BANK BY CA PRANAV CHANDAK Subscribe us on YouTube 46


Employer contribution to RPF in excess of 12% is taxable (3% of 18,00,000) [See Note below] 54,000
Rent free accommodation @ 15% of Rs. 20 lacs (basic salary + dearness allowance + bonus) 3.00,000
Less: Standard deduction u/s 16 (ia) (40,000)
Gross Salary 23,14,000
Less: Deductions under Chapter VI-A
(i) Section 80C Contribution to RPF: 1,50,000
(ii) Section 80D - Health insurance premium Family: 20,000
(iii) Parents (Senior Citizens) Section 80D: 28,000 (1,98,000)
Medical treatment of dependent brother with severe disability [Sec 80DD] (1,25,000)
Section 80E
Interest on loan taken for his son studying B.Com: 24,000
Interest on loan taken for a student studying B.Sc. for whom he is legal guardian: 20,000 (44,000)
Total Income 19,47,000

Q45. X was employed with ABC Ltd. He retired w.e.f. 1.2.2019 after completing a service of 24 years & 4 months.
▪ Basic Salary: Rs. 10,000 per month (at the time of retirement)
▪ Dearness Allowance: 120% of Basic Salary (40% of basic salary forms part of salary).
▪ Last increment: Rs. 1,000 w.e.f. 1.7.2018
▪ His pension was determined at Rs. 6,000 per month.
▪ He got 50% of the pension commuted w.e.f. 1.3.2019 & received Rs. 2,00,000 as commuted pension.
▪ In addition to this, he received a gratuity of Rs. 2,40,000 & leave encashment amounting to Rs. 1,12,000 on
account of accumulated leave of 240 days.
▪ He was entitled to 40 days leave for every year of service.
Compute his Gross Salary for AY 2019-20 assuming that he is not covered under Payment of Gratuity Act.
Solution: Computation of Gross Salary of X (For the AY 2018-19)
Basic Salary (Rs.9,000 x 3 + 10,000 x 7) 97,000
Dearness Allowance (120%) 1,16,400
Uncommuted Pension (Rs.6,000 x 1 + 3,000 x 1) 9,000
Commuted Pension
Amount Received 2,00,000
Less- Exempt (Rs. 2,00,000/50% × 1/3) (1,33,333) 66,667
Gratuity-
Amount received 2,40,000
Less: Exempt (Note 1) 1,62,960 77,040
Leave Encashment
Amount received 1,12,000
Less: Exempt (Note 2) Nil 1,12,000
Gross Salary 4,78,107
Working Notes-
1. Gratuity shall be exempt to the extent of minimum of following limits-
(i) Actual amount received Rs.2,40,000
(ii) Half month's average salary for each completed year of service = 13580/2* 24 = 162960.
(iii) Specified amount Rs. 10,00,000.
AMS = 97000 + 38800 /10 = Rs. 13,580.

2. Leave encashment shall be exempt to the extent of minimum of following limits-


(i) Actual amount received Rs. 1,12,000
(ii) 10 months average salary (10 x 13,580) Rs.1,35,800
(iii) Cash for unavailed leave calculated on basis of 30 days leave Nil
(iv) Amount specified Rs.3,00,000
Total leave entitlement on basis of 30 days = 720 days – leaves availed = 720 days.
Entitlement to encashment on basis of 30 days = Nil.

Q46. Mr. X, an Accounts Manager, has retired from JK Ltd. on 15.01.2019 after rendering services for 30 years 7
months. His salary is Rs. 25,000/- p.m. upto 30.09.2018 & Rs. 27,000 p.m. thereafter. He also gets Rs. 2,000/- p.m.
as dearness allowance (55% of it is a part of salary for computing retirement benefits).
He is not covered by the payments of Gratuity Act, 1972. He has received Rs. 8 Lacs as gratuity from the employer.
Solution: Computation of gratuity taxable in hands of Mr. X for PY 2018-19
As per section 10(10), gratuity received by an employee would be exempt upto the least of the following limits -

DT QUESTION BANK BY CA PRANAV CHANDAK Subscribe us on YouTube 47


(i) Gratuity received = Rs. 8,00,000
(ii) Half-month’s salary for every year of completed service = % x 26,700 x 30 = Rs. 4,00,500
(iii) Monetary limit = Rs. 10,00,000.
Taxable Gratuity = Gratuity Received – Gratuity Exempt = Rs. 8,00,000 – Rs. 4,00,500 = Rs. 3,99,500.
Note:
1. One of the limits for calculation of gratuity exempt u/s 10(10) is one-half-month’s salary for each year of
completed service (fraction of a year to be ignored), calculated on the basis of average salary for the ten months
immediately preceding the month of retirement.
Retirement month is January 2019. Therefore, average salary for March 2018 to Dec 2018 have to be considered.
The salary is Rs. 25,000 p.m. upto 30.09.2018 & Rs. 27,000 p.m. from 01.10.2018.
Hence, average salary = Rs. 26,700 [(Rs. 25,000 x 7) + (Rs. 27,000 x 3) + (Rs. 2000 x 55% x 10)]/10.
Note: ½ month’s salary × 30 (completed Years of service).

Q47. Mrs. X (aged 40 years) is working with ABC Company Ltd., a manufacturer of tyres based at Mumbai, has
received the following payments during the financial year 2018-19 from her employer:
▪ Basic salary: Rs. 60,000 per month.
▪ Dearness allowance: 40% of basic salary.
▪ She has paid professional tax of Rs. 6,000 during financial year 2018-19.
▪ She is owning only one house & payment of interest of Rs. 1,75,000 & principal of Rs. 1,00,000 was made
for housing loan taken from SBI for purchase of house.
▪ She has also taken a loan of Rs. 2,00,000 from her employer for study of her son. SBI rate for such loan is
10%. Her employer has recovered Rs. 10,000 as interest from her salary for such loan during the year.
Compute Total Income & Tax Liability for AY 2019-20.
Answer: Computation of Taxable Income of Mrs. X for the AY 2019-20
Basic salary (60,000 x 12) 7,20,000
Dearness Allowance (7,20,000 x 40%) 2,88,000
Accommodation at concessional rent {Sec 17(2)(ii) Rule 3(1)} 84,000
Perquisite of Interest on loan {Sec 17(2)(viii) Rule 3(7)(i)} [(2,00,000 x 10%)-10,000] 10,000
Gross Salary 11,02,000
Less: Standard Deduction u/s 16(ia) (40,000)
Less: Professional Tax u/s 16(iii) (6,000)
Income under the head Salary 10,56,000
Computation of income under the head House Property
Gross Annual Value (15,000 x 12) 1,80,000
Less: Municipal Tax Nil
Net Annual Value 1,80,000
Less: 30% of NAV u/s 24(a) (54,000)
Less: Interest on capital borrowed u/s 24(b) (1,75,000)
Loss under the head House Property (49,000)
Gross Total Income 10,07,000
Less: Deduction u/s
80C-Repayment of principal (1,00,000)
Total Income 9,07,000

Q48. Mr. X an employee of XYZ Co. Ltd. at Mumbai & covered by Payment of Gratuity Act, retires at the age of 64
years on 31.12.2018 after completing 33 years & 7 months of service.
▪ On retirement, his employer pays Rs. 20,51,640 as Gratuity & Rs. 6,00,000 as accumulated balance of RPF
▪ He is entitled for monthly pension of Rs. 8,000.
▪ He gets 75% of pension Commuted for Rs. 4,50,000 on 1.2 2019.
▪ Basic Salary (Rs. 80,000 x 9): Rs. 7,20,000; Bonus: Rs. 36,000.
▪ House Rent Allowance ( Rs. 15,000 x 9): Rs. 1,35,000
▪ Rent paid by Mr. X (710,000 x 12): Rs. 1,20,000
▪ Employer contribution towards RPF: Rs. 1,10,000.
▪ Professional Tax paid by Mr. X: Rs. 2,000.
Determine taxable salary of Mr. X for AY 2019-20. Note: Salary & Pension falls due on last day. [Nov - 14]
Solution: Computation of income under the head Salary
Basic Pay (80,000 x 9) 7,20,000
Bonus 36,000
House rent allowance {Sec 10(13A), Rule 2A} [WN 1] 1,17,000
Employer’s contribution to provident fund {Part A of schedule IV} [WN 2] 23,600

DT QUESTION BANK BY CA PRANAV CHANDAK Subscribe us on YouTube 48


Gratuity {Sec 10(10)} [WN 3] 10,51,640
Uncommuted Pension {Sec 17(1)} 12,000
Commuted Pension (Sec 10(10A)} 2,50,000
Gross Salary 22,10,240
Less: Standard Deduction u/s 16(ia) (40,000)
Less: Deduction u/s 16(iii) - Professional Tax (2,000)
Income under the head Salary 21,68,240

Working Note:
1. House Rent Allowance 2. Contribution to PF
Lower of below is exempt: Least of the following is exempt:
a. Rs. 90,000 - Rs. 72,000 = Rs. 18,000 Retirement benefit salary = Rs. 7,20,000
b. 50% of retirement benefit salary = 3,60,000 Employer contribution = Rs. 1,10,000
(Retirement Benefit Salary = 80,000 x 9 = 7,20,000) Allowed = 12% of retirement benefit salary = 86,400;
c. Amount Received = Rs. 1,35,000 Taxable = Rs. 23,600
Exempt = (Rs. 18,000); Taxable = Rs. 1,17,000
3. Gratuity 4. Uncommuted pension
Least of the following is exempt: January: 8,000 x 1 = 8,000
1. Rs. 20,51,640 Feb & March: 8,000 x 25% x 2 = Rs. 4,000
2. Rs. 10,00,000 Total = 12,000
3.15/26 x 80,000 x 34 = Rs. 15,69,230.77 5. Uncommuted pension
Received = Rs. 20,51,640; Exempt = ( Rs. 10,00,000); Received = 4,50,000; Exempt = 4,50,000/75% x 1/3 =
Taxable = Rs. 10,51,640. (2,00,000); Taxable = 2,50,000
Note: Rs. 6,00,000 as accumulated balance of Recognised Provident fund received from employer is exempt as
Mr. X has rendered continuous service of more than 5 Years with the employer.

Q49. Mr. Honey is working with a Domestic Company having a production unit in the U.S.A. for last 15 years, he
has been regularly visiting India for export promotion of Company's product. He has been staying in India for at
least 184 days every year. He submits the following information:
▪ Salary received outside India (For 6 Months) Rs. 50,000 p.m.
▪ Salary received in India (For 6 months) Rs. 50,000 p.m.
▪ He has been given rent free accommodation in U.S.A for which company pays Rs. 15,000 per month as
rent, but when he comes to India, he stays in the guest house of the company.
▪ During this period he is given free lunch facility, for which company incurred Rs. 48,000.
▪ He has been provided a car of 2,000 CC in USA which is used by him for both office & private purposes.
The actual cost of the car is Rs. 8,00,000. But when he is in India, the car is used by him & the members
of his family only for personal purpose. The monthly expenditure of car is Rs. 5,000.
▪ His elder son is studying in India for which his employer spends Rs. 12,000 per year where as his younger
son is studying in U.S.A & stays in a hostel for which Mr. Honey gets Rs. 3,000 p.m as combined allowance.
▪ The Company has taken an accident insurance policy & a life insurance policy. During the year the
company paid premium of Rs. 5,000 & Rs. 10,000 respectively.
Compute Mr. Honey's taxable income from salary for the Assessment Year 2019-2020. [May 18]
Solution: Computation of Income from Salary
Particulars Amount
Basic Salary [50,000 x 12] 6,00,000
RFA in USA [WN 1] 95,400
RFA in India [WN 2] Nil
Lunch Facility [(48,000) – (Rs. 50 x 184)] 38,800
Motor Car in USA (2400 x 6) 14,400
Motor Car in India (5000 x 6) + [8,00,000 x 10% x 6/12] 70,000
Education expenditure of Elder Son met in India – Taxable Perquisite 12,000
Education Allowance: Fully taxable – (3,000 x 12) – No exemption for allowance received for 36,000
education & hostel facility of children outside India.
Accident Insurance Life Insurance 10,000
Gross Salary 8,76,600
Less: Deductions u/s 16(ia) Standard Deduction (40,000)
Income under the head Salaries 8,36,600
Note:
1. Salary for the purpose of RFA = Basic Salary (6,00,000) + All taxable allowances (36,000) = 6,36,000.
2. Taxable RFA during Stay in USA = 15% of salary or Actual rent paid by Employer (whichever is lower) = 15%
on 6,36,000 = 95,400 or 15,000 x 12 = 1,80,000 whichever is lower = 95,400

DT QUESTION BANK BY CA PRANAV CHANDAK Subscribe us on YouTube 49


3. Taxable RFA during stay in India = since the accommodation is provided to him for his stay in India entirely
for official purpose, it is not taxable.

Q50. Mr. Anand, an employee of XYZ Co. Ltd at Mumbai & covered by the Payment of Gratuity Act retires at the age
of 64 years on 31.12.2018 after completing 33 years & 7 months of service. At the time of retirement, his Employer
pays Rs. 20,51,640 as Gratuity & Rs.6,00,000 as Accumulated Balance of Recognised Provident Fund. He is also
entitled for a monthly pension of Rs. 8,000. He gets 75% of Pension commuted for Rs.4,50,000 on 1st Feb 2019.
Determine taxable Salary of Mr. Anand for AY 2019-2020, with the help of the following information: [N 14]
Basic Salary (Rs. 80,000 x 9) Rs.7,20,000
Bonus Rs.36,000
House Rent Allowance ( Rs.15,000 x 9) Rs.1,35,000
Rent paid by Mr. Anand (Rs.10,000 x 12) Rs.1,20,000
Employer Contribution towards RPF Rs.1,10,000
Professional Tax paid by Mr. Anand Rs.2,000
Note: Salary & Pension falls due on the last day of each month.
Solution: Computation of Taxable Salary
Particulars Amount
Salary (80,000 x 9) 7,20,000
Pension (8,000 x 1) + (8,000 x 25% x 2) 12,000
Bonus Given 36,000
Taxable HRA (W.N.l) 1,17,000
Employers’ Contribution to Recognised PF above 12% (1,10,000-7,20,000 x 12%) 23,600
Taxable Gratuity (W.N.2) 10,51,640
Taxable Commuted Pension (W.N.3) 2,50,000
Accumulated Balance of Recognised PF Exempt u/s 10(12) since Service NIL
> 5 years
Gross Salary 22,10,240
Less: Deduction u/s 16 (ia) Standard Deductions (40,000)
Less: Deduction u/s 16 Profession Tax Given (2,000)
Taxable Salary 21,68,240

Working Notes:
1. Computation of Taxable HRA u/s 10(13A)
Particulars Amount Amount
Amount of HRA Received (15,000 x 9) 1,35,000
Less: Amount exempt u/s 10(13 A) = Least of the following -
(a) Actual HRA Received 1,35,000
(b) 50% of Salary (Being at Mumbai) = 50% of Rs. 7,20,000 3,60,000
(c) Rent Paid Less 10% of Salary = (10,000 x 9) – (10% of 7,20,000) 18,000 (18,000)
Taxable HRA 1,17,000
Note: Salary = Basic + DA (Considered for Retirement Benefits) + Commission as a percentage of Turnover. =
7,20,000 + Nil + Nil = Rs. 7,20,000.
Rent paid is considered only for the period during which HRA is received, i.e. 9 months.
2. Taxable Gratuity u/s 10(10) [covered under Payment of Gratuity Act]
Amount of Gratuity Received 20,51,640
Less: Amount exempt u/s 10(10) = Least of the following -
(a) Actual Gratuity received 20,51,640
(b) 15/26 x Last Drawn Salary x No. of completed years of Service or part 15,69,231
thereof (15/26 x 80,000 x 34 Years) [Salary = Basic + DA, taken as Rs. 80,000
26 pm]
(c) Notified Amount 20,00,000 (10,00,000)
Taxable Gratuity 10,51,640

3. Taxable Commuted Pension u/s 10(10A)


Particulars Amount
Commuted Pension Received 4,50,000
Less: Exempt u/s 10(10A) = l/3rd of Full Value of Pension (Non-Govt Employee in receipt of 2,00,000
Gratuity) (Note: Full Value of Pension = Rs.4,50,000 75% = 6,00,000)
Taxable Commuted Pension 2,50,000

DT QUESTION BANK BY CA PRANAV CHANDAK Subscribe us on YouTube 50


Q51. Dr. Manjula Grover received the following during PY 2017-18:
Holiday home facility maintained by the employer
- Expense of the employer 40,000
- Market value 50,000
Car of 1.4 litre engine capacity purchased from the employer
Purchase price 1,10,000
Car purchased by the employer used by her for 4 years & 4 months. During current FY, it was 400000
used by her for 7 months
Basic Salary 4,00,000
Dearness Allowance 50,000
City Compensatory Allowance 10,000
Compute her taxable income for the AY 2018-19.
Solution- Computation of taxable income of Dr. Manjula Grover (For AY 2018-19)
Basic Salary 4,00,000
Dearness Allowance 50,000
City Compensatory Allowance 10,000
Holiday home facility 40,000
Value of perquisite of car (Rs.1800 x 7) 12,600
Value of car sold at concessional value
WDV of car after 4 completed PYs - Price at which given to employee 1,63,840 – 1,10,000 53,840
Gross Salary 5,56,440
WDV (Depreciated value) of car
Cost of car 4,00,000
Less: First year depreciation @ 20% (80,000)
Less: Second year depreciation @ 20% on reducing balance (64,000)
Less: Third year depreciation @ 20% on reducing balance (51,200)
Less: Fourth year depreciation @ 20% on reducing balance (40,960) (1,63,840)

Q52. Determine the taxable value of the perquisite in the following cases-
1. R is employed by A Ltd. on 1.7.2018, the company gives an interest-free housing loan of Rs. 12,00,000. Loan is
repayable within 5 years. The lending rate of SBI for similar loan as on 1.4.2018 is 10.25%.
2. S is employed by B Ltd. On 1.4.2018, he takes a personal loan of Rs. 50,000 from B Ltd. B. Ltd. recovers interest
@ 7% p.a. from S. SBI lending rate for similar loan is 15%.
3. C Ltd. gives the following interest-free loan to T, an employee of the company - Rs. 12,000 for child's education
& Rs. 8,000 for purchasing a refrigerator. No other loan is given by C Ltd.
4. R purchases a Honda City 1.6 Lxi on 1.3.2018 from a loan of Rs. 12,00,000 taken at concessional rate of 7% p.a.
from his employer XYZ Ltd. As per the agreed terms of repayment, R is supposed to repay in monthly instalments
of Rs. 40,000 starting from 1.1.2019. Compute the taxable value of perquisite in respect of concessional loan for
the PY 2018-19. SBI lending rate for similar loan is 12%.
5. R taken an interest-free loan of Rs. 10,00,000 from his employer R Ltd., on 12.6.2018 for medical treatment of
his wife who is suffering from a disease specified in rule 3A. Mrs. R is also covered under a mediclaim insurance
cover. Insurance company reimburses her of the hospitalization charges of Rs. 4,00,000 on 1.1.2019. According to
terms of repayment of loan, R has to pay Rs. 20,000 p.m. on the seventh day of each month starting November
2019. The amount paid by the insurance company is retained by R. SBI lending rate is 16%.
Solution:
1. Rs. 92,250 (being interest @ 10.25% on Rs. 12,00,000 from 1.7.2018 to 31.3.2019) is taxable in the hands of R.
2. Rs. 4,000 [being interest @ 8% (i.e. 15% - 7%) on Rs. 50,000 for one year] is taxable in the hands of S.
3. Nothing is taxable in the hands of T as the aggregate amount of loan does not exceed Rs.20,000.
4. Maximum monthly outstanding amount for the various months in the PY 2018-19 is as follows-
Month Maximum monthly outstanding Interest
amount on last day of the month
30.4.2018 12,00,000 5,000 (Rs. 12,00,000 x 5% x 1/12)
31.5.2018 12,00,000 5,000
30.6.2018 12,00,000 5,000
31.7.2018 12,00,000 5,000
31.8.2018 12,00,000 5,000

DT QUESTION BANK BY CA PRANAV CHANDAK Subscribe us on YouTube 51


30.9.2018 12,00,000 5,000
31.10.2018 12,00,000 5,000
30.11.2018 12,00,000 5,000
31.12.2018 12,00,000 5,000
31.1.2019 11,60,000 4,833 (i.e., Rs.11,60,000 x 5% x 1/12)
28.2.2019 11,20,000 4,667 (i.e., Rs.11,20,000 x 5% x 1/12)
31.3.2019 10,80,000 4,500 (i.e., Rs. 10,80,000 x 5% x 1/12)
Taxable value of concessional loan is Rs. 59,000 for the PY 2018-19.
5. The perquisite in respect of interest-free loan provided for medical treatment in respect of a disease specified
in rule 3A is not taxable. However, it will not include that amount as has been reimbursed to the employee under
any medical insurance scheme. Therefore, out of loan of Rs. 10,00,000 granted to R, Rs. 4,00,000 reimbursed by
insurance company on 1.1.2019 will be taken into account for computing taxable value of perquisite. The SBI
lending rate for a similar loan is 16% on 1.4.2018.
Month Maximum monthly outstanding Interest
amount on last day of month
31.1.2019 3,80,000 5,067 (Rs. 3,80,000 x 16% x 1/12)
28.2.2019 3,60,000 4,800 (Rs. 3,60,000 x 16% x 1/12)
31.3.2019 3,40,000 4,533 (Rs. 3,40,000 x 16% x 1/12)
Taxable value of interest-free loan is Rs. 14,400 for PY 2018-19.

Q53. Compute Total Income & Tax Payable for AY 2019-20 by Mr. X with the help of following information:
(a) X retired on 31.12.2018 (age 58) after 25 years & 9 months of service, from a Pvt. Company situated at Mumbai.
(b) He was paid a Salary of 40,000 p.m. & HRA of 10,000 p.m. He paid Rent of 13,000 p.m. during his service.
(c) On retirement, he was paid a Gratuity of 6,00,000. He was not covered by the Payment of Gratuity Act.
(d) He had accumulated leave of 15 days per annum during the period of his service, this was encashed by Mr. X at
the time of his retirement. Rs. 5,15,000 was received by him in this regard. [Note: His Average Salary is Rs. 37,500]
(e) After retirement, he ventured into textile business & incurred a loss of Rs. 80,000 for the period upto 31.3.2019.
(f) Mr. X has invested Rs. 22,500 in Recognized Provident Fund, Rs.1,20,000 in PPF & Rs. 37,500 in NSC. [M 05]
Answer: 1. Computation of Total Income
Particulars Rs.
Salary (40,000 x 9) 3,60,000
Taxable HRA (WN 1) 9,000
Taxable Gratuity (WN 2) 1,31,250
Taxable Leave Encashment (WN 3) 2,15,000
Gross Salary 7,15,250
Less: Deductions u/s 16 (ia) Standard Deduction of Rs. 40,000 (40,000)
Income under the head “Salaries” 6,75,250
Income from Business or Profession (WN 4) Nil
Gross Total Income 6,75,250
Less: Deduction under Chapter VI A, i.e. Sec. 80C (WN 5) (1,50,000)
Total Income 5,25,250
Tax Payable = [12,500 + (5,25,250 – 5,00,000) x 20%] + HEC at 4% = 17,550 + 702 = 18,252.

Working Notes: 1. Computation of Taxable HRA u/s 10(13A)


Amount of HRA Received (10,000 x 9) 90,000
Less: Amount exempt u/s 10(13 A) = Least of the following -
(a) Actual HRA Received 90,000
(b) 50% of Salary (Being at Mumbai) = 50% of Rs.3,60,000 1,80,000
(c) Rent Paid Less 10% of Salary = (13,000 x 9) – (10% of 3,60,000) 81,000 (81,000)
Taxable HRA 9,000
Note: Salary = Basic + DA (Considered for Retirement Benefits) + Commission as a percentage of Turnover.

DT QUESTION BANK BY CA PRANAV CHANDAK Subscribe us on YouTube 52


2. Taxable Gratuity u/s 10(10)
Amount of Gratuity Received 6,00,000
Less: Amount exempt u/s 10(10) = Least of the following -
(a) Actual Gratuity received 6,00,000
(b) ½ x Avg. Salary of 10 months x Completed years of Service (1/2 x 37,500 x 25) 4,68,750
(c) Notified Amount 20,00,000 (4,68,750)
Taxable Gratuity 1,31,250

3. Computation of Taxable Leave Encashment


Amount of Leave Encashment Received 5,15,000
Less: Amount exempt u/s 10(10AA) = Least of the following -
(a) Amount actually received 5,15,000
(b) Average Salary of past 10 months’ Salary x 10 months (37,500 x 10) 3,75,000
(c) Notified Amount 3,00,000
(d) Leave Encashment Calculation on basis of 30 days credit for every completed 4,68,750 (3,00,000)
years of service (15 x 25 x 37,500/30)
Taxable Leave Encashment 2,15,000

4. Treatment of Loss: Loss u/h PGBP cannot be set off against Income under the head Salaries (Sec. 71). So, it
shall be carried forward u/s 72 for a period of 8 Assessment Years & can be set off only against Business Income.

5. Computation of Deduction u/s 80C


Contribution to RPF 22,500
PPF (Maximum Investment Rs. 1,50,000) 1,20,000
NSC 37,500
Total (Maximum deduction u/s 80C restricted to Rs.1,50,000) 1,80,000

Q54. Khanna, an Employee of IOL, New Delhi, Private Sector Company, received the following for PY 2018-2019:
Basic Salary Rs. 3,20,000, House Rent Allowance Rs. 90,000, Special Allowance Rs. 50,000. Khanna was residing
at New Delhi & was paying a rent of Rs. 10,000 a month.
(a) Compute eligible exemption u/s 10(13A) in respect of House Rent Allowance received.
(b) If Khanna opts for Rent Free Accommodation whereby IOL would be paying a rent of Rs. 10,000 per month,
to the Landlord, & recovers a sum of Rs.1,000 per month from Khanna which was in excess of his entitlement,
what will be the perquisite value in respect of such Rent-Free Accommodation?
(c) Which of the above would be beneficial to Khanna, i.e. House Rent Allowance or Rent-Free Accommodation?
Note: Students should analyse the Cash Flow (not just only Tax Liability) in case of HRA Vs RFA Decisions. [N 07]
Solution: Statement of Total Income & Tax Payable
Particulars HRA Option RFA Option
Basic Salary 3,20,000 3,20,000
Special Allowance 50,000 50,000
House Rent Allowance |WN 1] 2,000 N.A.
Perquisite Value of Rent-Free Accommodation [WN 2] N.A. 43,500
Gross Salary 3,72,000 4,13,500
Less: Deduction u/s 16(ia) Standard Deduction (40,000) (40,000)
Income under the head of “Salaries” 3,32,000 3,73,500
Less: Deduction under Chapter VI-A — —
Total Income 3,32,000 3,73,000
Tax on Total Income 4,100 6,175
Add: HEC at 4% 164 247
Total Tax Payable (Rounded Off) 4,164 6,422

Cash Flow Statement


Inflow – Salary [Basic + HRA received + Special Allowance] 4,60,000 3,70,000
Less: Outflow – Rent Paid (1,20,000) (12,000)
Tax on Total Income (4,164) (6,422)
Net Inflow 3,35,836 3,51,578
Decision: Since Net cash Inflow under RFA Rs. 3,51,578 is higher than HRA Rs. 3,35,836 RFA option is beneficial
for Khanna even though the Tax liability is higher under this option.

DT QUESTION BANK BY CA PRANAV CHANDAK Subscribe us on YouTube 53


Working Notes:
1. Computation of Taxable House Rent Allowance
Actual House Rent Allowance Received 90,000
Less: Exempt u/s 10(13 A) = Least of the following -
- 50% of Basic Pay – (3,20,000 x 50%) (1,60,000)
- Actual HRA (90,000)
- Rent paid Less 10% of salary [1,20,000 – 10% (3,20,000)] (88,000) (88,000)
Taxable HRA 2,000

2. Computation of Perquisite Value of Rent-Free Accommodation:


Lower of (i) Rent paid by Employer or (ii) 15% of Salary [Rs. 1,20,000 or 15% of 3,70,000] 55,500
Less: Amount recovered from Employee [1,000 x 12] (12,000)
Value of Perquisite 43,500
Note: For the purposes of Perquisite Valuation of Accommodation Facilities, Salary includes DA if considered for
retirement benefits, all Taxable Allowances, Bonus, Commission or Ex-gratia & other monetary payments.
Therefore, Salary for RFA = Rs.3,20,000 + Rs.50,000 = Rs. 3,70,000.

Q55. Mr. Mohit is employed with XY Ltd on a Basic Salary of Rs.10,000 p.m. He also got an increment of Rs. 1,000
per month in his Basic Salary with effect from 1.2.2019.
▪ He is entitled to DA at 100% of Basic Salary, 50% of which is included in Salary as per Terms of
Employment.
▪ Company gives him House Rent Allowance of Rs.6,000 p.m. which was increased to Rs. 7,000 p.m. w.e.f
1.1.2019.
▪ Rent paid by him during the Previous Year 2018-2019 is as under:
▪ April & May 2018: Nil, as he stayed with his parents
▪ June to Oct 2018: Rs. 6,000 p.m. for an accommodation in Ghaziabad
▪ Nov 2018 to March 2019: Rs. 8,000 p.m. for an accommodation in Delhi
Compute his Gross Salary for Assessment Year 2019-2020.
Solution: Computation of Gross Salary
Particulars Amount
1. Basic Salary [(10,000 x 10) + (11,000 x 2)] 1,22,000
2. Dearness Allowance (100% of 1,22,000) 1,22,000
3. House Rent Allowance (12,000 + 7,500 + 800) [WN 1 to 3] 20,300
Gross Salary 2,64,300
Less: Deductions u/s 16 (ia) Standard Deduction (40,000)
Taxable Salary 2,24,300

Working Notes:
1. For the period from 01.04.2018 to 31.05.2018, the Assessee stays with his parents, he does not incur any rent
expenditure. So, HRA received for this period is fully taxable. Taxable HRA = Rs. 6,000 x 2 = Rs. 12,000.
2. Computation of Taxable HRA for the period 1.6.2018 to 31.10.2018 in Ghaziabad – [For 5 Months]
(a) Computation of Salary for HRA Exemption
Basic Salary (10,000 x 5) 50,000
DA considered for Retirement Benefits (50,000 x 50%) 25,000
Commission forming part of Salary as a Fixed Percentage of Turnover of Employee Nil
Salary for HRA Exemption 75,000
(b) Taxable HRA
Amount Received during the financial year towards HRA (6,000 x 5) 30,000
Less: Exemption u/s 10(13A) is the least of the following -
(a) Actual amount of HRA received 30,000
(b) 40% of the Salary = 40% x 75,000 30,000
(c) Rent Paid Less 10% of Salary [(6,000 x 5) – 7,500] 22,500 (22,500)
Taxable Amount of HRA 7,500

3. Computation of Taxable HRA for the period 01.11.2018 to 31.03.2019 in Delhi – [5 Months]
(a) Computation of Salary for HRA Exemption
Basic Salary [(10,000 x 3) + (11,000 x 2)] 52,000
DA considered for Retirement Benefits (52,000 x 50%) 26,000
Commission forming part of Salary as a Fixed Percentage of Turnover of Employee Nil
Salary for HRA Exemption 78,000

DT QUESTION BANK BY CA PRANAV CHANDAK Subscribe us on YouTube 54


(b) Taxable HRA
Amount Received during the financial year towards HRA [(6,000 x 2)+(7,000 x 3)] 33,000
Less: Exemption u/s 10(13A) is the least of the following -
(a) Actual amount of HRA received 33,000
(b) 50% of the Salary = 50% x 78,000 (since located in Delhi) 39,000
(c) Rent Paid Less 10% of Salary (40,000 – 7,800) 32,200 (32,200)
Taxable Amount of HRA 800

Q56. R is offered an employment by ABC Ltd., Mumbai with the following two alternatives:
Particulars I II
Basic salary 40,000 40,000
Bonus 6,000 6,000
Education allowance for 2 children 10,200 -
Education facility for 2 children in an institution maintained by the employer - 10,200
Sweeper allowance 10,000 -
Free sweeper - 10,000
Entertainment allowance 6,000 -
Club facility - 6,000
Conveyance allowance for personal use 12,000 -
Free car facility for personal use - 12,000
Medical allowance 18,000 -
Medical facility for R & family members in its own hospital - 18,000
Allowance for gas, electricity & water supply 4,500 -
Free gas, electricity & water supply - 4,500
A rent-free unfurnished house- Fair rent 24,000 24,000
R in neither a director nor he has substantial interest in the company. Which of the two alternatives he should opt
for on the assumption that both employer & employee will contribute 10% of salary towards URPF?
Solution:
Particulars Alternative I Alternative II
Basic salary 40,000 40,000
Bonus 6,000 6,000
Education allowance (10,200 - 2,400) 7,800 NA
Education facility [Exempt upto Rs. 1,000 per month per child) NA Exempt *
Sweeper allowance 10,000 NA
Free Sweeper NA Exempt *
Entertainment allowance 6,000 Exempt
Club facility NA 6,000
Conveyance allowance 12,000 NA
Car facility NA Exempt *
Medical allowance 18,000 NA
Medical Facility in employer’s hospital NA Exempt
Allowance for gas/electricity/water 4,500 NA
Free facility for gas/electricity/water NA Exempt *
Rent free unfurnished house (Note 2) 15,645* 6,900*
Gross Salary 1,19,945 58,900
Less: Standard Deduction u/s 16(ia) 40,000 40,000
Income from Salary 79,945 18,900

Note:
1. Such facilities are exempt in case of Non-specified Employees.
2. Salary for the purpose of Rent-free accommodation =
Alternative I = 40,000 + 6,000 + 7,800 + 10,000 + 6,000 + 12,000 + 18,000 + 4,500 = 1,04,300.
= 15% of 1,04,300 = 15,645*
Alternative II (Non-specified employee): Rs.40,000 + 6,000 = Rs. 46,000 = 15% of Rs. 46,000 = Rs. 6,900*.

DT QUESTION BANK BY CA PRANAV CHANDAK Subscribe us on YouTube 55


4B. INCOME FROM HOUSE PROPERTY
Q1. X has built a house on a Leasehold Land. He has let-out the above property & claims that the rental Income
from such Property is taxable u/h "IFOS", & deducted Expenses on Repair, Security Charges, Insurance & Collection
Charges, in all amounting to 40% of receipts. State the head of income under which the receipt is to be assessed &
comment.
Answer:
▪ Mr. X is the owner of the house property.
▪ The fact that the house is built on leasehold land is irrelevant. Ownership of the building is relevant & not the
ownership of the land on which building stands.
▪ Thus Income from such house is liable to tax, u/h “Income from House Property”. Since it is chargeable under
the head House Property, deduction u/s 24 alone can be claimed & no other deductions are allowed.

Q2. Arvind commenced construction of a residential house intended exclusively for his residence, on
1.11.2017 He raised a loan of Rs. 7 Lacs @ 16% Interest for the purpose of construction on
1.11.2017 Finding that there was an overrun in the cost of construction he raised a further loan of Rs. 10
Lacs at the same rate of interest on 01.10.2018.
What is the interest allowable u/s 24, assuming that the construction was completed on 31.03.2019? [M 2000]
Solution: Computation of Interest u/s 24
1. Prior Period: 01.11.2017 to 31.03.2018
2. Prior Period Interest Calculation: Loan -1: Rs. 7,00,000 x 5/12 x 16% 46,667
3. 1/5th of Prior Period interest: (1/5 x 46,667) 9,333
4. Interest for Current Year: Loan – 1: Rs. 7,00,000 x 16% 1,12,000
Loan – 2: Rs. 10,00,000 x 16% x 6/12 80,000 1,92,000
5. Total Interest (3 + 4) 2,01,333
6. Deduction Allowable u/s 24 in respect of Interest on Borrowed Capital (Note) 2,00,000
Note: In case of Self Occupied Property, the maximum permissible interest deduction is Rs. 2,00,000.

Q3. Compute gross annual value in the following cases for the AY 2019-20: [Modified May 2012]
Particulars Situation 1 Situation 2 Situation 3 Situation 4
Fair Rent (p.m.) 9,000 13,000 12,000 16,000
Municipal Valuation (p.m.) 10,000 9,000 9,000 18,000
Standard Rent (p.m.) 12,000 11,000 7,000 16,000
Rent received/ receivable (p.m.) 7,000 11,500 20,000 16,500
Vacancy 1 month 1 month 2 month 2 months
Solution:
Situation 1
(a) Fair Rent (9,000 x 12) 1,08,000
(b) Municipal Valuation (10,000 x 12) 1,20,000
(c) Higher of (a) or (b) 1,20,000
(d) Standard Rent (12,000 x 12) 1,44,000
(e) Expected Rent {Lower of (c) or (d)} 1,20,000
(f) Actual Rent Received/Receivable (7,000 x 11) 77,000
▪ If there was no vacancy, in that case actual rent received/receivable would have been Rs. 84,000.
▪ Thus we can say that ER > ARR due to other reasons & thus GAV = ER = 1,20,000.
Situation 2
(a) Fair Rent (13,000 x 12) 1,56,000
(b) Municipal Valuation (9,000 x 12) 1,08,000
(c) Higher of (a) or (b) 1,56,000
(d) Standard Rent (11,000 x 12) 1,32,000
(e) Expected Rent {Lower of (c) or (d)} 1,32,000
(f) Actual Rent Received/Receivable (11,500 x 11) 1,26,500
▪ In this case, if there was no vacancy, ARR would have been Rs. 1,38,000.
▪ Thus we can say that ARR < ER due to vacancy & thus GAV = ARR = 1,26,500.

DT QUESTION BANK BY CA PRANAV CHANDAK Subscribe us on YouTube 56


Situation 3
(a) Fair Rent (12,000 x 12) 1,44,000
(b) Municipal Valuation (9,000 x 12) 1,08,000
(c) Higher of (a) or (b) 1,44,000
(d) Standard Rent (7,000 x 12) 84,000
(e) Expected Rent {Lower of (c) or (d)} 84,000
(f) Actual Rent Received/Receivable (20,000 x 10) 2,00,000
▪ Since ARR > ER despite vacancy, GAV = ARR = 2,00,000.

Situation 4
(a) Fair Rent (16,000 x 12) 1,92,000
(b) Municipal Valuation (18,000 x 12) 2,16,000
(c) Higher of (a) or (b) 2,16,000
(d) Standard Rent (16,000 x 12) 1,92,000
(e) Expected Rent {Lower of (c) or (d)} 1,92,000
(f) Actual Rent Received/Receivable (16,500 x 10) 1,65,000
▪ In this case, if there was no vacancy, ARR would have been Rs. 1,98,000.
▪ Thus we can say that ARR < ER due to vacancy & thus GAV = ARR = 1,65,500.

Q4. Mr. Ganesh owns a Commercial Building whose construction got completed in June 2017. He took a Loan of
Rs. 15 Lacs from his friend on 1.8.2016 & had been paying Interest calculated at 15% p.a.
Mr. Ganesh has let out the Commercial Building at a monthly rent of Rs. 40,000 during FY 2018-19. He paid
Municipal Tax of Rs. 18,000 each for PY 2017-18 & PY 2018-19 on 1.5.2018 & 5.4.2019 respectively. Compute
Income under the head 'House Property' of Mr. Ganesh for AY 2019-20. [May 17]
Solution:
Gross Annual Value = Actual Rent Receivable ( Rs. 40,000 x 12 months) 4,80,000
Less: Municipal Taxes (deductible only on actual payment basis, during PY i.e 01.05.2017 (18,000)
Net Annual Value 4,62,000
Less: Deductions u/s 24
(a) Standard Deduction at 30% of Net Annual Value (1,38,600)
(b) Interest on loan borrowed: Current Year Interest: Rs. 15,00,000 x 15% = 2,25,000
: Pre-Construction Interest at 1/5th of Rs. 1,50,000 = 30,000 (2,55,000)
Income from House Property 68,400
Note:
• Pre-Construction Period Interest = Rs. 15,00,000 x 15% 8/12 (from 01.08.2015 to 31.03.2016) = Rs. 1,50,000.
• This is deductible in 5 years form PY 2017-2018 onwards. Hence, Share of this Interest for PY 2018-2019 = 1/5th.

Q5. R owns a house property in Delhi, which is let out for Rs. 8,000 p.m. Municipal Value of the house is Rs. 60,000
& Fair Rent is Rs. 90,000. Standard Rent of the house property was Rs. 80,000. Municipal taxes of the house for the
relevant PY were Rs. 20,000 but the assessee paid the municipal taxes of the next five years also in advance. The
house remained vacant for March 2019. Compute the income under the head house property for the AY 2019-20.
The other particulars of the house were as under:
(i) Repairs & collection charges = Rs. 3,000; (ii) Interest on money borrowed for purchase: Rs. 10,000;
Solution:
Particulars Amount
Gross Annual Value [WN1] Rs. 88,000
Less: Municipal tax paid by the owner (Rs. 1,20,000)
Net Annual Value [GAV – MT] (Rs. 32,000)
Less: Deduction u/s 24
24(a): Std Deduction @ 30% Nil
24(b): Interest (10,000)
Income from House Property (42,000)
Working Note: Since ARR > ER, GAV = ARR; ER = 80,000; ARR: 8,000 x 11= Rs. 88,000; GAV = Rs. 88,000.

DT QUESTION BANK BY CA PRANAV CHANDAK Subscribe us on YouTube 57


Q6. For AY 2019-20, R submits the following information, Determine the taxable income of R for the AY 2019-20.
Particulars House A House B
Fair Rent 1,20,000 1,40,000
Actual Rent 1,32,000 1,06,000
Standard Rent 1,26,000 1,20,000
Municipal Tax due 18,000 20,000
Repairs 6,000 8,000
Insurance 3,000 4,000
Land revenue (paid) 4,000 2,000
Ground rent 2,000 5,000
Interest on capital borrowed by mortgaging A (funds are used for construction of B) 30,000
Solution: Computation of taxable income of X
Particulars House A House B
GAV 1,32,000 1,20,000
Less: Municipal taxes [Not paid] Nil Nil
NAV 1,32,000 1,20,000
Less: Deduction u/s 24
24(a): Standard deduction @ 30% (39,600) (36,000)
24(b): Interest - (30,000)
Income from house property 92,400 54,000
Income from house property [Rs. 92,400+ Rs. 54,000] = 1,46,400

Q7. R is the owner of a residential house whose construction was completed on 31.8.2008. It has been let out from
1.1.2009 for residential purposes. Its particulars for the financial year 2018-19 are given below:
Municipal valuation (p.a.) 65,000
Fair Rent (p.a.) 72,000
Standard rent under the Rent Control Act (p.m.) 7,000
Actual rent (p.m.) 7,000
Municipal taxes paid (including Rs. 5,000 paid by tenant) 20,000
Water/sewerage benefit tax, levied by State Government paid under protest 5,000
Interest on loan taken for the construction of the house. Interest has been paid o/s India to NR 15,000
without TDS (NR had agreed to pay income-tax on such Interest direct to the Government)
Legal charges for the recovery of rent 4,000
Stamp duty & registration charges incurred in respect of lease agreement of house 2,000
The unrealised rent for PY 2009-10 is recovered of Rs. 20,000 from the defaulting tenants. Compute Income from
house property for the AY 2019-20.
Solution: Computation of Income from House Property
Gross Annual rent 84,000
Less: Municipal taxes paid by owner 15,000
Net Annual value 69,000
Less: Deduction u/s 24
24(a) - Standard deduction @ 30% (20,700)
24(a) - Interest: Paid to NR without deducting TDS, hence, not deductible Nil (20,700)
48,300
Add: Unrealised rent recovered [20,000 * 70%] 14,000
Income from house property 62,300

Q8. Mr. X owns one residential house in Mumbai. The house is having 2 units. 1st unit of the house is self-occupied
by Mr. X & another unit is rented for Rs. 8,000 p.m. Rented unit was vacant for 2 months during the year. The
particulars of the house for the Previous Year 2018-2019 are as under:
Standard Rent Rs. 1,62,000 p.a.
Fair Rent Rs. 1,85,000 p.a
Municipal Value Rs. 1,90,000
Light & Water Charges Rs. 500 p.m.
Money Repairs Rs. 1,200 p.m
Municipal Tax paid 15 %
Interest on Borrowed Capital Rs. 1,500 p.m.
Compute Income from House Property of Mr. X for AY 2019-20. [NOV - 13/Nov 2008/Mod. Nov 2012]

DT QUESTION BANK BY CA PRANAV CHANDAK Subscribe us on YouTube 58


Solution: Computation of Income from House Property
Particulars 1st Unit SOP 2nd Unit LOP
Gross Annual Value NIL 80,000
Less: Municipal Taxes (assumed as paid during PY) [Rs. 1,90,000 x 15% x 50%] NIL (14,250)
Net Annual Value NIL 65,750
Less: Deduction u/s 24
Deduction at 30% of NAV (Rs. 65,750 x 30%) NIL (19,725)
Interest on Borrowed Capital (Rs. 750 p.m. x 12 months) each for 2 units (9,000) (9,000)
Income from House Property (9,000) 37,025
Taxable Income from House Property 28,025

Working Note:
1. GAV of 2nd Unit is determined as:
Expected Rent = Higher of MV (Rs. 95,000) or FR (Rs. 92,500) subject to Maximum of SR (Rs. 81,000) = Rs. 81,000.
Actual Rent Receivable = Rs. 80,000 (8,000 x 10)
If there would have been no vacancy, ARR would be Rs. 96,000. Thus, we can say that ARR < ER due to vacancy,
GAV = ARR = Rs. 80,000.
2. Annual Value of 1st Unit: Since the House Property is self-occupied by the Assessee, GAV is taken as NIL.
3. Light & Water Charges, Lease Money, Insurance charges & Repairs are not allowable as deduction u/s 24.
4. Set-off of Losses: Loss from one House property can be set off against Income from another Property, u/s 70.

*Q9. A owns a house property at Delhi. 60% of house property is self-occupied for residence & 40% is let out on
monthly rent of Rs. 5,000. Let-out portion was also self-occupied from 1.10.2018 to 31.12.2018. However, w.e.f.
1.1.2019, entire house was let out for Rs. 12,500 pm. Construction of house property was completed on 31.12.1998.
The following expenses were incurred for the above house property during the year ending on 31.3.2019.
Municipal Tax paid for FY 2016-17 : 5,000 FY 2017-18 : 10,000 FY 2018-19 : 15,000
Insurance premium paid: 3,000; Land revenue due: 6,000; Interest on money borrowed for construction: 18,000
Calculate income under the head house property of A for the AY 2019-20.
Solution: As both the unit are let out for the part of the year & self-occupied for some part of the year, benefit of
self-occupation for residential purpose u/s 23(2) will not be available & NAV of both units shall be determined as
per section 23(1).
(a) ER 12,500 x 12 1,50,000
(b) AR
Rs. 5,000 x 6 30,000
Rs. 12,500 x 3 37,500 67,500
Thus GAV = 1,50,000
Less: Municipal tax paid (5,000 + 10,000 + 15,000) (30,000)
NAV 1,20,000
Less: Standard deduction @ 30% (36,000)
Interest (18,000) 54,000
Income from House Property 66,000

*Q10. Ganesh has two houses, both of which are SOP. The particulars of the houses for the PY 2018-19 are as
under:
Particulars House I House II
Municipal valuation p.a. 1,00,000 1,50,000
Fair rent p.a. 75,000 1,75,000
Standard rent p.a. 90,000 1,60,000
Date of completion 31.3.1999 31.3.2001
Repairs Rs. 12,000 Rs. 4,000
Insurance Premium Rs. 1,500 Rs. 1,800
Municipal taxes paid during the year 12% 8%
Interest on money borrowed for repair of property during the current year - 55,000
Compute Ganesh’s income from house property for AY 2019-20 & suggest which house should be opted by Ganesh
to be assessed as self-occupied so that his tax liability is minimum. [May 14/SM]
Solution: Computation of income from house property of Ganesh for AY 2019-20
Let us first calculate the income from each house property assuming that they are deemed to be let out.
Particulars Amount in Rs.

DT QUESTION BANK BY CA PRANAV CHANDAK Subscribe us on YouTube 59


House I House II
GAV [ER = GAV since both are SOP [Higher of MV & FR, but restricted to SR] 90,000 1,60,000
Less: Municipal taxes (paid by the owner during the PY) 12,000 12,000
NAV 78,000 1,48,000
Less: Deductions u/s 24
(a) 30% of NAV 23,400 44,400
(b) Interest on borrowed capital - 55,000
Income from house property 54,600 48,600

OPTION 1 (House I: SOP & House II: DLOP)


If House I is opted to be self-occupied, the income from house property shall be –
Particulars Amount
House I (Self-occupied) Nil
House II (Deemed to be let-out) 48,600
Income from house property 48,600

OPTION 2 (House I: DLOP & House II: SOP)


Particulars Amount
House I (Deemed to be let-out) 54,600
House II (Self-occupied) (interest deduction restricted to Rs. 30,000) (30,000)
Income from house property 24,600
Since Option 2 is more beneficial, Ganesh should opt to treat House II as SOP & House I as DLOP. His income from
house property would be Rs. 24,600 for the AY 2019-20.
Note:
1. Gross Annual Value for SOP is Nil. Municipal Taxes paid for SOP shall not be allowed as deduction.
2. For Loan taken for repair, Interest on Housing Loan shall be allowed as deduction u/s 24 up to a maximum of
Rs. 30,000 in case of SOP.

Q11. Mrs. R is the owner of a two storied house in Madras. She gets a monthly rent Rs. 7,000 from her tenant in
the ground floor. The first floor, identical in all respect with the ground floor used to be occupied by a friend of
Mrs. R from whom she charged a rent of Rs. 5,000 per month. During the year ended 31.3.2019, the friend stayed
in Mrs. R house up to 31.12.2018. On 1.1.2019 it was again let out to tenant at a rent of Rs.7,000 per month.
Details of expenses incurred by Mrs. R during the year ending 31.3.2019 in respect of the house were as under:
(a) Cost of repairing ground floor: Rs. 7,500; (b) Cost of repairing first floor: Rs. 50,000
(c) Interest on loan taken for construction of first floor: Rs. 20,000;
(d) Municipal, tax paid by owner: Rs. 6,000
(e) Monthly salary of an employee for collecting rent: Rs. 1,000.
Compute Mrs. R's income from house property for the AY 2019-20.
Solution: Computation of Income from house property of Mrs. R
Ground Floor
GAV 84,000
Less: Municipal Taxes paid 3,000
NAV 81,000
Less: Deduction u/s 24
Standard deduction @ 30% 24,300
Income from ground floor 56,700
First Floor
GAV, higher of the following two:
ER (7,000 x 12) 84,000
AR (5,000 x 9) + (7,000 x 3) 66,000 84,000
Less: Municipal taxes paid 3,000
NAV 81,000
Less: Deductions u/s 24
(a) Standard deduction @ 30% 24,300
(b) Interest on borrowed capital 20,000 44,300
Income from first floor 36,700
Total income from House Property (Rs. 56,700 + Rs. 36,700) = Rs. 93,400.

DT QUESTION BANK BY CA PRANAV CHANDAK Subscribe us on YouTube 60


Q12. R has a property which consists of three identical flats. Construction of the property started in 1988 & on
completion the property was occupied for the first time on 5.9.1994. One flat is let-out on Rs. 5,000 p.m., & second
for Rs. 6,000 p.m., third is occupied by R himself. Other relevant details are given below:
(i) MV: Rs. 1,50,000; (ii) SR: Rs. 1,98,000; (iii) FR: Rs. 2,00,000; (iv) Municipal tax paid: Rs. 24,000;
(v) Ground rent paid: Rs. 3,000; (vi) Insurance premium on property: Rs. 1,500.
R had borrowed Rs. 1,00,000 against his fixed deposits to repair the tenanted portion of the flat which is let out
for Rs. 5,000 p.m. He paid interest of Rs. 12,000 on the said loan. He had spent Rs. 4,000 for collection of rent. The
second unit which was let for Rs.6,000 p.m. remained vacant for a period of 2 months during the year.
Compute the income under the head house property for the AY 2019-20.
Solution:
Unit I Unit II Unit III
LOP for Rs. 5,000 p.m. LOP for Rs. 6,000 p.m. SOP
Annual municipal valuation 50,000 50,000 50,000
FR 66,667 66,667 66,667
Standard Rent 66,000 66,000 66,000
AR exclusive of vacancy allowance 60,000 60,000 Nil
GAV 66,000 60,000 Nil
Less: Municipal taxes paid 8,000 8,000 Nil
58,000 52,000 Nil
Less: Deductions u/s 24
Standard deduction @ 30% 17,400 15,600 Nil
Interest on money borrowed for repairs 12,000 - -
Income from 3 units 28,600 36,400 Nil
Income from house property Rs. 28,600 + 36,400 = Rs. 65,000.

Q13. Prem owns a house in Madras. During PY 2018-19, 2/3rd portion of the house was self-occupied & 1/3rd
portion was let out for residential purposes at a rent of Rs. 8,000 p.m. MV = Rs. 3,00,000 p.a., FR = Rs. 2,70,000 p.a.
& SR = Rs. 3,30,000 p.a. He paid MT @10% during the year. A loan of Rs. 25,00,000 was taken by him during the
year 2013 for acquiring the property. Interest on loan paid during PY 2018-19 was Rs. 1,20,000. Compute Prem’s
income from house property for the AY 2019-20. [Similar to Nov 2012]
Solution:
▪ There are two units of the house. Unit I with 2/3rd area is used by Prem for self- occupation throughout the
year & no benefit is derived from that unit, hence it will be treated as SOP & its NAV will be Nil.
▪ Unit 2 with 1/3rd area is let-out throughout the previous year & its NAV has to be determined as per sec 23(1).
Computation of Income from house property of Mr. Prem for AY 2019-20
Unit I (2/3rd area: Self-occupied)
NAV Annual Value Nil
Less: Deduction u/s 24(b) 2/3rd of Rs. 1,20,000 (80,000)

Unit II (1/3rd area: Let out)


Particulars Reason/ Analysis Amount
Gross Annual Value Higher of ER = 1,00,000; ARR: 8,000 x 12 = Rs. 96,000 Rs. 1,00,000
Less: Municipal tax paid [1/3rd of (10% of Rs. 3 lacs)] = Rs. 30,000/3 (Rs. 10,000)
Net Annual Value GAV – MT Rs. 90,000
Less: Deduction u/s 24
24(a): Std Deduction @ 30% 30% of Rs. 90,000 (Rs. 27,000)
24(b): Interest 1/3rd of Rs. 1,20,000 (Rs. 40,000)
Income from Unit II (let-out) Rs. 23,000
▪ Total Income from House Property = (Rs. 80,000) + Rs. 23,000 = (Rs. 57,000).

Q13. X owns a building consisting of 3 identical units. Construction of the building was completed on 1.9.2018.
The building was occupied from 1.4.2018 onwards. Particulars for PY 2018-19 are given below:
Particulars Unit I Unit II Unit III
Fair Rent 60,000 60,000 60,000
Rent received - 72,000 -
Municipal taxes Paid 3,000 5,000 3,000
Municipal taxes Due but not yet paid 3,000 5,000 3,000
Land revenue due but outstanding 1,200 1,200 1,200
Ground rent due, but not yet paid 2,400 2,400 2,400

DT QUESTION BANK BY CA PRANAV CHANDAK Subscribe us on YouTube 61


Nature of occupation SOP (Residence) LOP (Residence) Business Use
On 1.4.2016, X had borrowed a sum of Rs. 7,50,000 bearing interest at 8% p.a for construction of this building. The
total cost of construction of the building was Rs. 12,00,000. Compute Income from HP" for AY 2019-20.
Solution:
Particulars I II III
SOP LOP Not
GAV Nil 72,000 taxable
Less: Municipal tax - 5,000 as used
NAV Nil 67,000 for
business
Less: Deductions u/s 24
(a) Standard deduction @ 30% - (20,100)
(b) Interest for each unit (8,000 (pre) + 20,000 (current year interest) (28,000) (28,000)
(28,000) 18,900
Income from House Property (Rs. 9,100)

Calculation of Interest Deductible for PY 2018-19:


(i) Current Year Interest:
Interest for PY 2018-19: Rs. 7,50,000 * 8% = Rs. 60,000.
Interest of Rs. 60,000 is for all 3 units. Thus Interest proportionate to Unit 1 = Rs. 60,000/3 = Rs. 20,000.

(ii) Pre-Construction Period Interest: From 1.4.2016 – 31.3.2018


Interest = Rs. 7,50,000 * 8% = Rs. 60,000 * 2 Years. = Rs. 1,20,000.
Interest of Rs. 1,20,000 is for all 3 units. Thus Interest proportionate to Unit 1 = Rs. 1,20,000/3 = Rs. 40,000.
Thus Interest Allowed as deduction in PY 2018-19 = 1/5th of Rs. 40,000 = Rs. 8,000.
Total Deduction of Interest in PY 2018-19 = Rs. 20,000 + Rs. 8,000 = Rs. 28,000.

Q14. K constructed a house property in old Hubli consisting of two units on a leasehold land. The construction was
completed on 30.6.2003 & its municipal valuation was at Rs. 80,000. He let out one of them for commercial
purposes & was himself residing in the other. The self-occupied portion was 2/5th of the house. Rent is fixed at Rs.
6,000 per month. His income from other sources is Rs. 15,000. His expenses for PY 2018-19 were as under:
(i) Municipal taxes: Rs. 6,000; (ii) Ground rent: Rs. 1,000; (iii) Insurance: Rs. 400; (iv) Collection charge: Rs. 2,000
Interest on loan for: (i) Construction of house: Rs. 30,000; (ii) Purchase of car: Rs. 20,000.
Calculate his income from house property.
Solution:
Let out unit
GAV higher of the following two:
(a) MV 3/5 of Rs. 80,000 = 48,000
(b) AR (Rs. 6,000 x 12) = 72,000 72,000
Less: Municipal taxes 3/5th 3,600
NAV 68,400
Less: Deductions u/s 24
(a) Standard deduction @ 30% 20,520
(b) Interest (60%) 18,000 38,520 29,880
Self-occupied unit
Annual value Nil
Less: Interest 12,000 (-) 12,000
Income from House Property 17,880

Q15. From the particulars given below compute income from house property:
Fair value Rs. 72,000
Date of completion 1.11.1999
Self-occupied 2/3 portion
Let-out 1/3 portion from 1.4.2018 to 31.8.2018 @ Rs. 2,500 p.m. & SOP from 1.9.2018.
Municipal taxes paid Rs. 15,000 p.a.
Insurance premium Rs.2,000 p.a.
Ground rent Rs.2,000 p.a.
Solution: Computation of House Property Income
(i) Self occupied 2/3rd portion Nil
(ii) Let out portion 1/3rd
Fair Value (72,000 x 1/3) 24,000

DT QUESTION BANK BY CA PRANAV CHANDAK Subscribe us on YouTube 62


Actual rent receivable (2500 x 5) 12,500
Annual rental value (treating as fully let-out during the year) 24,000
Less: Municipal taxes 5,000
NAV 19,000
Deductions
Standard deduction @ 30% 5,700
Income from l/3rd portion 13,300
Income from house property = Nil + 13,300 = Rs. 13,300.

Q16. R owns three houses properties. He has furnished the following particulars for the financial year 2018-19:
First House: Its Municipal valuation is Rs. 1,00,000. This is used by him for his residence. He has paid fire
insurance premium Rs. 2,000 & Municipal taxes Rs. 10,000. He has also paid interest on loan Rs. 16,000. This loan
was taken to repay another loan which he had taken for the construction of this house.
Second House: Its Municipal valuation is Rs.60,000 & it has been let-out at a rent of Rs.6,000 p.m. He has made
the following payments in respect of this house:-
Municipal taxes Rs.6,000; Repairs Rs.10,000; Land Revenue Rs.2,000; Legal expenses for getting the house vacated
Rs. 5,000. Annual charge Rs.30,000. The house remained vacant for 3 months.
Third House: The construction of this house was completed on 31.3.2005. Its Municipal valuation is Rs. 1,20,000.
It consists of two identical units, each of which has been let-out at a rent of Rs.7,500 p.m. He has made the following
payments in respect of in house: Municipal Taxes Rs. 12,000; Ground Rent Rs.6,000; Interest on loan taken for
construction Rs.40,000. The second unit remained vacant for two months. Compute his taxable income from house
property for AY 2019-20.
Solution: Income from House Property
H1 (SOP) H 2 (LOP) H 3 – Unit I (LOP) H 3 – Unit II (LOP)
GAV Nil 54,000* 90,000 75,000
Less: Municipal taxes N/A (6,000) (6,000) (6,000)
NAV Nil 48,000 84,000 69,000
Deduction u/s 24(a) @ 30% - (14,400) (25,200) (20,700)
Interest on loan u/s 24(b) (16,000) - (20,000) (20,000)
Income from HP (16,000) 33,600 38,800 28,300
Income from HP = (16,000) + 33,600 + 38,800 + 28,300 = Rs. 84,700.
* Actual rent received exclusive of vacancy 6,000 x 9 = Rs. 54,000.

Q17. Mr. X has let out one house along with generator facility & has charged Rs. 40,000 p.m. as rent, out of which
Rs. 3,000 p.m. is attributable to the generator. He has paid Rs. 2,300 & the tenant has paid Rs. 900 towards
municipal taxes. The interest on the capital borrowed for construction of the house is Rs. 7,000. Mr. X has paid
repair charge of the generator Rs. 3,400, fuel charges Rs. 5,600 & operator’s salary Rs. 300 p.m. Compute the tax
liability of Mr. X for AY 2019-20.
Solution: Computation of income under the head House Property
Gross Annual Value (37,000 x 12) 4,44,000
Less: Municipal Taxes (2,300)
Net Annual Value 4,41,700
Less: 30% of NAV u/s 24(a) (1,32,510)
Less: Interest on capital borrowed u/s 24(b) (7,000)
Income under the head House Property 3,02,190

Computation of Income u/h “Other Sources”


Income from generator (3,000 x 12) 36,000
Less: Repair charges + Fuel charges + Operator Salary (300x12) [3,400 + 5,600 + 3,600] (12,600)
Income under the head Other Sources 23,400

Computation of Total Income


Income under the head House Property 3,02,190
Income under the head Other Sources 23,400
Gross Total Income 3,25,590
Less: Deduction u/s 80C to 80U Nil
Total Income 3,25,590

DT QUESTION BANK BY CA PRANAV CHANDAK Subscribe us on YouTube 63


Q18. Two brothers Arun & Bimal are Co-Owners of a House Property with equal share. The property was
constructed during PY 1998 - 99. The property consists of eight identical units & is situated at Cochin.
During PY 2018 - 19, each Co-Owner occupied one unit for residence & the balance of six units were let out at a
rent of Rs. 12,000 per Month per unit. The Municipal Value of the House Property is Rs. 9,00,000 & the Municipal
Taxes are 20% of Municipal Value, which were paid during the year. Other expenses were as follows: (i) Repairs:
Rs. 40,000; (ii) Insurance premium (paid): Rs. 15,000; (iii) Interest payable on loan taken for construction of house:
Rs. 3 lacs.
One of the Let – Out units remained vacant for 4 months during PY. Arun could not occupy his unit for 6 months
as he was transferred to Chennai. He does not own any other house. The Other Income of Mr. Arun & Mr. Bimal are
Rs. 2,90,000 & Rs. 1,80,000 respectively for the Previous Year 2018-2019. Compute the Income under the head
Income from House Property & the Total Income of two brothers for AY 2019-20. [Nov 12]
Solution: 1. Computation of Income from Let-Out House Property (6 Units)
Gross Annual Value: Municipal Value or Actual Rent whichever is higher 8,16,000
Rs. 9,00,000 x 6/8 (OR) Rs. 12,000 x 12 months x 6 Units, 6,75,000 (or) 8,64,000
(But owing to vacancy in one of the let-out property, the Annual Value received with respect to that property is
reduced to actual rent received i.e. Rs. 12,000 x 8 = Rs. 96,000)
[Therefore, the Annual Value is ( Rs. 12,000 x 12 months x 5 units) + Rs. 96,000]
Less: Municipal Taxes Paid (20% on Municipal Value) [( Rs. 9,00,000 x 6/8) x 20%)] (1,35,000)
Net Annual Value 6,81,000
Less: Deductions u/s 24: (a) 30% of Net Annual Value (2,04,300)
(b) Interest ( Rs. 3,00,000 x 6/8) (2,25,000)
Income from House Property 2,51,700
2. Computation of Total Income of Mr. Arun
1. Income from House Property – Self Occupied, So, Annual Value u/s 22 NIL (30,000)
Less: Deduction u/s 24 – Interest [WN 1] (30,000) 1,25,850
Income from Self-occupied Property 2,90,000
Share of Income from Let-out Property ( Rs. 2,51,700 2)
2. Other Income
Total Income 3,85,850
3. Computation of Total Income of Mr. Bimal
1. Income from House Property – Self Occupied, So, Annual Value u/s 22 NIL
Less: Deduction u/s 24 – Interest [WN 1] (30,000)
Income from Self-occupied Property (30,000)
Share of Income from Let-out Property (Rs. 2,51,700 + 2) 1,25,850
2. Other Income 1,80,000
Total Income 2,75,850
Working Note:
1. Interest on Loan is Rs. 3,00,000 + 8 Units = Rs. 37,500. Since the loan is taken prior to 01.04.1999, maximum
allowable interest is only Rs. 30,000.
2. Repairs & Insurance Premium are not allowed as deduction.
3. Municipal Taxes paid is not allowed as deduction while computing income from Self-occupied property.

Q19. Mr. Raman is a Co-Owner of a House Property along with his brother. [Nov 09]
Municipal Value of the Property 1,60,000
Fair Rent 1,50,000
Standard Rent under the Rent Control Act 1,70,000
Rent Received 15,000 p.m.
The loan for the construction of this property is jointly taken & the interest charged by the bank is Rs. 25,000 out
of which 21,000 have been paid. Interest on the unpaid interest is 450. To repay this loan, Raman & his brother
have taken a fresh loan & interest charged on this loan is Rs. 5,000. Municipal Taxes of Rs. 5,100 have been paid
by the Tenant. Compute Income from this house property chargeable in the hands of Mr. Raman for AY 2019-20.
Solution: Shares of each co-owner is not given. If share of each Co-Owner of the property is not definite, then
Income from property will be determined & charged to tax in the capacity of an AOP (Sec. 26).
Computation of Income from House Property
Gross Annual Value [Note 1] 1,80,000
Less: Municipal Taxes paid [Note 2] (Nil)

DT QUESTION BANK BY CA PRANAV CHANDAK Subscribe us on YouTube 64


Net Annual Value of the House Property 1,80,000
Less: Deduction u/s 24(a): 30% of NAV = 30% of Rs. 1,80,000 (54,000)
Deduction u/s 24(b): Interest on original loan + Loan taken to repay the original loan (30,000)
Income from House Property 96,000

Note:
1. GAV = Higher of ER or ARR.
(i) ER = Higher of FR (1,50,000) & MR (1,60,000) sub to Max. of SR (1,70,000) = 1,60,000
(ii) ARR = Rs. 1,80,000 (15,000 x 12).
2. Since Municipal Taxes have been paid by the tenant, it shall not be allowed as a deduction to owner.
3. Interest on interest of Rs. 450 shall not be allowed as deduction u/s 24.
4. Interest shall be allowed on accrual basis even though it is not paid during the Relevant Previous Year.
5. It is assumed the interest of Rs. 25,000 does not include the interest on interest of Rs. 450.
6. If it is assumed that the Mr. Raman is an equal owner of the House Property, then the Taxable Income from
House Property in the hands of Mr. Raman is Rs. 48,000 (Rs. 96,000 + 2).

Q20. Mrs. Rohini Ravi, a Citizen of the USA is a Resident & Ordinarily Resident in India during the Previous Year
2018-2019. She owns a House Property at Los Angeles, U.S.A. which is used as her residence. The annual value of
the house is $ 15,000. The value of 1 USD ($) may be taken as Rs. 60. She took ownership & possession of a flat in
Chennai on 01.07.2018, which is used for Self- Occupation, while she is in India.
The flat was used by her for 7 months only during the year ended 31.03.2019. Whilst the Municipal Valuation is
Rs. 32,000 p.m, the Fair Rent is Rs. 4,20,000 p.a. She paid the following to Corporation of Chennai – Property Tax
Rs. 16,200 & Sewerage Tax Rs. 1,800.
She had taken a Loan from Standard Chartered Bank for purchasing this flat. Interest on Loan was
Period prior to 01.04.2018 49,200
01.04.2018 to 30.06.2018 50,800
01.07.2018 to 31.03.2019 1,31,200
She had a house property in Bangalore, which was sold in March, 2018. In respect of this house, she received
arrears of rent of Rs. 60,000 in March, 2019. This amount has not been charged to tax earlier. Compute the Income
from House Property of Mrs. Rohini Ravi for AY 2019-20 exercising the most beneficial option available. [May 09]
Solution: Computation of Income from House Property
Particulars Option I Option II
US (SOP) Indian DLOP US DLOP Indian SOP
Gross Annual Value Nil 3,15,000 9,00,000 Nil
1. Fair Rent or Municipal Value, whichever is higher.
India: 32,000x9 = 2,88,000 or (4,20,000x9/12)= 3,15,000
2. US: $15,000 x Rs. 60 (given)
Less: Municipal Taxes paid (16,200 + 1,800) NA (18,000) Nil NA
Net Annual Value Nil 2,97,000 9,00,000 Nil
Less: Deductions u/s 24
(a) 30% of Net Annual Value - (89,100) (2,70,000) -
(b) Interest on borrowings (Refer Note 2) - (1,91,840) - (1,91,840)
Income from House Property (A) Nil 16,060 6,30,000 (1,91,840)
(before considering Arrears of Rent) 16,060 4,38,160
Arrears of Rent received [70% of amount received) 42,000 42,000
Net Income from House Property (A+B) 58,060 4,80,160
Conclusion: Option I is chosen, due to lower chargeable Income. Hence, Income from House Property is Rs. 58,060.
Notes:
1. Since Mrs. Rohini Ravi is a resident in India, her global income is taxable in India.
2. Deductions for Interest on Capital borrowed:
▪ Prior Period Interest = 49,200/5 = Rs. 9,840 (assuming conditions u/s 24 are satisfied)
▪ Interest for the Indian Property = 9,840 + 50,800 + 1,31,200 = 1,91,840
▪ If the Indian Property is considered as Self Occupied, deduction for interest is restricted to Rs. 2,00,000
▪ If the Indian Property is considered as Deemed Let Out, then there is no ceiling limit for interest deduction.
3. Since the ownership of the Indian property is from 01.07.2018, Income shall be computed only for 9 months.

DT QUESTION BANK BY CA PRANAV CHANDAK Subscribe us on YouTube 65


Q21. R owns three house properties, which are situated in three different cities. All the house properties are meant
for self-occupation of the assessee. The particulars of the house properties are as under.
Particulars Property A Property B Property C
Municipal valuation 7,00,000 7,20,000 7,40,000
Fair Rent 7,40,000 7,50,000 7,60,000
Standard rent 7,30,000 7,60,000 7,50,000
Municipal taxes paid 30,000 20,000 80,000
Interest on money borrowed (on purchase/construction) 2,35,000 1,80,000 85,000
Ground rent due 5,000 - 8,000
Land revenue due - 6,000 -
Compute income u/h House Property by making assumption in such a manner that tax liability of R is minimum.
Solution: We shall first assume all the three houses deemed to be let out.
Particulars House A House B House C
GAV 7,30,000 7,50,000 7,50,000
Less: Municipal taxes paid 30,000 20,000 80,000
NAV 7,00,000 7,30,000 6,70,000
Less: Deduction u/s 24
Standard deduction @ 30% 30,000 39,000 21,000
Interest 2,35,000 1,80,000 85,000
Income from House Property 4,35,000 5,11,000 5,64,000
Option I: Assume House A as self-occupied:
Annual value of House A Nil
Less: Deduction u/s 24 Interest on money borrowed Rs. 2,35,000 limited to Rs. 2,00,000 (2,00,000)
Income from house property A 2,00,000
Income from House Property A + B + C = (2,00,000) + 5,11,000 + 5,64,000 = 8,75,000
Option II: Assume house B as self occupied:
Annual value of House B Nil
Less: Deduction u/s 24(b): Interest on money borrowed for construction of house (1,80,000)
Income from House B 1,80,000
Income from House Property A + B + C = (4,35,000 - 1,80,000 + 5,64,000) = 8,19,000

Option III: Assume house C as self occupied:


Annual value of house C Nil
Less: Deduction u/s 24(b) Interest on money borrowed (85,000)
Income from HP (85,000)
Income from House Property A + B + C = (4,35,000 + 5,11,000 - 85,000) = 8,61,000
Conclusion: As Income from house property is minimum under option II. Hence House B should be opted as self-
occupied & other two should be deemed let out property.

Q22. Mr. Raj has let out his house. The particulars are as follows: Annual Rent: Rs. 60,000; MV: Rs. 48,000;
Municipal taxes (paid by tenant): Rs. 4,000; Expenses incurred are: Repairs met by tenant: Rs. 3,000; Fire
Insurance premium paid: Rs. 1,500; Electricity & water charges paid by Shri Goel: Rs. 5,400; Lift maintenance
charges paid by Shri Goel: Rs. 2,400; Collection charges (including Rs. 300 of litigation expenses): Rs. 750. During
PY 2010-11, he had claimed a deduction of unrealised rent of Rs. 22,500 out of which Rs. 16,500 was allowed as
deduction for that year. On 10.8.2018, he recovered Rs. 10,500 from defaulting tenant. Rs. 60,000 is a composite
rent of property, as well as amenities provided to the tenant. Compute "income from HP" for AY 2019-20.
Solution:
Composite Rent of Property + Amenities Rs. 60,000
Less: Amenities Charges (Electricity & water charges + lift charges) (Rs. 7,800)
GAV of the House property Rs. 52,200
Less: Municipal taxes (not allowed as paid by tenant) Nil
NAV 52,200
Less: Deductions u/s 24: Standard deduction @ 30% (Rs. 15,660)
Rs. 36,540
Add: Unrealized Rent recovered in the PY @ 70% Rs. 3,150
Income from House Property Rs. 39,690

DT QUESTION BANK BY CA PRANAV CHANDAK Subscribe us on YouTube 66


Q23. Miss Charlie, an American national, got married to Mr. Radhey of India in USA on 2.3.2018 & came to India
for the first time on 16.03.2018. She left for USA on 23.09.2018. She returned to India again on 27.03.2019. While
in India, she had purchased a show room in Mumbai on 22.04.2018, which was leased out to a company on a rent
of Rs. 25,000 p.m. from 01.05.2018. She had taken loan from a bank for purchase of this show room on which bank
had charged interest of Rs. 97,500 upto 31.03.2019.
She had received the following gifts from her relatives & friends during 01.04.2018 to 30.06.2018:
- From parents of husband: Rs. 51,000.
- From married sister of husband: Rs. 11,000.
- From two very close friends of her husband: Rs. 1,51,000 & Rs. 21,000 = Rs. 1,72,000.
Determine her residential status & compute total taxable income for AY 2019-20. [MAY - 2007]
Solution:
▪ Her stay in India during the previous year 2018-19 & in the preceding four years is as under:-
▪ PY 2018-19: 176 days (1.4.2018 to 23.9.2018) + 5 days (27.3.2019 to 31.03.2019) = 181 days
▪ Since Miss Charlie is not able to comply with any of the basic condition, she is a non-resident in PY 2018-19.
Stay in India in 4 preceding PYs
PY 2017- 2018 [16.3.2018 to 31.3.2018] = 16 days; PY 2016- 2017 [1.4.2016 to 31.3.2017] = Nil
PY 2015- 2016 [1.4.2015 to 31.3.2016] = Nil; PY 2014- 2015 [1.4.2014 to 31.3.2015] = Nil
Total 16 days
Computation of total income of Miss. Charlie for AY 2019-20
(I) Income from house property
Gross Annual Value [25,000 x 11] 2,75,000
Less: Municipal taxes Nil
Net Annual Value 2,75,000
Less: Standard deduction 30% of NAV u/s 24(a) (82,500)
Less: Interest on loan 24(b) (97,500)
Income under the Head House Property 95,000
(II) Income from other sources
- Rs. 51,000 received from parents of husband – Exempt: Nil
- Rs. 11,000 received from married sister of husband – Exempt: Nil
- From 2 friends of husband Rs. 1,51,000 & Rs. 21,000: Rs. 1,72,000 1,72,000
Total Income 2,67,000

Note:
1. Actual rent received has been taken as the gross annual value in the absence of other information (i.e. Municipal
value, fair rental value & standard rent) in the question.
2. Rebate u/s 87A is not allowed to non-resident.

Q24. R has the following properties:


(a) Flat in Mumbai purchased on 1.6.2014 which is let out on a monthly rent of Rs. 4,000. The building in which
the flat is located was completed on 1.5.2014.
(b) Flat in Delhi (construction completed on 10.5.2015) which is self-occupied.
(c) Godown in Kolkatta constructed in 1999 which is let out on a monthly rent of Rs. 8,000. The expenses actually
incurred during the year against rental income are:
Particulars Mumbai Delhi Kolkatta
Municipal taxes actually paid during PY ending 31.3.2019 7,000 2,400 11,000
Building co-operative maintenance charges 2,000 900 -
Electricity charges - 4,200 6,800
Fire insurance premium - - 2,600
Collection charges 750 - 1,400
Repairs 320 1,900 14,000
The following further information is given:
(i) The flat in Delhi, if let out, would fetch a monthly rent of Rs.7,000; however, standard rent of the house according
to the Delhi Rent Control Act is Rs. 6,000 per month.
(ii) R carries on business in which he suffered a loss of Rs. 600 during year ended on 31.3.2019.
(iii) R received a consolidated salary of Rs. 4,000 per month during the year from a part-time employment.
(iv) R took a loan of Rs. 3,60,000 on 1.4.2011 from a bank at 12% interest p.a, to construct the house in Delhi.
However, on 5.5.2015 it is repaid along with interest. Compute R's total taxable income for AY 2019-20.

DT QUESTION BANK BY CA PRANAV CHANDAK Subscribe us on YouTube 67


Solution:
Income from salary
Gross salary 48,000
Income from House Property at Mumbai
GAV (4,000 x 12) 48,000
Less: Municipal Taxes 7,000
NAV 41,000
Less: Deductions u/s 24
Standard deduction @ 30% 12,300
Income from flat at Mumbai 28,700
Income from let out godown at Kolkata:
GAV ( Rs. 8000 x 12) 96,000
Less: Municipal taxes 11,000
NAV 85,000
Less: Deductions u/s 24
Standard deduction @ 30% 25,500
Income from godown at Kolkatta 59,500
Income from self-occupied property in Delhi
Annual value Nil
Less: Deduction u/s 24(b) = 1/5th of 1,72,800 = 34,560 [Restricted to maximum Rs. 30,000] 30,000
Pre-construction period interest 3,60,000 x 12% x 4 = 1,72,800
Income from self-occupied property (-) 30,000

Computation of taxable income of R


Salary 48,000
Income from house property
Flat at Mumbai 28,700
Flat at Delhi (-) 30,000
Godown at Kolkatta 59,500 58,200
Business loss (-) 600
Total taxable income 1,05,600
Note: Since the construction of flat in Delhi was completed after 3 years from the end of the financial year in which
money was borrowed, deduction of interest shall be limited to Rs.30,000.

Q25. A is employed in B Ltd. on a salary of Rs. 10,000 p.m. He owns a house at Delhi whose construction was
completed on 31.12.2016. He has let out this house from 1.4.2018 @ Rs. 6,300 p.m. The house is vacated by the
tenant on 1.9.2018. From 1.11.2018, he has let out the house to his employer B Ltd. @ Rs.7,000 p.m. The company
has allotted the house to A. Other particulars in respect of the house for the financial year 2018-19 are given below.
MV Rs. 60,000 p.a.; Municipal tax paid Rs.6,000; He borrowed Rs. 1,25,000 from HUDCO on 1.4.2012 @ 12% p.a.
interest for the construction of the house. He refunded entire loan in lump sum on 31.12.2018. Compute the total
income of A for the AY 2019-20.
Solution:
Salary: 10,000 x 12 1,20,000
Value of rent-free house: 15% of salary for 5 months (10,000 x 5) 7,500
Gross Salary 1,27,500
Less: Deduction u/s 16 (40,000)
Income under the head salary 87,500
Income from house property:
Gross rental value exclusive of vacancy period (6,300 x 5 + 7,000 x 5) 66,500
Less: Municipal taxes paid (6,000)
Annual value 60,500
Deduction u/s 24:
(i) Standard deduction @ 30% (18,150)
(ii) Interest for pre-construction period (1/5th of Rs. 45,000) (9,000)
(iii) Interest for current year for 9 months (11,250) (20,250) 22,100
Total income 1,49,600

DT QUESTION BANK BY CA PRANAV CHANDAK Subscribe us on YouTube 68


*Q26. Mrs. Deepali (aged 40 years) is working with M/s Good Company Ltd, a manufacturer of tyres based at
Mumbai, has received the following payments during the Previous Year 2018-2019 from her employer:
(a) Basic Salary – Rs. 60,000 per month; (b) Dearness Allowance – 40% of Basic Salary.
Her employer has taken on rent her own house on a monthly rent of Rs. 15,000 & the same has been provided for
residence of Mrs. Deepali. Company is recovering Rs. 2,000 per month as rent of house.
Mrs. Deepali has further furnished the following details:
(i) Contribution to PPF Rs. 60,000.
(ii) She has paid Professional Tax of Rs. 6,000 during Financial Year 2018-2019.
(iii) She is owning only one house & payment of Interest of Rs. 1,75,000 & Principal of X 1,00,000 was made for
Housing Loan taken for purchase of House.
(iv) She has also taken a Loan of Rs. 2,00,000 from her employer for study of her son. SBI Rate for such Loan is
10%. Her employer has recovered Rs. 10,000 as Interest from her Salary for such Loan during the year.
Compute Taxable Income & Tax Liability for Assessment Year 2019-2020. [Nov 11]
Solution
Computation of Total Income & Tax Liability
Particulars Rs. Rs.
Income under the Head Salary - Basic (60,000 x 12) 7,20,000
DA (40% of 7,20,000) 2,88,000
Value of Accommodation taken on Lease by the Employer Least of: 1,27,200
(i) Rent paid by employer (15,000 x 12) 1,80,000
(ii) 15% of Salary (15% of 10,08,000) 1,51,200
Less: Amount Recovered from Employee (2,000 x 12) (24,000)
Taxable value of Concessional Loan (Loan o/s @ SBI Rate – Actual Rate is Taxable) = 10,000
(2,00,000 x 10%)- 10,000
Gross Salary 11,45,200
Less: Deductions u/s 16(ia) Standard Deduction (40,000)
Less: Deduction u/s 16(iii) – Professional Tax (6,000) 10,99,200
Income u/h House Property: Let out Property
Gross Annual Value (15,000 x 12) 1,80,000
Less : Deduction u/s 24 @ 30% of NAV (54,000)
Interest on Housing Loan (1,75,000) (49,000)
Gross Total Income 10,50,200
Less: Deduction under Chapter VI A
U/s 80C - Housing Loan Principal 1,00,000 + NSC 60,000 (Max. 1,50,000) (1,50,000)
Total Taxable Income 9,00,200

Tax on Total Income = [12,500 + (9,00,200 – 5,00,000) x 20%] + HEC @ 4% = 92,540 + 3,702 = Rs. 96,240.

DT QUESTION BANK BY CA PRANAV CHANDAK Subscribe us on YouTube 69


4C. PROFITS & GAINS OF BUSINESS & PROFESSION
Q1. Mr. Pandey, a resident individual, aged 45 years, is a Chartered Accountant in practice. He maintains his
accounts on cash basis. His profit & Loss Account for the year ended 31st march, 2019 is as follows: [NOV-2017]
Profit & Loss Account for the year ending March 31, 20 19
Expenditure Amount Income Amount
Staff salary 18,25,000 Fees earned
Rent of the office 6,00,000 (i) Audit 43,00,000
Premises (ii) Taxation 34,50,000
Administrative expenses 5,75,000 (iii) Consultancy services relating 10,00,000
Stipend to Articled clerks 1,85,000 to syndication of loan
Meeting, seminars & conferences 36,500 Gifts 1,00,000
Depreciation 55,000 Dividends from Indian Company 12,00,000
Printing & Stationery 8,75,000 Interest on Gold deposit 15,000
certificates
Net Profit 59,13,500
1,00,65,000 1,00,65,000
Other Information:
(1) Depreciation allowable under Income-Tax Act Rs. 1,25,000.
(2) Administrative expenses include Rs. 55,000 paid to a tax consultant in cash for assisting Mr. Pandey.
(3) Gifts represent fair market value of a LED TV which was given by one of the clients for successful presentation
of case in the Income Tax Appellate Tribunal.
(4) Last month’s rent of Rs. 50,000 was paid without deduction of tax at source.
(5) Mr. Pandey had taken a loan of Rs. 32,00,000 for the purchase of a house property valuing Rs. 45,00,000 from
a recognized financial institution on 1st May, 2018. He repaid Rs. 1,50,000 on 31 st March, 2019 out of which Rs.
1,00,000 is towards principal payment & the balance is for interest on loan . The possession of the property will
be handed over to him in October 2019.
(6) Mr. Pandey paid medical insurance premium of his parents (senior citizens & not dependent on him) by cheque
of Rs. 27,000. He also paid Rs. 8,500 by cash towards preventive health checkup for himself & his spouse.
Compute the total income of Mr. Pandey & tax payable by him for Assessment Year 2019-20.

Solution:
Income under the head Business/Profession
Net Profit as per profit & loss account 59,13,500
Add: Expenses disallowable
Depreciation as per books of accounts 55,000
Administrative expense paid in cash in excess of Rs. 10,000 section 40A (3) 55,000
Rent Paid without deduction of TDS Section 40(a)(ia) (Disallowed 30%) 15,000
Less: Expenses allowable/ Income considered in other heads etc.
Dividends (12,00,000)
Interest on deposit certificate issued under Gold Monetization scheme (15,000)
Depreciation as per Income Tax Act (1,25,000)
Income under the head Business/Profession 46,98,500
IFOS: Dividend Income (12,00,000 -10,00,000) as per section 115BBDA 2,00,000
Interest on deposit certificate issued under Gold Monetization scheme Nil
Income under the head other sources 2,00,000
Gross Total Income 48,98,500
Less: Deduction u/s 80D: Medical Insurance of Parents (27,000)
PHC of himself limited to Rs. 5,000 (5,000)
Total Income 48,66,500

Note:
1. Deduction u/s 80C is not allowed for repayment of loan as possession of property will be given in next year.
2. Deduction of payment made in cash for PHC is allowed u/s 80D.
3. Gift received from Client shall be considered as Professional Income as per section 28.
4. Interest income from certificate issued under Gold Monetization scheme is exempt u/s 10(15).
5. Dividend from Indian Company is exempt u/s 10(34) but to the extent of Rs. 10,00,000.

DT QUESTION BANK BY CA PRANAV CHANDAK Subscribe us on YouTube 70


Q2. Ms. Rekha a resident individual aged 50, provides the following information for the financial year 2018-19:
(i) She is a partner in AK & CO. & received the following amounts from the firm:
Share of profit from the firm Rs. 35,000
Interest on capital @ 15% p.a. Rs. 3,00,000
Salary as working partner Rs. 1,00,000 (fully allowed in the hands of the firm)
(ii) She is running a rice mill as proprietor. The Net profit as per Profit & Loss Account is 4,50,000.
The following items are debited to Profit & Loss account:
• Advance Income-tax paid: Rs. 1,00,000
• Personal drawings: Rs. 5 0,000
The following items are credited to Profit & Loss Account:
• Interest on savings bank account with SBI: Rs. 12,000
• Interest on savings account with post office: Rs. 5,000
• Dividend from listed Indian Company (DDT): Rs. 80,000
(iii) She owned a house property in Mumbai which was sold in January, 2017. She received Rs. 90,000 by way of
arrear rent in respect of the said property in October, 2018.
(iv) She made the following investments:
Life insurance premium on a policy in the name of her daughter Rs. 60,000. (The policy was taken on 1- 10-
2014 sum assured being Rs. 5,00,000).
Health insurance premium on a policy covering her mother aged 75. She is not dependant on Ms. Rekha.
Premium paid by cheque: Rs. 35,000.
Compute the Total Income of Ms. Rekha for the Assessment year 2019-20. [MAY-2017]
Solution: Computation of Total Income of Ms. Rekha for AY 2019-20
(i) Income under the head Business Profession
Share of Profit from Partnership Firm (Exempt u/s 10(2A)) Nil
Interest on capital (3,00,000 x 12%/15%) 2,40,000
Salary from Partnership firm 1,00,000
Net Profit as per Profit & loss Account 4,50,000
Add: Income Tax Paid 1,00,000
Add: Personal Drawings 50,000
Less: Interest on saving bank account with SBI (12,000)
Less: Interest on saving bank account with Post Office (5,000)
Less: Dividend from Indian Company (80,000)
Income from Rice Mill 5,03,000
Income under the head Business Profession 8,43,000
(ii) Income u/h “other Sources”
Interest on saving bank account with SBI 12,000
Interest on saving bank account with Post Office 5,000
Less: Exempt u/s 10(15) (3,500)
Interest on saving bank account with Post Office 1,500
Dividend from Indian Company (Exempt u/s 10(34)) Nil
Income under the head other Sources 13,500
(iii)Income u/h House Property: Arrear of Rent (Sec 25A) - 70 % of Rent received is taxable 63,000
Gross Total Income 9,19,500
Less: Deduction u/s 80C: For life Insurance of married daughter (10% of 5,00,000) (50,000)
Less: Deduction u/s 80D: For health Insurance of mother (30,000)
Less: Deduction u/s 80TTA: For Interest on saving account (10,000)
Total Income 8,29,500

Note:
1. As per section 40(b) Interest is allowed to the partner’s maximum at a rate of 12% p.a.
2. Interest from post office savings bank account is exempt upto Rs. 3500.
3. Arrears of rent shall be taxable under the head house property even if the assessee donot have any house
property in his name.
4. If life policy is taken w.e.f. 1st April 2012 onwards, deduction is allowed maximum @ 10% of sum assured.
5. As per section 80D, maximum deduction allowed for senior citizen shall be Rs. 30,000.

DT QUESTION BANK BY CA PRANAV CHANDAK Subscribe us on YouTube 71


Q3. Net profit as per the profit & loss account of Mr. PC is Rs. 7,70,000 for the year ending 31 st March, 2019.
The following information is noted from the accounts:
(a) Advertisement expenditure debited to profit & loss account includes the following:
(i) Expenditure incurred outside India: Rs. 56,000 (Tax has been deducted at source & paid during the year)
(ii) Articles presented by way of advertisement (60 articles cost of each being Rs. 700, & 36 articles cost of
each being Rs. 1,500);
(iii) Rs. 20,000 being the cost of advertisement which appeared in a newspaper owned by a political party;
(iv) Rs. 14,400 being capital expenditure on advertisement; (eligible for dep. @ 25%)
(v) Rs. 9,000 paid in cash
(vi) Rs. 9,000 paid to a concern in which X has substantial interest (Excessive amount is Rs. 1,800)
(b) Out of salary to the employees debited to the profit & loss account:
(i) Rs. 60,000 is employee’s contribution to the recognized provident fund, Rs. 47,500 of which is credited in
the employee’s account in the relevant fund before the due date for provident fund;
(ii) Rs. 58,000 is bonus which is paid on 13th November, 2019;
(iii) Rs. 44,000 is commission which is paid on 1st December, 2019;
(iv) Rs. 25,000 is incentive to workers, which is paid on 10th December, 2019.
(v) Rs. 46,000 is paid outside India in respect of which tax is not deducted at source;
(vi) Rs. 6,000 being capital expenditure for promoting family planning amongst employees; &
(vii) Rs. 55,000 being entertainment allowance given to employees.
(c) Entertainment expenses debited to profit & loss account is Rs. 12,000.
Determine the Total Income & Tax Liability of Mr. X for the AY 2019-20.
Solution: Computation of Total Income
Net profit as per profit & loss account 7,70,000
Add: Inadmissible items
Advertisement in a newspaper owned by a political party (Sec 37(2B)) 20,000
Capital expenditure on advertisement 14,400
Excess amount paid to a concern in which ‘X’ has substantial interest 1,800
Employee contribution to recognised provident fund
(to the extent not deposited before the due date) 12,500
Bonus being paid to employee after the due date of filing the return 58,000
Commission to employee after the due date of filing the return 44,000
Salary paid outside India in respect of which tax is not deducted at source 46,000
Capital expenditure for promoting family planning
amongst employees (allowed only to a company assessee) 6,000
Less: Depreciation on capital expenditure on advertisement @ 25% of Rs. 14,400 (3,600)
Income under the head Business/Profession 9,69,100

Q4. ABC Ltd., engaged in manufacture of medicines furnishes the following information for the PY 2018-19:
(i) Municipal tax relating to office building Rs. 51,000 not paid till 30.09.2019.
(ii) Patent acquired for Rs. 20,00,000 on 01.09.2018 & used from the same month.
(iii) Capital expenditure on scientific research Rs. 10,00,000 which includes cost of land Rs. 2,00,000.
(iv) Amount due from customer X outstanding for more than 3 years written off as bad debt - Rs. 5,00,000.
(v) Income tax paid Rs. 90,000 by the company in respect of non-monetary perquisites provided to its employees.
(vi) Employee’s contribution towards PF - Rs. 5,50,000 of salary of July 2018 was deposited on 27th Aug 2018.
(vii) Expenditure towards advertisement in souvenir of a political party Rs. 1,50,000.
(viii) Refund of GST Rs. 75,000 received during the year, which was claimed as expenditure in an earlier year.
State with reasons the taxability or deductibility of the items given above. [NOV-2011]
Solution:
(i) Municipal taxes relating to office building not paid till the last date of filing of the return is not allowed as
deduction as per sec 43 B.
(ii) Patent acquired for 20,00,000. Depreciation shall be allowed @ 25% of 20,00,000 = Rs. 5,00,000.00
(iii) Capital expenditure on scientific research shall be allowed Rs. 8,00,000 u/s 35. Further deduction allowed
shall be 150%. No deduction shall be allowed for land.
(iv) Amount written off as bad debts shall be allowed as deduction as per section 36(1).
(v) Income Tax paid Rs. 90,000 shall not be allowed as deduction u/s 40(a).
(vi) Provident fund paid on 27th August 2018 is not allowed because employer should pay such contribution within
15 days of subsequent month. In this case, amount is remitted on 27.8.2018, hence expenditure is not allowed.
(vii) Expenditure towards advertisement in souvenir of political party is not allowed as deduction u/s 37(2B).
(viii) As per sec 41(1), refund of GST shall be treated as income since it was claimed as expenditure in earlier year.

DT QUESTION BANK BY CA PRANAV CHANDAK Subscribe us on YouTube 72


Q5. State with reasons allowability of following expenses while computing income from “PGBP” for AY 2019-20:
(i) Provision made on the basis of actuarial valuation for payment of gratuity Rs. 5,00,000. However, no payment
on account of gratuity was made before due date of filing return.
(ii) Purchase of oil seeds of Rs. 50,000 in cash from a farmer on a banking day.
(iii) Payment of Rs. 50,000 by using credit card for fire insurance.
(iv) Salary payment of Rs. 2,00,000 by a company outside India without deduction of tax.
(v) GST deposited in cash Rs. 5 0,000 with State Bank of India.
(vi) Payment made in cash Rs. 30,000 to a transporter in a day for carriage of goods.
Answer:
(i) Allowed, provision made on the basis of actuarial valuation is allowed as business expense.
(ii) Allowed, cash payment of Rs. 50,000 for purchase of oil seeds is allowed as business expense.
(iii) Allowed, payment through credit card is allowed.
(iv) If tax is neither deducted nor paid, it is not allowed. In the given question it is not mentioned that tax is paid
hence it is presumed that tax is not paid & in that case, it is disallowed.
(v) Allowed, payment of GST in cash shall be allowed & deductible as per Rule 6DD.
(vi) Allowed, as per section 40A(3), payment in cash to a transporter upto Rs. 35,000 is allowed.

Q6. State under which heads the following incomes are taxable:
(i) Rental income in case of dealer in property
(ii) Dividend on shares in case of a dealer in shares
(iii) Salary by a partner from his partnership firm
(iv) Rental income of machinery
(v) Winnings from lotteries by a person having the same as business activity
(vi) Salaries payable to a Member of Parliament
(vii) Receipts without consideration
(viii) In case of retirement, interest on employee’s contribution if provident fund is unrecognized.
Answer:
Particulars Head of Income
(i) Rental income in case of dealer in property Income from house property
(ii) Dividend on shares in case of a dealer in shares Income from other sources
(iii) Salary by partner from his partnership firm PGBP
(iv) Rental income of machinery (See Note below) IFOS/PGBP
(v) Winnings from lotteries by a person having the same as business activity Income from other sources
(vi) Salaries payable to a Member of Parliament Income from other sources
(vii) Receipts without consideration Income from other sources
(viii) Interest on employee’s contribution from URPF. Income from other sources
Note: As per section 56, rental income of machinery would be chargeable to tax u/h “Income from Other Sources”,
if the same is not chargeable to income-tax under the head “Profits & gains of business or profession”.

Q7. Following is the profit & loss account of Mr. X for PY 2018-19:
Particulars Rs. Particulars Rs.
To Repair on Building 1,81,000 By Gross Profit 6,01,000
To Amount paid to IIT Mumbai for 1,00,000 By Income Tax Refund 8,100
approved Scientific research u/s 35(2AA)
To Interest 1,10,000 By Interest on Company Deposits 6,400
To Travelling 1,30,550
To Net Profit 93,950
Additional information:
(1) Repairs on building includes Rs. 1,00,000 being cost of laying a toilet roof.
(2) Interest payment include Rs. 50,000 on which TDS has not been deducted & penalty for contravention of GST
Act of Rs. 24,000.
Mr. X is required to gets his accounts audited in the preceding year 2017-18.
Compute the income chargeable u/h “PGBP” of Mr. X for PY 2018-19 ignoring depreciation.
Solution: Computation of Income under the head Business/ Profession for the AY 2019-20
Net profit as per P & L A/c 93,950
Add: Expenses Disallowed
(i) Repair of building(capital expenditure) 1,00,000
(ii) Interest paid without TDS (50,000 x 30%) 15,000
(iii) GST Penalty 24,000 1,39,000

DT QUESTION BANK BY CA PRANAV CHANDAK Subscribe us on YouTube 73


Less: Incomes not taxable u/h PGBP or Exempt Incomes
(i) IT Refund ( not an income) 8,100
(ii) Interest on deposit (taxable u/h other sources) 6,400 (14,500)
Less: Deduction u/s 35(2AA) (amount paid to IIT) 150 % - 100 % = 50 % (50,000)
Income under the head Business/Profession 1,68,450

Q8. ABC Ltd., a manufacturing company, which maintains accounts under mercantile system has disclosed a net
profit of Rs. 12.50 Lacs for the year ending 31st March, 2019. You are required to compute the total Income & Tax
Liability of the company for the AY 2019-20, after considering the following information, duly explaining the
reasons for each item of adjustment:
(i) Advertisement expenditure includes the sum of Rs. 60,000 paid in cash to the sister concern of a director, the
market value of which is Rs. 52,000.
(ii) Repairs of plant & machinery includes Rs. 1.80 Lacs towards replacement of worn out parts of machineries.
(iii) A sum of Rs. 6,000 on account of liability foregone by a creditor has been taken to general reserve. The same
was charged to the revenue account in the AY 2016-17.
(iv) Sale proceeds of import entitlements amounting to Rs. 1 lacs has been credited to P&L A/c, which the company
claims as capital receipt not chargeable to income tax.
(v) Company has donated Rs. 2,00,000 to National Urban Poverty Eradication Fund which is debited to P&L A/c.
(vi) Being also engaged in biotechnology business, company incurred the following expenditure on in-house
research & development as approved by the prescribed authority:
(a) Research equipment purchased Rs. 1,50,000; (b) Remuneration paid to scientists Rs. 50,000.
The total amount of Rs. 2,00,000 is debited to the profit & loss account.
Solution: Computation of Total Income of ABC Ltd.
Net profit as per profit & loss account 12,50,000
Add:
(i) Payment of advertisement expenditure 60,000
(a) Rs. 8,000, being the excess payment to a related disallowed u/s 40A(2) 8,000
(b) As the payment is made in cash & since the remaining amount of Rs. 52,000 exceeds Rs. 60,000
10,000, shall be disallowed u/s 40A(3)
(ii) U/s 31, expenditure relatable to repairs of plant, machinery or furniture is allowed.
(iii) Liability foregone by creditor [Taxable u/s 41(1)] 6,000
(iv) Sale proceeds of import entitlement licence. The sale of the rights gives rise to profits or gains
taxable u/s 28. As the amount has already been credited to profit & loss a/c, no further
adjustment is necessary.
(v) Donation to National Urban Poverty Eradication Fund is allowed u/s 35CCA
Less:
(vi) Expenditure on in house research & development is entitled 150% deduction of both capital
& revenue expenditure u/s 35 = 2,00,000 x 1.5 = 3,00,000. Expenditure Rs. 2,00,000 already
debited to profit & loss account, additional deduction of Rs. 1 Lac is further allowed. (1,00,000)
Income under the head business/profession 12,16,000

*Q9. Mr. PC submits the profit & loss account for the year ending 31.03.2019 as under:
(Debits) Rs. (Credits) Rs.
Household expense 20,000 Gross Profit 5,28,500
Interest on loan taken from Mrs. X 2,000 Income tax refund 3,000
Income tax 12,000 Interest on income tax refund 300
Interest on loan for payment of income tax 1,200 GST refund 1,000
Contribution to Unrecognised Provident Fund 4,000 Interest on GST refund 400
Expenditure on advertisement (revenue) 25,000 Bad debts recovered 5,000
Public provident fund contribution 7,000 Dividends from foreign company 3,000
Investment in post- office saving bank account 12,000
Purchase of car (paid by A/c payee cheque) 2,45,000
Purchase of computer (paid by A/c payee cheque) 35,000
Purchase of plant (paid by A/c payee cheque) 23,000
Net Profit 1,55,000
5,41,200 5,41,200
Addition Information: Car, computer & P&M were purchased on 1.10.2018 & were put to use on the same date.
Compute Total Income of Mr. PC for the AY 2019-20.

DT QUESTION BANK BY CA PRANAV CHANDAK Subscribe us on YouTube 74


Solution: Computation of Total Income
Net Profit as per profit & loss account 1,55,000
Add: Inadmissible Expenses
• Household expenses 20,000
• Income tax 12,000
• Interest on loan for payment of income tax 1,200
• Contribution to Unrecognised provident fund 4,000
• Contribution to public provident fund 7,000
• Investment in post office saving bank account 12,000
• Purchase of car 2,45,000
• Purchase of computer 35,000
• Purchase of plant 23,000
Less:
• Income tax refund (3,000)
• Interest on Income tax Refund (300)
• Dividends (3,000)
• Depreciation on car (2,45,000 x 15%) (36,750)
• Depreciation on computer (35,000 x 40%) (14,000)
• Depreciation (23,000 x 15%) on plant (3,450)
Income under the head Business/Profession 4,53,700
Income under the head Other Sources
Interest on income tax Refund 300
Dividends from foreign company 3,000
Income under the head Other Sources 3,300
Gross Total Income 4,57,000
Less: Deductions u/s 80C (7,000)
Total Income 4,50,000

Q10. Mr. X, aged 75 years, has submitted his profit & loss account for the year ending 31.03.2019 as given below:
Particulars Amount Particulars Amount
Opening Stock 13,50,000 Sales 105,00,000
Purchases 75,00,000 Gift from friend 1,200
Franchises 1,00,000 Bad debts recovered 2,900
Advertisement 9,000 Rental income from House Property 1,40,000
Income Tax of the PY 2017-18 7,000 Income tax refund 700
Advance Tax 1,200 Dividends from a foreign company 3,000
Addition to the office building 45,000 Closing stock 1,80,000
Investment in public provident fund 70,000
Net Profit 17,45,600
108,27,800 108,27,800
Additional information:
1. Opening & closing stocks are undervalued by 10%.
2. Franchises were purchased on 01.07.2018 & were put to use on 03.10.2018.
3. Advertisement expenditure relates to a neon sign board which was purchased & put to use on 1.8.2018.
4. Office building has WDV of Rs. 56 lacs on 1.4.2018 & addition was made to the building by constructing
additional room on the roof. Construction was completed on 1.11.2018 & it was put to use on the same date.
Expenditure of Rs. 45,000 includes cost of wiring & switches of Rs. 4,500. No depreciation has been debited with
regard to the building.
5. Sale includes sale of Rs. 1,20,000 to the proprietor & cost of these goods was Rs. 1,00,000 & FMV is Rs. 1,25,000.
6. Bad debts recovered were allowed earlier.
Mr. X has not opted for presumptive taxation of Income u/s 44AD. Compute his Taxable Income for AY 2019-20.
Solution:
Net Profit as per profit & loss account 17,45,600.00
Add: Inadmissible Expenses already debited to P & L A/c
1. Franchises, being capital expenditure 1,00,000.00
2. Advertisement, being capital expenditure 9,000.00
3. Income tax (income tax is not allowed as per sec 40(a)) 8,200.00
4. Addition to office building, being capital expenditure 45,000.00
5. Investment in public provident fund (not a revenue expenditure) 70,000.00

DT QUESTION BANK BY CA PRANAV CHANDAK Subscribe us on YouTube 75


Add: Incomes not credited to profit & loss account
Closing stock undervalued by 10% (1,80,000 x 10/90) 20,000.00
Deduct expenditures not debited to profit & loss account (1,50,000.00)
Opening stock undervalued by 10% (13,50,000 x 10/90) Depreciation (5,88,150.00)
Working Note:
1. Franchises 1,00,000 x 25% 25,000
2. Furniture/fixture @ 10%
- Neon sign board: 9,000 x 10% 900
- Wirings etc. in the building: 4,500 x 5% 225
3. Office building: 56,00,000 x 10% + Addition: 40,500 x 5% 5,62,025
Total Depreciation 5,88,150
Less: Incomes credited to P&L A/c but not taxable u/h PGBP/ are Exempt
1. Gift from friend (only gift received from client is taxable u/h PGBP) (1,200.00)
2. Rental income from House Property (1,40,000.00)
3. Income tax refund (700.00)
4. Dividends from a foreign company (to be taxed under the head other sources) (3,000.00)
5. Sale to the proprietor should be at cost price (20,000.00)
Income under the head business/profession 10,94,750
Income under the head other sources: Dividends from a foreign company 3,000
Income under the head house property
NAV = GAV – Municipal Tax paid = Rs. 1,40,000 – Nil = Rs. 1,40,000 1,40,000
Less: 30% of NAV u/s 24(a) (42,000)
Income under the head House Property 98,000
Gross Total Income 11,95,750
Less: Deduction u/s 80C (70,000)
Total Income 11,25,750

Q11. Mr. X submits his profit & loss account for year ending 31st March 2019. Computed net profit after debiting
the following amounts to Rs. 87,000.
1. Provisions for doubtful debts 16,000
2. Depreciation reserve 21,000
3. Household expenses 20,000
4. Donations to poor persons 10,000
5. Other charitable donations 20,000
6. Cash payment for purchases 80,000
7. Advertisement expenses: Rs. 5,000 spent on neon sign board purchased & put to use on 1.7.2018 &
advertisement gifts to 50 customers at a cost of Rs. 100 each.
8. Audit fee charged Rs. 5,000, including expenses on income-tax proceedings Rs. 3,000.
9. Patents purchased for Rs. 70,000 (paid by A/c payee cheque) on 01.10.2018 & put to use on 7.10.2018.
10. Preliminary expenses covered u/s 35D: Market survey expenses: 5,000; feasibility report expenses: 10,000.
Project cost: Rs. 10,00,000.
Incomes credited to profit & loss account were:
(i) Interest on company deposit Rs. 50,000.
(ii) Opening stock is valued at cost plus 15% basis, whereas closing stock was valued at cost minus 15% basis.
Opening stock valued was Rs. 1,15,000; closing stock valued was Rs. 1,70,000.
Compute his Taxable Income for the AY 2019-20.
Solution: Computation of Business Income
Net Profit as per profit & loss account 87,000
Add: Inadmissible Expenses
• Provision for doubtful debts 16,000
• Depreciation Reserve 21,000
• Household Expenses 20,000
• Donations to poor persons 10,000
• Other charitable donations 20,000
• Cash purchases in excess Rs. 10,000 80,000
• Cost of neon sign board (capital expenditure) 5,000
• Patents purchased 70,000
• Installment for preliminary expenses u/s 35D (15,000 - 3,000) (Refer Note Below) 12,000
• Opening stock overvalued [1,15,000 x 15/115] 15,000
• Closing stock undervalued [1,70,000 x 15/85] 30,000

DT QUESTION BANK BY CA PRANAV CHANDAK Subscribe us on YouTube 76


Less:
• Interest on company deposit (50,000)
• Depreciation on neon sign @ 10% on Rs. 5,000 (500)
• Depreciation on patents @ 12.5% on Rs. 70,000 (8,750)
Income under the head Business/Profession 3,26,750
Income under the head Other Sources [Interest on company deposit] 50,000
Gross Total Income 3,76,750
Note: Eligible Expenditure = 15,000; Max. ceiling = 5% x 10 lacs = 50,000; Deduction u/s 35D = 15,000/5 = 3,000.

Q12. The profit & loss account of Mr. X for the previous year ending 31st March, 2019 is as follows:
Particulars Amount Particulars Amount
Cost of Goods Sold 105,45,000 Sales 109,70,000
Remuneration to Proprietor 3,00,000 Dividend from Indian Company 30,000
Remuneration to Employees 1,70,000 Long term capital gain 1,90,000
Interest to proprietor 40,000
Other Expenses 1,00,000
Income Tax Paid 10,000
Net Profit 25,000

Additional information is given below:


(1) Other expenses include the following:
(i) Entertainment expenses incurred for business purpose Rs. 20,000
(ii) VIP bags, Costing Rs. 1,500 each, given to 10 dealers under the sales promotion scheme.
(iii) Employer’s contribution to recognized provident fund amounting to Rs. 10,000 was paid on 20.04.2019.
(iv) Rs. 30,000 paid in cash to a supplier who refused to accept payment by a cheque.
You are required to compute Tax Liability for the AY 2019-20.
Solution: Computation of total income of Mr. X
Net Profits as per P & L A/c 25,000
Add: Inadmissible Expenses
• Remuneration given to proprietor, not allowed 3,00,000
• Interest given to proprietor, not allowed 40,000
• Cash payment to a supplier 30,000
• Income Tax Paid 10,000
Less:
• Dividend from Indian company (30,000)
• Long Term Capital Gains (1,90,000)
Income under the head Business/Profession 1,85,000
Income from Capital Gains (Long Term Capital Gains) 1,90,000
Income from Other Sources [Dividend from Indian company is exempt u/s 10(34)] Nil
Gross Total Income 3,75,000

Q13. Income & Expenditure A/c of Dr. Sagar for the PY 2018-19 is as follows:
Expenditure Amount Income Amount
To Medicines Consumed 2,52,000 By Fee Receipts 8,49,500
To Staff Salary 1,55,000 By Rental income from house property 29,000
To Hospital Consumables 48,500 By Dividend from Indian companies 15,000
To Rent Paid 60,000
To Administrative Expenses 1,28,000
To Net Income 2,50,000
Additional Information:
(i) Rent paid includes rent for his residential accommodation of Rs. 38,000 (paid in cash).
(ii) Medicines consumed include medicines (cost) Rs. 12,000 used for Dr. Sagar’s family.
(iii) Rent received relates to a property situated at Mysore. Municipal tax of Rs. 3,500 paid in December, 2018
has been included in the “administrative expenses.”
(iv) He received Rs. 10,000 p.m as salary from ‘Full Cure Hospital’. This has not been included in “Fee Receipts”.
(v) Hospital equipment (eligible for depreciation @ 15%): 1.4.2018: Opening WDV: Rs. 5,50,000;
Acquired by A/c payee cheque (cost) on 7.12.2018 & put to use on the same date: Rs. 2,50,000.
Compute Dr. Sagar’s Taxable Income for PY 2018-19.

DT QUESTION BANK BY CA PRANAV CHANDAK Subscribe us on YouTube 77


Solution: Computation of Income of Dr. Sagar
Net profit as per profit & loss account 2,50,000
Add: Inadmissible expenses
• Rent for residential accommodation 38,000
• Medicines for personal use 12,000
• Municipal taxes 3,500
Less: Admissible Expenses/Exempt Incomes or not taxable u/h PGBP but other heads
• Depreciation on hospital equipment [Note 1] (1,01,250)
• Rental income from house property (29,000)
• Dividend from Indian companies (15,000)
Income under the head Business/Profession 1,58,250
Income from Salary
Salary (10,000 x 12) 1,20,000
Less: Standard deduction (40,000)
Income under the head Salary 80,000
Income from House Property
Gross Annual Value 29,000
Less: Municipal Taxes (3,500)
Net Annual Value 25,500
Less: 30% of NAV u/s 24(a) (7,650)
Income under the head House Property 17,850
Income under the head Other Sources - Dividend from Indian company [Exempt u/s 10(34)] Nil
Gross Total Income 2,96,100
Note: Computation of Depreciation: Rs. 5,50,000 @ 15% + Rs. 2,50,000 @ 7.5% = 82,500 + 18,750 = 1,01,250.

Q14. Mr. X, a Chartered Accountant & has prepared the following income & expenditure account as on 31.03.2019.
Income & Expenditure Account
Expenditure Amount Income Amount
Office expenses 12,000 Professional fee 65,00,000
Employee’s salary 20,000 Consultancy Fee 55,000
Magazines & newspapers 800 Dividend from Indian co. 8,500
Entertainment Expenses (Personal) 17,500 Profit on sale of debentures (STCG) 8,450
Donation for a charity show 600 Gift from father in-law 6,050
Interest on loan for professional purpose 800
Income Tax (advance tax) 5,000
Car Expenses 2,500
Purchase of books 2,000
Stationery 21,000
Diwali gift to employees 1,000
Rent of own building 60,000
Municipal tax 1,000
White washing & Painting of building 2,000
Expenses incurred on opening ceremony 3,000
You are required to compute his Total Income for AY 2019-20 considering the following points -
1. The car is used equally for official & personal purposes.
2. Rs. 1,500 for domestic servant’s salary is included in employee’s salary.
3. Books were purchased on 01.09.2018 & were put to use on the same date.
4. Payment of stationery Rs. 20,500 was made by a bearer cheque & Rs. 500 was paid in cash.
5. Mr. X is owner of a building. Its written down value is Rs. 90,000 on 01.04.2018. The building is used for official
purposes. No depreciation is claimed.
6. Furniture having written down value of Rs. 30,000 as on 01.04.2018 is also used for profession. Office chairs &
tables were purchased & put to use on 30.03.2019 for the purpose of a new office which has been inaugurated on
31.03.2019. No depreciation has been debited to the profit & loss account. Actual cost Rs. 20,000
7. Employee’s salary includes bonus of Rs. 5,000 which was paid to one of the employees on 1.7.2019.

DT QUESTION BANK BY CA PRANAV CHANDAK Subscribe us on YouTube 78


Solution: Computation of professional income as per income & expenditure account
Net profit as per profit & loss account 64,28,800.00
Add: Inadmissible expenses
Domestic servant salary 1,500.00
Entertainment expenses 17,500.00
Donation for charity show 600.00
Income tax 5,000.00
Car expenses 1,250.00
Books purchased 2,000.00
Stationery 21,000.00
Rent of own building 60,000.00
Less:
Dividend {Exempt u/s 10(34)} (8,500.00)
Profit on sale of debentures (8,450.00)
Gift from father in law (6,050.00)
Depreciation on building ( Rs. 90,000 @ 10%) (9,000.00)
Depreciation on books (2,000 @ 40%) (800.00)
Depreciation on furniture
(Rs. 30,000 @ 10%) (3,000.00)
(Rs. 20,000@5%) (1,000.00)
Income under the head Business/profession 65,00,850.00
Income under the head Capital Gains (STCG) 8,450.00
Gross Total Income 65,09,300.00
Less: Deduction u/s 80C to 80U Nil
Total Income 65,09,300.00
Note: Expenses on opening ceremony are allowed u/s 37(1).

Q15. From the following P & L A/c of Mr. X for the PY 2018-19, compute his total income for AY 2019-20:
Debits Rs. Credits Rs.
Opening Stock 9,50,000 Sales 101,06,000
Purchases 80,50,000 Closing Stock 3,60,000
Salaries 7,00,000 LTCG on sale of house property 36,000
Rent, rates & taxes 1,25,000 Dividends from foreign company 12,000
Deposit in National Saving Certificate 42,000 Winnings of a lottery (gross) 5,00,000
Miscellaneous Expenses 21,000
Provision for Income Tax 31,000
Provision for gratuity 24,000
Provision for GST 45,000
Salary to Mrs. X 48,000
Purchased a computer on 1.11.2018 & put 40,000
to use on the same date
Net Profit 9,38,000
Additional information:
(i) Purchases include
(a) Purchase of Rs. 1,00,000 from a relative (market price Rs. 80,000) & payment was made in cash.
(b) Purchase of Rs. 25,000 being the products manufactured without aid of power in a cottage industry & the
payment was made to its producer & payment was made in cash.
(c) Purchases of Rs. 35,000 from a person residing in village having no bank & payment was made in cash.
(ii) Opening & closing stock were overvalued by 10%.
(iii) Salary includes Rs. 25,000 being bonus paid to the staff on 01.11.2019 on the occasion of Diwali.
(iv) Rent, rates & taxes include Municipal tax paid on 01.11.2019 Rs. 30,000
(v) Provision for Gratuity is on actuarial basis.
(vi) Mrs. X is a housewife & payment is excessive by Rs. 48,000.
Mr. X has not opted for presumptive taxation of Income u/s 44AD.

DT QUESTION BANK BY CA PRANAV CHANDAK Subscribe us on YouTube 79


Solution:
Net profit as per profit & loss account 9,38,000.00
Add: Expenses debited to P & L A/c but not allowable
Deposit in NSC (not an expenditure) 42,000.00
Provision for income tax 31,000.00
Provision for GST 45,000.00
Salary to Mrs. X [Sec 40A(2)] 48,000.00
Purchase of computer (capital expenditure) 40,000.00
Purchase from relative [Sec 40A(2)] 20,000.00
Payment in cash [Sec 40A(3)] 80,000.00
Adjustment for opening stock (9,50,000 x 10 / 110) 86,363.64
Bonus paid after due date (Sec 43B) 25,000.00
Municipal tax paid after due date (Sec 43B) 30,000.00
Less: Permissible Expenses
Depreciation on computer (40,000 x 40% x ½) (8,000.00)
Closing stock overvalued (3,60,000 x 10/110) (32,727.27)
Less: Incomes taxable under other head but not u/h PGBP
Long term capital gain (36,000.00)
Dividend from foreign company (12,000.00)
Winnings of lottery (5,00,000.00)
Business income 7,96,636.37
Income from Other Sources
Dividend from foreign company 12,000.00
Winnings from lottery 5,00,000.00
Income from Other Sources 5,12,000.00
Income under the head Capital Gains (LTCG) 36,000.00
Gross Total Income 13,44,636.37
Less: Deduction u/s 80C [Deposit in NSC] (42,000.00)
Total Income (rounded off u/s 288A) 13,02,640.00

Q16. ABC Ltd. has computed its income to be Rs. 20 lacs & some of the entries omitted from P&L A/c are given:
1. The company has received Rs. 13 lacs for not carrying out a particular business activity
2. Company has debited the amount of opening stock Rs. 33,00,000 which is overvalued by 10%.
3. The company has paid Premium on Life of Mr. X, who is the director of the company in PY 2016-17. In PY
2018-19, it has received the maturity amount of that policy of Rs. 10 lacs.
4. Company has received duty drawbacks of Rs. 7,00,000 but the amount has not been credited to the profit &
loss account.
5. The company has received import entitlement license from the Government, & it was sold it at a profit of Rs.
5,00,000. The amount has not been credited to the profit & loss account.
6. Company was engaged in the business of electric gadgets. During the PY, Company has used one electronic
gadget for its business which was purchased for Rs. 3,00,000 but market value is Rs. 5,00,000.

Solution: In this case company’s tax liability shall be:


Net profit as per profit & loss account Rs. 20,00,000
Add: Income not credited to P&L A/c
Non-compete fees received Rs. 13,00,000
Opening stock overvalued (33,00,000 /110 x 10) Rs. 3,00,000
Keyman Insurance policy – received by employer (WN1) Rs. 10,00,000
Duty drawback received Rs. 7,00,000
Sale of import entitlement license Rs. 5,00,000
Conversion of SIT into Capital asset (WN2) Rs. 2,00,000
Income under the head business/profession Rs. 40,00,000
Tax Liability (30% + 4%) Rs. 12,48,000

Working Notes:
1. Insurance paid on keyman insurance policy is deductible u/s 37. But since it was paid in PY 2016-17, it must
have been allowed as deduction in PY 2016-17 itself. Thus no adjustment is needed for the premium paid in PY
2016-17.
2. Cost of the asset to be included in Block of asset for charging depreciation = FMV on the date of conversion of
SIT into capital asset = Rs. 5,00,000.

DT QUESTION BANK BY CA PRANAV CHANDAK Subscribe us on YouTube 80


Q17. State with reasons, the allowability of the following expenses incurred by XYZ Limited, a wholesale dealer of
commodities while computing Profit & Gains from business or profession for AY 2019-20.
(i) Construction of school building in compliance with CSR activities amounting to Rs. 5,60,000.
(ii) Purchase of building for setting up a warehousing facility for storage of food grains amounting to Rs. 4,50,000.
(iii) Interest on loan paid to Mr. X (a resident) Rs. 50,000 on which tax has not been deducted.
(iv) Commodity transaction tax paid Rs. 20,000 on sale of bullion. [NOV-2015]
Solution:
(i) Any expenditure incurred by an assessee on the activities relating to corporate social responsibility referred
to in section 135 of the Companies Act, 2013 shall not be deemed to be an expenditure incurred by the assessee
for the purposes of the business or profession.
(ii) As per section 35AD, assessee shall be allowed to debit 100% of the expenditure incurred in connection with
warehousing facility or agricultural produce hence assessee shall be allowed to debit 4.5 Lacs x 100% = 4.5 Lac.
(iii) Payment of Interest is subject to TDS. Since no tax is deducted at source, the expenditure of Rs. 15,000 (i.e.
30% of Rs. 50,000) is disallowed u/s 40(a) & balance 70% is allowed.
(iv) An amount equal to commodity transaction tax paid by the assessee shall be allowable as deduction, u/s
36(l)(xvi), if the income arising from taxable commodities transactions is included in the income computed
under the head “Profits & gains of business or profession”. In the given case, MN Limited, is entitled to claim
deduction in respect of commodity transaction tax of Rs. 20,000 paid by him.

Q18. ABC Ltd. is engaged in manufacturing & has submitted information as given below:
1. Factory Building - Written down value on 01.04.2018 12,00,000
2. Plant & Machinery (Rate 15%) - Written down value on 01.04.2018 8,70,000
3. Purchase of New Plant on 30.6.2018 (Put to use on 1.7.2018) 1,20,000
4. Purchase of New Plant on 31.12.2018 (Put to use on 1.1.2019) 1,10,000
5. Sale of old Plant on 1.12.2018 6,40,000
6. Motor Car (Rate 15%) - Written down value on 1.4.2018 1,20,000
7. Sale of Car on 30.9.2018 1,50,000
Compute depreciation allowed for the AY 2019-20.
Solution:
Factory Building - Depreciation @ 10%
Written down value on 01.04.2018 12,00,000
Depreciation @ 10% 1,20,000
Plant & Machinery - Depreciation @ 15%
Written down value on 01.04.2018 8,70,000
Purchase on 30.06.2018, put to use on 01.07.2018 1,20,000
Purchase on 31.12.2018, put to use on 01.01.2019 1,10,000
Sale of old plant on 1.12.2018 (6,40,000)
Written down value on 31.3.2019 4,60,000
Depreciation @ 15% on Rs. 3,50,000 52,500
Depreciation @ 7.5% on Rs. 1,10,000 8,250

Additional Depreciation = 1,20,000 x 20% + 1,10,000x 10% = 24,000 + 11,000 35,000

Motor Car - Depreciation @ 15%


Written down value on 01.04.2018 1,20,000
Sale on 30.09.2018 (1,50,000)
Short term capital gain 30,000

*Q19. PC Ltd. has started a new business of manufacturing paints on 1.4.2018. The company has purchased the
following assets during FY 2018-19.
Asset Actual COA Date of purchase ROD Date when it is put to use
Furniture 6,00,000 20.04.2017 10% 20.04.2017
AC installed in office 3,00,000 10.06.2017 15% 22.06.2017
Motor Car 24,00,000 15.07.2017 15% 16.07.2017
Plant A 1,50,00,000 22.04.2017 15% 25.04.2017
Plant B 60,00,000 14.09.2017 15% 14.11.2017
Plant C 2,40,000 05.08.2017 100% 18.09.2017
Computer installed in office 3,00,000 09.07.2017 60% 09.07.2017
Computer for factory 4,50,000 08.07.2017 60% 12.07.2017
Compute normal & additional depreciation allowable for AY 2019-20 to PC Ltd.

DT QUESTION BANK BY CA PRANAV CHANDAK Subscribe us on YouTube 81


Solution: Computation of Normal & Additional depreciation AY 2019-20
Asset Furniture Plant Plant& Car (15%) Plant
(10%) (60%) (100%)
Opening WDV as on 1.4.2018 Nil Nil Nil Nil
Add: Cost of Assets acquired during PY 6,00,000 7,50,000 2,37,00,000 2,40,000
WDV as on 31.3.2019 6,00,000 7,50,000 2,37,00,000 2,40,000
Less: Normal Depreciation 60,000 4,50,000 (WN 2) 31,05,000 (WN 3) 2,40,000
Additional Depreciation Nil (WN 1) 90,000 (WN 2) 36,00,000 (WN 4) Nil
WDV as on 1.4.2019 5,40,000 2,10,000 1,69,95,000 Nil
Working Note:
1. Additional depreciation is not available on furniture as the same is not covered u/s 32 (1)(iia).
2. Block of Plant also consists of computer. Computer installed in office is not eligible for additional depreciation.
Normal depreciation @ 60% on Rs. 7,50,000 Rs. 4,50,000
Additional Depreciation @ 20% on Rs. 4,50,000 Rs. 90,000
3. Normal Depreciation on Plant (inclusive of Motor-Car) has been calculated as under:
Depreciation @ 15% on Rs. 1,77,00,000 Rs. 26,55,000
Depreciation @ 7.5% on Rs. 60,00,000 (as put to use for less than 180 days) Rs. 4,50,000
Total Depreciation Rs. 31,05,000
4. Additional Depreciation on Plant (inclusive of Motor-Car) as under:
Asset Plant A Plant B Plant C
Actual cost Rs. 1,50,00,000 Rs. 60,00,000 -
Additional Depreciation Rs. 30,00,000 Rs. 6,00,000 Nil
As per third proviso to sec.32(1)(ii) inserted by Finance Act, 2015 the balance of additional depreciation of Rs.
6,00,000 being 50% of Rs. 12,00,000 (20% of Rs. 60,00,000) would be allowed as deduction in the AY 2018-19.

Q20. Mr. X is engaged in the business of generation & distribution of electric power. He always claims depreciation
on written down value. From the following details, compute depreciation allowable for AY 2019-20: [NOV-2013]
(i) Opening WDV of block (15% rate) 42 Lacs
(ii) New machinery purchased on 12.10.2018 10 Lacs
(iii) Machinery imported from Colombo on 12.04.2018. This machine had been used only in 9 lacs
Colombo earlier & the assessee is the first user in India.
(iv) New computer installed in generation wing of the unit on 15.07.2018 2 lacs

Solution: Computation of Depreciation allowable u/s 32 for AY 2019-20


Particulars Rs.
Opening WDV as on 01.04.2018 42,00,000
Add: New Machinery purchased & put to use on 12.10.2018 10,00,000
Add: Machinery imported from Colombo purchased & put to use on 12.04.2018 9,00,000
WDV as on 31.03.2019 61,00,000
Normal Depreciation @ 7.5% on Rs. 10,00,000 75,000
Normal Depreciation (5) 15% on Rs. 51,00,000 7,65,000
Total Normal Depreciation 8,40,000
Additional depreciation @ 10% on Rs. 10,00,000 1,00,000
Total Depreciation 9,40,000

Block 40%
New Computer in Generation wing purchased & put to use on 15.07.2018 2,00,000
Normal Depreciation @ 40% on Rs. 2,00,000 80,000
Additional depreciation @ 20% on Rs. 2,00,000 40,000

Q21. Venus Ltd., engaged in manufacture of pesticides, furnishes the following particulars relating to its
manufacturing unit at Chennai (for the year ending 31.03.2019):
Opening WDV of Plant & Machinery 20 Lacs
New machinery purchased on 01.09.2018 10 Lacs
New car purchased on 01.12.2018 8 Lacs
Computer purchased on 03.01.2019 4 Lacs

DT QUESTION BANK BY CA PRANAV CHANDAK Subscribe us on YouTube 82


Additional information:
• All assets were put to use immediately.
• Computer has been installed in the office.
• During the year ended 31.03.2018, a new machinery had been purchased on 5.10.2017, for Rs. 10 lacs. Additional
depreciation, besides normal depreciation, had been claimed thereon.
• Depreciation rate for machinery may be taken as 15%.
Compute the depreciation available & WDV of different blocks of assets on 31.03.2019. [MAY-2016]
Solution: Computation of depreciation allowable for AY 2019-20
Block of Assets Rs.
Block 1: Plant - rate 40%
Computer 4,00,000
Less: depreciation 4,00,000 @ 40% x ½ (80,000)
WDV as on 31.03.2019 3,20,000
Block 2: Plant & Machinery - Rate 15%
Opening WDV of the block 20,00,000
Add: Machinery purchased on 01.09.2018 10,00,000
Less: Depreciation 30,00,000 @ 15% (4,50,000)
Less: Additional depreciation 10,00,000 @ 20% (2,00,000)
Less: Additional depreciation 10,00,000 @ 10% (1,00,000)
WDV as on 31.03.2019 22,50,000
Block 3: Motor Car - rate 15%
Opening WDV of the block Nil
Add: Car purchased on 01.12.2018 8,00,000
Less: Depreciation - 8,00,000 @ 15% x ½ (60,000)
WDV as on 31.03.2019 7,40,000
Note:
1. Additional depreciation shall not be allowed on any machinery installed in office & road transport vehicles.
2. If asset is put to use for less than 180 days then additional depreciation is allowed at half of the normal rate in
the year of purchase & balance shall be allowed in the subsequent year.
3. If asset is used for less than 180 days then depreciation shall be allowed at half of the normal rate.

Q22. X Ltd. set up a manufacturing unit in notified backward area in the state of Telangana on 1.6.2018. It invested
Rs. 30 crores in new plant & machinery on 01.06.2018. Further, it invested Rs. 25 crores in plant & machinery on
1.11.2018, out of which Rs. 5 crores was second hand plant & machinery. Compute the depreciation allowable u/s
32. Is X Ltd. entitled for any other benefit in respect of such investment? If so, what is the benefit available?
Solution: Computation of depreciation u/s 32 for X Ltd. for AY 2019-20
Particulars Rs. (in cr)
Plant & machinery acquired on 01.06.2018 30,000
Plant & machinery acquired on 01.11.2018 25,000
WDV as on 31.03.2019 55,000
Less: Depreciation @ 15% on Rs. 30 crore 4,500
Depreciation @ 7.5% (50% of 15%) on Rs. 25 crore 1,875
Additional Depreciation @ 35% on Rs. 30 crore 10,500
Additional Depreciation @ 17.5% (50% of 35%) on Rs. 20 crore 3,500
WDV as on 01.04.2019 34,625

Deduction u/s 32AD @ 15% on Rs. 50 crores = 7.50 crores

Q23. Lights & Power Ltd. engaged in the business of generation of power, furnishes the following particulars
pertaining to P.Y. 2018-19. Compute the depreciation allowable u/s 32 for AY 2019-20. The company has opted
for the depreciation allowance on the basis of written down value.
Opening Written down value of Plant & Machinery (15% block) as on 01.04.2018 5,78,000
Purchase of second hand machinery (15% block) on 29.12.2018 for business purpose & put to 2,00,000
use on the same date
Machinery Y (15% block) purchased & installed on 12.7.2018 for power generation 8,00,000
Acquired & installed for use as new air pollution control equipment (40% block) on 31.7.2018 2,50,000
New air conditioner purchased & installed in office premises on 08.09.2018 3,00,000
New machinery Z (15% block) acquired & installed on 23.11.2018 for generation of power 3,25,000
Sale value of an old machinery X, sold during the year 3,10,000

DT QUESTION BANK BY CA PRANAV CHANDAK Subscribe us on YouTube 83


Solution: Computation of depreciation allowance u/s 32 for AY 2019-20
Particulars P & M (15%) P & M (40%)
Opening WDV as on 1.4.2018 5,78,000
Add: P&M acquired during the year
- Second hand machinery 2,00,000
- Machinery Y 8,00,000
- Air conditioner for office 3,00,000
- Machinery Z 3,25,000
- Air pollution control equipment - 2,50,000
Less: Asset sold during the year 13,10,000 Nil
Written down value before charging depreciation 18,93,000 2,50,000
Normal depreciation: - 1,00,000
40% on air pollution control equipment
Depreciation on plant & machinery put to use for less than 180 39,375
days @ 7.5% (2,00,000 + 3,25,000)
15% on Rs. 13,68,000 2,05,200
Additional depreciation 1,60,000
- Machinery Y ( Rs. 8,00,000 x 20%)
- Machinery Z ( Rs. 3,25,000 x 10%) 32,500
Total depreciation 4,37,075 1,00,000

Q24. A Ltd. engaged in the business of generation & distribution of power has claimed depreciation on SLM
basis. You are informed that a second hand Diesel Electric & Gas Plant was acquired for Rs. 60 lacs& depreciation
at 8.24% was claimed for 2 years on SLM basis. Assuming that the said Plant is sold for a consideration of
(i) Rs. 36,00,000: or (ii) Rs. 55,00,000; or (iii) Rs. 62,00,000 during PY 2017-18. Examine the treatment.
Solution: Depreciation = (Rs. 60,00,000 X 8.24% X 2) = Rs. 9,88,880.
(i)Sale price Rs. 36,00,000
The actual cost of the plant is Rs. 60,00,000. The depreciation claimed is Rs. 9,88,800 & consequently, the
depreciated value after 2 years is Rs. 50,11,200. The sale consideration of Rs. 36,00,000 being less than the
depreciated value, the short fall of Rs. 14,11,200 shall be allowed to be written off under clause (iii) of sub-
section (1) of Sec. 32. This is normally known as Terminal Depreciation.
(ii) Sale price Rs.55,00,000
The sale consideration of Rs. 55,00,000 is more than the depreciated value of Rs. 50,11,200 but not more than
the actual cost of Rs. 60,00,000. Therefore, the surplus of Rs. 4,88,800 shall be assessed to tax u/s. 41 (2) as
balancing charge.
(iii) Sale price Rs. 62,00,000
The sale consideration of Rs. 62,00,000 is not only more than the depreciated value of Rs. 50,11,200 but also
more than the actual cost of Rs. 60,00,000. Therefore, the surplus of Rs. 9,88,800 ( Rs. 60,00,000 less Rs.
50,11,200) shall be assessed to tax u/s. 41 (2) as balancing charge under the head “Profits & gains of business or
profession” & the remaining surplus of Rs. 2,00,000 (Rs. 62,00,000 – Rs. 60,00,000) shall be taxed as STCG.

Q25. Rahul Ltd. has a block of assets (depreciation @ 15%) WDV of which on 1.4.2018 was Rs. 10,00,00. It
purchased another asset on 1.1.2019 of the same block for Rs. 2,88,000 & put it to use immediately. Rahul Ltd.
amalgamated with Geeta Ltd. on 2.3.2019 & the block of assets were transferred to Geeta Ltd. at Rs. 20,00,000.
Compute the depreciation allowable to Rahul Ltd. & Geeta Ltd for PY ended on 31.3.2019.
Solution: As per 6th proviso to section32, depreciation shall be calculated first as if no amalgamation has taken
place & then aggregate depreciation shall be apportioned b/w amalgamated company & amalgamating company:
WDV as on 1.4.2018 10,00,000
Add: Actual cost of asset acquired during the year for less than 180 days 2,88,000
WDV for charging Depreciation 12,88,000
Depreciation @ 7.5 % on Rs. 2,88,000 21,600
Depreciation @ 15 % on Rs. 10,00,000 1,50,000
Total Depreciation for the PY 2018-19 1,71,600

Apportionment of depreciation Between two companies:


(i) For the amalgamating company Rahul Ltd.
Rs. 1,50,000 × 335/365 = Rs. 1,37,671 + Rs. 21,600 × 60/90 = Rs. 14,400 Rs. 1,52,071
(ii) For the amalgamated company Geeta Ltd.
Rs. 1,50,000 × 30/365 = Rs. 12,329 + Rs. 21,600 × 30/90 = Rs. 7,200 Rs. 19,529

DT QUESTION BANK BY CA PRANAV CHANDAK Subscribe us on YouTube 84


Q26. Opening WDV of the Block Assets of Old Ltd. is Rs. 38,00,000 as on 1.4.2018. It acquired another asset for Rs.
7,00,000 on 1.6.2018. Old ltd is amalgamated with Strong Ltd. on 30.8.2018. In Strong Ltd, opening WDV of such
block was Rs. 20,00,000. It acquired another asset in the block on 31.12.2018 for Rs. 4,00,000. Old Ltd has sold the
block of asset to Strong Ltd. for Rs. 50,00,000. Rate of Depreciation is 15%. Explain the tax implications.
Solution:
▪ As per 6th proviso to section 32, depreciation shall be calculated first as if no amalgamation has taken place &
then the aggregate depreciation shall be apportioned between the firm & the company.
▪ Depreciation shall be first calculated as if the amalgamation has not taken place:
15 % of Rs. 38,00,000 = Rs. 5,70,000;
15 % of Rs. 7,00,000 = Rs. 1,05,000.
Apportionment of Depreciation between “predecessor & successor”
Old Ltd: Rs. 5,70,000 × 151/365 = Rs. 2,35,808 + Rs. 1,05,000 × 90/304 = Rs. 31,086 = Rs. 2,66,894.

Strong Ltd: Rs. 5,70,000 × 214/365 = Rs. 3,34,192 + Rs. 1,05,000 × 214/304 = Rs.73,914 = Rs.4,08,106.

For amalgamating Company (Old Ltd)


▪ No Capital gains shall arise to the amalgamating company as per section 47 (vi).
▪ Depreciation allowable to amalgamating company = Rs. 2,66,894

For amalgamated Company (Strong Ltd)


Opening WDV as o 01-04-2016 20,00,000
Add: Actual cost of the assets acquired during the previous year 4,00,000
Add: Actual cost of assets acquired from the amalgamating company
As per Explanation 7 to section 43 (1) 7,00,000 7,00,000
As per Explanation 2 to section 43 (6) 38,00,000
Less: Depreciation allowable to amalgamating company (Old Ltd) 2,66,894 42,33,106

WDV for AY 2017-18 66,33,106


Less: Depreciation for Strong Ltd
@ 15% on Rs. 20,00,000 3,00,000
@ 7.5% on Rs. 4,00,000 30,000
On Rs. 45,00,000 apportioned depreciation 4,08,106
WDV as on 01-04-2017 58,95,000

Q27. Harsh Ltd. has a block of assets (depreciation @ 15%), WDV of which on 1.4.2018 was Rs. 12 lacs. It
purchased & put to use another asset on 1.11.2018 for Rs. 3 lacs. Harsh Ltd. is converted into Harsh LLP on
1.1.2019 & the block of assets were transferred to Harsh LLP at Rs. 18 lacs. Though the conversion into LLP took
place on 1.1.2019, but assets were put into use by Harsh LLP on 1st March 2019. Explain the tax implications.
Solution: Depreciation shall be calculated first as if no conversion has taken place & then the aggregate
depreciation shall be apportioned between the predecessor company & the successor LLP.
WDV as on 1.4.2018 12,00,000
Add: Actual cost of asset acquired during the year [Put to use for less than 180 days] 3,00,000
WDV on 31.3.2019 15,00,000
Depreciation @ 15% on Rs. 12,00,000 [WDV of assets used for more than 180days] 1,80,000
Depreciation @ 7.5% on Rs. 3,00,000 [WDV of assets used for more than 180days] 22,500
Depreciation for PY 2018-19 2,02,500
Apportionment of Depreciation between Predecessor & successor
1. Harsh Ltd: (Note 1) = [(1,80,000 × 275/306 = ) Rs. 1,61,765 + (22,500 × 61/92 = ) 14,918] = Rs. 1,76,683
2. Harsh LLP (Note 1) = [(1,80,000 × 31/306 = ) Rs. 18,235 + (22,500 × 31/92 = ) Rs. 7,582] = Rs. 25,817
Calculation of Depreciation for Harsh LLP
Actual Cost of Assets acquired from Harsh Ltd. (WDV on the Date of conversion)
[(WDV of Block of assets on date of conversion) = (Rs. 12,00,000 + Rs. 3,00,000)] 15,00,000
Less: Depreciation allowable to Harsh Ltd (1,76,683)
WDV for AY 2019-20 13,23,317
Less: Depreciation for Harsh LLP on Rs. 15,00,000 - Apportioned depreciation (25,817)
WDV as on 1.4.2019 12,97,500
Note: (1) The number of days of month of January & February has not been considered for apportioning the
depreciation as the Harsh LLP has put to use assets only on 1st March, 2018 i.e. after 2 months from Conversion.
(2) Price at which the assets are transferred (Rs. 18,00,000) has no implication in computing depreciation.

DT QUESTION BANK BY CA PRANAV CHANDAK Subscribe us on YouTube 85


Q28. Mr. A Converted the following assets used for personal purposes into his business assets on 01.04.2018.
Particulars COA DOA FMV on 1.4.2018 Value recorded in books on 1.4.2018
Building 20,00,000 1.4.2016 30,00,000 30,00,000
LAND 10,00,000 1.4.2015 50,00,000 10,00,000
Furniture 5,00,000 1.4.2016 2,00,000 3,00,000
Solution: Computation of Actual Cost of the asset of Mr. A
Particulars Rs.
Cost of asset as on 1.4.2016 20,00,000
Less: Notional depreciation @ 5% for the year 2016-17 (1,00,000)
WDV as on 31.03.2017 19,00,000
Less: Notional depreciation @ 5% for the year 2017-18 (95,000)
Cost to be capitalized as per Explanation 5 Sec. 43(1) 18,05,000
Explanation 5 to Sec. 43 requires the notional deprecation to be computed only in the case of building & not in
case of any other asset. Therefore, the value of other fixed assets namely LAND & Furniture shall get recorded at
Rs. 10 Lacs & Rs. 5 Lacs respectively. However, in case of building, cost to be adopted for computing depreciation
shall be Rs. 18,05,000 as given above & not at Rs. 30 Lacs as adopted by Mr. A.

Q29. Mr. Praveen Kumar has furnished the following particulars relating to payments made towards scientific
research for the year ended 31.3.2019:
SN Particulars (Rs. In Lacs)
(i) Payments made to K Research Ltd. 20
(ii) Payment made to LMN College 15
(iii) Payment made to OPQ College 10
(iv) Payment made to National Laboratory 8
(v) Machinery purchased for in-house scientific research 25
(vi) Salaries to research staff engaged in in-house scientific 12
Note: K Research Ltd. & LMN College are approved research institutions & these payments are to be used for the
purposes of scientific research. Compute the amount of deduction available u/s 35. [SM + May 2011]
Solution:
Particulars Amount Sec % of Deduction Deduction
Payment for scientific research
K Research Ltd (WN 2) 20 35(1)(ii) 100% 20.00
LMN College 15 35(1)(ii) 150% 22.50
OPQ College 10 - Nil Nil
National Laboratory 8 35(2AA) 150% 12.00
In-house research [WN 2]
Capital expenditure 25 35(1)(iv) r.w. 35(2) 100% 25.00
Revenue expenditure 12 35(1)(i) 100% 12.00
Deduction allowable u/s 35 91.50
Notes:
1. Deduction for in-house research & development: Only company assessees are entitled to weighted deduction
@150% u/s 35(2AB) in respect of in-house research & development expenditure incurred. However, in this case,
the assessee is an individual. Thus he would be entitled to deduction @100% of the revenue expenditure
incurred u/s 35(1)(i) & 100% of the capital expenditure incurred u/s 35(1)(iv) read with section 35(2).

Q30. XYZ Ltd., a paper manufacturing concern, purchases a machine on 1 March 2016 for Rs. 6,10,000 for its
laboratory with a view to improving the quality of art paper manufactured by the company.
1. What will be the amount of deduction u/s 35 on account of capital expenditure of Rs. 6,10,000 for PY 2018-19?
2. If the research activity ceases in 2017 & machinery is brought into business on 1.11.2017 (FMV: Rs. 2,30,000);
depreciation @ 15%: Depreciated value of relevant block of assets on 1.4.2017 is Rs. 14,07,860; scientific research
machine is sold for Rs. 1,90,000 on 4.4.2018, what will be the depreciation for AY 2019-20 & profit u/s 41(3)?
3. If research activity ceases on 1.12.2017 (FMV of the machine: Rs. 2,30,000) & the machine is sold on 1.4.2018
without using it for another purpose [Sale price = Rs. 190000 or Rs. 1500000].
Solution:
1. 100% of capital expenditure (Rs. 6,10,000) will be allowed as deduction in AY 2016-17 u/s 35 (PY 2015-16).
2. The machine is brought into business on 1.11.2017. Tax treatment if depreciation will be as under:
Depreciated value of the block of assets on 1.4.2017 14,07,860
Add: Cost of machine transferred on 1.11.2017 (since 100% deduction already claimed) Nil
Written down value of the block after addition of Machinery 14,07,860

DT QUESTION BANK BY CA PRANAV CHANDAK Subscribe us on YouTube 86


Less: Depreciation for PY 2017-18 (15% of Rs. 14,07,860) (2,11,179)
Depreciated value of the block on April 1, 2018 11,96,681
Less: Sale proceeds of machine sold on 4 April 2018 (1,90,000)
Written down value after sale of machinery 10,06,681
Less: Depreciation for the PY 2018-19 (15% of Rs. 10, 06,681) (1,51,002)
Depreciated value of the block on April 1, 2019 8,55,679
Note: There will be no capital gain or loss in this case since the block is in existence & WDV is also not “Zero”
3. Tax treatment if machine is sold without using it for another purpose:
Particulars Sold @ 1.9 Lacs Sold @ 15 Lacs
Taxable amount u/s 41(3) 1,90,000 6,10,000
[Lower of (i) Sale proceeds (ii) Deduction allowed u/s 35]
Capital Gain u/s 45
Sale proceeds 1,90,000 15,00,000
Less: Cost of acquisition [Depreciable Asset & thus always STCA] 6,10,000 6,10,000
Short term capital gain (-) 4,20,000 8,90,000

Q31. An assessee, who is already in the business of trading in textiles, commences the business of cold chain facility
w.e.f. 01.07.2018 & has incurred the following expenditure:
Machinery purchased on 26.02.2018 & capitalized in the books of accounts 4,00,000
LAND purchased on 1.4.2017 & capitalized 6,00,000
Building constructed on 30.6.2018 10,00,000
Goodwill purchased on 5.6.2018 2,00,000
Machinery purchased on 20.1.2019 3,00,000
Calculate the deduction allowed u/s 35AD for AY 2018-2019.
Solution:
▪ Deduction of 100% of capital expenditure incurred wholly for cold chain business shall be allowed during the
year in which the expenditure is incurred.
▪ All capital expenditures are eligible for 100% deduction (except LAND, GW & any financial instrument).
▪ Land, Goodwill will not qualify for deduction.
▪ Capital expenditure incurred before commencement shall be deductible in the year of commencement, if
expenditure is capitalized in books of account on date of commencement.
▪ Total deduction allowed for PY 2018-19 = 100% of [Rs. 4L (machinery purchased before commencement of
business since capitalized ) + Rs. 10 L (building) + Rs. 3 L (machinery) = Rs. 17,00,000.

Q32. MNP Ltd. Commenced operations of the business of a new four-star hotel in Chennai on 01.04.2018. The
company incurred capital expenditure of Rs. 40 Lac during the period January, 2018 to March, 2018 exclusively
for the above business & capitalized the same in its books of account as on 1 st April, 2018. Further, during the
Previous Year 2018-19, it incurred capital expenditure of Rs. 2.5 crore (out of which Rs. 1 crore was for acquisition
of land) exclusively for the above business. Compute the income under the heading “profit & gains of business or
profession” for the AY 2019-20, assuming that MNP Ltd. has fulfilled all the conditions specified for claim of
deduction u/s 35AD. The profits from the business of running this hotel (before claiming deduction u/s 35AD) for
the AY 2019-20 is Rs. 80 Lacs. Assume that the company also has another existing business (specified business)
of running a four-star hotel in Kanpur, which commenced operations 2 years back, the profits from which was Rs.
130 Lacs for AY 2019-20. [May 2012]
Solution: Deduction allowable u/s 35AD
Profit of specified business 80,00,000
Less: Capital Expenditure before commencement (40,00,000)
Less: Capital Expenditure during the year (250 Lacs - 100 Lacs) (150,00,000)
Loss from specified business of new hotel in Chennai (110,00,000)
Net Income (Loss can be set off from income of other specified business) [130 lacs - 110 lacs] 20,00,000

Q33. ABC Ltd. commenced business of operating 3-star hotel in Tirupati on 1.4.2018. It furnishes following details:
(i) Cost of land (acquired in June 2016) Rs. 60 lacs
(ii) Cost of construction of hotel - FY 2016-17 & 2017-18 [Rs. 30 lacs & Rs. 150 lacs resp] Rs. 180 lacs
(iii) Plant & machineries (all new) Acquired during financial year 2017-18 Rs. 30 lacs
Net profit before depreciation for the financial year 2018-19 Rs. 80 lacs
[All the above expenditures were capitalized in the books of the company]
Determine the amount eligible for deduction u/s 35AD for AY 2019-20. [MAY-2011]

DT QUESTION BANK BY CA PRANAV CHANDAK Subscribe us on YouTube 87


Solution: Business of operating of three-star hotel is a specified business & 100% of capital expenditure shall be
allowed as deduction including expenditure incurred before commencement of business except cost of land.
Computation of Income
Profits before debiting the expenses Rs. 80,00,000
Cost of land (not eligible for deduction u/s 3 5 AD) Nil
Cost of construction of hotel building ( Rs. 30 Lacs + Rs. 150 Lacs) (180,00,000)
Cost of plant & machinery (30,00,000)
Loss from specified business (130,00,000)
As per section 73A such loss can be set off from income of any other specified business & its carry forward is
allowed for unlimited period & even in the subsequent years it can be set off from income of specified business.

Q34. X Ltd. is incorporated in Mumbai on 6 September 2018. It commences production on 15 March 2019. The
following expenses are incurred by the company before commencement of business-
(a) Expenses on incorporation , issue of shares, etc : Rs. 92,000.
(b) Preparation of feasibility report & conducting market survey: Rs. 1,40,000.
(c) Engineering services (work is carried on by a concern which is unapproved by Board): Rs. 1,30,000.
Determine deduction u/s 35D. Particulars as on last day of PY in which business is commenced are:
Cost of fixed asset 55 Lacs
Share capital 40 Lacs
Debentures 12 Lacs
Long-term borrowing from a financial institution (repayable for not less than 7 years) 8 Lacs
Solution:
Cost of Project 55,00,000
Capital Employed (i.e. Rs. 40 lacs + Rs. 12 lacs + Rs. 8 lacs) 60,00,000
(i) Eligible Expenditure
Expenses on incorporation (These are included even if the work is undertaken by a 92,000
person not approved by the Board)
Preparation of feasibility report, project report & conducting market survey 1,40,000
Engineering services (Since it is completed by unapproved concern, it is not included) Nil
Total Eligible Expenditure Rs. 2,32,00
(ii) Maximum Limit (5% of Rs. 55 lacs or Rs. 60 lacs, whichever is higher) Rs. 3,00,000
Qualifying Amount for Deduction Lower of (i) or (ii) Rs. 2,32,000
Amount deductible in 5 years for AY 2019-20 to 2023-24 Rs. 46,400

Q35. XLtd. made payment of VRS to its employee Y as under:


PY 2018-19: Rs. 4,00,000 PY 2019-20: Rs. 3,00,000 PY 2020-21: Rs. 1,40,000
How deduction of above expense will be claimed by X Ltd. as per Income Tax Act?
Also calculate how much deduction will be allowed to X Ltd. for AY 2019-20 & 2020-21 in respect of the VRS?
Solution: Deduction of VRS Expenditure:
AY Payment 1/5th Deduction from PY of payment Period of 5 years from year of payment
2019-20 4,00,000 80,000 AY 2019-20 to 2023-24
2020-21 3,00,000 60,000 AY 2020-21 to 2024-25
2021-22 1,40,000 28,000 AY 2021-22 to 2025-26
(i) Total Deduction u/s 35DDA for AY 2019-20 = Rs. 80,000.
(ii) Total Deduction u/s 35DDA for AY 2020-21 = Rs. 80,000 + Rs. 60,000 = Rs. 1,40,000.

Q36. ABC limited is a company engaged in the business of biotechnology. The net profit of the company for the
financial year ended 31.03.2019 is Rs. 15,25,890 after debiting the following items:
1. Purchase price of raw material used for the purpose of in-house research & development 1,80,000
2. Purchase price of asset used for in-house research & development
(a) Land 5,00,000
(b) Building 3,00,000
3. Expenditure incurred on notified agricultural extension project 1,50,000
4. Expenditure on notified skill development project:
(a) Purchase of land 2,00,000
(b) Expenditure on training for skill development 2,50,000
5. Expenditure incurred on advertisement in the souvenir published by a political party 75,000
Compute the income u/h “Profits & gains of business or profession” for the AY 2019-20 of ABC Ltd.
The company is eligible to claim 150% of expenditure incurred u/s 35(2AB).

DT QUESTION BANK BY CA PRANAV CHANDAK Subscribe us on YouTube 88


Solution: Computation of Income u/h “Profits & gains of business or profession” for AY 2019-20
Net profit as per profit & loss account 15,25,890
Less: Purchase price of raw material (balance amount u/s 35(2AB) (90,000)
Add: Purchase price of Land used in in-house research & development 5,00,000
Less: Expenditure incurred on notified Agricultural Extension Project
(Balance - 1,50,000 x 0.5 u/s 35CCC) (75,000)
Add: Expenditure incurred on notified skill development project - Purchase of land
- being capital expenditure not qualifying for deduction u/s 35CCD 2,00,000
Less: Expenditure incurred on notified skill development project
(balance amount u/s 35CCD 2,50,000 x 0.5) (1,25,000)
Add: Expenditure incurred on advertisement in the souvenir published by a political party 75,000
not allowed as deduction as per section 37(2B)
Profits & gains of business/profession 20,10,890
Note: The expenditure incurred on advertisement in the souvenir published by a political party is disallowed as
per section 37(2B) while computing income under the head “Profit & Gains of Business or Profession”.

Q37. During the financial year 2018-19, the following payments/expenditure were made/incurred by Mr. X, a
resident individual (whose turnover during the year ended 31.03.2018 was Rs. 99 lacs):
(i) Interest of Rs. 12,000 was paid to ABC & Co., a resident partnership firm, without deduction of tax at source;
(ii) Interest of Rs. 4,000 was paid as interest to Mr. Y, a non-resident, without deduction of tax at source;
(iii) Rs. 3,00,000 was paid as salary to a resident individual without deduction of tax at source;
(iv) He had sold goods worth Rs. 5 lacs to Mr. D. He paid Commission of Rs. 15,000 to Mr. Z on 02.07.2018. In none
of these transactions, tax was deducted at source.
Briefly discuss whether any disallowance arises u/s 40(a) of the Income tax Act, 1961. [MAY-2011]
Solution:
(i) Since Mr. X was not liable to tax audit in the preceding year hence expenditure is allowed even if tax has not
been deducted at source.
(ii) In the case of interest paid to a non-resident, there is obligation to deduct tax at source u/s 195, hence non-
deduction of tax at source will attract disallowance.
(iii) Disallowance u/s 40(a) is attracted for failure to deduct tax at source u/s 192 from salaries. 70% of salary i.e.
Rs. 3,00,000 x 70% = Rs. 2,10,000 shall be allowed in previous year 2018-19 & balance i.e. Rs. 90,000 is disallowed.
(iv) Since Mr. X was not liable to tax audit in the preceding year hence expenditure is allowed even if tax has not
been deducted at source.

Q38. M/s ABC Ltd., submits the following details of expenditures pertaining to the financial year 2018-19:
(i) Payment of professional fees to Mr. M Rs. 50,000. Tax not deducted at source.
(ii) Interior works done by Mr. H for Rs. 2,00,000 on a contract basis. Payment made in the month of March
2019. Tax deducted in March 2019, was paid on 30.06.2019.
(iii) Factory Rent paid to Mrs. R Rs. 15,00,000. Tax deducted at source & paid on 01.10.2019.
(iv) Interest paid on Fixed Deposits Rs. 2,00,000. Tax deducted on 31.12.2018 & paid on 28.09.2019.
(v) Payment made to M/s G & Co. towards import of Raw Materials Rs. 25,00,000. No tax was deducted at
source. The supplier G & Co. is located in London.
Examine the above with reference to allowability of the same in AY 2019-20. [NOV-2010]
Solution: Allowability of expenses of M/s. ABC Ltd for the AY 2019-20
(i) Payment of professional fees is subject to TDS u/s 194J. Since no tax is deducted at source, the expenditure of
Rs. 15,000 (i.e. 30% of Rs. 50,000) is disallowed u/s 40(a) & balance 70% is allowed.
(ii) Since the tax was deducted in March, 2019 & paid on or before the due date of filing the return (i.e., on or before
September 30th, 2019), the expenditure on interior works will be allowed as deduction.
(iii) The maximum time allowable for deposit of tax deducted at source is upto the due date of filing of return i.e.,
30th September, 2019. In this case, since tax deducted u/s 194-1 was paid after the due date of filing the return,
the expenditure of Rs. 4,50,000 (30% of Rs. 15,00,000) is disallowed in previous year 2018-19 & balance 70% is
allowed in previous year 2018-19.
(iv) Since the tax was deducted in December, 2018 & paid on or before the due date of filing the return (i.e., on or
before September 30th, 2019), the interest paid on fixed deposits will be allowed as deduction.
(v) Since payment towards import of raw materials does not attract the provisions of deduction of tax at source,
the expenditure will be allowed as deduction.

Q39. X & Y, a partnership firm consisting of 2 partners, reports net profit of 7,00,000 before deduction of following:
(1) Salary of 20,000 each p.m payable to 2 working partners of firm (as authorized by the deed of partnership).
(2) Depreciation on plant & machinery u/s 32 (computed): Rs. 1,50,000.
(3) Interest on capital at 15% p.a (as per p’ship deed). Amount of capital eligible for interest is Rs. 5,00,000.

DT QUESTION BANK BY CA PRANAV CHANDAK Subscribe us on YouTube 89


Compute:
(i) Book-profit of the firm u/s 40(b) of the Income-Tax Act 1961.
(ii) Allowable working partner salary for AY 2019-20 as per section 40(b). [NOV-2011]
Solution: (i) Computation of Book- Profits u/s 40(b) of Income Tax Act 1961
Net Profit 7,00,000.00
Less: Depreciation u/s 32 (1,50,000.00)
Less: Interest on capital (60,000.00)
(5,00,000 x 12%)
Book Profit as per section 40(b) 4,90,000.00

(ii) Calculation of allowable salary to partners


(a) Book Profit 4,90,000.00
(b) Allowable Salary
On first 3,00,000 of book profit: 90% of book profits or 1,50,000 whichever is higher 2,70,000
On balance book profit: 60% on balance book profit (1,90,000 x 60/100) 1,14,000
Hence, salary as per section 40(b) would be 3,84,000

Q40. Mr. X has computed his income Rs. 3,50,000 & some of the amounts debited to P&L A/c are as given below:
1. Household expense Rs. 5,000
2. Rent for own building Rs. 1,20,000 (half of the building is in personal use & balance half in business use).
3. Municipal tax of the building Rs. 3,000 (amount was paid on 01.04.2019)
4. Expenditure on repairs of the building Rs. 4,000.
5. Premium paid for insurance of the building Rs. 2,000.
6. Mr. X has purchased one motor car for Rs. 3,00,000 on 01.01.2019 & it was put to use on the same date. The
car was use for personal purpose as well as official use (50% official & 50% personal). Assessee has also debited
petrol expenses of Rs. 5,000.
7. He has debited Rs. 20,000 being the amount invested in public provident fund.
His tax liability for the AY 2019-20 shall be computed in the manner given below:
Solution:
Net income as per profit & loss account 3,50,000.00
Add: Inadmissible Expenses
1. Household expenses 5,000.00
2. Rent of the own building 1,20,000.00
3. Municipal tax 1,500.00
4. Repairs 2,000.00
5. Insurance 1,000.00
6. Capital expenditure on motor car 3,00,000.00
7. Petrol 2,500.00
8. Public Provident Fund 20,000.00
Less: Admissible Expenses
Depreciation of motor car [3,00,000 x 15% x 54 x 54] (11,250.00)
Income under the head Business/Profession 7,90,750.00
Gross Total Income 7,90,750.00
Less: Deduction u/s 80C (20,000.00)
Total Income 7,70,750.00

Q41. Mr. X engaged in Retails Trade, reports a turnover of Rs. 58,50,000 (all payments received in account payee
cheque) for the financial year 2018-19. His income from the said business as per books of account is computed at
Rs. 2,90,000. Retail trade is the only source of income for Mr. X.
(i) Is Mr. X eligible to opt for presumptive determination of his income chargeable to tax for AY 2019-20?
(ii) Is so, determine his income from retail trade as per the applicable presumptive provision.
(iii) In case, Mr. X has not opted for presumptive taxation of income from retail trade, what are his obligations
under the Income-tax Act, 1961?
(iv) What is the ‘due date’ for filing his return of income, under both the options? [MAY-2011]
Solution:
(i) Yes. Since his total turnover for the F.Y.2018-19 is below Rs. 200 Lacs, he is eligible to opt for presumptive
taxation scheme u/s 44AD in respect of his retail trade business.
(ii) His income from retail trade, applying the presumptive tax provisions u/s 44AD, would be Rs. 3,51,000,
being 6% of Rs. 58,50,000.
(iii) In case he has not opted for the presumptive taxation scheme u/s 44AD, & claims that his income is Rs.
2,90,000 (which is lower than the presumptive business income of Rs. 3,51,000), he has to maintain books of

DT QUESTION BANK BY CA PRANAV CHANDAK Subscribe us on YouTube 90


account as required u/s 44AA & also get them audited & furnish a report of such audit u/s 44AB, since his total
income > BEL of Rs. 2,50,000. & he is not eligible to claim the benefit of presumptive taxation for 5 AYs.
(iv) In case he opts for the presumptive taxation scheme u/s 44AD, due date would be 31st July, 2019.
If he has not opted for presumptive scheme & claims that his income is Rs. 2,90,000, then he has to get his books
of account audited u/s 44AB, in that case the due date for filing of return would be 30 th September, 2019.

Q42. Mr. Subramany is engaged in the business of producing & selling toys. During PY 2018-19, his turnover was
Rs. 1.80 crores. He opted for paying tax as per presumptive taxation scheme laid down in section 44AD. He has no
other income during the previous year. Is he liable to pay advance tax & if so, what is the minimum amount of
advance tax to be paid & the due date for payment of such advance tax? [Nov - 17]
Answer:
▪ As per section 211, An assessee who declares income u/s 44AD, 100% Advance tax shall be paid on or before
15th March of the financial year.
▪ Thus Mr. Subramany is liable to pay 100% advance tax in 1 instalment on or before 15th March 2019.
Income under the head business/profession
As per section 44AD, Business Income = 8% of Turnover = 8% x 1,80,00,000 = 14,40,000
Computation of Tax Liability: Tax on Rs. 14,40,000 at slab rate = 2,44,500 + 4 % HEC = Rs. 2,54,280
Thus, Advance Tax Payable before 15th March 2019 = Rs. 2,54,280

Q43. Mr. X had 4 heavy goods vehicles having gross vehicle weight of 15 ton each as on 01.04.2018. He acquired 7
heavy goods vehicles having gross vehicle weight of 14 ton each on 27.06.2018. He sold 2 heavy goods vehicles
having gross vehicle weight of 15 ton each on 31.05.2018.
He has brought forward business loss of Rs. 50,000 relating to AY 2015-16 of a discontinued business. Compute
his Total Income for AY 2019-20 if he opts for presumptive taxation of income u/s 44AE [MAY-2011]
Solution: Computation of Total Income of Mr. X for AY 2019-20
Presumptive business income u/s 44AE
2 heavy goods vehicles for 2 months (2 x 15 x Rs. 1,000 x 2) 60,000
2 heavy goods vehicles for 12 months (2 x 15 x Rs. 1,000 x 12) 3,60,000
7 heavy goods vehicles for 10 months (7 x 14 x 1,000 x 10) 9,80,000
Business Income 14,00,000
Less: Brought forward business loss of discontinued business (50,000)
Gross Total Income 13,50,000
Less: Deduction u/s 80C to 80U Nil
Total Income 13,50,000

Q44. M/s. Ram Ltd. purchased an asset worth US $ 1 Million from Germany. The asset was delivered in India on
1.7.2017. The sum was paid in 4 equal instalments, each falling due on a 6 months interval starting from the date
on which the asset was delivered in India. The exchange rates prevailing on each of the instalments are Rs. 63.56,
Rs. 63.89, Rs. 64.01 &Rs. 63.15 respectively. The asset was put to use on 10.10.2017. Compute the cost of asset &
the depreciation allowable for AY 2018-19 & AY 2019-20.
Solution: Computation of cost of Asset & depreciation allowable of M/s. Ram Ltd.
Particulars Rs.
Purchase cost – US$1 Million x Rs. 63.56 6,35,60,000
Add: Forex variation (US$ 0.25 M x (Rs. 63.89-Rs.63.56) on payment made on 01.01.17 82,500
6,36,42,500
Less: Depreciation (Less than 180 180days) – (6,36,42,500 x 15 % x 50 %) + (6,36,42,500 x
20% x 50%) (less than 180days) [Refer Note] 1,11,37,438
WDV as on 31.03.2017 5,25,05,062

Computation of cost of Asset & depreciation allowable of M/s. Ram Ltd.


Particulars Rs.
Opening WDV as on 01.04.2017 5,25,05,062
Add: Forex variation [Payment on 1.7.2017 (US$ 0.25 M x (Rs. 64.01 - Rs. 63.56)] 1,12,500
5,26,17,562
Less: Forex variation [Payment on 1.1.2018 – (US$ 0.255 M x (Rs. 63.15 - Rs. 63.56)] 1,02,500
Less: Depreciation (entire year) – (Rs. 5,25,15,062 x 15 %) + (6,36,42,500 x 20% x 50%) 1,42,41,509
WDV as on 31.03.2018 3,82,73,553
Note: It is assumed that Ram Ltd. is engaged in the business of manufacture or production of an article or thing or
in the business of generation, transmission or distribution of power & eligible for additional depreciation.

DT QUESTION BANK BY CA PRANAV CHANDAK Subscribe us on YouTube 91


Q45. Explain the provision regarding the taxability of LLP under the Income-Tax Act, 1961.
Answer:
(i) The taxation scheme of LLPs in the Income-tax Act, 1961 is on the same lines as applicable for general
partnerships, i.e. tax liability would be attracted in the hands of the LLP & tax exemption would be available to the
partners.
Therefore, the same tax treatment would be applicable for both general partnerships & LLPs.
(ii) The rate of income-tax applicable to LLPs is the same as the rate applicable for firms i.e. 30% of total income.
(iii) The provisions of section 40(b) requiring payment of remuneration only to working partner in accordance
with the terms of the partnership deed for a period commencing on or after the date of the partnership deed, would
apply to LLPs as well. Further, disallowance of interest in excess of 12% per annum & salary exceeding the
prescribed limits would also be applicable in the case of LLPs.
(iv) However, whereas a partnership firm can opt for presumptive taxation scheme u/s 44AD, an LLP cannot opt
for such scheme.

Q46. State the head of income in the following cases under which the receipt is to be assessed & comment.
(A) A uses his property for his own business. Can he claim depreciation?
(B) B Lets out his property to X. X sublets it. How is sub-letting to be assessed in the hands of X?
(C) C has built a house on a leasehold land. He has let out the property & claims the rent as income from house
property & deducted expenses on repairs, security charges, insurance & collection charges totaling to 40% of
receipts.
Answer:
(A) Yes, Mr. A can claim depreciation, since the property is an asset used for business purposes. Section 22, which
is the charging section for “Income from house property” specifically excludes from its scope, property which an
assessee, as an owner, occupies for the purpose of any business or profession carried on by him.
(B) In the hands of Mr. B, income from letting out of property to Mr. X is chargeable u/h “Income from house
property”, since Mr. B is the owner of the property.
However, since Mr. X is not the owner of the house property, the income from sub-letting will not be chargeable
u/h “Income from house property”. It will be assessed as “Income from other sources” in the hands of Mr. X.
Alternate Answer: It would be assessed u/g PGBP if X is engaged in the business of sub-letting.
(C) Income from letting out of a house built on leasehold land is taxable u/h “Income from house property” in the
hands of Mr. C. Ownership of land is not a pre-requisite for charge of income under the head “Income from house
property”.
The annual letting value (higher of municipal value & fair rent, but restricted to standard rent) or actual rent,
whichever is higher, would be the gross annual value of the house property. The municipal taxes paid by Mr. C
during the year is to be deducted from gross annual value to arrive at the net annual value. Deduction of 30% of
net annual value is allowable u/s 24 to arrive at the income chargeable under the head “Income from house
property”. No other deduction is allowable in respect of repairs, insurance, security & collection charges.

Note - Interest on loan borrowed for construction of house is deductible u/s 24(b) from Net Annual Value to arrive
at the Income from house property, in addition to the statutory deduction of 30%.

Q47. Mr. X engaged in various types of activities gives the following information for PY 2018-19.
Loss from automobile business (Total turnover Rs. 6,00,000) 1,10,000
Profit from wholesale trade in furniture items @ Prescribed % of turnover as per section 44AD 4,00,000
Brought forward loss relating to discontinued textile business (discontinued w.e.f. 01.06.2012) 2,00,000
Short term capital loss on sale of vacant site during the year 70,000
Profit from speculation business related to oil seeds 1,10,000
Loss from speculation business brought forward & related to cotton (b/f from AY 2018-19) 50,000
Brought forward unabsorbed depreciation of trade in furniture items related to AY 2018-19 60,000
Compute the Total Income of Mr. X for the Assessment Year 2019-20.
Solution: Computation of Total Income of Mr. X for AY 2019-20
Income under the head Business/Profession
Income from wholesale trade in furniture 4,00,000
Less: Loss from automobile business (current year) (1,10,000)
Less: Brought forward loss relating to discontinued textile business (P.Y. 2012-13) (2,00,000)
Income from wholesale trade in furniture 90,000
Income from Speculation Business 1,10,000
Less: Loss from speculation business brought forward & related to cotton (50,000)
Income from Speculation Business 60,000
Income under the head Business/Profession 1,50,000
Gross Total Income 1,50,000

DT QUESTION BANK BY CA PRANAV CHANDAK Subscribe us on YouTube 92


Note:
(i) Brought forward unabsorbed depreciation of trade in furniture items related to AY 2018-19 is not allowed to
be adjusted.
(ii) STCL on sale of vacant site of Rs. 70,000 to be carried forward for 8 years starting from AY 2020-21.

Q48. Mr. X a resident individual & practicing CA furnishes the following for the Financial Year 2018-19.
Receipts & Payments Account
Receipts Rs. Payments Rs.
Opening balance (1.4.2018) 2,000 Staff salary, bonus & stipend to articled 1,50,000
Cash in hand & at Bank clerks
Fee from professional services 9,38,000 Other administrative expenses 48,000
Rent 60,000 Office rent 30,000
Motor car loan from Canara Bank (@ 2,50,000 Housing loan repaid to SBI (includes 1,88,000
9% per annum) interest of Rs. 88,000)
Life insurance premium 24,000
Motor car (acquired in Jan. 2019) 4,25,000
Medical insurance premium (for self & 18,000
wife) paid by cheque
Books bought on 20.04.2018 20,000
Computer acquired on 01.11.2018 & put 30,000
to use on same date (for professional
use)
Domestic drawings 2,72,000
Public provident fund subscription 20,000
Motor car maintenance 10,000
Closing balance (31.3.2019) Cash + 15,000
Bank
12,50,000 12,50,000
Additional Information:
(1) He occupies 50% of the building for own residence & let out the balance for residential use at a monthly rent
of Rs. 5,000. The building was constructed during the year 1998-99.
(2) Motor car was put to use both for official & personal purpose. One-fifth of the motor car use is for personal
purpose. No car loan interest was paid during the year.
(3) The written down value of assets as on 01.04.2018 are given below:
Furniture & fittings Rs. 60,000
Plant & Machinery (Air-conditioners, Photocopiers, etc.) Rs. 80,000
Computers Rs. 50,000
Mr. X has not opted for presumptive taxation of Income u/s 44ADA. Compute Total Income Mr. X for AY 2019-20.
Note: Mr. X follows regularly the cash system of accounting.

Solution: Computation of Total Income of Mr. X for AY 2019-20


Income from house property
Self-occupied
Annual value Nil
Less: Deduction u/s 24(b)
Interest on housing loan 50% of Rs. 88,000 = 44,000 but limited to (30,000)
Loss from self-occupied property (30,000.00)
Let out property
Gross Annual value 60,000.00
Less: Municipal Tax Nil
Net Annual Value 60,000.00
Less: Deductions u/s 24
(a) 30% of Net Annual Value (18,000.00)
(b) Interest on housing loan (50% of Rs. 88,000) (44,000.00)
Loss from let out house property (2,000.00)
Total loss under the head House Property (32,000.00)
Profits & gains of business or profession
Fees from professional services 9,38,000.00
Less: Expenses
Staff salary, bonus & stipend (1,50,000.00)

DT QUESTION BANK BY CA PRANAV CHANDAK Subscribe us on YouTube 93


Other administrative expenses (48,000.00)
Office rent (30,000.00)
Motor car maintenance (10,000 x 4/5) (8,000.00)
Car loan interest - not allowable (since the same has Nil
not been paid & the assessee follows cash system of accounting)
Depreciation on Motor car Rs. 4,25,000 x 7.5% x 4/5 (25,500.00)
Depreciation on Books @ 40% on Rs. 20,000 (8,000.00)
Depreciation on Furniture & fittings @ 10% of Rs. 60,000 (6,000.00)
Depreciation on Plant & machinery @ 15% of Rs. 80,000 (12,000.00)
Depreciation on Computer @ 40% of Rs. 50,000 (20,000.00)
Depreciation on Computer (New) Rs. 30,000 @ 40% x 'A thereon (6,000.00)
Income under the head Business/Profession 6,24,500.00
Gross Total Income 5,92,500.00
Less: Deduction under Chapter VI-A Section 80C
Housing loan principal repayment (1,00,000.00)
PPF subscription (20,000.00)
Life insurance premium (24,000.00)
Section 80D
Medical insurance premium paid (18,000.00)
Total Income 4,30,500.00

Q49. Mr. Vidyasagar, Resident Individual aged 64, is a Partner in Oscar Musicals & Co, a Partnership Firm. He also
runs a wholesale business in medical products. The following details are made available for PY 2018-19:
Particulars Rs. Rs.
1. Interest on Capital received from Oscar Musicals & Co at 15% 1,50,000
2. Interest from Bank on Fixed Deposit (Net of TDS Rs. 1,500) 13,500
3. I.T. Refund received relating to AY2017-2018 including interest of Rs. 2,300 34,500
4. Net Profit from Wholesale Business 5,60,000
Amount debited include the following:
Depreciation as per Books 34,000
Motor Car Expenses 40,000
Municipal Taxes for the Shop (For two half years, payment for one half year made on 7,000
12.06.2019&for other, on 14.11.2019)
Salary to Manager for whom single Cash Payment was made for 21,000
5. WDV of Assets (as on 01.04.2018 used in above business is as under:
Computers 1,20,000
Motor Car (20% used for personal use) 3,20,000
6. LIP paid for major son 60,000
PPF of his wife 70,000
NSC 30,000
Compute the Total Income of the Assessee for the Assessment Year 2019-2020. The computation should show the
proper heads of Income. Also compute the WDV of the different Blocks of Assets as on 31.03.2019. [M11]
Solution: Computation of Total Income

Particulars Rs. Rs.


Net profit as per Profit & Loss 5,60,000
Add: Depreciation as per books 34,000
Add: Depreciation as per IT act 7,000
Add: Salary to manager 21,000
Add: Interest on capital upto 12% taxable (1,50,000 x 12%/15%) 1,20,000
Add: Motor car expense (20% for personal purpose disallowed) 8,000
Less: Municipal tax (86,400 + 3,500) (89,900) 1,00,100
Profits & Gains from Business or Profession 6,60,100

DT QUESTION BANK BY CA PRANAV CHANDAK Subscribe us on YouTube 94


Income from Other Sources: Interest from Fixed Deposits (13,500 + 1,500) 15,000
Interest on I.T Refund 2,300 17,300
Gross Total Income 6,77,400
Less: Deductions under Chapter VI-A
(i) U/s 80C - LIP for Major Son – irrespective of dependent or not: 60,000
(ii) U/s 80C - NSC: 30,000
(iii) U/s 80C - Contribution to PPF for his wife 70,000 Restricted to Rs. 1,50,000 (1,50,000)
Total Income 5,27,400

Working Notes:
1. Computation of WDV
Particulars Computer (40%) Motor Car (15%)
Opening WDV as on 01.04.2018 1,20,000 3,20,000
Less: Current Year Depreciation (1,20,000 x 40%) = 48,000 (3,20,000 x 15%) = 48,000
Closing WDV as on 31.03.2019 72,000 2,72,000
Total Depreciation for the year = Rs. 48,000 (computer)+ 80% of Rs. 48,000 (Car)= Rs. 86,400. 20% of Depreciation
on Motor Car used for Personal Purpose is not allowed as deduction.
2. Payment made in cash: U/s 40A(3), where an Assessee incurs any expenditure in respect of which aggregate of
payments on a single day in excess of Rs. 10,000 is made, otherwise than by an Account Payee Cheque drawn on a
Bank or an Account Payee Bank Draft, whole of such expenditure shall not be allowed as a deduction. In the given
case, salary payment of Rs. 21,000 to Manager is made in cash, hence the entire amount is not allowed as a
deduction.
3. Income Tax Refund: It is not income u/h “PGBP”. Interest on Income Tax Refund is taxable u/h “IFOS”.

Q50. Mr. Y carries on his Own Business. An analysis of his Trading & Profit & Loss for PY 2018-19 revealed the
following information:
(a) The Net Profit was Rs. 1,11,20,000.
(b) The following incomes were credited in the Profit & Loss Account:
(a) Dividend from UTI Rs. 2,20,000.
(b) Interest on Debentures Rs. 1,75,000.
(c) Winnings from Races Rs. 1,50,000
(c) It was found that some Stocks were omitted to be included in both the Opening & Closing Stocks, the value of
which were: Opening Stock: Rs. 80,000 & Closing Stock: Rs. 1,20,000
(d) Rs. 10 lacs was debited in P&L Acc being contribution to a University approved & notified u/s 35(1)(ii).
(e) Salary includes Rs. 2,00,000 paid to his brother which is unreasonable to the extent of Rs. 25,000.
(f) Advertisement Expenses include 15 packets of dry fruits of 10,000 per piece presented to important Customers.
(g) Total Expenses on Car = 7,80,000. Car was used for Business & Personal purposes. 3/4th is for Business.
(h) Miscellaneous Expenses included 30,000 paid to A & Co., a Goods Transport Operator in Cash on 31.1.2019 for
distribution of the Company's Product to the Warehouse.
(i) Depreciation debited in the Books was Rs. 5,50,000. Depreciation allowed as per IT Rules was Rs. 7,50,000.
(j) Drawing Rs. 1,00,000.
(k) Investment in NSC Rs. 1,50,000.
Compute the Total Income of Mr. Y for AY 2019-2020. [M12]
Solution: Computation of Taxable Income & Tax Liability
1. Profits & Gains of Business or Profession
Net Profit as per Profit & Loss A/c 1,11,20,000
Less: Incomes taxable under other head or Exempt Incomes
Dividend Received from UTI (Exempt) (2,20,000)
Interest on Debentures (considered u/h “IFOS”) (1,75,000)
Winning from Races (considered u/h “IFOS”) (1,50,000) (5,45,000)
Less: Undervalued Opening Stock (Earlier omitted, to be included) (80,000) (80,000)
Add: Closing Stock (Earlier omitted, to be included) 1,20,000 1,20,000
Add: Indamissible expenses under PGBP
Amount paid to Approved University u/s 35(1)(ii) 10,00,000
[Salary to Brother [unreasonable amount disallowed u/s 40A(2)] 25,000

DT QUESTION BANK BY CA PRANAV CHANDAK Subscribe us on YouTube 95


Car Expenses disallowed to the extent of Personal use (7,80,000 x 1/4) 1,95,000
Depreciation as per Books of A/c 5,50,000
Drawings disallowed being personal in Nature 1,00,000
Investment in NSC 1,50,000 20,20,000
Less: Admissible Expenses: Depreciation as per IT Act (7,50,000) (7,50,000)
Less: Deduction u/s 35(1)(ii) [150% of Rs. 10 Lacs] (15,00,000) (15,00,000)
Income under the head “PGBP” 1,03,85,000
2. Income from Other Sources
(i) Dividend from UTI – Exempt u/s 10(35) Nil
(ii) Interest on Debentures 1,75,000
(iii) Winnings from Races 1,50,000 3,25,000
Gross Total Income 1,07,10,000
Less: Deduction under Chapter VI-A: U/s 80C – Investment in NSC (1,50,000)
Total Income 1,05,60,000

Notes:
1. As per Sec. 40A(3), any payment in Cash made to any Transporter for the purpose of plying, hiring or leasing
goods carriage shall be disallowed if it exceeds Rs. 35,000 on any day. In the given case, assessee makes a payment
of Rs. 30,000 in cash to A & Co., a Transport Operator on 31.01.2019. It is an allowable expenditure.
2. Advertisement Expense is incurred for purpose of Business & hence allowed u/s 37. So, no adjustment is
required to be made for the same.

Q51. Mr. Raghuveer, a Resident Individual aged 35 years, furnished the following information from his
Profit & Loss Account for the year ended 31st March 2019:
(i) The Net Profit was Rs. 6,50,000.
(ii) The following Incomes were credited in the Profit & Loss Account:
(a) Interest on Government Securities Rs. 25,000.
(b) Dividend from a Foreign Company Rs. 18,000.
(c) Gold Coins worth Rs. 55,000 received as Gift from his father.
(iii) Depreciation debited in books of a/c was Rs. 85,000. Depreciation allowed as per IT Act, 1961 was Rs. 96,000.
(iv) Interest on Loan amounting to Rs. 68,000 was paid in respect of Capital borrowed for purchase of New Asset
which has not been put to use till 31st March 2019.
(v) General Expenses included:
(a) An expenditure of Rs. 20,500 which was paid by a Bearer Cheque.
(b) Compensation of Rs. 4,500 paid to an Employee while terminating his services in Business Unit.
(vi) He contributed the following amounts by Cheque:
(a) Rs. 45,000 in Sukanya Samridhi Scheme in the name of his minor daughter Alpa.
(b) Rs. 20,000 to the Swachh Bharat Kosh set up by the Central Government.
(c) Rs. 28,000 towards Premium for Health Insurance & Rs. 2,500 on account of Preventive Health Check up
for Self & his wife.
(d) Rs. 35,000 on account of Medical Expenses of his father aged 82 years (no Insurance Scheme had been
availed on the health of his father).
You are required to compute the Total Income of Mr. Raghuveer. [N 16]
Solution
Computation of Taxable Income & Tax Liability
Particulars Rs.
1. Profits & Gains of Business or Profession
Net Profit as per Profit & Loss A/c 6,50,000
Less: Income taxable under other heads or Exempt Incomes
Interest on Government Securities (considered u/h ’”IFOS”) (25,000)
Dividend from Foreign Company (considered u/h “IFOS”) (18,000)
Gold Coins from Father (considered u/h ’”IFOS”) (55,000) (98,000)
Add: Inadmissible Expenses u/h “PGBP”
Depreciation as per Books of A/c 85,000

DT QUESTION BANK BY CA PRANAV CHANDAK Subscribe us on YouTube 96


Interest on Loan [Disallowed u/s 36(1)(iii)] [See Note 1] 68,000
General Expenses [Disallowed u/s 40A(3)] [See Note 2] 20,500 1,73,500
Less: Depreciation as per IT Act 96,000 (96,000)
Income under the head “PGBP” 6,29,500
2. Income from Other Sources
Interest on Government Securities 25,000
Dividend from Foreign Company [See Note 3] 18,000
Gold Coin from Father (not taxable since, it is received from Relative) [Note 4] Nil 43,000
Gross Total Income 6,72,500
Less: Deduction under Chapter VI-A
(a) Sukanya Samridhi Scheme [u/s 80C] (45,000)
(b) For Assessee & Spouse (Assuming Premium Payment by A/c Payee Cheque) (25,000)
[Preventive Checkup is also included within the overall limit of Rs. 25,000] – 80D
(c) Medical Expense of Father (82 Yr) (assumed to be paid by A/c Payee Cheque)- 80D (35,000)
(d) Contribution to Swachh Bharat Kosh [100% allowed without restriction] – 80G (20,000) (1,25,000)
Total Income 5,47,500
Notes:
1. As per Sec. 36(1)(iii), Interest on Capital borrowed for the purchase of asset, paid from the date on which the
capital was borrowed upto the date such asset was first put to use, shall not be allowed as a deduction.
2. As per Sec.40A(3), Expenditure in respect of which aggregate payments made to a person in a day, in excess of
Rs. 10,000, made otherwise than by way of Account Payee Cheque / Demand Draft is disallowed in full.
3. Dividend from Indian/Domestic Companies exempt from tax u/s 10(34). But Dividend from Foreign Company
is Taxable under the Head “Income from Other Sources”.
4. As per Sec.56, Gift received from a Relative (Father) is exempt from tax.
5. Compensation to Employees for Termination is incurred for business & thus allowed u/s 37.

Q52. Mr. Rajiv (age 50) Resident Individual & Practicing CA gives you Receipts & Payments A/c for PY 2018-19:
Receipts Rs. Payments Rs.
Op. Bal – Cash in Hand & Bank 12,000 Staff Salary, Bonus & Stipend to Articled 1,50,000
Clerks
Fee from Professional Services 9,38,000 Other Administrative Expenses 48,000
Car Loan @ 9% p.a from Bank 2,50,000 Office Rent 30,000
Rent 50,000 Housing Loan repaid to SBI (Inch Int. of 1,88,000
Rs. 88,000)
Life Insurance Premium 24,000
Motor Car (acquired in January 2018) 4,25,000
Medical Insurance Premium (for Self & 18,000
Wife)
Books bought of Annual Publications 20,000
Computer acquired on 01.11.2018 30,000
(Professional use)
Domestic Drawings 2,72,000
Public Provident Fund subscription 20,000
Motor Car Maintenance 10,000
Cl. Bal – Cash in Hand & Cash at Bank 15,000
Following further information is given to you:
(1) He occupies 50% of the building for own residence & let-out the balance for residential use at a monthly rent
of Rs. 5,000. The building was constructed during the year 1998-1999.
(2) Motor Car was put to use both for Official & Personal purpose. One-Fifth of the Motor Car use is for personal
purpose. No Car Loan Interest was paid during the year.
(3) WDV of Assets on 1.4.2018 are: F&F: 60,000; P&M (AC, Photocopiers): 80,000; Computers: 50,000.
Mr. Rajiv follows regularly Cash System of Accounting. Compute the Total Income of for the AY 2019-2020. [M 11]
Solution: Computation of Taxable Income & Tax Liability
Particulars Rs.
1. Income from House Property [WN 1] (39,000)
2. Profits & Gains from Business or Profession (9,38,000 – 3,38,500) 6,24,500

DT QUESTION BANK BY CA PRANAV CHANDAK Subscribe us on YouTube 97


Particulars Deduction Addition
Fees from Professional Services 9,38,000
Staff Salary, Bonus & Stipend paid to Articled Clerks 1,50,000
Other Administrative Expenses 48,000
Office Rent 30,000
Depreciation [W.N. 2) 77,500
Motor Car Maintenance [10,000 x 4/5] 8,000
Total 3,13,500 9,38,000
Gross Total Income 5,85,500
Less: Deductions under Chapter VI-A [WN 3] (1,62,000)
Total Income 4,23,500
Tax payable = ( 4,23,500 – Rs. 2,50,000) x 5% 8,675
Add: HEC@ 4% 347
Net tax liability (Rounded off) 9,000

Working Notes:
1. Income from House Property:
Nature: 50% Self Occupied
i. Annual Value u/s 23(2) Nil
Less: Deduction u/s 24: Interest on Borrowed Capital (88,000/2 = 44,000) (30,000) (30,000)
(Since Construction is prior to 01.04.1999, Maximum Interest allowable is Rs. 30,000)
Nature: 50% Let-Out 50,000
Annual Value u/s 23(1)(a)/(b) = Actual Rent (5,000 x 12 = 60,000. It is assumed that the
House Property is vacant for 2 months. Therefore owing to vacancy the Annual Value is
reduced to Rs. 50,000)
Less: Municipal Taxes NIL
Net Annual Value 50,000
Less: Deduction u/s 24(a) 30% of Net Annual Value (15,000)
Less: Deduction u/s 24(b) Interest on borrowed Capital ( Rs. 88,000 2) (44,000) (9,000)
Net Income from House Property (39,000)

2. Computation of Depreciation:
Particulars Rs.
(a) Books bought of Annual Publication at 40% ( Rs. 20,000 x 40%) 8,000
(b) Furniture & Fitting at 10% ( Rs. 60,000 * 10%) 6,000
(c) Plant & Machinery at 15% ( Rs. 80,000 x 15%) 12,000
(d) Computers: Opening 50,000 at 40% (+) Additions 30,000 (Less than 180 days) at 50% of 40%. 26,000
(e) Motor Car at 15% (Less than 180 days). Therefore 4,25,000 * 15% x 50% x 4/5 for Official Use 25,500
Total 77,500

3. Deduction under Chapter VI-A:


Particulars Rs. Rs.
(a) U/s 80C: Housing Loan Repayment (Principal Portion = 1,88,000 - 88,000) 1,00,000
Life Insurance Premium 24,000
PPF 20,000 1,44,000
(b) u/s 80D: Lower of Medical Insurance Premium of Rs. 18,000, or Rs. 25,000 18,000
Total 1,62,000
4. Motor Car Loan is not considered since it is a Capital Receipt. Interest is not paid during the year. Since the
Assessee follows Cash Basis of Accounting, it is not considered in the Profits & Gains of Business or Profession.
5. Domestic Drawings is not considered since it is a Personal Expenditure.

DT QUESTION BANK BY CA PRANAV CHANDAK Subscribe us on YouTube 98


4D. CAPITAL GAINS
Q1. Mrs. X, an Individual Resident Woman, wanted to know whether tax is attracted on sale of gold & jewellery
gifted to her by her parent at the occasion of her marriage in the year 2000 which was purchased at a total cost of
Rs. 2,00,000? [NOV 08]
Answer:
▪ Jewellery & Gold being a capital asset as defined u/s 2(14) will be liable to capital gains.
▪ Since Mrs. X has acquired the property by way of Gift before 01.04.2001, cost of acquisition shall be higher of
(i) Cost of Purchase of the Jewellery by the Previous Owner (or) (ii) FMV of the Asset on 1.4.2001.
▪ As the asset is a Long-Term Capital Asset & is acquired before 01.04.2001, the Assessee shall be eligible for
Indexation Benefit as applicable to the Previous Year 2001-2002, i.e. 100.

Q2. A land was purchased in April 2011 & the building was constructed on it during 2017-2018. The building was
sold in June 2018. Is the Capital Gain arising out of sale, Long-Term or Short-Term Capital Gain?
Answer:
▪ Land & Building are two separate Capital Assets for the purpose of computation of Capital Gains.
▪ So, composite consideration received for Land & Building should be apportioned b/w Land & Superstructure.
▪ In the instant case, Capital Gain arising out of Land will be LTCG (since Holding Period is > 24months).
▪ Capital Gain arising out of super-structure will be STCG (since Holding Period is < 24 months).

Q3. Mr. X sold a house, held as a capital asset, to his friend Mr. Y on 1.12.2018 for 25 lacs. SDV was 45 lacs. Mr. X
preferred an appeal to Revenue Divisional Officer, who fixed the value of the house @ 35 lacs (22 lacs for land &
balance for building portion). Differential stamp duty was paid, accepting the said value determined. Mr. X had
purchased land on 1.62006 for 5,19,000 & completed the construction of the house on January, 2017 for 14 lacs.
Compute the capital gains chargeable to tax. [CII for FY 2006-07: 122; & FY 2018-19: 280] [NOV-13]
Solution:
▪ As per section 50C, FVC shall be taken as Rs. 22 lacs for land & 13 lacs for building.
▪ In the given problem, land has been held for > 24 months & building for a period less than 24 months.
▪ Therefore, land is a long-term capital asset, whereas building is a short-term capital asset.
Computation of Capital Gains chargeable to tax
Particulars Rs. Rs.
(i) Long term capital gain on sale of land
Full Value of consideration 22,00,000
Less: Indexed COA [Rs. 5,19,000 /122 x 280] (11,91,148)
Long-term capital gain on sale of Land 10,08,852
(ii) Short-term capital loss on sale of building
Full Value of consideration 13,00,000
Less: COA (14,00,000)
Short term capital loss (B) (1,00,000)
Taxable Long-term capital gain (A - B) 9,08,852

Q4. Mr. X (62 years old), pledged his residential house to a bank under a notified reverse mortgage scheme. He
was getting loan from bank in monthly instalments. Mr. X did not repay the loan on maturity & hence gave
possession of the house to the bank to discharge his loan. How will the treatment of long-term capital gain be on
such reverse mortgage transaction? or Explain Reverse Mortgage.
Answer:
▪ As per section 47, reverse mortgage shall not be considered to be transfer for the purpose of capital gain. Under
reverse mortgage, a senior citizen can mortgage his house property to the bank & the bank shall grant a loan
against the security of house property & such loan shall be given in monthly installments & the amount so
received shall not be considered to be income of the mortgagor u/s 10(43).
▪ The purpose of the scheme is to make available regular amount to the persons who do not have regular income
but are the owners of the house property.
▪ In general, the mortgagors repay the loan in installments but in this case mortgagee i.e. bank is paying
installment to the mortgagor & hence it is called reverse mortgage.
▪ After the death of the mortgagor the bank shall have right to sell off the property & shall adjust loan & interest
& shall compute capital gains for the deceased person & shall pay tax to the government.

Q5. Examine, with reasons, whether the following statements are True or False:
(i) Alienation of a residential house in a transaction of reverse mortgage under a scheme made & notified by the
Central Government is treated as “transfer” for the purpose of capital gains.

DT QUESTION BANK BY CA PRANAV CHANDAK Subscribe us on YouTube 99


(ii) ZCBs means a bond on which no payment & benefits are received or receivable before maturity or redemption.
(iii) Zero coupon bonds of eligible corporation, held for more than 12 months, will be LTCAs.
(iv) Where an urban agricultural land owned by an individual, continuously used by him for agricultural purposes
for a period of two years prior to the date of transfer, is compulsorily acquired under law & the compensation
is fixed by the State Government, resultant capital gain is exempt.
Answer:
(i) False: As per section 47(xvi), such alienation in a transaction of reverse mortgage under a scheme made &
notified by the Central Government is not regarded as “transfer” for the purpose of capital gains.
(ii) True: As per section 2(48), ‘Zero Coupon Bond’ means a bond issued by any infrastructure capital company
or infrastructure capital fund or a public sector company, or Scheduled Bank on or after 1st June 2005, in
respect of which no payment & benefit is received or receivable before maturity or redemption from such
issuing entity & which the Central Government may notify in this behalf.
(iii) True: Section 2(42A) defines the term ‘short-term capital asset’. Under the proviso to section 2(42A), zero
coupon bond held for not more than 12 months will be treated as a short-term capital asset. Consequently,
such bond held for more than 12 months will be a LTCA.
(iv) False: As per section 10(37), where an individual owns urban agricultural land which has been used for
agricultural purposes for a period of two years immediately preceding the date of transfer, & the same is
compulsorily acquired under any law & the compensation is determined or approved by the Central
Government or the Reserve Bank of India, resultant capital gain will be exempt.

Q6. Mr. X purchased 2,000 equity shares of ABC Ltd. (a listed company) on 01.04.2014 at Rs. 20 per share. He sold
all the shares on 01.06.2018 at Rs. 50 per share. He also had to pay securities transaction tax (STT) on the same.
Explain the taxability in the hands of Mr. X in the year of transfer i.e. AY 2019-20.
Answer: In the given problem, since the listed equity shares of ABC Ltd. are being sold after 12 months & also STT
has been paid, it is exempt upto Rs. 1,00,000 u/s 112A & excess over Rs. 1,00,000 is taxable @ 10%.

Q7. Will your answer be different if these shares were preference shares & not equity shares? Explain.
Answer: Since section 112A is not applicable to preference shares, capital gains shall be taxable @ 20%.

Q8. Will you answer be different if these shares were not listed in a recognised stock exchange? Explain.
Answer: Since the shares are not listed, section 112A is not applicable, capital gains shall be taxable @ 20%.

Q9. Capital gain of Rs. 75 lacs arising from transfer of long-term capital assets will be exempt from tax if such
capital gain is invested in the bonds redeemable after three years, issued by NHAI u/s 54EC of the Act. [NOV-2008]
Answer. The statement is false.
▪ Exemption u/s 54EC has been restricted, by limiting the maximum investment in long term specified assets (i.e.
bonds of NHAI or RECL, redeemable after 5 years) to Rs. 50 Lac during any financial year.
▪ Therefore, in this case, the exemption u/s 54EC can be availed only to the extent of Rs. 50 Lac.

Q10. What are the circumstances under which the Assessing Officer can make reference to the Valuation Officer
u/s 55A of the Income Tax Act, 1961?
Answer: Refer page no. 131 in the “Concept Book”

Q11. Compute capital gains of Mr. X in the following Individual situations for the AY 2019-20
Asset Gold Land Residential House
Date of purchase 01.07.1990 01.04.1992 01.07.1994
Cost price 4,00,000 6,00,000 8,00,000
Cost of improvement 1,00,000 2,00,000 4,00,000
Year of improvement 1999-2000 2000-01 2005-06
Fair market value on 01.04.2001 30,00,000 60,00,000 5,00,000
Date of Sale 01.01.2019 01.01.2019 01.01.2019
Full value of consideration 200,00,000 300,00,000 400,00,000
Solution:
Asset Gold Land Residential House
Full value of consideration 200,00,000 300,00,000 400,00,000
Less: Indexed cost of acquisition 84,00,000 (168,00,000) (22,40,000)
(30,00,000/100 x 280) (60,00,000/100 x 280) (8,00,000/100 x 280)
Less: Indexed cost of improvement - - 9,57,265
(4,00,000/117 x 280)
Long term capital gain 116,00,000 132,00,000 368,02,735

DT QUESTION BANK BY CA PRANAV CHANDAK Subscribe us on YouTube 100


Q12. Mrs. Padmini owned two motor-cars, which were mainly used for business purposes. The Written Down
Value on 01.04.2018 of the Block of Assets comprising of only these two cars, both of which were purchased in
May 2008, was Rs.1,81,000. These two cars were sold in June 2018, for X 1.50.000. In February 2019, she sold
1,000 Shares in X Ltd (unlisted), an Indian Company, for Rs. 3,50,000. She had purchased the same during
January2017 for Rs. 2,44,000. A House Plot purchased by her in March 2008 for Rs.2,73,000 was sold by her for
Rs. 6,50,000 on 18.01.2019. Compute Capital Gains for the AY 2019-2020. [Nov 93]
Solution: Computation of Capital Gain on Sale of Assets
Particulars Motor cars Share in X Ltd. House plot
Sale Consideration 1,50,000 3,50,000 6,50,000
Less: Expenses on Transfer NIL NIL NIL
Net Sale Consideration 1,50,000 3,50,000 6,50,000
Less: WDV/Indexed Cost of Acquisition (1,81,000) (2,58,788) (5,16,486)
(2,44,000 x 280/264) (2,73,000 × 280/148)
LTCG/STCG/(Loss) (31,000) 91,212 1,33,514
Net Taxable LTCG = Rs. 1,93,726
Notes:
1. Gain/Loss on Sale of Depreciable Assets will be treated as Short Term Capital Gain/Loss Only.
2. Shares in X Ltd, a Financial Asset, is held for more than 24 months & hence is a LTCA.
3.Current Year Short Term Capital Loss can be adjusted against any Capital Gain. (Sec. 70)

Q13. Mr. X owns a plot of land acquired on 01.06.2002 for a consideration of Rs. 2 Lacs. He enters into an
agreement to sell the property on 15.03.2019 for a consideration of Rs. 20 Lacs. In part performance of the
contract, he handed over the possession of land on 21.03.2019 on which date he received the full consideration.
As on 31st March 2019 the sale was not registered. Discuss the liability to capital gain for the AY 2019-20.
Solution: Computation of Capital Gains
Full value of consideration 20,00,000.00
Less: Indexed cost of acquisition = 2,00,000 / 105 x 280 = Rs. 5,33,333.33 (5,33,333.33)
Long Term Capital Gain 14,66,666.67

Q14. Mr. X purchased one house on 01.07.2002 for Rs. 3,50,000. He constructed its first floor on 01.10.2011 by
incurring Rs. 4 lacs & constructed its second floor on 01.10.2012 by incurring Rs. 6,00,000 & third floor on
1.10.2014 by incurring Rs. 7,00,000. Finally, sold the building on 01.01.2019 for Rs. 120,00,000 & selling expenses
were 2% of the sale price. He has deposited Rs. 1,00,000 in NSC. Compute his total income for AY 2019-20.
Solution: Computation of Capital Gains
Full value of consideration 120,00,000
Less: Selling Expenses = 2% of Rs. 120,00,000 (2,40,000)
Net Sale Consideration 1,17,60,000
Less: Indexed cost of acquisition = 3,50,000 / 105 x 280 = Rs. 9,33,333.33 (9,33,333.33)
Less: Indexed COI - Cost of constructing 1st floor = = 4,00,000 / 184 x 280 (6,08,695.65)
Less: Indexed COI - Cost of constructing 2nd floor = 6,00,000 / 200 x 280 (8,40,000)
Less: Indexed COI - Cost of constructing third floor = 7,00,000 / 240 x 280 (8,16,666.67)
Long Term Capital Gain 85,61,304.35
Income under the head Capital Gain (LTCG) 85,61,304.35
Gross Total Income 85,61,304.35
Less: Deduction u/s 80C to 80U Nil
Total Income (Rounded off u/s 288A) 85,61,300

Q15. Dinesh received a vacant site as Gift from his friend in November 2011. The site was acquired by his friend
for Rs. 6,00,000 in April 2006. Dinesh constructed a Residential Building during the year 2013-2014 in the said
site for Rs. 25,00,000. He carried out some Further extension of the construction in PY 2013-14 for Rs. 6,00,000.
Dinesh sold the Residential Building for Rs. 55,00,000 in August 2018 but the State Stamp Valuation Authority
adopted Rs. 65,00,000 as value for the purpose of Stamp Duty. Compute his LTCG for AY 2019-20.
[CII: FY 2006-2007: 122, FY 2011-2012:184, FY 2013-2014: 220, FY 2015-2016: 254, 2018 2019: 280] [M 12]
Solution:
▪ As per section 50C, where Sale Consideration is less than the value adopted or assessed or assessable by the
State Government Authority (referred to as “Stamp Valuation Authority”) for the purpose of payment of Stamp
Duty, Value adopted by the Stamp Valuation Authority is adopted as the Sale Consideration.
▪ Since the property was gifted to Mr. Dinesh, benefit of indexation in respect of Cost of Acquisition can be availed
from the date of original acquisition by his friend (Cost to the Previous Owner). [CIT vs Manjula J. Shah 204
Taxmann 691 (Bom.) & Arun Shungloo Trust Vs. CIT 205 Taxmann 456 (Del.)]

DT QUESTION BANK BY CA PRANAV CHANDAK Subscribe us on YouTube 101


Computation of Taxable Long-Term Capital Gains
Sale Consideration u/s 50C 65,00,000
Less: Indexed Cost of Acquisition = Rs. 6,00,000 × 280/122 (13,77,049)
Less: Indexed Cost of Improvement = Cost of Improvement x CII of year of Transfer / CII of year (13,45,454)
of Improvement = Rs. 25,00,000 x 280 /220 + Rs.6,00,000 x 280 /220 (31,81,818 + 7,63,636)
Long Term Capital Gains 11,77497
Note: For the Buyer of Property at a Consideration below SDV, difference is taxable u/s 56 as “IFOS”.

Q16. Mrs. X transferred a house to her friend Mrs. Y for Rs. 35,00,000 on 01.10.2018. The Sub Registrar valued
the land at Rs. 48,00,000. Mrs. X contested the valuation & the matter was referred to Divisional Revenue Officer,
who valued the house at Rs. 41,00,000. Accepting the said value, differential stamp duty was also paid & the
transferred was completed.
Total income of Mrs. X & Mrs. Y for AY 2019-20, before considering the transfer of said house are Rs. 2,80,000 &
Rs. 3,45,000, respectively. Mrs. X had purchased the house on 15th May 2011 for Rs. 25 lacs & registration expenses
paid at the time of purchase of the house were Rs. 1,50,000. Determine the total income of both Mrs. X & Mrs. Y
taking into account the above said transactions. CII for FY 2011-12: 184 & FY 2018-19: 280] [MAY – 2015]
Solution: Computation of Total Income of Mrs. X for AY 2019-20
Computation of Long-term Capital Gain
Sale consideration as per section 50C of the Act 41,00,000.00
Less: Indexed cost of acquisition = 26,50,000/184 x 280 (40,32,608.70)
Long term capital gain 67,391.30
Other Income 2,80,000.00
Gross Total Income 3,47,391.30
Note: If value by VO > Actual sale consideration but is less than SDV, then FVC = Value by VO.

Computation of Total Income of Mrs. Y for AY 2019-20


Income under the head other sources
Gift (41,00,000 - 35,00,000) 6,00,000.00
Other Income 3,45,000.00
Gross Total Income 9,45,000.00

Q17. Mr. X inherited a house in Jaipur under will of his father in May, 2003. The house was purchased by his father
in Jan 2001 for 2,50,000. FMV of house on 1.4.2001 was Rs. 2,70,000. He invested Rs. 7 lacs in construction of one
more floor in this house in June, 2005. The house was sold by him in November, 2018 for 37.5 lacs. Stamp duty
value was 47,25,000. But as per assessee’s request, AO made a reference to Valuation Officer. Value determined by
Valuation Officer was 47,50,000. Brokerage @ 1% of sale consideration was paid by Mr. X. Compute capital gain
AY 2019-20 [CII: FY 2018-19: 280; FY 2003-04: 109 & FY 2005- 06: 117. [NOV - 2007]
Solution: Computation of Long-term Capital Gain for AY 2019-20
Full Value of consideration (As per section 50C of the Act) 47,25,000
Less: @ 1% of sale consideration of Rs. 37.50 lacs (37,500)
Less: Indexed cost of acquisition = 2,70,000 /100 x 280 (7,56,000)
Less: Indexed cost of improvement = 7,00,000 /117 x 280 (16,75,213.68)
Long term capital gain 22,56,286.32

Q18. Anshu transfers Land & Building on 02.07.2018 & furnishes the following information.
(i) Net Consideration received 18,00,000
(ii) Value adopted by Stamp Valuation Authority 24,00,000
(iii) Value ascertained by Valuation Officer on reference by the Assessing Officer 24,00,000
(iv) This Land was acquired by Anshu on 01.04.2001. Fair Market Value as on 01.04.2001 5,10,000
(v) Building was constructed on Land by Anshu (construction completed during FY 2011-12) Rs. 4,20,000
(vi) Short Term Capital Loss incurred on Sale of Shares during FY 2013- 2014 b/f 50,000
Anshu seeks your advice to the amount to be invested in NHAI Bonds so as to be exempt from Capital Gain Tax
under IT Act. [CII for FY 2001-2002 = 100; CII for FY 2011-2012 = 184; CII for FY 2018-2019 = 280] [Nov 12]
Solution: Computation of Capital Gain
Sale Consideration Received (WN 1) 24,00,000
Less: Indexed COA = (Rs. 5,10,000 × 280/184) [Note: COA = COA to Previous Owner] (14,28,000)
Less: Indexed COI (Rs. 4,20,000 × 280/184) (6,39,130)
Long Term Capital Gains 3,32,870
Less: Short Term Capital Loss (WN 2) (50,000)

DT QUESTION BANK BY CA PRANAV CHANDAK Subscribe us on YouTube 102


Taxale Long Term Capital Gains 2,82,870
Conclusion: Since the Assessee has a LTCG of Rs. 2,82,870, he has to invest the same in NHAI Bonds u/s 54EC to
avail exemption from Capital Gain Tax. The Investment should be made within 6 months from the date of Transfer.
Working Notes:
1. If value by VO > Actual sale consideration but is less than SDV, then FVC = Value by VO.
2. STCL on Sale of Securities & Shares can be set off against STCG or LTCG & Unabsorbed Loss can be carried
forward for 8 AYs. AY 2012-2013 to 2018-2019 is less than 8 years & hence it can be set off.

Q19. Mr. Y bought a vacant Land for Rs. 80 Lacs in May 2004. Registration & other expenses were 10% of the cost
of land. He constructed a residential building on the said land for Rs. 100 Lacs during FY 2006-07.
He entered into an agreement for sale of the above said residential house with Mr. John (not a relative) in April
2016. Sale consideration was fixed at Rs. 700 Lacs & on 23.4.2016, Mr. Y received 20 lacs as advance.
Sale deed was executed & registered on 14.1.2019 for agreed consideration. However, Stamp Duty authority had
revised the values; hence the value of property for stamp duty purposes was Rs. 770 Lacs. Mr. Y paid 1% as
brokerage on sale consideration received. Subsequent to sale, Mr. Y made following investments:
(i) Acquired a residential house at Delhi for Rs. 110 Lacs.
(ii) Acquired a residential house at London for Rs. 190 Lacs.
(iii) Subscribed to NHAI capital gains bond for Rs. 45 Lacs on 29-3-2019 & for 50 Lacs on 12-5-2019.
Compute the income chargeable u/h ‘Capital Gains'. CII: FY 2004-05 = 113; FY 2006-07 = 122; FY 2018-19 = 280
Solution: Computation of Capital Gains of Mr. Y for AY 2019-20
Full value of consideration 770,00,000.00
Less: Indexed cost of acquisition
Indexed cost of land (88,00,000 / 113 x 280) (218,05,309.73)
Indexed cost of building (100,00,000 / 122 x 280) (229,50,819.67)
Less: Brokerage (7,00,000.00)
Long term capital gain 315,43,870.60
Less: Investment in house property section 54 (110,00,000.00)
Less: Investment in NHAI section 54EC (assumed redeemable after 5 years) (50,00,000.00)
Long Term Capital Gains 155,43,870.60

Note:
1. Registration & other expenses paid at the time of purchase shall be part of the cost.
2. SDV on date of actual sale shall be taken as consideration as per section 50C because advance was paid in cash.
3. Maximum Deduction allowed u/s 54EC during a particular previous year shall be Rs. 50,00,000.
4. Residential house purchased in India is eligible for exemption u/s 54. (Residential house purchased outside
India is not eligible for exemption u/s 54.)

Q20. Mr. X (age 55) owned a Residential House in Ghaziabad. It was acquired by Mr. X on 10.10.2006 for 6 lacs. He
sold it for 53 lacs on 4.11.2018. Stamp valuation authority of the state fixed value of the property at 70 Lacs.
Assessee paid 2% of the sale consideration as brokerage on the sale of the said property.
Mr. X Acquired a Residential House property at Kolkata on 10.12.2018 for Rs. 10,00,000 & deposited Rs. 4,00,000
on 10.4.2019 & Rs. 5,00,000 on 15.6.2019 in capital gains bonds of RECL Ltd. He deposited 4 lacs on 6.7.2019 & 3
lacs on 1.11.2019 in capital gain deposit scheme in Nationalized Bank for construction of additional floor on house
property in Kolkata. Compute Capital Gain for AY 2019-20. [CII for FY 2006-07 = 122; [MAY-2014]
Solution: Computation of Capital Gains in the hands of Mr. X for AY 2019-20
Full value of Consideration 70,00,000
Less: Brokerage @ 2% (1,06,000)
Less: Indexed cost of acquisition [ Rs. 6,00,000 /122 x 280] (13,77,050)
Long-term capital gain 55,16,950
Less: Exemption u/s 54 - Acquisition of residential house property at Kolkata (10,00,000)
Amount deposited in capital gains accounts scheme (4,00,000)
Exemption u/s 54EC: Amount deposited in capital gains bonds of RECL on 10.04.2019 (4,00,000)
Long-term capital gain 37,16,950
Total Income (rounded off u/s 288A) 37,16,950

Note:
▪ As per the decision of Gauhati High Court in CIT vs Rajesh Kumar Jalan (2006) & Punjab & Haryana High Court
in CIT vs Jagriti Aggarwal (2011), exemption u/s 54 is allowable even if the amount of capital gain is deposited
in CGAS after due date specified u/s 139(1) but before DD for filing a belated return u/s 139(4).

DT QUESTION BANK BY CA PRANAV CHANDAK Subscribe us on YouTube 103


▪ If we apply the above interpretation in this case, Mr. X would be eligible for exemption u/s 54 in respect of Rs.
3,00,000 deposited in Capital Gains Accounts Scheme on 01.11.2019 also, since the said date falls within the
time specified u/s 139(4). On the basis of this interpretation, taxable LTCG in hands of Mr. X = 34,16,950.

Q21. Mr. X, a resident individual, aged 55 years, purchased 10 Plots in FY 2003-04 for Rs. 12 Lac. On 1.4.2004, he
started a business of property dealing & converted all 10 plots into SIT of his business & recorded Rs. 40 Lac in his
books being FMV the said date.
On 31st March 2011, he sold all 10 Plots for Rs. 55 Lacs & purchased a residential house property for Rs. 50 Lacs.
He has constructed 2 rooms in this residential house in June 2011 & has spent 8 Lacs.
He sold the above residential house on 5.2.2019 for 80 Lacs. Stamp duty value was 105 Lacs. On the request of Mr.
X, AO made a reference to valuation officer. Valuation Officer determined the value at 108 Lac. Mr. X paid brokerage
1% of sale consideration. Compute Capital gains of Mr. X for AY 2019-20.
(CII: 2003-04-109; 2004-05-113; 2010-11-167; 2011-12-184; 2018-19-280.) [NOV-2016]
Solution: Computation of capital gains of Mr. X for AY 2019-20
Capital Gains on sale of residential house property
Full value of consideration [Note 1] 105,00,000
Less: Brokerage @ 1% of sale consideration (80,000)
Less: Indexed cost of acquisition ( Rs. 50,00,000 /167x 280) (83,83,234)
Less: Indexed cost of improvement ( Rs. 8,00,000 /184x 280) (12,17,391
Long-term capital gain 8,19,375
Note: If SC < SDV & SDV < Value by VO, SDV shall be taken as full value of consideration.

Q22. Mr. X sold his residential house property on 8.6.2018 for 70 Lacs which was purchased by him for 20 Lacs on
5.5.2006. He paid 1 Lac as brokerage for sale of said property. SDV assessed by sub registrar was Rs. 80 Lacs.
He bought another house property on 25.12.2018 for 11 Lacs. He deposited Rs. 8 Lacs on 10.11.2018 in the capital
gain bond of NHAI. He deposited another 10 Lacs on 10.7.2019 in capital gain deposit scheme for construction of
additional floor of house property. Compute income u/h “Capital Gains” for AY 2019-20. [CII: FY 2006-07 = 122].
Solution: Computation of Capital Gains of Mr. X for AY 2019-20
Full value of consideration 80,00,000
Less: Brokerage (1,00,000)
Less: Indexed cost of acquisition of building (20,00,000 / 122 x 280) (45,90,163.93)
Long term capital gain 33,09,836.07
Less: Investment in house property: Section 54 (11,00,000)
Less: Investment in NHAI section 54EC (assumed redeemable after 5 years) (8,00,000)
Less: Deposited in Capital Gain Account Scheme (10,00,000)
Long Term Capital Gains 4,09,836.07

Q23. Mr. Sunil entered into an agreement with Mr. Dhaval to sell his residential house located at Navi Mumbai on
16.8.2018 for Rs. 80,00,000. The sale proceeds was to be paid in the following manner:
(i) 20% through account payee bank draft on the date of agreement.
(ii) 60% on the date of the possession of the property.
(iii) Balance after the completion of the registration of the title of the property.
Mr. Dhaval was handed over the possession of the property on 15.12.2018 & registration process was completed
on 14.01.2019. He paid the sale proceeds as per the sale agreement.
SDV on 16.08.2018 was Rs. 90,00,000 whereas SDV on 14.1.2019 was Rs. 91,50,000.
Mr. Sunil had acquired the property on 1.4.2001 for Rs. 10,00,000. After recovering the sale proceeds from Dhaval,
he purchased another residential house property for Rs. 35,00,000.
Compute the income u/h “Capital Gains” for AY 2019-20. [CIIs: FY 2001-02: 100 FY 2018-19: 280] [NOV-17]
Solution:
▪ As per section 50C, if sale consideration < SDV, then FVC shall be taken to be Stamp duty value.
▪ If date of agreement & date of registration are different, value on date of agreement shall be taken into
consideration provided some advance was given otherwise than in cash on or before the agreement.
▪ In the given case 20% amount was paid on date of agreement through A/c payee bank draft. Thus value as on
the date of agreement shall be considered.
Computation of Capital Gains u/s 48
Full value of consideration 90,00,000.00
Less: Indexed cost of acquisition (10,00,000/100 x 280) (28,00,000.00)
Long term capital gain 62,00,000.00
Less: Exemption u/s 54 - Investment in House Property (35,00,000.00)
Long Term Capital Gains 27,00,000.00

DT QUESTION BANK BY CA PRANAV CHANDAK Subscribe us on YouTube 104


Q24. Mr. X purchased agricultural land in urban area on 1.4.2001 for Rs. 2,00,000. It was being used for agricultural
purposes since then & was sold by the assessee on 01.07.2018 for Rs. 123,00,000. He made following investments:
(i) Bonds of National Bank for Agriculture & Rural Development on 1.6.2018 for Rs. 1,50,000.
(ii) He purchased agricultural land on 01.09.2018 for Rs. 2,00,000.
(iii) He has invested Rs. 75,000 on 01.10.2018 in the bonds of NHAI redeemable after 5 years.
He sold the bonds of NHAI on 15.04.2019 for Rs. 3,00,000. Compute his capital gains for various years.
Solution: Computation of Capital gains [PY 2018-19]
Full value of consideration 123,00,000
Less: Indexed cost of acquisition [2,00,000/100 x 280 = Rs. 5,60,000] (5,60,000)
Long Term Capital Gain 117,40,000
Less: Exemption u/s 54B (2,00,000)
Less: Exemption u/s 54EC (75,000)
Long Term Capital Gain 114,65,000
Computation of Capital Gains [PY 2019-20]
Full value of consideration 3,00,000
Less: Cost of acquisition (75,000)
Short Term Capital Gain 2,25,000
Long Term Capital Gain (withdrawal of exemption) 75,000

Q25. Mr. Surinder furnishes the following particulars for PY 2018-19. He had a Residential House, inherited from
his father in December 2006, FMV of which on 1.4.2001 is Rs. 15 Lacs. In PY 2009-10, further construction &
improvements costed Rs. 10 Lacs. On 10.5.2018, House was sold for 75 Lacs. Transfer Expenses were 50,000. On
20.12.2018, he purchased a Residential House for Rs. 20 Lacs. Compute the taxable Capital Gain for AY 2019-20.
(b) What will be the taxable Capital Gain, if the cost of the new Residential House is only Rs.12 Lacs? [M 02]
Solution: Computation of Capital Gain on Sale of Inherited Property
Full value of consideration 75,00,000
Less: Expenses on Transfer (50,000)
Net Sale Consideration 74,50,000
Less: Indexed Cost of Acquisition [Rs. 15 00,000 x 280/100] (42,00,000)
Less: Indexed Cost of improvement [(1 10,00,000 × 280/148)] (18,91,892)
Long Term Capital Gains 13,58,108
Less: Exemption u/s 54 (13,58,108)
Taxable Long-Term Capital Gains Nil
(b) If Cost of New House is Rs. 12,00,000, exemption u/s 54 will be least of Rs. 13,58,108 & Rs. 12,00,000, i.e. Rs.
12,00,000. In such case, Taxable Capital Gains will be Rs. 1,58,108.
Note: Since Mr. Surinder inherited the property, indexation benefit for Cost of Acquisition can be availed from
the date of original acquisition by his father. [CIT vs Manjula J. Shah 204 Taxmann 691 (Bom.)]
Note:
1. U/s 54, newly acquired property shall be held by Assessee for 3 years from the date of acquisition/completion.
2. If new property is being transferred within 3 years, then STCG on newly acquired property shall be calculated
by reducing the amount of Capital Gains exempted u/s 54 from the cost of the newly acquired property.

Q26. A house was purchased on 1.05.2001 for Rs. 2 Lacs & was used as a residence by the owner. The owner had
contracted to sell this property in june 2012 for Rs. 8 lacs & he had received an advance of Rs. 50,000 towards sale.
The deal was not finalized & hence the amount was forfeited on August 2012. He again contracted to sell this
property & received an advance on 24.02.2015. However this deal was also not finalized & hence the amount was
forfeited on 30.04.2015. The property was sold in June 2018 to another buyer for Rs. 10 Lacs. Owner paid 2%
brokerage on sale of the house. Find the capital gain. [CII: FY 2001-02: 100; FY 2018-19: 280]
Solution: Computation of Capital Gain in the hands of owner for AY 2018-19
Full Value of consideration 10,00,000
Less: Expenses on transfer [Brokerage @ 2% of Sale Value] (20,000)
Net Sale Consideration 9,80,000
Less: (Cost of acquisition - Forfeited Advance) = (2L – 50,000) × 280/100) (4,20,000)
Long- Term Capital Gain 5,60,000
Note: Advance forfeited on or after 1.4.2001 shall be taxed u/h “IFOS” u/s 56(2)(ix).

DT QUESTION BANK BY CA PRANAV CHANDAK Subscribe us on YouTube 105


Q27. Mr. Rakesh purchased a House Property on 14.4.1996 for 2,50,000. He entered into an agreement with Mr.
B for sale of house on 15.9.1999 & received an Advance of 25,000. However, since Mr. B did not remit the balance
amount, Mr. Rakesh forfeited the advance. Later on, he gifted the House Property to his friend Mr. A on
15.6.2001. Following renovations were carried out by Mr. Rakesh & Mr. A to the House Property:
Particulars Amount
By Mr. Rakesh during FY 1996-1997 1,00,000
By Mr. A during FY 2005-2006 1,00,000
By Mr. A during FY 2007-2008 2,50,000
FMV of the Property on 1.4.2001 is Rs. 2,50,000. Mr. A entered into an agreement with Mr. C for sale of the House
on 1st June 2015 & received an Advance of Rs. 1,00,000. The said amount was forfeited by Mr. A, since Mr. C could
not fulfil the terms of the agreement. Finally, the House was sold by Mr. A to Mr. Sanjay on 2 nd Jan 2018 for Rs. 15
lacs. Compute taxable Capital Gains in hands of Mr. A for AY 2018-2019. [May 2011]
Solution: Computation of Capital Gain
Particulars Rs.
Sale Consideration 15,00,000
Less: Indexed Cost of Acquisition = (2,50,000 x 280/100) (7,00,000)
Less: Indexed Cost of Improvement [(1,00,000 x 280/117) + (2,50,000x 280/129)] (7,81,952)
Long Term Capital Gain 18,048
• Income from Other Sources: Advance Forfeited [Sec. 56(2)(ix)]: Rs. 1,00,000.
Notes:
1. As per Sec. 51, any Advance Money or any other sum received & retained by the Assessee will be treated as
Income from other Sources. The Cost of Acquisition shall not be reduced by that amount.
2. Advance Money received & forfeited by the Previous Owner shall not be considered u/s 51.
3. Improvement done by the previous owner is also considered if it is done after 1.4.2001.

Q28. The Assessee was a Company carrying on business of manufacture & sale of art-silk cloth. It purchased
Machinery worth Rs. 4 Lacs on 01.05.2014 & insured it with United India Assurance Ltd, against fire, flood,
earthquake, etc. Depreciation was granted at 15% for each assessment year. The Insurance Policy contained a
reinstatement clause requiring the Insurance Company to pay the value of the machinery, as on the date of fire etc,
in case of destruction or loss. A fire broke out in August 2018 causing extensive damage to the machinery of the
Assessee rendering them totally useless. Assessee Company received Rs. 6 Lacs from the Insurance Company on
15.03.2019. Discuss the issues arising on account of the transactions & their tax treatment. [May 2000]
Solution: 1. Computation of WDV of Machinery as on 01.04.2018
Cost of Machinery 4,00,000
Less: Depreciation for Previous Year 2014—2015 at 15% (60,000)
WDV on 01.04.2015 3,40,000
Less: Depreciation for Previous Year 2015-2016 at 15% (51,000)
WDV on 01.04.2016 2,89,000
Less: Depreciation for Previous Year 2016-2017 at 15% (43,350)
WDV on 01.04.2017 2,45,650
Less: Depreciation for Previous Year 2017-2018 at 15 % (36,848)
WDV on 01.04.2018 2,08,802

2. Computation of Capital Gains u/s 45(1A) read with Sec. 50


Insurance Compensation received 6,00,000
Less: WDV of the Machinery (2,08,802)
Short Term Capital Gain 3,91,198

Q29. Mr. A is an Individual carrying on business. His stock & machinery were damaged & destroyed in a fire
accident. The value of stock lost (totally damaged) was t 6,50,000. Certain portion of the Machinery could be
salvaged. The Opening WDV of the Block as on 01.04.2018 was Rs.10,80,000.
During the process of safeguarding machinery & in the fire fighting operations, Mr. A lost his gold chain & a
diamond ring, which he had purchased in April 2015, for Rs.1,20,000. Market Value of these two items on the date
of fire accident was Rs.1,80,000. Mr. A received the following amounts from the Insurance Company:
(a) Towards Loss of Stock: Rs. 4,80,000; (b) Towards Damage of Machinery: Rs. 6,00,000
(c) Towards Gold Chain & Diamond Ring: Rs. 1,80,000
Comment on the tax treatment of the above three items under the provisions of the Income Tax Act, 1961. [N-06]
Answer:
Stock ▪ Loss of Stock-in-Trade used for the purpose of business is a Business Loss.
▪ It is a loss incurred during the course of business & allowable u/s 37.

DT QUESTION BANK BY CA PRANAV CHANDAK Subscribe us on YouTube 106


▪ Loss in excess of the Insurance Compensation will be allowable as a deduction u/s 37.
▪ So, the amount of Rs.1,70,000 (Insurance Compensation Rs. 4,80,000 – Loss of Stock Rs.
6,50,000) is an allowable business expenditure u/s 37.
Machinery ▪ Machinery is a Capital Asset u/s 2(14) on which depreciation has been claimed.
▪ Compensation received from Insurance Company on destruction of Machinery by fire, is
taxable u/s 45(1 A), subject to Sec. 50.
▪ Since part of the Machinery is salvaged, the compensation received from the Insurers shall be
reduced from the WDV of the Block.
Gold ▪ Jewellery is a Capital Asset u/s 2(14).
Chain & ▪ Sec.45(lA) provides for tax consequences of compensation received from Insurers on account
Diamond of damage or destruction of Capital Assets due to accidental fire or explosion.
Ring ▪ However, damage or destruction does not include loss of assets other than accidents by way of
fire or explosion. So, the gains on account of compensation received from Insurers in this case,
shall not be brought to tax under the head Capital Gains u/s 45(1 A).
▪ Such compensation being a Capital Receipt is not chargeable to tax.

Q30. Aarav converts his plot of land purchased in July 2012, for Rs. 80,000 into Stock-in-Trade on 31st March
2018. Fair Market Value as on 31.03.2018 was Rs. 1,90,000. The Stock in Trade was sold for Rs. 2,25,000 in the
month of Ja 2019. Find out the Taxable Income if any, & if so under which Head of Income & for which AY? [M 11]
Solution:
(I) Income u/h PGBP: Business Income on Sale of Stock = Sale Value – FMV on date of Conversion = (Rs. 2,25,000-
Rs. 1,90,000) = Rs. 35,000.
(II) Income u/h ‘Capital Gains’
Full Value of Consideration [Fair Market Value on date of Conversion] 1,90,000
Less: Indexed Cost of Acquisition (Rs. 80,000 x 272/200) (1,08,800)
Long Term Capital Gains 81,200
Notes:
▪ Capital Gains shall be charged to tax as income of previous year in which the converted stock is sold.
▪ Indexation shall apply only on the basis of Year of Conversion.

Q31. Ms. Gunjan purchased a Land at a cost of Rs. 50 Lacs in PY 2008-09 & held the same as her capital asset till
31st Aug 2014. She started her Real Estate Business on 1st Sep 2014 & converted the said Land into Stock-in-Trade
of her business on the said date, when the Fair Market Value of the Land was Rs. 320 Lacs. She constructed 8 Flats
of equal size, quality & dimension. Cost of Construction of each Flat is Rs. 36 Lacs. Construction was completed in
January 2018. She sold 5 Flats at Rs.90 Lacs per Flat in April 2018. She invested Rs. 70 Lacs in Bonds issued by
National Highways Authority of India on 31st May 2018. Compute the Capital Gains & Business Income arising
from the above transactions in the hands of M.J. Gunjan for AY 2019-2020 indicating clearly the reasons for
treatment for each item. [CII: FY 2008-09: 137, FY 2014-15: 240, FY 2015-16: 254, FY 2018-2019:280] [RTP]
Solution: Computation of Capital Gain & business income of Ms. Gunjan
Full Value of Consideration [FMV of Land on date of conversion] 3,20,00,000
Less: Indexed Cost of Acquisition [Rs. 50,00,000 x 240/137] (87,59,124)
Capital Gains 2,32,40,876
Proportionate LTCG arising during AY 2019-2020 [Rs. 2,32,40,876 × 5/8] 1,45,25,548
Less: Exemption u/s 54EC (restricted to Rs. 50 Lacs) 50,00,000
Taxable LTCG 95,25,548
Business Income
Sale Price of Flats (5x Rs.90 Lacs) 4,50,00,000
Less: Cost of Flats: (a) FMV of Land on the date of conversion (Rs.3,20,00,000 x 5/8) (2,00,00,000)
(b) Cost of Construction of Flats (5 × Rs. 36 Lacs) (1,80,00,000)
Business Income 70,00,000
Note:
1. Conversion of Capital Asset into SIT is a transfer u/s 2(47). It would be treated as a transfer in PY in which
Capital Asset is converted into SIT. But Capital Gains will be taxable only in the PY in which SIT is sold.
2. Indexation is available only upto the year of conversion of Capital Asset to SIT & not upto year of sale of SIT
3. 5 flats out of 8 Flats is sold in PY 2018-19. So, only proportionate Capital Gains (5/8th) is taxable in AY 2019-20.
4. In case of conversion of Capital Asset into Stock-in-Trade & subsequent sale of Stock-in-Trade, the period of 6
months, for the purpose of exemption u/s 54EC, is to be reckoned from the date of sale of SIT. In this case, since
investment in bonds of NHAI has been made within 6 months of sale of flats, exemption u/s 54EC is available.

DT QUESTION BANK BY CA PRANAV CHANDAK Subscribe us on YouTube 107


Q32. Mr. X & Mr. Y are two partners of a firm X & Co. On 1.1.2019, Mr. Z joins the firm & brings shares in a company
as his capital contribution. Fair market value of these shares on 01.01.2019 is Rs. 5,00,000 whereas amount
credited in Mr. Z’s account in the firm is Rs. 4,00,000. Assuming that cost of acquisition in 2006-07 of these shares
was Rs. 48,000, find out the amount of chargeable capital gain for the AY 2019-20 in the hands of Mr. Z?
Solution: Computation of Capital Gains
Full value of consideration 4,00,000.00
Less: Indexed cost of acquisition [48,000/122 x 280 = 1,10,164] (1,10,164)
Long Term Capital gain 2,89,836

Q33. One partnership firm has purchased gold on 01.10.2005 for Rs. 5,00,000 & dissolution has taken place on
01.10.2018 & this gold was transferred to one of the partner in settlement of his claim of Rs. 25,00,000, though
the market value was Rs. 35,00,000. Compute capital gains for AY 2019-20.
Solution: Computation of Capital Gains
Full value of consideration 35,00,000.00
Less: Indexed cost of acquisition = 5,00,000/ 117x280 = Rs. 11,96,581.20 (11,96,581.20)
Long Term Capital Gain 23,03,418.80

Q34. Mr. X (Date of birth: 01.10.1946) has purchased one house on 01.04.1995 for Rs. 4 lacs & incurred Rs. 2 lacs
on its improvement on 1.10.1998. Its market value on 1.4.2001 was Rs. 3 lacs. This house was acquired by the
Government on 1.10.2013 & compensation fixed was Rs. 50 lacs & Government has paid half of the compensation
on 1.10.2018 & balance half on 1.10.2019.
Assessee has filed an appeal for increasing compensation & court has given decision on 31.03.2020 directing the
Government to pay additional compensation of Rs. 5,00,000.
The Government has paid half of the amount on 01.04.2021 & balance half on 01.04.2022.
He has invested Rs. 72,000 in NSC in previous year 2018-19.
Compute assessee’s tax liability for the AY 2019-20 & also capital gains for various years.
Solution: Computation of Capital Gains u/s 45(5)
Capital gain shall be computed in the year in which the asset was acquired by the Government i.e. in PY 2013-14
& shall be taxed in the year in which the first payment has been received by the assessee i.e. in the PY 2018-19.
Full value of consideration 50,00,000.00
Less: Indexed cost of acquisition = 4,00,000 /100 x 220 = Rs. 8,80,000 (8,80,000.00)
Long Term Capital Gain 41,20,000.00
Gross Total Income 41,20,000.00
Less: Deduction u/s 80C {Deduction u/s 80C is not allowed from LTCG} Nil
Total Income 41,20,000.00
Computation of Tax Liability
Tax on Rs. 38,20,000 ( Rs. 41,20,000 - Rs. 3,00,000) @ 20% 7,64,000.00
Add: HEC @ 4% 30,560.00
Tax Liability 7,94,560.00
[Since normal income is nil; as per section 112, unexhausted BEL of 3 lacs shall be allowed from LTCG]

Computation of Capital Gain for PY 2021-22: Long Term Capital Gain 2,50,000.00
Computation of Capital Gain for PY 2022-23: Long Term Capital Gain 2,50,000.00

Q35. Mr. X purchased 100 equity shares in ABC Ltd. on 01.10.1995 @ Rs. 10 per share. The company has issued
100 bonus shares on 01.10.1998 & market value of the shares on 01.04.2001 was Rs. 7 per share. The company
has again issued 100 bonus shares on 01.10.2012.
The company has offered 100 right shares on 01.04.2018 @ Rs. 140 per share though the market value is Rs. 250
per share. Mr. X purchased half of the shares & remaining half were renounced by him in favour of his friend Mr.
Y. He has charged Rs. 20 per share from Mr. Y for renouncing the right.
All the shares were sold by Mr. X & Mr. Y @ Rs. 300 per share on 01.01.2019 & STT has been paid.
Compute Capital gains of Mr. X for AY 2019-20.
Solution: Computation of Capital Gains Mr. X
Original Shares
Full value of consideration (100 x 300) 30,000
Less: Cost of Acquisition (100 x 10) (1,000)
Long Term Capital Gain u/s 112A 29,000

1st Bonus Shares 2nd Bonus Shares


Full value of consideration (100 x 300) 30,000 Full value of consideration (100 x 300) 30,000

DT QUESTION BANK BY CA PRANAV CHANDAK Subscribe us on YouTube 108


Less: Cost of Acquisition (100 x 7) (700) Less: Cost of Acquisition [After 1.4.2001] Nil
Long Term Capital Gain u/s 112A 29,300 Long Term Capital Gain u/s 112A 30,000

Right shares Sale of Rights


Full value of consideration (50 x 300) 15,000 Full value of consideration 1,000
Less: Cost of Acquisition (50 x 140) (7,000) Less: Cost of Acquisition Nil
Short Term Capital Gain u/s 111A 8,000 STCG (Taxable @ slab rate) 1,000
Note: No Indexation is available for LTCG u/s 112A.

Q36. Mr. X purchased 100 shares of M/s AB Ltd on 1.11.2016 @ Rs. 1,000 per Share in Public Issue of the Company.
Company allotted Bonus Shares in ratio of 1:1 on 1.12.2016. He received dividend of Rs. 10 per Share on 1.5.2018.
To celebrate his 75th birthday, Mr. X has sold all Shares on 1.10.2018 @ Rs. 3,300 per share through RSE & paid
brokerage of 1% & Securities Transaction Tax of 0.02%. FMV of these shares on 31.01.2018 is Rs. 2000 per share.
CIIs - FY 2016-17: 264; FY 2018-2019: 272. Compute his Total Tax Liability for AY 2019-2020. [Nov 2011]
Solution: Computation of Total Income & Tax Liability
Computation of capital gains on Original shares Bonus shares
Sale consideration 3,30,000 3,30,000
Less: Brokerage at 1% (Rs. 3,30,000 x 1%) (3,300) (3,300)
Net Consideration 3,26,700 3,26,700
Less: Cost of Acquisition (100 Shares x 2000) [FMV on 31.1.2018] (2,00,000) (2,00,000)
Long Term Capital Gain 1,26,700 1,26,700
Income from Other Sources:
Dividend Received (200 Shares x Rs. 10 per Share) - Exempt u/s 10(34) Nil
Gross Total Income 2,53,400
Tax on Total Income (Refer WN 2) Nil
Working Note 1:
(a) Shares purchased originally are held for more than 12 months & so they are Long-Term Capital Gains.
(b) Bonus Shares are held for more than 12 months & hence they are Long-Term Capital Gains.
(c) STT paid on Sale of Shares shall not allowed as deduction while computing Expenses for Transfer.
Working Note 2: Special Benefit for Resident Individuals / HUF u/s 112A:
▪ Applicability: Resident Individual or Resident HUF
▪ Condition: Total income excluding Long-Term Capital Gains is less than the basic exemption.
▪ Tax on LTCG is determined as follows: Tax on LTCG = 10% [TI including LTCG – BEL] – 1,00,000].

Q37. Mr. X started business on 01-04-2001 & Mr. Y started profession on 01-04-2001. Each one of them sold
goodwill on 01-07-2018 for Rs. 50,00,000. Compute Capital gains in both the cases.
Solution: 1. Capital Gains of Mr. X
Full value of consideration 50,00,000
Less: Indexed cost of acquisition Nil
Long Term Capital Gain 50,00,000
Note: Cost of acquisition of self - generated goodwill of business shall be taken to be Nil.
2. Capital Gains of Mr. Y: As per Supreme Court ruling CIT vs. B.C. Srinivasa, no capital gains shall be computed
in case of self-generated goodwill of profession.

Q38. Raju acquired & transferred the shares of X ltd in his Demat a/c as given below. Compute Capital Gains.
Details of DEMAT A/c
Acquisitions: Transfers:
Date of Entry in Demat A/c No. Cost Transfer No of shares Sold @
1.1.2007 1000 share 120/share 1.04.2011 2,500 shares 189/share
1.12.2009 3000 share 136/share 1.08.2015 5,000 shares 260/share
1.4.2013 (Acquired on 1.1.2002) 5000 share 45/share 1.10.2018 1,500 shares 340/share
Solution:
(i) Capital Gain on Transfer of 2,500 shares on 1.4.2011
Sale Consideration (2500 × 189) 4,72,500
Less: Indexed COA: (1000 × 120 × 184/122) (1,80,980)
(1500 × 136 × 184/148) (2,53,620) (4,34,600)
Long term Capital gain 37,900
(ii) Capital Gain on Transfer of 5,000 shares on 1.8.2015

DT QUESTION BANK BY CA PRANAV CHANDAK Subscribe us on YouTube 109


Sale Consideration (5,000 × 260) 13,00,0000
Less: Indexed COA: (1500 × 136 × 254/148) (3,50,110)
(3500 × 45 × 254/100) (1,57,500) (5,07,610)
Long term Capital gain 7,92,390
(iii) Capital Gain on Transfer of 1500 shares on 1.10.2018
Sale Consideration (1500 × 340) 5,10,000
Less: Indexed COA: (1500 × 45 × 280/100) (1,89,000)
Long term Capital gain 3,21,000

Q39. R, a NRI, remits US$ 40,000 to India on 16.9.2009. The amount is partly utilized on 3.10.2009 for purchasing
10,000 shares in A Ltd., an Indian company at the rate of Rs.12 per share. These shares are sold for Rs. 36 per share
on 30.3.2019. The telegraphic transfer buying & selling rate of US dollars adopted by the State Bank of India is:
Date TT Buying Rate TT Selling Rate Average TT Rate [ Buying + Selling]/2
16.9.2009 18 20 19.5
3.10.2009 19 21 20
30.3.2019 44 46 45
Compute capital gain chargeable to tax for the AY 2019-20 on the assumption that:
(a) These shares have not been sold through RSE. (b) These shares have been sold through RSE & STT was paid.
Solution: (a) Where shares are not sold through recognised stock exchange
Sale consideration (Rs. 3,60,000/45) US$ 8,000
Less: Cost of acquisition (Rs. 1,20,000/20) (US$ 6,000)
Long-term capital gain US$ 2,000
Long term Capital gain covered into Rupees (US$ 2,000 x Rs. 44/US$) Rs. 88,000

(b) Where shares are sold through a recognised stock exchange: Entire LTCG is exempt since it is < 10 Lacs.

Q40. Singhania & Co., a sole proprietorship owns 6 machines, put in use for business in March 2018. Rate of
Depreciation is 15%. WDV of these machines as on 1st April 2018 was Rs. 8,50,000. Three of the old machines were
sold on 10th June 2018 for Rs. 11,00,000. A second-hand plant was bought for Rs. 8,50,000 on 30th November 2018.
You are required to:
(i) Determine depreciation for AY 2019-20;
(ii) Compute Capital Gains for AY 2019-20.
(iii) If 3 machines are sold in June 2018 for Rs. 21 lacs, will there be any difference in your above workings?
Solution:
(i) Computation of depreciation for AY 2019-20
Particulars Rs.
W.D.V. of the block as on 1.4.2018 8,50,000
Add: Purchase of second-hand plant during the year 8,50,000
Less: Sale consideration of old machinery during the year 11,00,000
W.D.V of the block as on 31.03.2019 6,00,000
▪ Since, Second-hand machinery was put to use for less than 180 days, depreciation is restricted to 50% of 15%.
▪ Therefore, depreciation for the year = Rs. 45,000, being 7.5 % of Rs. 6,00,000. [Refer PGBP if required]

(ii) Section 50 on Capital Gains in case of depreciable assets is applicable only in the following circumstances:
(a) When one or some of the assets in the block are sold for consideration more than the value of the block.
(b) When all the assets are transferred for a consideration more than the value of the block.
(c) When all the assets are transferred for a consideration less than the value of the block.
▪ Since in 1 & 2, Sale consideration > WDV of the block, short term capital gains would arise.
▪ In 3, Since SC < WDV of block, short term capital loss would arise.
▪ In the given case, capital gains will not arise as the block of asset continues to exist, & some of the assets
are sold for a price which is lesser than the written down value of the block.

(iii) If 3 machines are sold in June 2018 for Rs. 21,00,000, short term capital gains would arise since sale
consideration is more than the aggregate of opening WDV of the block + Additions made during the year.
Particulars Rs. Rs.
Sale consideration 21,00,000
Less: WDV of the machines as on 1.4.2018 8,50,000
Purchase of second plant during the year 8,50,000 17,00,000
Short term capital gains 4,00,000

DT QUESTION BANK BY CA PRANAV CHANDAK Subscribe us on YouTube 110


Q41. X Ltd. has several undertakings carrying on several businesses. During PY 2018-19, company sold one of its
undertakings (as it was continuously generating loss since last 5 years) for a lump sum value of Rs. 300 lacs without
assigning value to individual asset & liabilities. Brokerage on transfer paid @ 5%. Compute capital gain.
Book value of sundry assets & liabilities of the undertaking as on the date of sale is as under:
Items Book Value FMV
LAND Rs. 50 lacs (Stamp duty Value = Rs.7000000) Rs.100 lacs
Machinery Rs. 30 lacs (WDV as per IT Act Rs.60 lacs) Rs.100 lacs
Furniture Rs. 50 lacs (WDV as per IT Act Rs.90 lacs) Rs.75 lacs
Stock Rs. 30 lacs Rs.35 lacs
Debtors Rs. 40 lacs Rs.40 lacs
Creditors Rs. 50 lacs -
Solution: Since the undertaking is owned by the company for more than 3 year hence the gain on transfer shall he
liable to long term. Calculation of cost of acquisition (i.e. Net worth)

Calculation of Net Worth

A. Value of Assets taken over: B. Value of liabilities taken over:


Asset Value Basis
Land Rs. 50 Lacs Book Value (i) Creditors = Rs. 50 Lacs – Basis of Book Value.
Machinery Rs. 60 Lacs WDV
Furniture Rs. 90 Lacs WDV
Stock Rs. 30 Lacs Book Value NET WORTH = Assets – Liabilities = A - B =
Debtors Rs. 40 Lacs Book Value Rs. 270 Lacs – Rs. 50 Lacs = Rs. 220 Lacs
TOTAL Rs. 270 Lacs

Computation of capital gains in the hands of X Ltd. for AY 2019-20


Sale consideration Rs. 300 lacs
Less: Expenses on transfer = (5% of Rs.300 lacs) Rs. 15 Lacs
Net sale consideration Rs. 285 lacs
Less: Cost of acquisition i.e. Net Worth: Calculated above (Rs. 220 lacs)
Less: Cost of improvement (Rs. Nil)
Long Term Capital Gain Rs. 65 Lacs

Q42. Star Enterprises has transferred its unit R to A Ltd by way of slump sale on January 23, 2019. The Using the
further information below, compute the Capital Gains arising from slump sale of Unit R for AY 2019-20
Liabilities Amount (Rs. in Lacs) Assets Amount (in Lacs)
Own Capital 1,750 Fixed Assets:
Accumulated P & L Balance 670 (i) Unit P 200
Liabilities: (ii) Unit Q 150
(i) Unit P 90 (iii) Unit R 600
(ii) Unit Q 160 Other Assets:
(iii) Unit R 140 (i) Unit P 570
(ii) Unit Q 850
(iii) Unit R 440
Total 2,810 Total 2,810
▪ Slump Sale consideration on transfer of Unit R was Rs. 930 Lacs.
▪ Fixed Assets of Unit R includes land which was purchased at Rs.110 Lacs in 2009 & was revalued at 140 Lacs.
▪ Other Fixed Assets are reflected at Rs. 460 Lacs, (i.e. Rs. 600 Lacs less value of land) which represents WDV of
those assets as per books. The written down value of these asset is Rs. 430 Lacs.
▪ Unit R was set up by Star Enterprises in Oct, 2007.
Note: CII for the FY 2007-2008 & FY 2018-19 are 129 & 280 respectively. [M 18]
Answer: Computation of Capital Gains
Particulars (In Lacs)
Consideration 930
Less: Expenses on transfer (Nil)
Net consideration 930

DT QUESTION BANK BY CA PRANAV CHANDAK Subscribe us on YouTube 111


Less: Net Worth (840)
Long term Capital Gain 90
Computation of Net Worth
Particulars Amt
Fixed Assets [Cost of land Rs.110 + Other depreciable assets @ WDV as per IT Act Rs. 430] 540
Other Assets 440
Total Assets taken over 980
Less: Liabilities of Unit R (140)
Net Worth 840
Notes:
1. No Indexation for Slump Sale transaction.
2. Revaluation effect shall be ignored for the purpose of computing Cost of Acquisition u/s 50B.

Q43. ABC Ltd. has issued 1 lacs shares of Rs. 10 each & the company goes into liquidation on 1.10.2018 &
distributable asset of the company are valued at Rs. 8 lacs. The company’s accumulated profits on the date of
liquidation are Rs. 3.5 Lacs which are included in Rs. 8 Lacs. Mr. X has purchased 100 shares in this company on
1.10.1998 for Rs. 10 each & market value of the shares on 1.4.2001 is Rs. 12 per share. Compute dividends in the
hands of Mr. X & also capital gains.
Solution:
▪ % Holding of Mr. X = 100 shares/1 lac shares = 0.1 %
▪ Share of Mr. X in the distributable profits = 0.1 % of Rs. 8,00,000 = Rs. 800.
▪ Share of Mr. X in the Accumulated profits = 0.1 % of Rs. 3,50,000 = Rs. 350.
▪ Dividends in the hands of Mr. X as per sec 2(22)(c) = Rs. 350.
Computation of capital gains as per section 46
Full value of consideration [ 800 – 350] since dividend u/s 2(22)(c) is exempt to shareholder 450
Less: Indexed COA = 1,200/100 x 280 (3,360)
Long-term capital loss (2,910)

Q44. Mr. PC purchased 10,000 equity shares of XYZ Co. Pvt. Ltd on 28.2.2007 for Rs. 1,20,000. The company was
wound up on 31.7.2018. The following is the summarized financial position of the company as on 31.7.2018.
Liabilities Rs. Assets Rs.
60,000 Equity shares 6,00,000 Agricultural lands 42,00,000
General Reserve 40,00,000 Cash at bank 6,50,000
Provision for taxation 2,50,000
48,50,000 48,50,000
Tax liability (towards DDT) was ascertained at Rs. 3 Lacs, after considering refund due to the company.
The remaining assets were distributed to the shareholders in the proportion of their shareholding.
The market value of the 6 acres of the agriculture land (in an urban area) as on 31.7.2018 is Rs. 10 Lacs per acre.
Agriculture land received above was sold by Mr. PC on 29.2.2019 for Rs. 15 Lacs. Discuss tax treatment. [M-08]
Solution:
1. CG arising to company on distribution of asset in kind to shareholders on liquidation is exempt u/s 46.
2. Computation of CG in the hands of Mr. PC
Sale consideration [refer note 1] 4,00,000
Less: Indexed cost of acquisition [1,20,000 × 280/122] (2,75,410)
Long term capital gain 1,24,590
3. CG arising on Sale of URBAN Agricultural land received in the hands Mr. PC
Sale consideration Rs. 15 Lacs
Less: COA [deemed to be FMV on date of distribution] Rs. 10 Lacs
Short term capital gain Rs. 5 Lacs
Working Note:
1. Calculation of Sale Consideration of Shares: Mr. PC holds 1/6th of shareholding of the company, so
Agriculture land received (60 Lacs/6) Rs. 10,00,000
Cash at bank (6,50,000 – 3 Lacs)/6 Rs. 58,333
Less: Deemed dividend u/s 2 (22)(c) (40 Lacs - 50,000)/6 ---- Exempt u/s 10(34) (6,58,333)
Sale consideration Rs. 4,00,000
2. Dividend u/s 2 (22)(c) i.e. Rs. 6,58,333 will be exempt.
3. Tax liability has been ascertained at Rs. 3 Lacs as against the provision of Rs. 2,50,000. Therefore Rs. 50,000
(Rs. 3 Lacs - Rs. 2,50,000) has to be reduced from general reserve for calculating deemed dividend u/s 2(22)(c).

DT QUESTION BANK BY CA PRANAV CHANDAK Subscribe us on YouTube 112


Q45. Mr. X & sons, HUF, purchased a land for Rs. 40,000 in 2001-02. In 2005-06, a partition takes place when Mr.
A, a coparcener, is allotted this plot valued at Rs. 2,00,000. In 2006-07, he had incurred expenses of Rs. 1,85,000
towards fencing of the plot. Mr. A sells this plot of land for Rs. 15,00,000 in 2018-19 after incurring expenses of
Rs. 20,000. You are required to compute the capital gain for the AY 2019-20.
Solution: Computation of taxable capital gains for the AY 2019-20
Particulars Rs.
Sale consideration 15,00,000
Less: Expenses incurred for transfer (20,000)
Net Sale consideration 14,80,000
Less: Indexed cost of acquisition = 40,000/ 100 x 280 (1,12,000.00)
Less: Indexed cost of improvement = 1,85,000/122 x 280 (4,24,590.16)
Long term capital gains 9,43,409.84

Q46. Mr. X has purchased 100 debentures in ABC Ltd. on 01.10.2002 @ Rs. 300 per debentures & subsequently
these debentures were converted into shares on 01.10.2017 & 3 shares were issued for each debenture. The
assessee has sold all the shares on 01.04.2018 @ Rs. 750 per share. Compute capital gains for the AY 2019-20.
Solution: Computation of Capital Gains
Full value of consideration (300 x 750) 2,25,000.00
Less: Indexed Cost of acquisition = 30,000 / 105 x 280 = Rs. 80,000.00 (80,000.00)
Long Term Capital Gain 1,45,000.00

*Q47. Mr. C inherited from his father 8 plots of Land in 1999. His father had purchased the plots in 1986 for 5 Lacs.
FMV of the Plots as on 1.4.2001 was Rs. 16 Lacs. (Rs. 2 Lac for each plot). On 1.6.2005, C started a Business of
dealer in plots & converted 8 plots as SIT of his business. He recorded the plots in his books at 64 Lacs being the
FMV on the date. In June 2009, C sold the 8 plots for Rs. 75 Lacs.
In the same year, he acquired a Residential House Property for Rs. 50 Lacs. He invested an amount of Rs. 5 Lacs in
construction of one more floor in his house in June 2010. The house was sold by him in June 2018 for Rs. 80 lacs.
Stamp Duty Value was Rs. 98,50,000. As per the Assessee's request, AO made a reference to a Valuation Officer.
Value determined by Valuation Officer was Rs. 99,20,000. Brokerage of 1% of Sale Consideration was paid by C.
CII: FY 2002-2003: 105; FY 2005-2006: 119; FY 2009- 10: 148; FY 2010-11: 167; FY 2018-19: 280. [N 12/N-16]
Solution:
1.Transfer: Conversion of a Capital Asset into Stock-in-Trade is a transfer liable for Capital Gains.
2. Year of Taxation: Capital Gains shall be taxable as Income of previous year in which the converted stock is sold.
3. Year for Indexation: Indexation shall apply only on the basis of Year of Conversion.
4. Indexed COA = Higher of Original Cost to Previous Assessee or FMV on 1.4.2001. Thus COA = 16 lacs.
5. CII should be based on the year in which the Asset is first held by the Previous Owner & not the Assessee.

PGBP: Business Income on Sale of Stock = Sale Value - FMV on conversion = 75 lacs – 50 lacs = 25 lacs.
Income u/h Capital Gains
Full Value of consideration [Section 50C is not applicable in case of deemed sale u/s 45(2)] 64,00,000
Less: Indexed Cost of Acquisition (Rs. 16,00,000 x 117/105) (17,82,857)
Long Term Capital Gain 46,17,143
Less: Exemption u/s 54: Lower of (i) Amount invested or (ii) Capital gains (46,17,143)
Taxable Long-Term Capital Gain Nil
Gross Total Income 25,00,000

Particulars
Sale Consideration (Note) 98,50,000
Less: Expenses (Brokerage at 1% on 80,00,000) 80,000
Net Consideration 97,70,000
Less: (a) Indexed Cost of Acquisition: 50,00,000 × 280/148 (94,59,459)
(b) Indexed Cost of Improvement: 5,00,000 × 280/167 (8,38,323)
Long Term Capital Loss (5,27,782)

*Q48. Sumit purchases 2,500 (non-listed) Shares in Amit Ltd on 16th August 2006 for Rs. 10,000. On 17th May
2008, he gets 500 Bonus Shares. On 20th October 2014, he acquires 1,500 Right Shares at the rate of Rs. 15 per
Share. He sells 4,500 (non-listed) Shares in Amit Ltd on 12thFebruary 2019 at the rate of Rs. 150 per Share
(Brokerage on Sale is 2%). He owns one Residential House Property. He purchases a Residential House on 29 th
June 2019 for Rs. 3,50,000. Ascertain the amount of Capital Gains chargeable to tax for AY 2019-2020.

DT QUESTION BANK BY CA PRANAV CHANDAK Subscribe us on YouTube 113


Solution:
Particulars Original shares Bonus share Right shares
Shares Quantity 2,500 Shares 500 Shares 1,500 Shares
Sale Consideration (Rs. 150 per Share) 3,75,000 75,000 2,25,000
Less: Expenses on Transfer (2%) 7,500 1,500 4,500
Net Sale Consideration 3,67,500 73,500 2,20,500
Less: Indexed Cost of Acquisition (22,931) Nil (26,250)
(10,000 x 280/122) (1,500 x 15 × 280/240)
Long term capital gains 3,44,549 73,500 1,94,250
[(LTCG / Net Sale Consideration) x 100] 93.75% 100% 88.09%
Rank of Claiming Exemption u/s 54F (Note) II, I III
Amount of Investment in House Property 2,76,500 73,500 Nil
out of sale proceeds (3,50,000 – 73,500)
Less: Exemption u/s 54F = 2,59,232 73,500 Nil
𝐴𝑚𝑜𝑢𝑛𝑡 𝐼𝑛𝑣𝑒𝑠𝑡𝑒𝑑
LTCG x
𝑁𝑒𝑡 𝑠𝑎𝑙𝑒 𝑐𝑜𝑛𝑠𝑖𝑑𝑒𝑟𝑎𝑡𝑖𝑜𝑛
Taxable LTCG 17,628 NIL 1,94,250
Note: Since exemption u/s 54F is given in the proportion of the LTCG to Net Sale Consideration, larger the
proportion, higher the exemption amount. Hence, ranks are allotted based on ratio of LTCG to Net Consideration.

Q49. ABC Ltd. purchased one commercial building on 01.07.1995 Rs. 2,00,000 & paid brokerage Rs. 20,000 & its
FMV on 1.4.2001 is Rs. 2,10,000. The company sold the building on 1.7.2018 for Rs. 500,00,000 & invested Rs.
60,00,000 in bond of NHAI redeemable after five years. Compute tax liability of the company for AY 2019-20.
Solution: Computation of Capital gains
Full value of consideration 5,00,00,000
Less: Indexed cost of acquisition = 2,20,000 / 100 x 280 = Rs. 6,16,000 (6,16,000)
Long Term Capital Gain 4,93,84,000
Less: Exemption u/s 54EC (exemption u/s 54EC cannot exceed Rs. 50,00,000) (50,00,000)
Long Term Capital Gain 4,43,84,000
Income under the head Capital Gain (LTCG) 4,43,84,000
Computation of Tax Liability
Tax on Rs. 443,84,000 @ 20% 88,76,800
Add: Surcharge @ 7% 6,21,376
Tax before health & education cess 94,98,176
Add: HEC @ 4% 3,79,927
Tax Liability 98,78,103

Q50. Anand, a Resident Individual & a Senior Citizen, purchased 25,000 shares of AB Ltd, an Unlisted Company at
Rs. 100 per Share, on 01.04.2005. On 01.10.2006, AB Ltd declared Bonus Share @ 20% on the original holdings.
On 01.04.2007, AB Ltd issued Rights Shares at one Share per every two Shares held at Rs.60 per Share. Anand
subscribed in full, his Rights Share entitlement. Anand sold all his original shares of AB Ltd on 30.04.2018 at Rs.
250 per Share, & Right & Bonus Shares for Rs. 200 per Share. He paid Brokerage @ 2% on the Sale Value of Shares.
Anand purchased a Residential Flat for Rs. 20 Lacs on 01.07.2018 as he did not own any house/ flat & invested Rs.
10 Lacs in REC Bonds (Capital Gain) out of the Net Sale Consideration of sale of shares. Anand has no other income
during PY 2018-19. Compute Capital Gains & total income & tax thereon for PY 2018-2019. [May 2010]
Solution: Computation of Capital Gains on Transfer of Shares & Tax Liability
Particulars Original Shares Bonus Shares Right Shares Total
Number of Shares 25,000 5,000 15,000 45,000
(25,000 x 20%) (30,000/2)
PP of Acquisition 2005-2006 2006-2007 2007-2008
Cost of Acquisition 25,00,000 NIL 9,00,000 34,00,000
Sale Consideration 62,50,000 10,00,000 30,00,000 1,02,50,000
Less: Brokerage at 2% (1,25,000) (20,000) (60,000) (2,05,000)
Net Consideration 61,25,000 9,80,000 29,40,000 1,00,45,000
Less: Indexed Cost of Acquisition (59,82,906) Nil (19,53,488) (79,36,394)
Long Term Capital Gains 1,42,094 9,80,000 9,86,512 21,08,608
Proportion of LTCG to NSC 2.31% 100% 33.55%
Ranking for Exemption u/s 54F III I II
Investment in New House Nil 9,80,000 (b/f) 10,20,000 20,00,000
Less: Exemption u/s 54F = Nil (9,80,000) (3,42,260) (13,22,260)

DT QUESTION BANK BY CA PRANAV CHANDAK Subscribe us on YouTube 114


Less: Exemption u/s 54EC – (REC (restricted to Nil Restricted (6,40,055)
Bonds) = Rs. 10 Lacs LTCG) (50,486) to (5,89,569)
Taxable LTCG Nil Nil Nil Nil
Tax Liability Nil

*Q51. Mr. X (age 82) purchased agricultural land on 1.10.2002 for Rs. 3,00,000 & it was being used for agricultural
purposes by him. It was sold on 01.01.2019 for Rs. 50,00,000. The assessee has purchased one agricultural land
in the rural area on 10.01.2019 for Rs. 10,00,000 & this land was sold by him on 11.2.2019 for Rs. 11,00,000 &
has invested Rs. 30,000 in National Saving Certificate. Compute taxable Capital gains for AY 2019-20.
(b) What if the land was purchased in the urban area instead of rural area.
Solution Computation of Capital Gains
Full value of consideration 50,00,000
Less: Indexed cost of acquisition = 3,00,000 / 105 x 280 = Rs. 8,00,000.00 (8,00,000)
Long Term Capital Gain 42,00,000
Less: Exemption u/s 54B (10,00,000)
Long Term Capital Gain 32,00,000
Note: If land is purchased in rural area, exemption is allowed u/s 54B but on its sale, exemption is not withdrawn.
(b) Computation of Capital Gains
Full value of consideration 50,00,000.00
Less: Indexed cost of acquisition = 3,00,000 / 105 x 280 = Rs. 8,00,000.00 (8,00,000.00)
Long Term Capital Gain 42,00,000.00
Assessee has the option either not to avail exemption u/s 54B or to avail exemption u/s 54B.
Option I Exemption is not availed:
Long Term Capital Gain: Rs. 42,00,000
Capital gains on sale of Urban agricultural land
Full value of consideration 11,00,000
Less: Cost of acquisition (10,00,000)
Short Term Capital Gain 1,00,000

Option II Exemption is availed


Long Term Capital Gain 42,00,000
Less: Exemption u/s 54B (10,00,000)
Long Term Capital Gain 32,00,000

Capital gains on sale of Urban agricultural land


Full value of consideration 11,00,000
Less: Cost of acquisition (10,00,000 - 10,00,000) Nil
Short Term Capital Gain 11,00,000

Hence the assessee should opt for option-I & his tax liability shall be 7,84,160.

*Q52. Mr. X owns several assets but does not own any residential house. He sells the following assets & requests
you to compute his tax liability for the AY 2019-20.
1. Shares (non-listed) purchased in April 2007 for Rs. 1,30,000 sold on 19.07.2018 for Rs. 12,00,000.
2. On 01.04.2001, he had agreed to sell the jewellery to Mr. Y for Rs. 3,50,000 which was purchased in 1996 for
Rs. 1,80,000. However, the sale could not be effected as Mr. Y backed out. He now sold the jewellery on
15.08.2018 for Rs. 18,00,000 & incurred Rs. 30,000 incidental selling expenses on account of brokerage &
commission.
In December 2018, he also purchased a small residential house for Rs. 2,00,000.
He has deposited Rs. 1,60,000 on 20.01.2019 in deposit account with a public sector bank under capital gains
deposit scheme for construction on the house which he has purchased in December 2018.
On 15.01.2019, he invested Rs. 2,50,000 in the bonds issued by National Highway Authority of India which are
redeemable after 5 years.
3. Debentures (unlisted) purchased in April 2017 for Rs. 80,000 sold on 31.12.2018 for Rs. 1,40,000.
4. Sold his motor car purchased in August 2007 for Rs. 1,50,000 on 15.03.2019 for Rs. 18,000.
5. He purchased equity shares of ABC Limited on 01.11.2017 for Rs. 2,00,000 & sold all the shares on 1.6.2018
for Rs. 10,00,000 & has paid STT @ 0.25% of sale price. Compute his income tax liability.

DT QUESTION BANK BY CA PRANAV CHANDAK Subscribe us on YouTube 115


Solution: Computation of capital gains
Option 1: Section 54F Exemption taken from Shares
1. Shares
Full value of consideration 12,00,000.00
Less: Indexed cost of acquisition = 1,30,000 / 129 x 280 = 72,82,170.54 (2,82,170.54)
Long term capital gain 9,17,829.46
Less: Exemption u/s 54F = 9,17,829.46 / 12,00,000 x 3,60,000 = 2,75,348.84 (2,75,348.84)
Less: Exemption u/s 54EC (2,50,000.00)
Long term capital gain 3,92,480.62
2. Jewellery
Full value of consideration 18,00,000.00
Less: Selling Expenses (30,000.00)
Less: Indexed cost of acquisition = 3,50,000 / 100 x 280 = 79,80,000 (9,80,000.00)
Long term capital gain 7,90,000.00
3. Debentures
Full value of consideration 1,40,000.00
Less: Cost of acquisition (80,000.00)
Short term capital gain 60,000.00
4. Motor car: is covered under the personal movable effects, hence, no capital gains shall be computed
5. Equity Shares
Full value of consideration 10,00,000.00
Less: Cost of acquisition (2,00,000.00)
Short term capital gain u/s section 111A 8,00,000.00
Income under the head Capital gain
Long Term Capital Gains 11,82,480.62
Short Term Capital Gains 8,60,000.00
Gross Total Income 20,42,480.62

Option 2: Section 54F Exemption taken from Jewellery


1. Shares
Full value of consideration 12,00,000.00
Less: Indexed cost of acquisition = 1,30,000/129 x 280 = Rs. 2,82,170.54 (2,82,170.54)
Long term capital gain 9,17,829.46
Less: Exemption u/s 54EC (2,50,000.00)
Long term capital gain 6,67,829.46
2. Jewellery
Full value of consideration 18,00,000.00
Less: Indexed cost of acquisition = 3,50,000 / 100 x 280 = Rs. 9,80,000 (9,80,000.00)
Less: Selling Expenses (30,000.00)
Long term capital gain 7,90,000.00
Less: Exemption u/s 54F = 7,90,000/17,70,000 x 3,60,000 = 1,60,677.97 (1,60,677.97)
Long term capital gain 6,29,322.03
3. Debentures
Full value of consideration 1,40,000.00
Less: Cost of acquisition (80,000.00)
Short term capital gain 60,000.00
4. Motor car: is covered under the personal movable effects, hence, no capital gains shall be computed
5. Equity Shares
Full value of consideration 10,00,000.00
Less: Cost of acquisition (2,00,000.00)
Short term capital gain u/s section 111A 8,00,000.00
Income under the head Capital gain
(i) LTCG = 12,97,151; (ii) STCG: Rs. 8,60,000. Thus Gross Total Income = Rs. 21,57,151.

Conclusion: Option 1 is better.

DT QUESTION BANK BY CA PRANAV CHANDAK Subscribe us on YouTube 116


4E. INCOME FROM OTHER SOURCES
Q1. Mr. PC (25 years) gives following information for PY 2018-19:
1. He gets gift of House A from his friend Goldy (stamp duty value is determined at Rs. 6,00,000).
2. He gets gift of House B from Mr. Raju (who is father-in-law of his younger brother) (SDV is Rs. 40,000 & FMV is
Rs. 65,000).
3. Mr. PC purchases a second-hand car for Rs. 70,000 from Babbu (FMV is Rs. 3,00,000).
4. Mr. PC purchases a work of art for Rs. 5,00,000 from Suraj (FMV is Rs. 5,30,000).
5. Mr. PC purchases jewellery for Rs. 7,00,000 from F (FMV is Rs. 7,25,000). F is not a registered dealer.
6. Mr. PC purchases a painting for Rs. 4,00,000 from GD (who is brother of Mrs. PC) (FMV is Rs. 7,00,000).
7. Mr. PC purchases a commercial property for Rs. 1,16 lacs from Bharat (FMV is Rs. 220 lacs).
8. Mr. PC gets a gift of 100 preference shares in A Ltd. from SS (Stock exchanges are closed on this date & lowest
quotation on immediately preceding working day in National Stock Exchange is Rs. 450).
9. Mr. PC gets a gift of Government security from K (K is husband of Mr. PC father's sister) (FMV is Rs. 4 lacs).
10. Mr. PC gets a gift cheque of Rs. 1,00,000 from his friend Gattu on his birthday.
11. Chota PC, minor son of Mr. PC gets the gift of Rs. 55,000 from elder brother of Mr. PC's Grandfather).
Determine the amount of income taxable u/h “IFOS” in the hands of Mr. PC for AY 2019-20.
Solution: Computation of income taxable u/h "Income from other sources":
Category Transactions Taxable
1. Cash Gifts (i) Gift from a friend : Rs. 1,00,000
(ii) Gift received by minor son (taxable in hands of Mr. PC): Rs.
55,000 – Rs. 1500 Exempt u/s 10(32) Rs. 1,53,500
2A. Movable Property (i) Gift of Government securities - It is gift received from a
received without "relative" & not taxable.
consideration (ii) Gift of 100 preference shares : Rs. 45,000.
(As there is no other gift chargeable to tax & aggregate FMV is
not more than Rs. 50,000, it is not chargeable to tax) Nil
2A. Movable Property (i) Purchase of car - It is not covered & thus not taxable.
received for (ii) Purchase of painting from relative – Not taxable
inadequate (iii) Purchase of work of art: Rs. 30,000 [530000 – 500000]
consideration (iv) Purchase of jewellery : Rs. 25,000 [725000 – 700000]
Total Discount: Rs. 55,000 (Aggregate discount > Rs. 50,000, Rs. 55,000
discount is taxable)
3. Gift of House A Received from a friend & SDV > Rs. 50,000, SDV is taxable Rs. 6,00,000
4. Gift of House B Received from C who is not "relative" of X. However, nothing is Nil
taxable, as stamp duty value is not more than Rs. 50,000
5. Purchase of Purchase of an immovable property for a consideration which is Rs. 104 lacs
commercial property lower than stamp duty value, (stamp duty value exceeds 105% of
for inadequate consideration) is chargeable to tax.
consideration Discount = Rs. 220 lacs – Rs. 116 lacs = Rs. 104 lacs is taxable
Total Rs. 1,12,08,500

Q2. Smt. Laxmi reports the following transactions -


1. Received Cash Gifts on her marriage on 18.7.2018 of 1,20,000 including 20,000 received from non-relatives.
2. On 1.8.2018 being her birthday, she received a gift of 40,000 by cheque from her mother's maternal uncle.
3. On 1.12.2018, she acquired a vacant site from her friend for Rs. 1,05,000. SDV is Rs. 2,80,000.
4. She bought 100 Equity Shares of a Listed Company from another friend for Rs. 60,000. FMV was Rs. 1,15,000.
5. A cell phone worth 21,000 is gifted by his friend on 16.8.2018.
Determine the amounts chargeable to tax in the hands of Smt. Laxmi for AY 2019-20.
Answer:
1. Gift received from Relatives or Friends on the occasion of marriage of individual is not taxable. Thus Nothing
will be taxable in this case.
2. Gift received from a Relative is exempt from tax. However, Relative does not include mother’s maternal uncle.
But sum of money does not exceed Rs. 50,000. Hence Nothing will be taxable.
3. If Immovable Property is acquired for inadequate consideration, discount is taxable if the difference between
SDV & Consideration exceeds Higher of (i) Rs. 50,000 or (ii) 5% of Consideration.
Hence, taxable amount is Rs. 2,80,000- Rs. 1,05,000 = Rs. 1,75,000.
4. If any property is acquired for inadequate consideration, it is taxable if the difference between its FMV &
Consideration exceeds Rs. 50,000. Hence, taxable amount is Rs. 1,15,000 – Rs. 60,000 = Rs. 55,000.
5. Cell phone being a personal effect is not a Movable Property as defined u/s 56(2)(x). Thus nothing is taxable.

DT QUESTION BANK BY CA PRANAV CHANDAK Subscribe us on YouTube 117


Q3. Discuss the taxability of the given transactions under the Income Tax Act , 1961.
1. Cash Gift of Rs. 51,000 received from her friend on the occasion of her "Shastiaptha Poorthi", a wedding
function celebrated on her husband completing 60 years of age. This was also her 25 th Wedding Anniversary.
2. On the above occasion, a diamond necklace worth Rs. 2 Lacs was presented by her sister living in Dubai.
3. When she celebrated her daughter's wedding on 21.02.2019, her friend assigned in Mrs. Hemali's favour a
Fixed Deposit held by the said friend in a Scheduled Bank, the value of the Fixed Deposit & the accrued interest
on the said date was Rs. 51,000.
4. Received a car from his friend on payment of Rs. 2,50,000, the FMV of which was Rs. 5,50,000.
5. Rs. 51,000 received from his sister living in US on 1.6.2018.
Compute the Income assessable u/h “Income from Other Sources” for AY 2019-20.
Answer: Computation of Income from Other Sources
1. Gift received from any other person other than a Relative is taxable. So, gift received from friend is taxable.
(Shastiaptha Poorthi is not a marriage occasion). Rs. 51,000 is taxable.
2. Gift received from a Relative is exempt from tax.
3. Gift received from Relatives or Friends on the occasion of marriage of individual is not taxable. However, here
the gift is received on the marriage occasion of Assessee’s daughter. Hence Rs. 51,000 is taxable in the hands of
Assessee.
4. Motor Car is not a Movable Property as defined u/s 56(2)(x). Thus nothing will be taxable in this case.
5. Gift received from any other person other than a Relative is taxable. Since, gift is received from Sister, the same
is not taxable.

Q4. Discuss the taxability of the following in the hands of the recipient u/s 56(2)(x): [NOV 13]
(i) Shivam HUF gifted a car to daughter of Karta for winning the first prize in all India music competition.
(ii) Mrs. Parth received 200 shares of ABC Ltd. from her friend as a gift on occasion of her 50th marriage
anniversary. The FMV on that date was 150 per share. She also received jewellery worth Rs. 35,000 (FMV) from
her niece on the same day.
(iii) Manish HUF received Rs. 55,000 in cash from nephew of Manish (son of Manish’s sister). Manish is the Karta
of Manish HUF.
(iv) Shivang, a member of his father’s HUF, transferred a house property to the HUF without consideration. The
stamp duty value of the house property is 12,00,000.
(v) Sharma’s son transferred shares of XYZ Ltd. to Sharma HUF without any consideration. The FMV of the shares
is 3.25 Lac.
(vi) Mr. Tejpal received a painting by M. F. Hussain worth 2 Lac from his nephew on his 10th wedding anniversary.
Answer:
(i) “Car” is not included in the definition of property for Sec 56(2)(x), therefore, the same shall not be taxable.
(ii) As per the provisions of section 56(2)(vii), in case the aggregate FMV of property of the movable property
received without consideration exceeds Rs. 50,000, Aggregate FMV shall be taxable.
In this case, aggregate FMV of the shares (Rs. 30,000) & jewellery (Rs. 35,000) exceeds Rs. 50,000 (Niece is not
covered within the definition of relative), hence entire amount of Rs. 65,000 shall be taxable.
(iii) Sum of money exceeding Rs.50,000 received without consideration from a non-relative is taxable u/s
56(2)(x). Son of Mr. Manish’s sister is not a relative of Manish HUF, since he is not a member of Manish HUF.
(iv) Immovable property received without consideration by a HUF from its relative is not taxable u/s 56(2)(x).
Since Shivang is a member of the HUF, he is a relative of the HUF.
(v) Any property received without consideration by a HUF from its relative is not taxable u/s 56(2)(x).
Since Sharma’s son is a member of Sharma HUF, he is a “relative” of the HUF. Therefore, if Sharma HUF
receives any property (shares, in this case) from its member, i.e., Sharma’s son, without consideration, then,
the FMV of such shares will not be chargeable to tax in the hands of the HUF, since gift received from a
“relative” is excluded from the scope of section 56(2)(x).
(vi) Paintings are included in the definition of “property”. Thus when paintings are received without
consideration & aggregate FMV of paintings exceed Rs. 50,000, FMV shall be taxable u/s 56(2)(x).
Therefore, Rs. 2,00,000, being the value of painting gifted by his nephew, would be taxable u/s 56(2)(x) in the
hands of Mr. Tejpal, since “nephew” is not included in the definition of “relative”.

Q5. Mr. Chezian is employed in a Company with Taxable Salary Income of Rs. 5,00,000. He received a Cash Gift of
1 lac from Atma Chartiable Trust (registered u/s 12AA) in December for meeting his Medical Expenses. Is the Cash
Gift so received from the Trust chargeable to Tax in the hands of Mr. Chezian? [May-11]
Answer:
▪ Amount received as Gift from any Trust / Institution registered u/s 12AA is exempt u/s 56.
▪ So the amount of Rs. 1,00,000 is exempt from tax.

DT QUESTION BANK BY CA PRANAV CHANDAK Subscribe us on YouTube 118


Q6. Mr. Vishnu received Gift (Scholarship) of Rs. 2 lacs received from Young CA’s association towards financial
assistance for pursuing CA classes . The association is not registered u/s. 12AA of the Income – tax Act. Discuss.
Answer:
▪ Gift received from registered charitable trust is not taxable.
▪ However in this case, the association is not registered & thus the amount of Rs. 2 lacs is taxable in the hands of
Mr. Vishnu.

Q7. Discuss the taxability of the above transactions in case of the company assessee: [MAY 15]
(i) Sunnyvale (P) Ltd. purchased 10,500 equity shares of Solid (P) Ltd. at 95 per share. FMV of the share on the
date of transaction is Rs. 115.
(ii) Balaji (P) Ltd. issued 26,000 equity shares of Rs. 10 each at a premium of 7. FMV on the date of issue is Rs. 13.
*(iii) RAU (P) Ltd. issued 30,000 equity shares of 10 @ Rs. 9. FMV of each share on the date of issue is Rs. 5.
Answer:
(i) Difference b/w aggregate FMV of shares of closely held company & consideration paid for purchase of
such shares = Income of the purchasing company u/s 56(2)(viia), if the difference exceeds Rs. 50,000.
Accordingly, in this case, the difference of Rs. 2,10,000 [i.e.,(Rs. 115 –Rs. 95) × 10,500] is taxable u/s
56(2)(viia) in the hands of Sunnyvale (P) Ltd.
(ii) The provisions of section 56(2)(viib) are attracted since the shares of a closely held company are issued at
a premium & issue price exceeds the FMV of such shares.
Consideration received by the company in excess of FMV of the shares would be taxable u/s 56(2)(viib).
Therefore, Rs. 1,04,000 [i.e., Rs.17 – Rs.13) x 26,000 shares] shall be the income chargeable u/s 56(2)(viib) in the
hands of Balaji (P) Ltd.
(iii) The provisions of section 56(2)(viib) are not attracted in this case since shares of a closely held company
are not issued at a premium.
Thus even if issue price exceeds the FMV of the shares, nothing shall be taxable since section 56(2)(viib) is not
attracted.

Q8. During PY 2017-18, Mr. Gagan received Rs. 5,32,000 towards interest on enhanced compensation from State
Government in respect of compulsory acquisition of his land effected during FY 2012-13.
The above amount of interest include interest relating to the following financial years:
FY 2014-15 1,58,000 FY 2015-16 1,78,000 FY 2016-17 1,96,000
He incurred Rs. 75,000 by way of legal expenses in FY 2016-17 to receive interest on such enhanced compensation.
Determine how much of interest on enhanced compensation would be taxable for AY 2019-20? Can he claim
deduction in respect of legal expenses from the amount of interest on enhanced compensation? [N14/M17 &M14]
Solution:
▪ Section 145A provides that interest received by the assessee on enhanced compensation shall be deemed to be
the income of the assessee of the year in which it is received, irrespective of the method of accounting followed
by the assessee & irrespective of the financial year to which it relates.
▪ Section 56(2)(viii) states that such income shall be taxable as ‘Income from other sources’
▪ 50% of such income shall be allowed as deduction by virtue of section 57(iv).
▪ Therefore, he cannot claim deduction in respect of legal expenses incurred to receive the interest on enhanced
compensation from such income.
Computation of interest on enhanced compensation taxable u/h “IFOS” for AY 2017-18:
Particulars Rs.
Interest on enhanced compensation taxable u/s 56(2)(viii) 5,32,000
Less: Deduction u/s 57(iv) (50% x Rs. 5,32,000) 2,66,000
Taxable interest on enhanced compensation 2,66,000

Q9. Mr. A received an Advance of Rs. 50,000 on 1.9.2018 against the sale of his house. However, due to non-
payment of instalment in time, the Contract has cancelled & the amount of Rs. 50,000 was forfeited.
Answer: Advance Forfeited on failed negotiation for transfer of Capital Asset is Taxable under “Income from
Other Sources” u/s 56 in the hands of Mr. A.

Q10. Rahul holding 28% of Equity Shares in a Company took a Loan of Rs. 5,00,000 from the same Company. On
the date of granting the Loan, the Company had accumulated Profit of Rs. 4,00,000. The Company is engaged in
some Manufacturing Activity.
(i) Is the amount of Loan taxable as Deemed Dividend in the hands of Rahul, if the Company is a Company in
which the Public are Substantially Interested?
(ii) What would be your answer, if the lending Company is Private Limited Company (i.e) a Company in which
the Public are not Substantially Interested? [May 2011]
Answer:

DT QUESTION BANK BY CA PRANAV CHANDAK Subscribe us on YouTube 119


(i) The amount of Deemed Dividend is not taxable in the hands of Rahul, since the Company is not a Company in
which the Public are not Substantially Interested.
(ii) In this case, the Company is a Company in which Public are not Substantially Interested & Rahul holds more
than 10% of Equity Shares (i.e. Voting Power). Hence, the amount is Taxable as Deemed Dividend u/s 2(22)(e) to
the extent of accumulated profits i.e. Rs. 4,00,000.

Q11. Mr. Rakesh has 15% Shareholding in RSL (P) Ltd & has also 50% Share in Rakesh & Sons, a Partnership
Firm. The Accumulated Profit of RSL(P) Ltd is Rs. 20 Lacs. Rakesh & Sons had taken a Loan of Rs. 25 Lacs from
RSL (P) Ltd. Explain, whether the above loan is treated as Dividend, as per the provisions of Income Tax Act,
1961. [NOV 16]
Answer:
▪ RSL(P) Ltd is a Company in which Public are not substantially Interested. It has Accumulated Profit of 20 Lac.
▪ Mr. Rakesh’s Shareholding in RSL(P) Ltd > 10% & Mr. Rakesh’s Share in the Partnership Firm > 20%.
▪ Thus all the conditions given u/s 2(22)(e) are satisfied.
▪ Out of the Loan of Rs.25 Lac, Rs.20 Lac shall be treated as Deemed Dividend u/s 2(22)(e).

*Q12. Mr X has the following income for AY 2018-19. Compute the tax payable by Mr. X. [PC]
Salary: Rs 6,00,000 (computed) House Property: Rs. 3,00,000
Dividends from domestic company: Rs. 14 Lacs LTCG( STT paid): Rs. 50,000
LTCG (STT not paid): Rs. 3 Lacs. STCG : 2,00,000
Solution: Computation of total income of X for AY 2018-19
Income under the head salary (computed) 6,00,000
Income under the head house property (computed) 3,00,000
IFOS (Dividend) 14,00,000
Less: Exemption u/s 10(34)/ Section 115BBDA (10,00,000) 4,00,000
Capital Gains
LTCG (STT paid) – Exempt since < Rs. 1,00,000 Nil
LTCG (STT not paid) 3 Lacs
STCG (Normal) 2 Lacs 5,00,000
Gross total income 18,00,000
Computation of Tax payable
Tax on Dividends exceeding Rs. 10,00,000 [Section 115BBDA] (400000 x 10%) 40,000
Tax on LTCG @ 20% = (3 lacs * 20%) 60,000
Tax on balance total income of Rs. 11,00,0000 1,42,500
Total Tax payable 2,42,500

Q13. Bharat Hurkat, a resident individual, submits the following particulars of his income for PY 2018-19.
(i) Royalty from a coal mine Rs. 15,000
(iii) Salary as a member of Parliament Rs. 5,00,000
(iv) Daily allowance as a member of Parliament Rs. 2,50,000
(v) Dividend received from a cooperative society Rs. 50,000
(vi) He has incurred the following expenses
(a) Paid collection charges for collecting dividends Rs. 1,000
(b) Amount spent for earning & collecting royalty income Rs. 4,000
Compute his “Income from other sources” for the AY 2019-20.
Solution:
Royalty from coal mine Rs. 15,000
Less: Collection charges (Rs. 4,000) Rs. 11,000
Salary as a member of Parliament Rs. 5,00,000
Daily allowance as a member of Parliament – Exempt Nil
Dividend from a Co-operative Society Rs. 50,000
Less: Collection charge (Rs. 1,000) Rs. 49,000
Total Income u/h IFOS Rs. 5,60,000

Q14. Mr. Hari, a property dealer, sold a building in the course of his business to his friend Rajesh, who is a dealer
in automobile spare parts, for Rs. 90 lac on 1.1.2019, when the SDV was Rs. 150 lac. The agreement was, however,
entered into on 1.9.2018 when the SDV was Rs. 140 lac. Mr. Hari had received a down payment of Rs. 15 lac by a
crossed cheque from Rajesh on the date of agreement. Discuss the tax implications in the hands of Hari & Rajesh,
assuming that Mr. Hari has purchased the building for Rs. 75 lac on 12th July, 2017.
Would your answer be different if Hari was a share broker instead of a property dealer?

DT QUESTION BANK BY CA PRANAV CHANDAK Subscribe us on YouTube 120


Solution:
Case 1: Tax implications if Mr. Hari is a property dealer
In the hands of Mr. Hari In the hands of Mr. Rajesh
Section 43CA would be attracted, since Since Mr. Rajesh is a dealer in automobile spare parts, building
the building represents his SIT & he has purchased would be a capital asset in his hands.
transferred it for less than SDV on the The provisions of section 56(2)(x) would be attracted in the hands of
date of agreement. Mr. Rajesh who has received immovable property, being a capital
Therefore, 65 lac, being difference b/w asset, for inadequate consideration.
SDV on date of agreement [(Rs. 140 lac) Therefore, Rs. 60 lac, being difference b/w SDV of the property on the
& the purchase price (Rs. 75 lac)], would date of registration (i.e., Rs. 150 lac) & the actual consideration (i.e.,
be taxable as business income in the Rs. 90 lac) would be taxable u/s 56(2)(x) in the hands of Mr. Rajesh,
hands of Mr. Hari. since the payment is made by crossed cheque & not A/c payee
cheque/draft or ECS.

Case 2: Tax implications if Mr. Hari is a stock broker


In the hands of Mr. Hari In the hands of Mr. Rajesh
In case Mr. Hari is a stock broker & not a property dealer, the There would be no difference in the taxability
building would represent his capital asset & not stock-in-trade. in the hands of Mr. Rajesh, whether Mr. Hari is
In such a case, the provisions of section 50C would be attracted a property dealer or a stock broker.
in the hands of Mr. Hari & Rs. 75 lac, being the difference b/w
Therefore, the provisions of section 56(2)(x)
SDV on date of registration (i.e., Rs. 150 lac) & purchase price
would be attracted in the hands of Mr. Rajesh
(i.e., Rs. 75 lac) would be taxable as STCG.
who has received immovable property, being a
Note: It may be noted that u/s 50C, the option to adopt the SDV capital asset, for inadequate consideration.
on the date of agreement can be exercised only if whole or part
Therefore, Rs. 60 lac, being difference b/w SDV
of the consideration has been received on or before the date of
of the property on the date of registration (i.e.,
agreement by way of account payee cheque or draft or by use of
Rs. 150 lac) & actual consideration (i.e., Rs. 90
ECS through a bank account on or before the date of agreement.
lac) would be taxable u/s 56(2)(x) in the hands
In this case, since the payment is made by crossed cheque, the
of Mr. Rajesh.
option cannot be exercised.
Note:
▪ As per section 43CA, SDV on the date of agreement can be adopted, if whole or part of consideration is received
otherwise than by way of cash on or before the date of agreement.
▪ However, both section 50C & 56(2)(x) permit adoption of stamp value duty on the date of agreement only if
whole or part of consideration is received/paid, as the case may be, by way of account payee cheque or account
payee bank draft or by use of ECS through a bank account.

Q15. Ms. Chhaya transferred a vacant site to Ms. Dayama for Rs. 4,25,000. The Stamp Valuation Authority fixed
the value of vacant site for Stamp Duty purpose at Rs. 6,00,000. The Total Income of Chhaya & Dayama before
considering the transfer of vacant site are Rs. 50,000 & Rs. 2,05,000 respectively. The Indexed Cost of Acquisition
for Ms. Chhaya in respect of vacant site is Rs. 4,00,000 (computed). Determine the Total Income of both Ms.
Chhaya & Ms. Dayama taking into account the above said transaction. [May 11]
Solution: Computation of Total Income of Ms. Chayya
I. Long Term Capital Gains:
Sale Consideration [Amount fixed by the Stamp Valuation Authority] 6,00,000
Less: Indexed Cost of Acquisition – Computed 4,00,000
Long-term Capital Gains 2,00,000
II. Other income 50,000
Total income 2,50,000
Computation of Total Income of Ms. Dayama
Income from Other Sources: Gift (See Note) [6,00,000 – 4,25,000] 1,75,000
Other Income 2,05,000
Total Income 3,80,000
Note: If Immovable Property is received for inadequate consideration, & shortfall / inadequacy exceeds Rs.50,000,
then, the Taxable Value of Gift is the difference between the Stamp Duty Value & Consideration paid.

Q16. Mr. Y submits the following information pertaining to the year ended 31.03.2019:
(a) On 30.11.2018, when he attained the age of 60, his friends in India gave a fiat at Surat as a gift, each
contributing a sum of Rs. 20,000 in cash. The cost of the flat purchased using the various gifts was Rs. 3.4 Lacs.
(b) His close friend in abroad sent him a Cash Gift of Rs. 75,000 through his relative, for the above occasion.

DT QUESTION BANK BY CA PRANAV CHANDAK Subscribe us on YouTube 121


(c) Mr. Y sold the above flat on 30.01.2019 for Rs. 3 Lacs. The Registrar's valuation for stamp duty purposes was
Rs. 3.7 Lacs. Neither Mr. Y nor the buyer, questioned the value fixed by the Registrar.
(d) He purchased some Equity Shares in X Pvt Ltd (unlisted)on 05.02.2018 for Rs. 3.5 Lacs, which were sold on
15.03.2019 for Rs. 3.20 Lacs.
You are requested to calculate the Total Income of Y for AY 2019-2020. [M 05]
Solution: Computation of Total Income
1. Short Term Capital Gain:
Full Value of consideration [SDV using Section 50C] 3,70,000
Less: Cost of Acquisition (Cost to Previous Owner since the asset is received by Gift) (3,40,000) 30,000
2. Long Term Capital Gain:
Full Value of Consideration 3,20,000
Less: Indexed Cost of Acquisition (Rs. 3,50,000 x 280/272) (3,60,294)
Long Term Capital Loss (To be carry fwd. & set-off only against LTCG) (40,294)
Income from Other Sources
Gift Received by way of Flat (Gift received in Kind or Cash is taxable) 3,40,000 4,15,000
Gift from Friend (Fully taxable as aggregate amount exceeds Rs. 50,000) 75,000
Gross Total Income 4,45,000
Less: Deduction under Chapter VI-A NIL
Total Income 4,45,000
Note: LTCL can be set-off only against LTCG. It shall be carried forward for 8 AYs & can be set-off only against LTCG

Q17. Following is the Profit & Loss Account of Mr. A, a Dealer in Shares & Securities for year ended 31.03.2019 -
Particulars Rs. Particulars Rs.
To Trading Expenses 1,87,80,000 By Sales 2,17,62,000
To Administrative Expenses 3,15,000 By Interest on FD with Bank 49,500
To Financial Expenses 1,44,795 By Dividend from Indian Co 1,93,080
To Demat & Delivery Charges 13,050 By Interest on IT Refund (AY 2012-13) 690
To Securities Transaction Tax 16,500
To Net Profit before Depreciation 27,35,925
Compute the Tax Liability of the Assessee. [N06]
Solution: Computation of Total Income & Tax Liability
Particulars Rs. Rs. Rs.
1. Profits & Gains from Business or Profession
Net Profit before Depreciation 27,35,925
Less: Items to be considered u/h “IFOS”
Interest on FD (49,500)
Dividend from Indian Company (1,93,080)
Interest on IT Refund (690) (2,43,270) 24,92,655
2. Income from Other Sources
(i) Income from Dividend – Exempt u/s 10(34) Nil
(ii) Interest on FD 49,500
(iii) Interest on IT Refund 690 50,190
Gross Total Income 25,42,845
Less: Deduction under Chapter VI-A Nil
Total Income (Rounded off) 25,42,850
Tax on Total Income [1,12,500 + (25,42,850 - 10,00,000) x 30%] 5,75,355
Add: HEC at 4% 23,014.4
Net Tax Payable (Rounded off) 5,98,370
Note: Securities Transaction Tax is allowed as a deduction u/s 36(1)(xv).

DT QUESTION BANK BY CA PRANAV CHANDAK Subscribe us on YouTube 122


MASTER QUESTION ON INCOME FROM OTHER SOURCES
MQ18. Mr. X furnishes the following particulars of his incomes for the PY 2018-19. Compute his IFOS.
Dividend on equity shares Rs. 600
Dividend on preference shares Rs. 3,200
Dividend from a foreign company Rs. 5000
Dividend from UTI Rs. 2,000
Dividends received from Assam Tea Ltd. (60% of income is agricultural Income). Rs. 3,600
Income from agricultural land in Pakistan Rs. 10000
Interest on Government Securities (Net) Rs. 18000
Interest on deposits with Industrial Finance Corporation of India Rs. 4500
Winning from Horse Race (Gross) Rs. 13000
Rent from sub-letting a house Rs. 26250
Rent payable by Mr. Mohan for the sub-let house Rs. 12000
Other expenses on sub-let-house Rs. 1000
Income from letting on hire of building & machinery under one composite lease Rs. 17,000
Interest on Bank Deposits Rs. 2,500
Directors sitting fees received Rs. 1,200
Ground rent received from Land in Pathankot Rs. 600
Income from undisclosed sources Rs. 10,000
Amount received on account of winnings from lotteries Rs. 14,000
Remuneration for taking Lectures in Seminars Rs. 15,000
Interest on loan given to a relative Rs. 12,000
Interest on debenture of a Listed Company (Net of Taxes) Rs. 36,000
Interest on Post Office Saving Bank Account Rs. 5,000
Interest on Deposit Certificate issued under the Gold Monetisation Scheme Rs. 11,000
Interest on Monthly Income Scheme of Post Office Rs. 1500
The following deductions are claimed by him
(a) Collection charges of preference dividend Rs. 200
(b) Allowable depreciation on Building & Machinery Rs. 4,000
(c) Fire Insurance on Building & Machinery Rs. 100
(d) Amount spent for buying lottery ticket Rs. 500
Solution: Computation of Income u/f IFOS in the hands of Mr. X
Dividend on equity & preference shares Exempt
Dividend from a foreign company Rs. 5,000
Dividend from UTI Exempt
Dividends received from Assam Tea Ltd. (Since it is Indian company). Exempt
Income from agricultural land in Pakistan Rs. 10,000
Interest on Securities (after grossing up @ 10%) [18000/90%] Rs. 20,000
Interest on deposits with Industrial Finance Corporation of India [4500/90%] Rs. 5,000
Winning from Horse-Race [Gross amount is given, so no need to gross up again] Rs. 13,000
Rent from sub-letting a house [ Rs 26250-12000 -1000] Rs. 13,250
Income from letting of machinery & building after Expense [Rs.17,000 - (4,000 + 100)] Rs. 12,900
Interest on Bank Deposits Rs. 2,500
Directors sitting fees received Rs. 1,200
Ground rent received from Land in Pathankot Rs. 600
Income from undisclosed sources Rs. 10,000
Amount received on account of winnings from lotteries Rs. 14,000
Remuneration for taking Lectures in Seminars Rs. 15,000
Interest on loan given to a relative Rs. 12,000
Interest on debenture of a Listed Company [Rs. 36,000/90%] Rs. 40,000
Interest on Post Office Saving Bank Account [Rs. 5,000 - Exempt u/s 10(15)] Rs. 1,500
Interest on Deposit Certificate issued under the Gold Monetisation Scheme - Exempt u/s 10(15) Nil
Interest on Monthly Income Scheme of Post Office Rs. 1500
Notes:
(i) Amount spent for buying lottery ticket is not deductible. (Winning from lottery is taxable @ 30%)
(ii) Collection charges of preference dividend are not deductible since dividend income is exempt.
(iii) Agricultural income from land situated in India is exempt u/s 10.
(iv) HEC @ 4% is not to be considered for TDS on payments to Residents.
(v) Interest on Debentures: Interest on Debentures is grossed up as TDS is deductible @ 10% on such interest.

DT QUESTION BANK BY CA PRANAV CHANDAK Subscribe us on YouTube 123


5. CLUBBING OF INCOME
Q1. Mr. Madhav made a gift of Rs. 2,50,000 to his handicapped son, Master Tapan (age 12 years) which he
deposited in a fixed deposit A/c @ 10% interest p.a. compounded annually. The balance in this account as on 1 st
April, 2018 was Rs. 2,75,000 & bank credited Rs. 27,500 as interest on 31st March 2019.
Madhav's father gifted equity shares worth Rs. 50,000 of an Indian company to Master Manan, another son of Mr.
Madhav (Date of Birth: 10.4.2011) in July 2011 which were purchased by him on 8 th Dec 2005 for Rs. 80,000.
Manan received a dividend of Rs. 5,000 on these shares in Oct 2018.
He sold these shares on 1.11.2018, for Rs. 5,00,000 & deposited Rs. 3,00,000 in a company at 15% interest p.a.
Mr. Madhav has a taxable income of Rs. 3,50,000 from his profession during PY 2018-19.
Compute his GTI for AY 2019-20. [CII: FY 2005-06: 117; FY 2011-12: 184; FY 2018-19: 280] [MAY-18]
Solution: Computation of Total Income
Particulars Rs
Income from Profession 3,50,000
Interest Income of Master Tapan [No clubbing since suffering from disability u/s 80U] Nil
Dividend Income of Master Manan [Dividend is exempted u/s 10(34), hence no clubbing] Nil
Capital Gain Income of Manan [Refer Note Below] 3,08,547
Interest Income of Manan (Rs. 3,00,000 x 15% × 5/12) 18,750
Less: Exemption u/s 10(32) (1,500)
Gross Total Income 6,75,800
Note: Computation of Capital gains [It is assumed that shares are not transacted in RSE & no STT is paid]
Full value of Consideration 5,00,000
Less: Indexed Cost of Acquisition (80,000 x 280/ 117) 1,91,453
Long Term Capital Gains 3,08,547

Q2. Kamal gifted Rs. 10 Lacs to his wife, Sulochana on her birthday on, 1st Jan 2018. Sulochana lent Rs. 5,00,000
out of the gifted amount to Krishna on 1st April, 2018 for 6 months on which she received interest of Rs. 50,000.
The said sum of Rs. 50,000 was invested in shares of a listed company on 15 th Oct 2018, which were sold for Rs.
75000 on 30th Dec 2018. STT was paid on such sale. The balance amount of gift was invested as capital by
Sulochana in a business. She suffered loss of Rs. 15,000 in the business in FY 2018-19. In whose hands the above
income & loss shall be included in AY 2019-20. [NOV-2017]
Answer: As per section 64(1),
▪ If any person has transferred any asset to his or her spouse without adequate consideration in such case Income
shall be clubbed in the income of the transferor, hence Interest income of Rs. 50,000 shall be clubbed in the
income of Mr. Kamal.
▪ If asset received by the spouse has been invested in the proprietor business, income from the business shall be
clubbed in the income of transferor & if there is any loss, it will also be clubbed.
▪ In the given case there is a loss of Rs. 15,000 from business, such loss shall be clubbed in the income of Mr.
Kamal.
▪ If any person has transferred the asset to the spouse, income from the asset shall be clubbed but if same income
is invested further, any subsequent income shall not be clubbed as decided in the case of M.P. BIRLA.
▪ In the given case, Mrs. Sulochana has invested interest income in the shares & there was capital gain on the sale
of shares, such capital gain shall not be clubbed rather it will be taxable in the hands of Mrs. Sulochana.

Q3. Mrs. X transferred her immovable property to ABC Co. Ltd. subject to a condition that out of the rental income,
a sum of Rs. 36,000 per annum shall be utilized for the benefit of her son’s wife.
Mrs. X claims that the amount of Rs. 36,000 (utilized by her son’s wife) should not be included in her total income
as she no longer owned the property. State with reasons whether the contention of Mrs. X is valid in law. [N-10]
Answer:
▪ Clubbing provisions are attracted in case of transfer of any asset otherwise than for adequate consideration, to
any person to the extent to which income from such asset is for immediate or deferred benefit of son’s wife.
▪ Such income shall be included in computing the total income of the transferor-individual.
▪ Therefore, income of Rs. 36,000 for benefit of daughter-in-law is taxable in hands of transferor (Smt. X).
▪ The contention of Smt. X is, hence, not valid in law.

Q4. Mr. X transferred 2,000 debentures of Rs. 100 each of Wild Fox Ltd. to Mrs. X on 03.04.2018 without
consideration. The company paid interest of Rs. 30,000 in September, 2018 which was deposited by Mrs. X with
Kartar Finance Co. in October, 2018. Kartar Finance Co. paid interest of Rs. 3,000 upto March, 2019. How would
both the interest income be charged to tax in assessment year 2019-20.

DT QUESTION BANK BY CA PRANAV CHANDAK Subscribe us on YouTube 124


Answer:
▪ As per section 64(1), income arising from assets transferred without adequate consideration by an individual
to his spouse is liable to be clubbed in the hands of the individual, but if there is any further income from such
income, it will not be clubbed.
▪ Therefore, Rs. 30,000, being the interest on debentures received by Mrs. X in September, 2018 will be clubbed
with the income of Mr. X, since he had transferred the debentures of the company without consideration to her.
▪ However, the interest of Rs. 3,000 upto March 2019 earned by Mrs. X on the interest of the debentures
deposited by her with Kartar Finance Company shall be taxable in her individual capacity & will not be clubbed
with the income of Mr. X.

Q5. Ms. X was born on 7.6.2002. Her betrothal took place on 5.3.2017 & she received cash gifts of Rs. 1,00,000
each from her father, father’s mother, father’s father, mother, mother’s mother & mother’s father. All 6 relatives
made similar gifts on the day of marriage on 1.4.2018. The amount so received is deposited in a private limited
company. For the year ending 31.03.2019, the company has paid her interest @ 8% i.e, Rs. 96,000. Discuss.
Answer:
▪ As per sec 56, Gift from relatives is not taxable & Gift from anyone on the occasion of marriage is also not taxable.
▪ There are 3 situations which need to be discussed in this question as detailed here below:
(i) Gifts received from relatives on the occasion of betrothal – Not taxable.
(ii) Gifts received from relatives on the occasion of Marriage: Not taxable.
(iii) Interest accrued on deposits made out of above gifts: As X is a minor child as on 31.03.2019, Rs. 96,000
being income accrued for the year ending 31.03.2019, relevant to AY 2019-2020 shall be included in the hands
of her father or mother whose total income, excluding this income, is greater.

Q6. Mrs. Satya Yadav received the following amounts during FY 2018-19:
Gross Salary 5,30,000
Family Pension (10,000 x 12) 1,20,000
Income of a minor child 49,000
Accumulated balance in PF of her husband after his death 1,00,000
Gratuity received after the death of husband 1,00,000
Calculate taxable income of Mrs. Satya Yadav & tax liability for AY 2019-20.
Solution: Computation of taxable income of Mrs. Satya Yadav for AY 2019-20
(i) Income from Salary
Gross salary 5,30,000
Less: Standard Deduction u/s 16(ia) (40,000) 4,90,000
(ii) Income from other sources
Family pension 1,20,000
Less: Deduction u/s 56 (Lower of 1/3 of amount received or 15,000)
rd (15,000) 1,05,000
Income of a minor child - Less: Exemption u/s 10(32) [49,000 - 1,500] 47,500
Gross total income 6,42,500
Note: Accumulated balance in PF & amount of gratuity received after the death of husband is exempt from tax.

Q7. Mr. B is the Karta of a HUF, whose members derive income as given below:
(a) Income from B's profession 4,50,000
(b) Mrs. B's salary as fashion designer 7,60,000
(c) Minor son D (interest on fixed deposits with a bank which were gifted to him by his uncle) 10,000
(d) Minor daughter P's earnings from sports 95,000
(e) D's winnings from lottery (gross) 1,95,000
Discuss the tax implications in the hands of Mr. & Mrs. B. [NOV-2012]
Solution: Tax Implications
(a) Income of Rs. 4,50,000 from Mr. B's profession shall be taxable in the hands of Mr. B u/h “PGBP”.
(b) Salary of Rs. 7,60,000 received by Mrs. B as a fashion designer shall be taxable as "Salaries" in hands of Mrs. B.
(c) Income from fixed deposit of Rs. 10,000 arising to the minor son D, shall be clubbed in the hands of the mother,
Mrs. B as Income u/h "IFOS", since her income is greater than income of Mr. B before including the income of the
minor child. As per Section 10(32), income of a minor child is exempt upto Rs. 1,500 per child (if clubbed).
(d) Income of Rs. 95,000 arising to the minor daughter P from sports shall not be included in the hands of the
parent, since such income has arisen on account of an activity involving application of her skill.
(e) Income of Rs. 1,95,000 arising to minor son D from lottery shall be included in the hands of Mrs. B as "IFOS",
since her income is greater than the income of Mr. B before including the income of minor child.
Note: She can reduce the tax deducted at source from such lottery income while computing her net tax liability.

DT QUESTION BANK BY CA PRANAV CHANDAK Subscribe us on YouTube 125


Computation of income of Mr. B & Mrs. B
Particulars Mr. B Mrs. B
Income from X’s Business 4,50,000
Salary as fashion designer - 7,60,000
Bank Interest to Minor Son D (10,000-1,500) Rs. 1,500 exempt u/s 10(32) - 8,500
Income of Minor Daughter from Sports (since she is earning income from her - -
(own talent, sports, income is not to be clubbed)
Lottery income to minor son D - 1,95,000
Total 4,50,000 9,63,500
Note: Whether exemption u/s 10(32) shall be allowed from casual income or not is controversial.

Q8. Mr. Dhaval & his wife Mrs. Hetal furnish the following information:
(i) Salary income (computed) of Mrs. Hetal 4,60,000
(ii) Income of minor son 'B' who suffers from disability specified in Section 80U 1,08,000
(iii) Income of minor daughter 'C' from singing 86,000
(iv) Income from profession of Mr Dhaval 750,000
(v) Cash gift received by C on 2.10.2018 from a friend of Mrs. Hetal on winning the competition 48,000
(vi) Income of minor married daughter 'A' from company deposit. 30,000
Computation of total income of Mr. Dhaval & Mrs. Hetal for AY 2019-20.
Solution:
Particulars Mr. Dhaval Mrs. Hetal
Salary income - 4,60,000
Income from profession of Mr. Dhaval 7,50,000 -
Income of minor married daughter 'A' from company deposits Rs. 30,000 -
Less: Exemption U/s 10(32) (1500) -
Total income 7,78,500 4,60,000
Note:
(a) U/s 56(2)(vi), cash gifts received from any person/persons exceeding Rs. 50,000 during the year in aggregate
is taxable. Since the cash gift in this case does not exceed Rs. 50,000 the same is not taxable.
(b) The clubbing provisions are attracted even in respect of income of minor married daughter. Hence, income of
minor married daughter 'A’ from company deposit shall be clubbed in the hands of the Mr. Dhaval.

Q9. Mr. X has 4 minor children consisting of three daughters & a son. Their annual income for AY 2019-20 are:
First daughter (including Scholarship received Rs. 5,000) 10,000
Second Daughter 8,500
Third Daughter (Suffering from disability specified U/s 80U) 4,500
Minor Son 40,000
Mr. X gifted 2 lac to his minor Son who invested them in the business & derived income of 20,000 which is included
above. Compute the Income earned by Minor Children to be clubbed in the hands of Mr. X. [N-14]/[ M-12]
Solution: Computation of Income of minor children to be clubbed in income of Mr. X
(i) Income of First Daughter 10,000
Less: Scholarship received exempt u/s 10(16) (assumed received for education) (5,000)
Less: Exempt u/s 10(32) (1,500)
Income to be clubbed 3,500
(ii) Income of Second Daughter 8,500
Less: Exempt u/s 10(32) (1,500)
Income to be clubbed 7,000
(iii) Income of Third Daughter who is suffering from disability shall not be clubbed
(iv) Income of Son 40,000
Less: Exempt u/s 10(32) (1,500)
Income to be clubbed 38,500
Total Income to be clubbed (3,500 + 7,000 + 38,500) 49,000

Q10. Compute the total income of Mr. & Mrs. X from the following information: [NOV – 2005]
Salary Income (computed) of Mrs. X 2,30,000
Income from profession of Mr. X 3,90,000
Income of minor son A from company deposit 15,000
Income of minor daughter B from special talent 32,000
Interest from bank received by B on fixed deposit made out of her special talent 3,000
Gift received by B on 30.09.2018 from friend of Mrs. X 2,500

DT QUESTION BANK BY CA PRANAV CHANDAK Subscribe us on YouTube 126


Solution: Since income of Mr. X is higher, income shall be clubbed in the income of Mr. X
Interest income (minor son A) (15,000 - 1,500) 13,500
Income of minor daughter B from special talent shall not be clubbed Nil
Interest income of minor daughter B (3,000 - 1,500) 1,500
Total income of Mr. X (3,90,000 + 13,500 + 1,500) 4,05,000
Total income of Mrs. X shall be 2,30,000

Q11. Determine Gross Total Income of Mr. X & his wife from the following particulars for PY 2018-19:
(i) X & his wife are partners in a firm carrying on cloth business, their shares of profit being 78,000 & 60,000.
(ii) Their 16 years old son has been admitted to the benefits of another firm, from which he received Rs. 80,000 as
his share of profit & Rs. 90,000 as interest on capital. Capital was invested out of minor’s own funds of Rs. 9,00,000.
(iii) A house property in the name of X was transferred to his wife on 1.12.2018 for adequate consideration. The
property has been let at a rent of 30,000 p.m.
(iv) Debentures of a company of 1,40,000 & 1,12,000 purchased two years ago are in the names of X & his wife
respectively, on which interest is receivable at 10% p.a. His wife had in the past transferred 70,000 out of her
income to X for the purchase of the debentures in X’s name.
(v) X had transferred 50,000 to his wife in the year 2009 without any consideration which she gave as a loan to Y.
She earned 20,000 as interest during the earlier PYs which was also given on loan to Y. During FY 2018-19, she
received interest at 10% p.a. on 70,000.
(vi) X transferred 75,000 to a trust, the income accruing from its investment as interest amounted to 7,500, out of
which 5,000 shall be utilized for the benefit of his son’s wife & 2,500 for the benefit of his son’s minor child.
Solution: Computation of Gross Total Income of X for AY 2019-20
Income from House Property:
Rental value for 8 months (i.e., before transfer) (8 x 30,000) 2,40,000
Less: Deduction u/s 24(a) @ 30% 72,000 1,68,000
Profit from Business:
(i) Share from firm (Exempt) Nil
(ii) Minor Son’s share in another firm (Exempt) Nil
(iii) Interest on minor’s capital with firm (90,000 - 1,500) 88,500 88,500
Income from other Sources:
(i) Interest @ 10% on 70,000 (only half of 1,40,000 were bought by own funds) 7,000
(ii) Interest received by his wife @ 10% on 50,000 (without any consideration) 5,000
(iii) Interest on 50,000 from trust (Interest income utilized for benefit of son’s wife) 5,000 17,000
Gross Total Income 2,73,500

Computation of Gross Total Income of Mrs. X for AY 2019-20


1. Income from House Property
Rental value for 4 months (i.e. after transfer) (30,000 x 4) 1,20,000
Less: Deduction u/s 24(a) @ 30% 36,000 84,000
2. Income from business: Share from firm (Exempt) Nil

3. Income from Other Sources: (i) Interest on 1,120,000 10% Debentures 11,200
(ii) Interest on 10 % 70,000 debentures in husband’s name but funds invested by her 7,000
(iii) Interest on 20,000 @ 10% 2,000 20,200
(This interest is on accrued income of 50,000, which have been transferred to her by the husband & interest
on such accrued income is treated as the income of the transferee, although the income on the transferred
amounts is treated as the income of the transferor as it was transferred without any consideration.)
Gross Total Income 1,04,200
Notes:
1. Shares of profit from a firm is fully exempt u/s 10(2A) in the hands of the partners. Even in a case where one
spouse gifts some amount to the other spouse to be invested as capital in the firm, the clubbing provisions though
applicable, it will not affect the Total Income since the share of the profit is itself exempt. However, if interest on
capital contribution is received, it will be clubbed to the extent of the amount invested as capital contribution out
of the transfer made without adequate consideration.
2. Minor son’s income though clubbed, but as share of profit from firm is exempt, will not affect the Total Income.
3. If asset is transferred to a Trust for benefit of son’s wife, income from such asset is taxable in hands of transferor.
However, income utilised for the benefit of son’s minor son shall be clubbed in the hands of that parent of the son’s
minor son, whose income is greater. Therefore it shall not be clubbed in the hands of transferor (i.e. X).

DT QUESTION BANK BY CA PRANAV CHANDAK Subscribe us on YouTube 127


Q12. Shri Madan (age 67 years) gifted a building owned by him to his son's wife Smt. Hema on 1.10.2018. The
building fetched a rental income of Rs. 10,000 per month throughout the year. Municipal tax for the first half-year
of Rs. 5,000 was paid in June 2018 & the municipal tax for the second half-year was not paid till 30.9.2019.
Incomes of Shri Madan & Smt. Hema other than income from house property are given below:
Name Business income Capital gain Other sources
Shri. Madan 1,00,000 50,000 (long-term) 1,50,000
Smt. Hema -75000 2,00,000 (short-term) 50,000
Compute the total income of Shri. Madan & Smt. Hema for AY 2019-20. [Nov 11]
Note: Capital gain does not relate to gain from shares & securities.
Solution: Computation of Total Income of Shri Madan for AY 2019-20
Income from house property (Refer Note 1) 80,500
Business Income 1,00,000
Long-term Capital Gains 50,000
Income from Other Sources 1,50,000
Total Income 3,80,500

Computation of Total Income of Smt. Hema for AY 2019-20


Particulars Rs. Rs.
Short-term Capital Gains 2,00,000
Less: Business loss (75,000) 1,25,000
Income from Other Sources 50,000
Total Income 1,75,000
Working Note:
1. As per section 64(1)(vi), the income arising to the son's wife of an individual, directly or indirectly, from assets
transferred to her, otherwise than for adequate consideration, by such individual, shall be included in the total
income of the individual. Therefore, the rental income from building transferred by Shri Madan to his son's wife
Smt. Hema without consideration on 1.10.2018 is includible in the hands of Shri Madan.
Computation of Income from House property
Particulars Madan Hema
(1.4.2018 -30.9.2018) (1.10.2018 - 31.3.2019)
Gross Annual value (Rs. 10,000 x 6 months) 60,000 60,000
Less: Municipal taxes paid 5000 Nil
Net Annual Value (NAV) 55,000 60,000
Less: Deduction u/s 24(a), 30% of NAV 16,500 18,000
Income from House Property 38,500 42,000
Income of Hema clubbed in hands of Madan -Sec 64(1)(vi) 42,000
Income from house property of Mr. Madan 80,500

Q13. Mr. A is an employee of Larsen Ltd & has substantial interest in the company. His salary is Rs. 25,000 p.m.
Mrs. A also is working in that company at a salary of Rs. 10,000 p.m. without any professional qualification.
Mr. A also receives Rs. 30,000 as income from securities.
Mrs. A owns a house property which she has let out. Rent received from such house property is Rs. 12,000 p.m.
Mr. & Mrs. A have three minor children-two twin daughters & one son. Income of each twin daughters is Rs. 2,000
p.a. & that of his son is Rs. 1,200 p.a. Compute the income of Mr. A & Mrs. A. [May 2013]
Solution: Computation of Total Income of Mr. A & Mrs. A for AY 2019-20
Particulars Mr. A Mrs. A
Income from Salaries
Salary income of Mr. A (Rs. 25,000 x 12) 3,00,000
Salary income of Mrs. A (Rs. 10,000 x 12) [Note 1] 1,20,000 -
Income from House Property: Rent received (Rs.12,000 x 12) 1,44,000
Less: Deduction u/s 24 @ 30% (43,200) 1,00,800
Income from other sources
Income from securities 30,000
Income before including income of minor children u/s 64(1A) (Note 2) 4,50,000 1,00,800
Income of twin daughters (Rs. 2,000 per child x 2) Rs. 4,000
Less: Exempt u/s 10(32) (Rs.1,500 x 2) (Rs. 3,000) 1,000
Income of the minor son Rs. 1,200
Less: Exempt u/s 10(32) Rs. 1,200 Nil
Total Income 4,51,000 1,00,800

DT QUESTION BANK BY CA PRANAV CHANDAK Subscribe us on YouTube 128


Note:
1. As per section 64(1), the salary of Rs. 10,000 p.m. received by Mrs. X from the company has to be included in
the total income of Mr. X, as Mrs. X does not possess any technical or professional qualification for earning such
income & Mr. X has substantial interest in the company.
2. As per section 64(1 A), the income of a minor child is to be included in the total income of the parent whose
total income (excluding the income of minor child to be so clubbed) is greater. Further, as per section 10(32),
income of a minor child which is includible in the income of the parent shall be exempt upto Rs. 1,500 per child.

Q14. Mr. A has 3 children of 19 years, 15 years & 5 years respectively. The first child derives Rs. 100000 incomes
every year. The income details of Mr. A, his second & third child are as follows:
Particulars Mr. A Second child Third child
Business Income 50,000 -- --
Interest on FD invested out of gifts -- 15,000 --
Bank Interest 7,000 8,000 1,000
Salary earned on application of skills 48,000 24,000 --
Interest on salary income saved & invested 8,000 2,000 --
Determine the Gross total income.
Solution: Computation of Gross Total Income of Mr. A & second child for AY 2019-20
Particulars Mr. A Second Child
1. Salary 48,000 24,000
2. Income from business 50,000
3. Income from other sources
(i)Bank interest 7,000
(ii) Interest on Investment 8,000 15,000
4. Income to be clubbed
(a) Second child’s income
(i) Interest On FD 15,000
(ii) Bank interest 8,000
(iii) Interest on Investment 2,000
Less: Exempt u/s. 10(32) (1,500) 23,500
(b) Third child’s Income
(i) Bank interest 1,000
Less: Exemption Restricted to Rs. 1,000 (1,000) Nil
Gross Total Income 1,36,500 24,000
Note:
▪ Income of minor child is clubbed even if it is earned on investment made out of salary.
▪ First child is a major & thus, income not clubbed.

Q15. Mr. Ramesh gifted a sum of Rs. 5 lacs to his brother's minor son on 16.4.2018. On 18.4.2018, his brother gifted
debentures worth Rs. 6 lacs to Mrs. Ramesh. Son of Mr. Ramesh' brother invested the amount in fixed deposit with
Bank of India @ 9% p.a. ROI & Mrs. Ramesh received interest of Rs. 45,000 on debentures received by her. Discuss.
Answer:
▪ In the given case, Mr. Ramesh gifted a sum of Rs. 5 lacs to his brother's minor son on 16.4.2018 &
simultaneously, his brother gifted debentures worth Rs. 6 lacs to Mr. Ramesh's wife on 18.4.2018.
▪ Mr. Ramesh's brother's minor son invested the gifted amount of Rs. 5 lacs in fixed deposit with Bank of India.
▪ These transfers are in the nature of cross transfers.
▪ Accordingly, income from assets transferred would be assessed in the hands of the deemed transferor because
the transfers are so intimately connected to form part of a single transaction & each transfer constitutes
consideration for the other by being mutual or otherwise.
▪ If two transactions are inter-connected & are part of same transaction in such a way that it can be said that the
circuitous method was adopted as a device to evade tax, implication of clubbing provisions would be attracted.
▪ Accordingly, the interest income arising to Mr. Ramesh's brother's son from fixed deposits would be included
in total income of Mr. Ramesh's brother, assuming that Mr. Ramesh's brother's total income is higher than his
wife's total income, before including minor's income. Mr. Ramesh's brother can claim exemption of Rs. 1,500.
▪ Interest on debentures arising in the hands of Mrs. Ramesh would be taxable in the hands of Mr. Ramesh.
▪ This is because both Mr. Ramesh & his brother are the indirect transferors of the income to their spouse &
minor son, respectively, with an intention to reduce their burden of taxation.
▪ In the hands of Mr. Ramesh, interest received by his spouse on debentures of Rs. 5 lacs alone would be included
& not entire interest income on debentures of 6 lacs, since the cross transfer is only to the extent of Rs. 5 lacs.
▪ Hence, only proportional interest (5/6th of interest) Rs. 37,500 would be includible in the hands of Mr. Ramesh.

DT QUESTION BANK BY CA PRANAV CHANDAK Subscribe us on YouTube 129


Q16. X & Y form a partnership from 1 April 2018 (PSR - 2: 3) by investing Rs. 10 Lacs & Rs. 15 Lacs respectively.
The investment has been financed from the following sources.
Particulars X Y
Gift from Mrs. X 6,60,000 -
Gift from Mrs. Y - 8,00,000
Past savings of X & Y 3,40,000 7,00,000
For the year ending March 31, 2019, Share of Profit from the firm is as follows:
Interest on Capital @ 12% 1,20,000 1,80,000
Salary as working partner 24,000 24,000
Share of Profit 1,08,000 1,62,000
Find out the Income chargeable to tax in the hands of X & Mrs. X
Solution:
Particulars Mr. X Mrs. X
Share of profit [Exempt u/s 10(2A)] Nil -
Salary from Firm 24,000 -
Interest on Capital [*(Rs. 1,20,000 × Rs.6.6 lacs)/Rs. 10 lacs] 40,800 79,200*
Business income 64,800 79,200

Q17. Mr. Vaibhav started a proprietary business on 1.04.2017 with a capital of Rs. 5,00,000. He incurred a loss of
Rs. 2,00,000 during PY 2017-18. To overcome the financial position, his wife Mrs. Vaishali, a software Engineer,
gave a gift of Rs. 5,00,000 on 1.4.2018, which was immediately invested in the business by Mr. Vaibhav. He earned
a profit of Rs. 4,00,000 during PY 2018-19. Compute the amount to be clubbed in the hands of Mrs. Vaishali for AY
2019-20. If Mrs. Vaishali gave the said amount as loan, what would be the amount to be clubbed? [NOV-2011]
Solution: Income to be clubbed in the hands of Mrs. Vaishali for AY 2019-20:
▪ Since Mr. Vaibhav has incurred loss of Rs. 2,00,000, remaining amount of Rs. 3,00,000 is the amount of Capital
he has in the business on 1.4.2018.
▪ Total Capital Contribution on 1.4.2018 = Rs. 8,00,000 [Includes Rs. 5,00,000 taken from his wife]
▪ Profit of PY 2018-19 = Rs. 4,00,000.
𝐼𝑛𝑣𝑒𝑠𝑡𝑚𝑒𝑛𝑡 𝑚𝑎𝑑𝑒 𝑜𝑢𝑡 𝑜𝑓 𝑆𝑝𝑜𝑢𝑠𝑒 𝑀𝑜𝑛𝑒𝑦
▪ Amount to be clubbed in the hands of Mrs. Vaishali = [ × 𝑃𝑟𝑜𝑓𝑖𝑡 ]
𝑇𝑜𝑡𝑎𝑙 𝐼𝑛𝑣𝑒𝑠𝑡𝑚𝑒𝑛𝑡 𝑜𝑛 𝑓𝑖𝑟𝑠𝑡 𝑑𝑎𝑦 𝑜𝑓 𝑃𝑌
𝑅𝑠. 5,00,000
=[ × 𝑅𝑠. 4,00,000 ] = Rs. 2,50,000
𝑅𝑠. 8,00,000

▪ Thus Rs. 2,50,000 shall be clubbed in the hands of Mrs. Vaishali.

Q18. The following details are furnished in respect of Mr. X & his family members. Determine their GTI:
Particulars Mr. X Mrs. X Minor Child
Income as child Artist in films - - 60,000
Business Income (Own) (40,000) - -
Salary from X Ltd. in which Mr. X holds 25% Voting power - 30000 -
Share of profit from Firm AB & Co. 80,000 (40%) - 20,000 (10%)
Commission from AB & Co. - 20,000 --
Interest Income 8000 5000 4000
Note:
1. Mrs. X possesses B.Com degree & works as accountant of X Ltd.
2. Mrs. X does not render any services to M/s. AB & Co.
3. Interest income received by Mrs. X is from an investment of Rs. 40,000 gifted by Mr. X & Rs. 40,000 invested
from her own resource.
Solution: Computation of Gross Total Income for AY 2019-20
Particulars Mr. X Mrs. X Minor Child
Salaries: Salary from X Ltd - 30,000 --
Profits & gains from business/ Profession: Income /(Loss) (40,000) -- 60,000
Income from other sources:
(i) Interest Income Own (Mr. X) 8,000 2,500
Add: Spouse – Sec 64(1) [5000 × 40,000/80,000] 2,500 10,500 -- --
Total Income for Clubbing of Minor’s Income (29500) 32,500 -
Interest Income of Minor Child Rs. 4,000 -- 2,500 --
Less: Exempt u/s. 10(32) (Rs. 1,500) 2,500
Commission income of spouse u/s. 64(1) 20,000 -- --
Gross Total Income (9,500) 35,000 60,000

DT QUESTION BANK BY CA PRANAV CHANDAK Subscribe us on YouTube 130


Notes:
1. Share of profit from firm is exempt from tax u/s. 10(2A).
2. Income of the minor child will be clubbed in the hands of the parent whose income before such clubbing is
greater. In this case, thus the interest income of minor child is clubbed in the hands of Mrs. X.

Q19. Mr. X commenced a proprietary business in the year 2013. His capital as on 01.04.2017 was Rs. 6,00,000.
On 10.04.2017 his wife gifted Rs. 2,00,000 which he invested in the business on the same date.
Mr. X earned profit from his proprietary business as given below:
Previous year 2017-18: Profit = Rs. 3,00,000 Previous year 2018-19: Profit = Rs. 4,40,000
During PY 2018-19, he sold a vacant site which resulted in LTCG of Rs. 5,00,000 (computed). The vacant site was
sold on 20.12.2018. Compute the total income & tax liability of Mr. X. [MAY – 2010]
Solution: Computation of business income of Mr. X
Capital as on 01.04.2017 6,00,000
Add: Gift from wife (10.04.2017) 2,00,000
Add: Profit for the year ended 31.03.2018 3,00,000
Capital as on 01.04.2018 11,00,000
Profit for PY 2018-19 4,40,000
Income to be clubbed in the hands of Mrs. X u/s 64(1) = (Rs. 4,40,000/11,00,000 x 2,00,000) 80,000
Income assessable in the hands of Mr. X (4,40,000 - 80,000) 3,60,000
Total Income of Mr. X
Income from Business 3,60,000
Long term capital gain 5,00,000
Gross Total Income 8,60,000

Q20. Mr. X is a trader. Particulars of his income & those of the members of his family are given below. These
incomes relate to the previous year ended 31st March, 2019: [N-98/N-99]
(i) Income from business of Mr. X 9,00,000
(ii) Salary derived from an educational institution by Mrs. X. (She is the principal) 5,00,000
(iii) Interest on company deposits derived by master Deep Singh (minor son). These deposits 12,000
were made in the name of master Deep Singh by his father’s father about 6 years ago (Gross)
(iv) Receipts from sale of paintings & drawings made by minor Dipali Singh (minor daughter & 60,000
noted child artist)
(v) Income by way of lottery earnings by Master Dipindar Singh (minor son) 6,000
Discuss whether the above will form part of the assessable income of any individual & also compute the assessable
income of Mr. X.
Solution: Since income of Mr. X is higher, income shall be clubbed in the income of Mr. X & such incomes shall be
Interest income of master Deep Singh (12,000 - 1,500) 10,500
Income of minor daughter Dipali Singh shall not be clubbed Nil
Lottery income of master Dipindar Singh (6,000 - 1,500) 4,500
Total income of Mr. X shall be (9,00,000 + 10,500 + 4,500) 9,15,000
Note: Whether exemption u/s 10(32) shall be allowed from casual income or not is controversial.

Q21. On 21.3.2018, Mr. Janak gifted to his wife Mrs. Thilagam 200 Listed Shares which had been bought by him on
19.04.2017 at Rs. 2,000 per Share. On 1.6.2018, Bonus Shares were allotted in the ratio of 1:1. All these Shares
were sold by Mrs. Thilagam as under:
Date of Sale Manner of sale No. of Shares Net Sales Value
21.05.2018 Sold in Recognized Stock Exchange, STT paid 100 2,20,000
21.07.2018 Private Sale, to an Outsider All Bonus Shares 1,25,000
28.02.2019 Private Sale, to her friend Mrs. Hema 100 1,70,000
(Market Value on this date was Rs. 2,10,000)
Briefly state the Tax consequences in respect of the sale of the shares by Mrs. Thilagam, showing the person in
whose hands the same is chargeable, the quantum & Head of Income in respect of the above transactions. [M-11]
Solution:
▪ Sale of 100 Shares on 21.05.2018 for Rs. 2,20,000 & another 100 Shares on 28.02.2019 for Rs. 1,70,000 shall be
clubbed with the Total Income of Mr. Janak u/h “Capital Gains”. Capital gain > 1 lac is taxed @ 10% u/s 112A.
▪ STCG from Private Sale of 200 Bonus Shares (i.e. Bonus in ratio 1:1) is taxable in hands of Mrs. Thilagam u/h
“Capital Gains”. It shall not be clubbed with Total Income of Mr. Janak, since it is income on income from
transferred assets (MSS Rajan 252 ITR 126 (Mad.)].

DT QUESTION BANK BY CA PRANAV CHANDAK Subscribe us on YouTube 131


Q22. During the previous year 2018-19 the following transactions occurred in respect of Mr. X.
(a) Mr. X had a fixed deposit of 5 lacs in BOI. He instructed the bank to credit interest on deposit @ 9% from
1.4.2018 to 31.3.2019 to savings A/c of Mr. B, son of his brother, to help him in his education.
(b) Mr. X holds 75% share in a partnership firm. Mrs. X received a commission of Rs. 25,000 from the firm for
promoting the sales of the firm. Mrs. X possesses no technical or professional qualification.
(c) Mr. X gifted a flat to Mrs. X on April 1, 2018. During the previous year the flat had income under the head House
Property Rs. 52,000 to Mrs. X.
(d) Mr. X gifted Rs. 2,00,000 to his minor son who invested the same in a business & he got a share income of Rs.
20,000 from the investment.
(e) Mr. X’s minor son derived an income of 20,000 through an activity involving application of his skill & talent.
During the year Mr. X had no other income. Mrs. X received salary of Rs. 20,000 per month from a part time job.
Discuss tax implications of each transaction & compute total income of Mr. X, Mrs. X & their minor child. [M-12]
Answer:
(a) Mr. X transferred interest on fixed deposit to Mr. B (son of his brother) without transferring the fixed deposit,
such income shall be clubbed in the hands of Mr. X as per sec 60. Amount to be clubbed = Rs. 5 lacs x 9 % = 45,000
(b) As per Section 64(1), if any person is getting salary, commission etc. from a concern in which his or her spouse
has substantial interest & further salary etc. is received without any professional or technical qualification, in such
case, salary etc. so received shall be clubbed in the income of the spouse having substantial interest.
In the given case Mr. X is having substantial interest in the partnership firm & Mrs. X received a commission of Rs.
25,000 from the firm for promoting the sales of the firm without any technical or professional qualification. So the
commission shall be clubbed in the hands of Mr. X
(c) As per section 27, An individual who transfers otherwise than for adequate consideration any house property
to his or her spouse, not being a transfer in connection with an agreement to live apart shall be deemed to be the
owner of the house property so transferred.
In the given case Mr. X transfers flat to Mrs. A without adequate consideration on April 1, 2018.
So Mr. X shall be deemed to be the owner of house property & Rs. 52,000 shall be considered as income of Mr. X.
(d) As per section 64(1A), if any income accrues or arises to a minor child, such income shall be clubbed in the
income of mother or father having higher income before taking in to consideration the income to be clubbed.
Amount to be clubbed = 20,000 - 1500 = Rs. 18,500 in the hands of father or mother having higher income.
(e) Clubbing provision is not applicable as Mr. X’s minor son derived an income of Rs. 20,000 through an activity
involving application of his skill & talent.

Q23. Mr. X gifts Rs. 1 Lac to his wife Mrs. X on April 1, 2018 which she invests in a firm on interest rate of 14% per
annum. On January 1, 2019, Mrs. X withdraws the money & gift it to her son’s wife. She claims that interest which
has accrued to the daughter-in-law, from January 1, 2019 to March 31, 2019 on investment made by her is not
assessable in her hands but in the hands of Mr. X. Is this correct Rs. What would be the position, if Mrs. X has gifted
the money to minor grandson, instead of the daughter-in- law.
Answer:
▪ Section 64(1) provides that in computing the total income of any individual, there shall be clubbed all such
income as arises directly or indirectly to the son’s wife, of such individual, from assets transferred directly or
indirectly to the son’s wife by such individual otherwise than for adequate consideration.
▪ There is an indirect transfer by Mr. X to the daughter-in-law & therefore, the interest income shall be clubbed
with income of Mr. X.
▪ If Mrs. X had gifted the money to her minor grandson, then the interest income arising to the minor shall be
clubbed u/s 64(1 A) in the total income of that parent (son/daughter-in-law) whose total income (before
including such income) is higher.

Q24. Write a short note on “Cross Transfers”.


Answer: Refer Page no. 176 of “Concept Book”.

DT QUESTION BANK BY CA PRANAV CHANDAK Subscribe us on YouTube 132


6. SET OFF & CARRY FORWARD OF LOSSES
Q1. Mrs. X has income & losses as given below:
1. Income under the head Salary Rs. 5,00,000
2. Loss under the head House Property Rs. 10,00,000
3. Income under the head Business/Profession Rs. 12,00,000
4. Income from STCG Rs. 2,00,000
5. Income from STCG u/s 111A Rs. 10,00,000
6. Casual Income Rs. 3,00,000
7. Brought forward Business/Profession loss for
- Previous year 2009-10 Rs. 3,00,000
- Previous year 2011-12 Rs. 6,00,000
- Previous year 2012-13 Rs. 3,00,000
Deduction u/s 80C to 80U Rs. 1,00,000
Compute tax liability of Mrs. X for Assessment Year 2019-20.
Solution: Option 1: Loss of house property is set off from normal income
Income under the head Salary 5,00,000
Less: Loss of house property (2,00,000)
Income under the head Salary 3,00,000
Income under the head Business/Profession 12,00,000
Less: Brought forward business/profession loss P.Y. 2011-12 (6,00,000)
Less: Brought forward business/profession loss P.Y. 2012-13 (3,00,000)
Income under the head Business/Profession 3,00,000
Short term capital gain 2,00,000
Short term capital gain u/s 111A 10,00,000
Casual Income 3,00,000
Gross Total Income 21,00,000
Less: Deduction u/s 80C to 80U (1,00,000)
Total Income 20,00,000
Computation of Tax Liability
Tax on STCG u/s 111A Rs. 10,00,000 @ 15% 1,50,000
Tax on Casual income Rs. 3,00,000 @ 30% 90,000
Tax on Normal income Rs. 7,00,000 at slab rate 52,500
Tax before HEC + HEC @ 4% [2,92,500 + 4 %] 3,04,200

Option II: Loss of house property is set off from STCG u/s 111 A
Income under the head Salary 5,00,000.00
Income under the head Business/Profession 12,00,000.00
Less: Brought forward business/profession loss P.Y. 2011-12 (6,00,000.00)
Less: Brought forward business/profession loss P.Y. 2012-13 (3,00,000.00)
Income under the head Business/Profession 3,00,000.00
Short term capital gain 2,00,000.00
Short term capital gain u/s 111A 10,00,000.00
Less: loss of house property (2,00,000.00)
Short term capital gain u/s 111A 8,00,000.00
Casual Income 3,00,000.00
Gross Total Income 21,00,000.00
Less: Deduction u/s 80C to 80U (1,00,000.00)
Total Income 20,00,000.00
Computation of Tax Liability
Tax on STCG u/s 111A Rs. 8,00,000 @ 15% 1,20,000.00
Tax on Casual income Rs. 3,00,000 @ 30% 90,000.00
Tax on Normal income Rs. 9,00,000 at slab rate 92,500.00
Tax before HEC + HEC @ 4% [3,02,500 + 4 %] 3,14,600

Conclusion: Option I is better.

DT QUESTION BANK BY CA PRANAV CHANDAK Subscribe us on YouTube 133


Q2. Mr. B, a resident individual, furnishes the following particulars for the PY 2018-19:
Income from salary (Net) 45,000
Income from house property (24,000)
Income from business – non-speculative (22,000)
Income from speculative business (4,000)
Loss from Specified Business (60,000)
Short-term capital losses (25,000)
Long-term capital gains 19,000
What is the total income chargeable to tax for the AY 2019-20? [Mod. ICAI -SM]
Solution: Total income of Mr. B for AY 2019-20
Particulars Amount Amount
Income from salaries 45,000
Income from house property (24,000) 21,000
Profits & gains of business & profession
Business loss to be carried forward [Note 1] (22,000)
Speculative loss to be carried forward [Note 2] (4,000)
Capital Gains
LTCG 19,000
STCL (25,000)
STCL to be carried forward [Note 3] (6,000)
Taxable income 21,000
Note
1. Business loss cannot be set-off against salary income. Therefore, loss of Rs. 22,000 from the non-speculative
business cannot be set off against the income from salaries. Hence, such loss has to be carried forward to the next
year for set-off against business profits, if any.
2. Loss of Rs. 4,000 from the speculative business can be set off only against the income from the speculative
business. Hence, such loss has to be carried forward.
3. STCL can be set off against both STCG & LTCG. Therefore, STCL of Rs. 25,000 can be set- off against long-term
capital gains to the extent of Rs. 19,000. The balance STCL of Rs. 6,000 cannot be set-off against any other income
& has to be carried forward to the next year for set-off against capital gains, if any.
4. Loss from specified business can be set off against specified business income only. Thus, it will be carried
forward to next year & set off against specified business income only.

Q3. Compute the gross total income of Mr. F for the AY2019-20 from the information given below: [ICAI – PM]
Particulars Rs.
Net income from house property 1,25,000
Income from business (before depreciation) 1,35,000
Short term capital gains on sales of shares 56,000
Long term capital loss from sale of property (brought forward from AY 2018-19) (90,000)
Income from tea business 1,20,000
Dividend from Indian companies carrying on agricultural operations 80,000
Current year depreciation 26,000
Brought forward business loss (loss incurred six years ago) (45,000)
Solution: Computation of Gross total income of Mr. F for AY 2019-20
Particulars Rs. Rs.
Income from house property 1,25,000
Income from business: Profits before depreciation 1,35,000
Less: Current year depreciation (26,000)
Less: Brought forward business loss (incurred 6 years ago) (45,000)
Total Income from Business 64,000
Income from tea business (40% is business income) [1,20,000 × 40 %] 48,000 1,12,000
Income from the capital gains: Short term capital gains 56,000
LTCL from property (cannot be set off against STCG & thus c/f) - 56,000
Gross Total Income 2,93,000
Note: Dividend from Indian companies is exempt. 60% of income from tea business is treated as agricultural
income & therefore, exempt from tax.

DT QUESTION BANK BY CA PRANAV CHANDAK Subscribe us on YouTube 134


Q4. Determine the total income of Mr. X from the following information for AY 2019-20:
Interest received on enhanced compensation (It relates to transfer of land in FY 2013-14) 4,00,000
Out of the above Rs. 65,000 relates to FY 2018-19 & the balance relate to preceding years)
Business loss relating to discontinued business of AY 2013-14 brought forward 1,50,000
Current year business income (i.e. financial year 2018-19) (Computed) 1,10,000
Solution: Computation of total income of Mr. X for AY 2019-20
Particulars Amount Amount
Profits & gains of business or profession 1,10,000
Current year business income
Less: B/f business loss of discontinued business Rs. 1,50,000 (1,10,000) Nil
Income from other sources
Interest on enhanced compensation taxable on receipt basis u/s 56(2) 4,00,000
Less: Deduction u/s 57 @ 50% (2,00,000) 2.00,000
Total Income 2.00,000
Note: Unabsorbed business loss of Rs. 40,000 ( Rs. 1,50,000 - Rs. 1,10,000) of AY 2013-14 relating to discontinued
business will be carried forward for set-off against income from any business in the next year i.e. AY 2020- 21.

Q5. Mr. Shyam, a resident of Chandigarh, provides the following information for PY 2018-19:
Particulars Rs.
Income from textile business 4,60,000
Income from speculation business 25,000
Loss from gambling 12,000
Loss on maintenance of race horse 15,000
Eligible current year depreciation of textile business not adjusted in income given above 5,000
Unabsorbed depreciation of Assessment year 2018-19 brought forward 10,000
Speculation business loss of Assessment year 2018-19 30,000
Compute GTI of Mr. Shyam for AY 2019-20 & any other loss eligible for carry forward. [M-17]
Solution: Computation of Gross Total Income
Income from Textile Business 4,60,000
Less: Current year depreciation (5,000)
Less: Unabsorbed depreciation (10,000)
Income from Textile Business 4,45,000
Income from speculation business 25,000
Less: Brought forward speculation loss (Section 73) (25,000)
Income from Speculation business Nil
Gross Total Income 4,45,000
Note:
1. As per sec 73, Unadjusted Brought Forward Speculation loss of AY 2018-19 shall be carried forward of 5,000.
2. Loss from Gambling shall not be treated as loss & have no treatment.
3. Loss on maintenance of race horse shall be allowed to be set off from income of maintenance of race horse only
& unadjusted loss of Rs. 15,000 shall be carried forward for 4 years as per section 74A.

Q6. Mr. X provides the following details for the previous year ending 31.03.2019.
(i) Salary from XYZ Ltd. 50,000 p.m
(ii) Interest on FD with SBI for FY 2018-19 (Net) 72,000
(iii) LTCL of AY 2017-18 96,000
(iv) Long term Capital gain 75,000
(v) Loss of minor son (Mr. X transferred his own house to his minor son without adequate 90,000
consideration few years back & minor son let it out & suffered loss)
(vi) Loss of his wife’s business (She carried business with funds which Mr. X gifted to her) (2,00,000)
You are required to compute taxable income of Mr. X for the AY 2019-20. [N-15]
Solution: Computation of taxable income of Mr. X for AY 2019-20
Income under the head Salary 6,00,000
Less: Loss under the head house property (Loss of minor son) (90,000)
Income under the head Salary 5,10,000
Income under the head capital Gains
Long Term Capital Gain 75,000
Less: Loss from Business of his wife (75,000)
Income under the head capital Gains Nil
Income under the head other sources

DT QUESTION BANK BY CA PRANAV CHANDAK Subscribe us on YouTube 135


Interest Income from Fixed Deposit 80,000
Less: Loss from Business of his wife (80,000)
Income under the head other sources Nil
(Balance Loss of Rs. 45,000 of his wife is to to be carried forward)
Gross Total Income 5,10,000
Note:
1. X shall be deemed owner of house property transferred to minor son. Thus it will be considered as X’s Loss.
2. Loss from business of Mrs. X shall also be clubbed.
3. Brought Forward LTCL of AY 2017-18 to be carried forward Rs. 96,000.

Q7. Mr. X furnishes you the following details for PY 2018-19:


Income (loss) from house property
House - 1 36,000
House - 2 (Self occupied) (20,000)
House - 3 60,000
Profits & gains from Business or Profession
Textile Business 2,00,000
Automobile Business (3,00,000)
Speculation Business 2,00,000
Capital Gains
Long-term capital gain from sale of shares (STT paid) 1,50,000
Long-term capital gain from sale of vacant site 2,00,000
Short-term capital loss from sale of building 1,00,000
Other sources:
Gift from a Friend (non-relative) on 05.06.2018 60,000
Gift from Maternal Uncle on 25.02.2019 1,00,000
Gift from Grandfather’s Younger Brother on 10.02.2019 1,00,000
Compute the total income of Mr. X for AY 2019-20. [MAY-2010]
Solution: Computation of total income of Mr. X for AY 2019-20
Income (loss) House property
House -I 36,000
House-2 -Self occupied (20,000)
House-3 60,000
Income from House Property 76,000
Profits & gains of business & profession
Textile business 2,00,000
Automobile business (3,00,000)
Speculation business Income from business or profession representing 2,00,000
speculation business profit (after set off of loss of automobile business) 1,00,000
Capital Gains
LTCG from sale of shares (STT paid) [Taxable over Rs. 1 lac u/s 112A] 50,000
Long term capital gain from sale of vacant site 2,00,000
Less: STCL set off against LTCG from sale of vacant site (1,00,000) 1,50,000
Income from Other sources
Gift from a friend on 05.06.2018 60,000
Gift from maternal uncle (on 25.2.2019) Rs. 1,00,000, not taxable since Nil
maternal uncle is a ‘relative’ given in explanation to section 56(2)
Gift from grandfather’s younger brother on 10.02.2019 – (taxable as 1,00,000 1,60,000
grandfather’s younger brother is not covered by the definition of ‘relative’)
Gross Total Income 4,86,000
Less: Deduction u/s 80C to 80U Nil
Total income 4,86,000

Q8. Mr. X an assessee aged 61 years gives the following information for the previous year 31.03.2019:
Loss from profession 1,05,000
Capital loss on the sale of property-short term 55,000
Capital gains on sale of shares-long term 2,05,000
Loss in respect of self-occupied property 15,000
Loss in respect of let out property 30,000

DT QUESTION BANK BY CA PRANAV CHANDAK Subscribe us on YouTube 136


Share of loss from firm 1,60,000
Income from card games 55,000
Winnings from lotteries 1,00,000
Loss from horse races in Mumbai 40,000
Medical insurance premium paid by cheque 18,000
Compute the total income of Mr. X for AY 2019-20. [N-12]
Solution:
(i) Income under the head Capital Gains
Long term capital Gain 2,05,000
Less: Short term capital loss on sale of property (55,000)
Less: Loss from profession (1,05,000)
Less: Loss from House Property (45,000) Nil
(ii) Income under the head Other Sources
Winning from lottery 1,00,000
Income from card game 55,000 1,55,000
Gross Total Income 1,55,000
Less: Deduction u/s 80D (Deductions are not allowed from casual income) Nil
Total Income 1,55,000
Working Notes:
1. Share of loss from firm is not allowed to be set off by the partner.
2. Loss from races can neither be set off nor be carried forward.

Q9. Mr. X furnishes the following details for year ended 31.03.2019.
Short term capital gain 1,40,000
Loss from speculative business (60,000)
Long term capital gain on sale of land 30,000
Long term capital loss on sale of shares (securities transaction tax not paid) 1,00,000
Income from business of textile (after allowing current year depreciation) 50,000
Income from activity of owning & maintaining race horses 15,000
Income from salary 1,00,000
Loss from house property (40,000)
Following are the carried forward losses:
(i) Losses from activity of owning & maintaining race horses-pertaining to AY 2016-17 Rs. 25,000.
(ii) Carry forward loss from business of textile Rs. 60,000- Loss pertains to AY 2011-12.
Compute gross total income of Mr. X for AY 2019-20. [N-11]
Solution: Calculation of Gross Total Income of Mr. X for AY 2019-20
Income under the head Salary
Salary Income 1,00,000
Less: Loss from house property (40,000) 60,000
Income under the head Business/Profession
Income from Business of textile 50,000
Less: Loss Carried forward from textile business (AY 2011-12) (50,000)
(Balance loss of Rs. 10,000 shall lapse)
Income under the head Capital Gains
Short Term Capital Gains 1,40,000
Long Term Capital Gains 30,000
Less: Long term loss (Balance of loss of 70,000 shall be carried forward) (30,000) 1,40,000
Income under the head Other Sources
Income from owning & maintaining race horses 15,000
Less: Loss carried forward to be adjusted (AY 2016-17) (15,000) Nil
(Balance b/f loss of Rs. 10,000 to be c/f to next year)
Gross Total Income 2,00,000
Note: Loss from speculative business of AY 2019-20: Rs. 60,000 to be c/f for 4 AYs starting from AY 2020-21.

Q10. Write a note on “Unabsorbed Depreciation” u/s 32(2).


Answer:
▪ An assessee having business or profession shall debit all expenditures of business/profession before debiting
depreciation i.e. depreciation shall be debited at the end.
▪ If there is a loss by debiting other expenditure, it will be called loss under the head business/profession.

DT QUESTION BANK BY CA PRANAV CHANDAK Subscribe us on YouTube 137


▪ Depreciation shall be debited only if income is available under the head business/profession & the depreciation
which cannot be debited shall be called unabsorbed depreciation & it will be allowed to be adjusted from any
income under any head except casual income.
▪ If it cannot be adjusted in the same year, its carry forward is allowed for unlimited period & in the subsequent
years, it can be set off from any income under any head except casual income.
▪ If any assessee has brought forward business loss as well as depreciation, business loss shall be adjusted first
& depreciation afterwards.

Q11. Mr. PC submits the following information for PY 2018-18 relevant to the AY 2019-20:
1. Profit from Business X situated in Bangalore 2,80,000
2. Profit from Business Y situated in Hyderabad 1,25,000
3. Loss from Business Z carried in Germany (business is controlled from India but profits 85,000
are not received in India)
4. Unabsorbed depreciation of business Z 45,000
5. Income from house property situated in India 30,000
6. Income from house property situated in London (rent received in London) 50,000
Find out the GTI of Mr. PC for AY 2019-20 if he is (a) ROR (b) RNOR & (c) NR.
Solution:
Particulars ROR RNOR NR
(i) Business Income
Business X (Profit) 2,80,000 2,80,000 2,80,000
Business Y (Profit) 1,25,000 1,25,000 1,25,000
Less: Business Z (Loss); [controlled from India but received o/s India] (85,000) (85,000) Nil
Less: Unabsorbed depreciation of business Z (45,000) (45,000) Nil
Total 2,75,000 2,75,000 4,05,000
(ii) Income from house property -
Property in India 30,000 30,000 30,000
Property in London 50,000 - -
Gross total income 3,55,000 3,05,000 4,35,000

Q12. Income from business = Rs. 1.5 lacs for AY 2019-20 without making following adjustments.
(i) Depreciation for the current year 30,000
(ii) Unabsorbed depreciation brought forward from AY 2018-19 15,000
(iii) Long-term capital loss for the current year 12,000
(iv) Unabsorbed business loss brought forward from AY 2009-10 50,000
(v) Unabsorbed speculation loss brought forward from AY 2017-18 15,000
(vi) Short-term capital loss for the current year 24,000
Compute total income for AY 2019-20 & the loss to be carried forward to next year.
Solution:
Business Income 1,50,000
Less: Depreciation (30,000)
Less: Unabsorbed depreciation b/f from AY 2018-19 (15,000)
Total income 1,05,000

Amount to be carried forward to the AY 2020-21:


1. LTCL & STCL for AY 2019-20 can be set off against income u/h capital gains only. It can be carried forward
upto 8 years commencing from the AY 2020-21.
2. Unabsorbed speculation loss brought forward from AY 2017-18 will be carried forward to next AY. It can be
set-off against speculation profit only in the next year.
Note: Unabsorbed business loss of Rs. 50,000 b/f from AY 2009-10 can neither be set-off against business profits
of AY 2018-19 nor can it be carried forward to next year because 8 years has already expired.

Q13. A share broker's firm derives brokerage income from shares sold & purchased on behalf of others. Firm also
enters into transactions of purchase & sale of shares on its own account without actual delivery in which it derived
loss. Advise whether the loss in share dealing can be allowed to be set off against income from brokerage.
Answer:
▪ As per section 43(5), a transaction in which a contract for sale or purchase of any commodity is settled otherwise
than by actual delivery, such transaction is called a speculative transaction.
▪ Income from activity of purchase/sale of share on its own account without actual delivery is speculative Income.
▪ Brokerage is a normal business income. Hence, loss in share dealing cannot be set off against brokerage.

DT QUESTION BANK BY CA PRANAV CHANDAK Subscribe us on YouTube 138


Q14. R discontinued his proprietary business in the AY 2018-19 for which year he had unabsorbed depreciation
of Rs. 75,000. For the AY 2018-19, his only source of income is from other sources. Can R claim the set off of
unabsorbed depreciation carried forward against income from other sources in the AY 2018-19? Discuss.
Answer:
▪ For set-off of unabsorbed depreciation brought forward from PY, it is not necessary that business in which the
depreciation was originally computed should be continued in the year in which set-off is claimed.
▪ Also, unabsorbed depreciation can now be set off from any head of income as it will form part of current
depreciation.

Q15 From the following information compute the total income of R for the AY 2018-19.
1. Income from House Property 60,000
2. Business Profits (before making the following deductions) 1,15,000
(i) Current depreciation allowance 30,000
(ii) Current scientific research revenue expenses 20,000
He brought forward the following losses/allowances:
(i) Unabsorbed business loss of the PY 2007-08 Rs. 25,000
(ii) Unabsorbed business loss of the PY 2016-17 Rs. 30,000.
(iii) Unabsorbed depreciation allowance of the PY 2015-16 Rs. 25,000
(iv) Unabsorbed depreciation allowance of the PY 2016-17 Rs. 24,000.
(v) Unabsorbed capital expenses on scientific research of the PY 2016-17 Rs. 25,000.
(iv) Unabsorbed loss from House Property of the PY 2016-17 Rs. 35,000.
Solution:
1. Income from house property 60,000
Less: Brought forward loss 35,000 25,000
2. Business Profits 1,15,000
Less: Current depreciation 30,000
Less: Scientific research expenses 20,000
Less: Unabsorbed business loss 2007-08
(period of set-off 8 years expired) Nil
Less: Unabsorbed business loss 2016-17 30,000
Less: Unabsorbed depreciation 2015-16 25,000
Less: Unabsorbed depreciation 2016-17 (can be set off from other income) 24,000
Less: Unabsorbed capital expenses of scientific research 25,000
(Set off allowed to the extent of other income)
(39,000) - 25,000
Total income Nil
R will carry-forward unabsorbed capital expense on scientific research Rs. 14,000 to set-off against income in the
subsequent assessment year.

Q16. X, a resident individual, submits the following information, relevant to PY 17-18:


(1) Income from salary (computed) 60,000
(2) Income from house property
House I 12,000
House II (50,000)
House III (Self-occupied) (10,000)
(3) Profit & gains of business or profession
Business I 8,000
Business II (12,000)
Business III (Speculative) (64,000)
Business IV (Speculative) 36,000
(4) Capital gains
Short-term capital loss (6,000)
Long-term capital gains on transfer of shares 5,400
(5) Income from other sources (computed):
Income from card games 36,000
Income from betting 24,000
Loss on maintenance of race horses (4,600)
Determine the gross total income for the AY 2018-19.

DT QUESTION BANK BY CA PRANAV CHANDAK Subscribe us on YouTube 139


Solution:
(i) Income from salary 60,000
Less: House property loss set off (48,000) 12,000
(ii) Income from House Property
House I 12,000
House II (50,000)
House III (10,000) (48,000)
Less: Set off from salary 48,000 Nil
(iii) Profit & gains from B/P
Business I 8,000
Business II (12,000) Nil
Business loss carried forward: Rs. (4,000)
Business III (Speculation) (64,000)
Business IV (Speculation) 36,000 Nil
Speculation loss carried forward: (28,000)
(iv) Capital Gain
Short-term capital loss (6,000)
Long-term capital gain 5,400
Capital Gains C/f to next year: (600)
(v) Income from other sources
Card Game (Computed) 36,000
Betting (Computed) 24,000 60,000
Loss on maintenance of race horse c/f: (4,600)
Gross total income 72,000

Q17. B submits the following particulars of his income & loss for the AY 2018-19:
(1) Income from house property (computed) 7,000
(2) Income from interest from a firm 1,500
(3) Profit from cloth business (before depreciation) 40,000
(4) Income from speculation business 3,200
(5) Long-term capital gain 9,100
(6) Dividend from UTI (units are held as stock-in-trade) (Gross) 2,000
(7) Current year's depreciation 2,000
The following items have been brought forward from the preceding year:
(i) Loss from cloth business: Rs. 10,000; (ii) unabsorbed depreciation: Rs. 7,500;
(iii) Loss from speculation: Rs.7,000; (v) STCL: Rs.4,200; (v) LTCL: Rs. 11,400.
You are required to compute his GTI & deal with the carry forward losses.
Solution:
(i) Income from House Property 7,000
(ii) Profit & Gains from Business & Profession
Interest from firm 1,500
Profit from Cloth business 40,000
Less: Depreciation (2000) 39,500
Less: Brought forward loss of cloth business (10,000)
Unabsorbed depreciation (7,500) (17,500) 22,000
Speculation Business 3,200
Less: B/f speculation loss (limited to 3,200) (3,200) Nil Nil
(iii) Capital Gains
Long-term capital gains 9,100
Less: Brought forward capital loss (limited to 9,100) (9,100) Nil
(iv) Income from other sources
Dividend from UTI Exempt
Gross total income 29,000

Losses to be carried forward


(a) Loss from speculation business 7,000 - 3,200 Rs.3,800
(b) Long-term capital loss 11,400-9,100 Rs.2,300
(c) Short-term capital loss Rs.4,200
Note: He should set off b/f LTCL first & c/f STCL of Rs.4,200 as it can be set off both from STCG & LTCG.

DT QUESTION BANK BY CA PRANAV CHANDAK Subscribe us on YouTube 140


Q18. R submits the following information relevant for PY 2017-18.
(a) Income from salary (computed) 62,000
(b) Interest on securities 2,000
(c) Income from House Property
House No. 1 12,000
House No. 2 ( 50,000)
House No. 3 (10,000)
(d) Profit & Gains from Business:
Business No. 1 16,000
Business No. 2 (12,000)
Business No. 3 (speculative) (64,000)
Business No. 4 (speculative) 36,000
(e) Capital gains:
Short-term capital gain (computed) ( 60,000)
Long-term capital gain (computed) 54,000
(f) Income from card games & betting (gross) 60,000
Loss from maintenance of race horses (46,000)
Income from maintaining horses & winning of races 20,000
Determine the total income of R for the AY 2018-19.

Solution: Computation of Total Income of R for AY 2018-19


(i) Income from salary 62,000
Less: House property loss (48,000) 14,000
(ii) Income from House Property [12,000 - 50,000 -10,000] (48,000)
Less: Set off from salary 48,000 Nil
(iii) Income from Profits & Gains from business
Non-Speculative Business [16,000 - 12,000] 4,000
Speculative Business (3) (64,000)
Speculative Business (4) 36,000 (28000) Nil
Carried forward of Speculation business loss: Rs (28000)
(iv) Income from capital gain
Short-term capital loss (60,000)
Long-term capital gain 54,000 Nil
Carried forward capital loss: Rs. (6,000)
(v) Income from other sources
Interest on securities 2,000
Income from card game & betting 60,000 62,000
Income from maintaining horses 20,000
Loss from maintenance of race horses (46,000) Nil
To be carried forward: Rs (26,000)
Gross total income 80,000

Q19. R furnishes the following particulars of his incomes & losses for PY 2017-18.
(i) Taxable income from salary 2,25,000
(ii) Interest on securities (Gross) 2,000
(iii) Income from house property (Net) 8,000
(iv) Profit from cloth business 20,000
(v) Speculation profit 5,000
(vi) Long-term capital gain 12,000
(vii) Short-term capital gain 4,000
(viii) Share of profit from a partnership firm 2,600
(ix) Current year depreciation 1,500

The items brought forward from PY 2017-18 are as follows:


(i) Unabsorbed depreciation 5,000
(ii) Speculation loss 15,000
(iii) Loss from stationary business (discontinued) 8,000
(iv) Long-term capital loss 15,000
(v) Short-term capital loss 6,000
Compute GTI of R for AY 2018-19 & give details of losses to be set-off & c/f by him.

DT QUESTION BANK BY CA PRANAV CHANDAK Subscribe us on YouTube 141


Solution: GTI of R (For AY 2018-19)
(i) Income from Salary 2,25,000
(ii) Income from House Property 8,000
(iii) Income from Business & Profession
Profit from cloth business 20,000
Less: Current depreciation (1,500)
Less: B/f loss from stationary business (8,000)
Less: Unabsorbed depreciation (5,000) (14,500) 5,500
Speculation profit 5,000
Less: B/f Speculation loss (5,000) Nil
Speculation loss of Rs. Rs. 10,000 is to be carried forward
(iv) Income from capital gain
Long-term capital gain 12,000
Short-term capital gains 4,000 16,000
Less: B/f Capital loss (LTCL 12,000 + STCL 4,000) (16,000) Nil
LTCL of Rs. 3000 & STCL of Rs. 2000 is to be carried forward
(v) Income from Other Sources
Interest on securities 2,000
Gross total income 2,40,500

Q20. Compute Total Income of Mr. Mohit for FY 2018-19 & ascertain losses which will be c/f to next year.
House No. 1 – Income after all statutory deductions 80,000
House No. 2 – Current Year Loss (38,000)
Income from Business
1. Textile Business: Discontinued from 30th September 2018
(a) Current Year's Loss 40,000
(b) Brought Forward Loss of AY 2015-2016 95,000
2. Chemical Business – since discontinued: Nil
(a) Bad Debts allowed in earlier years recovered during the year 35,000
(b) Brought Forward Business Loss of AY 2017-2018 50,000
3. Leather Business – Profit for the current year 1,00,000
4. Capital Gains:
(a) Short Term Capital Gains 60,000
(b) Long Term Capital Loss 35,000
Share of Profit in a Firm in which he is Partner since 2006 16,550

Solution: Computation of Total Income


Particulars Rs. Rs.
1. Income from House Property:
Income from House 1: 80,000
Less: Loss from House 2 (38,000) 42,000
2. Profits & Gains of Business or Profession:
(a) Income from Leather Business 1,00,000
Less: Current Year Loss of Textile Business (WN 1) (40,000)
Less: B/f Business Loss of Textile Business of AY 2015-16: (WN 2) (60,000)

(b) Chemical Business:Bad Debts recovered deemed as Business income u/s 41(4) 35,000
Less: B/f Business Loss of AY 2017-18: 50,000 restricted to Bad Debts recovery. (35,000)
(c) Share of Profit in a Firm – Exempt u/s 10(2A) Nil Nil
3. Capital Gains:
(a) Short Term Capital Gains 60,000
(b) Long Term Capital Loss – Can only be set off against LTCG. - 60,000
Gross Total Income 1,02,000
Working Note:
1. Current Year Business Loss can be set off against any Head of Income except Salaries.
2. Brought Forward Business Loss can be set off only against Profits & Gains of Business or Profession.
3. Losses to be carried forward to next AY:
(a) Loss from Textile Business: Rs. 35,000; (b) Loss from Chemical Business: Rs. 15,000
4. Loss of Discontinued Business shall be set off against Deemed Business Income u/s 41(4) & 41(5).

DT QUESTION BANK BY CA PRANAV CHANDAK Subscribe us on YouTube 142


Q21. Mr. Sohan submits the following details of his Income for the Assessment Year 2019-2020.
Particulars Rs. Particulars Rs.
Income from Salary 3,00,000 Winning in Card Games 6,000
Loss from Let Out House 40,000 Agricultural Income 20,000
Property 50,000 LTCG from Shares (STT paid) 10,000
Income from Sugar Business 60,000 Short Term Capital Loss u/s 111 10,000
Short Term Capital Loss 40,000 Bank Interest 5,000
Long Term Capital Gain Dividend 5,000 B/f Loss from Iron Ore Business 1,20,000
Lottery Winning (Gross) 50,000 (discontinued in 2012-2013)
Calculate Gross Total Income & Losses to be carried forward.
Solution: Computation of Taxable Income & Tax Liability
(i) Income from Salaries 3,00,000
Less: Deductions u/s 16(ia) Standard Deduction (40,000)
Income under the head “Salaries” 2,60,000
(ii) Income from House Property: Loss (40,000)
(iii) Profits & gains of Business or Profession
Income from Sugar Business 50,000
B/f Loss of Iron ore Business (Restricted to Rs. 50,000) [WN 1] (50,000) Nil
(iv) Capital Gains
LTCG from Sale of Shares (STT paid) taxable u/s 112A 10,000
Other Long-Term Capital Gains 40,000
Short Term Capital Loss [W.N. 2] – Restricted to Rs. 40,000 (40,000) Nil
Short Term Capital Loss u/s 111A [WN 3] Nil Nil
(v) Income from Other Sources
Dividend 5,000
Less: Exemption u/s 10(34) (5,000) Nil
Less: Income from Lottery Winning 50,000
Less: Winning in Card Games 6,000
Less: Bank Interest 5,000 61,000
Gross Total Income 2,91,000
Less: Deduction under Chapter VI-A
Sec 80TTA: Interest on Deposits in Savings Account (5,000)
Total Income 2,86,000
Computation of Tax Liability:
Special Rates: For Winning from Lottery & Card Games [Rs. 56,000 × 30%] 16,800
Tax on other income @ slab rate: Nil
Less: Rebate u/s 87A (2,500) 14,300
Add: HEC @ 4% 572
Total Tax Payable (Rounded Off) 14,880
Working Notes:
1. Brought Forward Loss from Iron Ore Business (being discontinued Business) can be set-off only to the extent of
Profits available. The Balance [ Rs. 70,000( Rs. 1,20,000 – Rs. 50,000)] is carried forward to the next AY.
2. STCL can be set-off against any Capital Gains. Balance of Rs. 20,000 is carried forward to next AY.
3. STCL u/s 111 is carried forward for 8 Assessment Year & can be set-off against any Capital Gains.
4. LTCG on shares u/s 112A shall be taxed at 10% for the amount in excess of Rs. 1 Lac. Hence, the tax is nil.
5. Agricultural Income is exempt u/s 10(1).
6. Winnings from Lottery & Card Games are subject to Tax at 30%.
7. When Total Income of Resident Individual does not exceed Rs. 3.5 Lacs, Rebate u/s 87A = 100% of Tax payable
or Rs. 2,500 whichever is less.

DT QUESTION BANK BY CA PRANAV CHANDAK Subscribe us on YouTube 143


7. DEDUCTIONS
Q1. Mr. X furnishes you the following information:
Raw material purchased Rs. 5,00,000
Manufacturing expenses (revenue nature) Rs. 2,00,000
Sale price Rs. 18,00,000
Plant & machinery acquired [Depreciation @ 15%. Rs. 2,60,000
He has made the investments as given below:
(i) Fixed deposit with State Bank for two years Rs. 5,000.
(ii) Investment in National Saving Certificates Rs. 5,000.
(iii) Deposit in Public Provident Fund Account in the name of major married independent son Rs. 5,000.
(iv) Deposit in Public Provident Fund Account in the name of minor son Rs. 5,000.
(v) Payment of premium for LIC policy in name of major married independent daughter on 15.09.2018 Rs. 5,000.
(sum assured Rs. 1,00,000).
(vi) Payment of premium for LIC policy in name of major married independent son on 11.11.2018 Rs. 5,000. (sum
assured Rs. 20,000)
(vii) Investment in Home Loan Account Scheme of National Housing Bank Rs. 5,000 (Investment was made out of
past savings).
(viii) Investment in units of Mutual Funds notified u/s 10(23D) Rs. 5,000. (Investment was made out of current
income exempt from income tax).
(ix) Investment in Equity Shares of Infrastructure Companies Rs. 5,000.
(x) Payment of Tuition fees of his son to a private coaching centre for coaching in taxation Rs. 5,000.
Compute his income & tax liability for AY 2019-20.
Solution: Computation of income under the head Business/profession
Sale price 18,00,000
Less: Purchase Price (5,00,000)
Less: Manufacturing expenses (2,00,000)
Less: Depreciation on plant & machinery (2,60,000 x 15%) (39,000)
Income under the head Business/profession/ Gross Total Income 10,61,000
Less: Deduction u/s 80C
National Saving Certificate (5,000)
Public Provident Fund (10,000)
LIC policy in name of major married independent daughter (5,000)
LIC policy in name of major married independent son (2,000)
Home Loan Account Scheme (5,000)
Units of Mutual Funds (5,000)
Equity Shares of Infrastructure Companies (5,000)
Total Income 10,24,000

Computation of Tax Liability


Tax on Rs. 10,24,000 at slab rate 1,19,700
Add: HEC @ 4% 4,788
Tax Liability 1,24,488
Rounded off u/s 288B 1,24,490

Q2. Compute the eligible deduction u/s 80C for AY 2019-20 in respect of life insurance premium paid by Mr. X
during the PY 2018-19, the details of which are given hereunder: [Mod. NOV-2014]
Date of issue of Person insured Actual capital sum Insurance premium
Policy Assured paid during 2018-19
1.6.2011 Mr. X 4,00,000 75,000
1.5.2017 Mrs. X, his wife 1,00,000 25,000
1.7.2018 Ms. Y, his handicapped daughter 5,00,000 60,000
(section 80U disability)
1.7.2018 Mr. Z, his son 1,00,000 25,000
1.10.2018 Mr. A, his father 2,00,000 60,000
Total Premium paid 1,85,000
Solution: Computation of Eligible Deduction u/s 80C for AY 2019-20

DT QUESTION BANK BY CA PRANAV CHANDAK Subscribe us on YouTube 144


Date of issue of Person insured Actual capital Insurance Restricted to % Deduction u/s 80C
policy sum assured premium paid of sum assured) for AY 2019-20
during 2018-19
1.6.2011 Mr. X 4,00,000 75,000 20% 75,000
1.5.2017 Mrs. X 1,00,000 25,000 10% 10,000
1.7.2018 Handicapped 5,00,000 60,000 15% 60,000
Daughter
1.7.2018 Son 1,00,000 25,000 10% 10,000
Total 1,55,000
Maximum deduction u/s 80C restricted to 1,50,000
Note: No deduction is available in case of payment of Life insurance premium on the life of parent u/s 80C.
In respect of policies issued Maximum deduction u/s 80C (% of actual
capital sum assured)
Between 01.04.2003 & 31.03.2012 20%
Between 01.04.2012 & 31.03.2013 10%
on or after 01.04.2013
- Insurance on life of person with disability u/s 80U 15%
- Others 10%

Q3. The particulars of income of Mrs. X. aged 55 years for the financial year 2018-19 are given below:
Gross salary received from M/s ABC Ltd. for the year 4,00,000
Rental income received from a commercial complex 1,44,000
Arrears of rent received from the complex, which were not taxed in any earlier years 30,000
Interest paid on loan taken from bank for purchase of house used as residence 30,000
Repayment of instalments of loan taken from bank for purchase of above property 60,000
Deposits in PPF A/c Out of current year’s income 40,000
Investment made in units of a mutual fund approved by the board u/s 80C 40,000
Compute the total income of Mrs. X & the tax payable thereon in respect of AY 2019-20. [MAY – 2001]
Solution: Computation of total income & tax liability Mrs. X Income from salary
Income under the head Salary 4,00,000
Income from house property
Let out commercial complex
Gross Annual Value (12,000 x 12) 1,44,000
Less: Municipal taxes Nil
Net Annual Value 1,44,000
Less: 30% of NAV u/s 24(a) (43,200)
Less: Interest on capital borrowed u/s 24(b) Nil
Income from let out property 1,00,800
Property self- occupied for residence Nil
Net Annual Value Nil
Less: Interest on capital borrowed u/s 24(b) (30,000)
Income from self-occupied property (30,000)
Arrears of rent Section 25A [70% taxable] 21,000
Income u/h House Property 91,800
Gross Total Income 4,91,800
Less: Deduction u/s 80C (1,40,000)
Repayment of loan taken to purchase residential house property 60,000
Deposit in public provident fund out of current income 40,000
Investment made in units of mutual fund for infrastructure facility 40,000
Total Income 3,51,800

Computation of Tax Liability


Tax on Rs. 3,51,800 at slab rate 5,090
Add: HEC @ 4% 204
Tax Liability 5,294
Rounded off u/s 288B 5,290

DT QUESTION BANK BY CA PRANAV CHANDAK Subscribe us on YouTube 145


Q4. Kalpesh borrowed a sum of Rs. 30 Lacs from the National Housing Bank for purchase of a Residential Flat. The
Loan amount was disbursed directly to the Flat Promoter by the Bank. Though the construction was completed in
May 2019, repayments towards Principal & Interest had been made during the year ended 31.03.2019.
In the light of the above facts, state:
(a) Whether Kalpesh can claim deduction u/s 24 in respect of interest for AY 2019-2020?
(b) Whether deduction u/s 80C can be claimed for the above AY, even though the construction was completed only
after the closure of the year? [May 08]
Answer:
(a) Prior Period Interest means the interest from the date of borrowal of the loan upto the end of the financial year
immediately preceding the financial year in which acquisition was made or construction was completed.
(b) Prior Period Interest shall be allowed in five equal instalments commencing from the financial year in which
the property was acquired or construction was completed.
▪ In the given case, Kalpesh cannot claim deduction u/s 24 in respect of Interest for the AY 2019-2020. He can
claim interest paid as Prior Period Interest (in 5 equal instalments) from AY 2020-2021.
▪ Deduction u/s 80C in respect of Housing Loan Repayments can be claimed in respect of a Residential House
Property, the income from which is chargeable to tax under the head ‘Income from House Property’. Unless the
construction is completed, a property is not chargeable u/s 23.
▪ Therefore, deduction u/s 80C can be claimed only if the construction of the house is completed. Hence, Kalpesh
cannot claim deduction in respect of principal repayment during AY 2019-2020.

*Q5. From following particulars, compute deduction u/c VI-A for AY 2019-2020 of Mr. H, employed with Arun Ltd.
(a) Contribution to PPF 20,000
(b) Life Insurance Premium for self (Policy taken on 01.05.2012, Sum Assured Rs. 1,00,000) 35,000
(c) Deposit in 5-Year Term Deposit with Bank 20,000
(d) Contribution to NPS (15% of his Salary, Matching Contribution was made by Arun Ltd) 1,80,000
(e) Subscription to Sukanya Samriddhi Account paid by Mr. H for his girl 10,000
(f) Payment of tuition fees to Apeejay School, New Delhi, for education of his son studying in XI 20,000
(g) Repayment of housing loan taken from Standard Chartered Bank 15,000
(h) Contribution to approved pension fund of LIC 5,000
Solution: Computation of Deduction under Chapter VI-A
Particulars Rs. Rs.
Sec. 80C: Deposit in Public Provident Fund 20,000
LIC Premium (Restricted to 10% of Sum Assured) 10,000
5-Year Term Deposit with Bank 20,000
Subscription to Sukanya Samriddhi Account 30,000
Payment of tuition fees for education of his son studying in Class XI 20,000
Repayment of housing loan taken from Standard Chartered Bank 15,000
Total of above u/s 80C 1,15,000 1,15,000
80CCC: Contribution to approved pension fund of LIC 5,000 5,000
80CCD(1): Contribution to NPS by Mr. H (upto 10% of Salary)(
1,80,000
) × 10% 1,20,000 30,000
15%
(Note 1)
80CCE: Total Deduction u/s 80C + 80CCC + 80CCD(1) cannot Exceed 150000 1,50,000
80CCD(1B): Additional deduction: Contribution to NPS by Mr. H (Note 2) 50,000
80CCD(2): Contribution to NPS by Arun Ltd (
1,80,000
) × 10% (Note 2 & 3) 1,20,000
15%
Total Deduction under Chapter VI-A 3,20,000

Note:
1. U/s 80CCE, Maximum Deduction u/s 80C, 80CCC & 80CCD(1) cannot exceed Rs. 1,50,000. Since deduction u/s
80C & 80CCC is Rs. 1,20,000 (being the maximum limit), only Rs. 30,000 will be allowed as deduction u/s
80CCD(1). However, Employee can claim additional deduction of Rs. 50,000 u/s 80CCD(1B) for balance amount.
2. Eligible contribution u/s.80CCD( 1B) & 80CCD(2) are not covered in overall limit of Rs. 1,50,000 u/s 80CCE.
3. Employer’s contribution to NPS has been included as income of employee u/h ‘Salaries’ while computing his
Gross Total Income & then, deduction u/s 80CCD(2) can be claimed.

DT QUESTION BANK BY CA PRANAV CHANDAK Subscribe us on YouTube 146


Q6. Mr. X, Mr. Y & Mr. Z, new retail investors for PY 2016-17 have made the following investment in equity
shares/units of EOF of Rajiv Gandhi Equity Savings Scheme for PY 2018-19 as below:
Particulars Mr. X Mr. Y Mr. Z
Investment in listed equity shares 20,000 45,000 30,000
Investment in units of equity-oriented fund 40,000 - 10,000
GTI (comprising of salary income & bank interest) 11,25,000 12,15,000 11,80,000
All has claimed deduction u/s 80CCG for AY 17-18 & AY 18-19. Compute deduction u/s 80CCG for AY 19-20.
Solution: Computation of Deduction u/s 80CCG for the AY 2019-20
Particulars Mr. X Mr. Y Mr. Z
Deduction u/s 80CCG Rs. 25,000 NIL Rs. 20,000
for AY 2019-20 (50% of 65,000 = Rs. 32,500 (Since GTI > Rs. 12 lacs) (50% of 40,000)
but restricted to Rs. 25,000)
Note: Mr. X & Mr. Z are eligible for deduction u/s 80CCG for AY 2019-20, since he has claimed deduction for AY
2017-18 & AY 2018-19 & fulfills the conditions for claim of deduction in AY 2019-20.

Q7. Mr. A, aged 40 years, paid medical insurance premium of Rs. 20,000 during PY 18- 19 to insure his health as well
as the health of his spouse. He also paid medical insurance premium of Rs. 47,000 during the year to insure the
health of his father, aged 63 years, who is not dependent on him. He contributed Rs. 3,600 to CG Health Scheme
during the year. He has incurred Rs. 3,000 in cash on preventive health check-up of himself & his spouse & Rs.
4,000 by cheque on preventive health check-up of his father. Compute deduction u/s 80D for AY 2019-20.
Solution: Deduction allowable u/s 80D for the AY 2019-20
Particulars Amt Paid Deduction
1 Premium paid & medical expenditure incurred for self & spouse
(a) Medical insurance premium paid for self & spouse 20,000 20,000
(b) Contribution to CGHS 3,600 3,600
(c) Exp. on preventive health check-up of self & spouse 3,000 1,400
26,600 25,000
2 Premium paid & medical expenditure incurred for father, who is a senior citizen
(a) Medi-claim premium paid for father, who is over 60 years of age 47,000 47,000
(b) Expenditure on preventive health check-up of father 4,000 3,000
51,000 50,000
Total deduction u/s 80D (Rs. 25,000 + Rs. 50,000) 75,000
Notes:
1. Total deduction for 1(a),(b),(c) above should not exceed Rs. 25,000. Therefore, expenditure on preventive
health check-up for self & spouse would be restricted to Rs. 1,400 (Rs. 25000 - Rs. 20000 - Rs. 3600).
2. Total deduction for 2(a)&(b) above should not exceed Rs. 50,000. Therefore, expenditure on preventive
health check-up for father would be restricted to Rs. 3,000 (Rs. 50,000 - Rs. 47,000).
3. In this case, total deduction allowed on account of expenditure on preventive health check-up of self, spouse
& father is Rs. 4400 (Rs. 1400 + Rs. 3000), which is less than the maximum permissible limit of Rs. 5,000.

Q8. Mr. Y, aged 40 years, paid medical insurance premium of Rs. 22,000 during PY 2018-19 to insure his health as
well as the health of his spouse & dependent children. He also paid medical insurance premium of Rs. 33,000
during the year to insure the health of his mother, aged 67 years, who is not dependent on him. He incurred medical
expenditure of Rs. 20,000 on his father, aged 71 years, who is not covered under mediclaim policy. His father is
also not dependent upon him. He contributed Rs. 6,000 to Central Government Health Scheme during the year.
Compute deduction allowable u/s 80D for the AY 2019-20.
Solution: Deduction allowable u/s 80D for the AY 2019-20
Particulars Rs. Deduction
Medical Insurance Premium paid for self, spouse & Dependent children Rs. 22,000
Contribution to CGHS Rs. 6,000
Total Expenditure for Family Rs. 28,000 Rs. 25,000
Medi-claim premium paid for mother, who is over 60 years Rs. 33,000
Medical expenditure incurred for father, who is over 60 years of age & not Rs. 20,000
covered by any insurance
Total Expenditure for Parents Rs. 53,000 Rs. 50,000
Total Deduction u/s 80D = Rs. 25,000 + Rs. 50,000 = Rs. 75,000.

DT QUESTION BANK BY CA PRANAV CHANDAK Subscribe us on YouTube 147


Q9. Mr. Arjun (52 years old) furnishes the following particulars in respect of the following payments:
Premium paid for insuring the health of -
• Self 10,000
• Spouse 8,000
• Dependant son 4,000
• Mother 18,000
Paid for Preventive Health Check-up of
• himself 2,000
• spouse 1,500
• mother 4,000
Incurred medical expenditure of Rs. 25,000 & Rs. 15,000 for his mother, aged 80 years & father, aged 85
years. Both mother & father are resident in India.
Compute the deduction available to Mr. Arjun u/s 80D for the AY 2019-20.

Solution: Computation of deduction u/s 80D for the AY 2019-20


Particulars Amt Amt
(a) Deduction In respect of premium paid for insuring the health of -
▪ Self 10,000
▪ Spouse 8,000
▪ Dependent Son 4,000 22,000
(b) In respect of expenditure on preventive health check-up of –
▪ Self 2,000
▪ Spouse 1,500
Restricted to [25,000 - 22,000] = Rs. 3,000 since maximum deduction is 25,000] 3,500 3,000
Aggregate of deduction (a+b) under (1) is restricted to 25,000
(a) Payment towards health insurance premium for his mother 18,000
(b) Preventive health check-up of his mother: 4000 restricted to 2000 (5000 -3000)
[since maximum deduction for preventive health check-up u/s 80D is Rs. 5,000] 2,000
(c) Medical expenditure for father would only be eligible for deduction [Note below] 15,000 35,000
Total deduction u/s 80D [(1) + (2)] 60,000
Note: Irrespective of the fact that the mother of Arjun is a very senior citizen, deduction u/s 80D would not
available to him in respect of medical expenditure incurred for his mother, since Mr. Arjun has taken a health
insurance policy for his mother.

Q10. GTI of A for PY 2018-19 is Rs. 3,50,000 which includes LTCG of Rs. 2,80,000 & STCG of Rs. 10,000. Beside this
(a) He has deposited Rs. 12,000 to effect a contract for Annuity Plan of LIC.
(b) He paid the following premium to New India Assurance Co Ltd for Mediclaim scheme for himself & his relatives.
(i) His own health 1,000
(ii) For health of Spouse 600
(iii) Major Son not dependent on him 800
(iv) Mother dependent on him 1,200
(v) Brother dependent on him 1,100
(c) His brothers is totally blind & dependent on him for medical treatment. Mr. A spends 10,000 on his blind
brother. He has also deposited Rs. 25,000 in a Scheme framed by UTI for maintenance of his dependent brother.
Compute his Total Income & Tax Payable for the AY 2019-2020.
Solution: Computation of Total Income & Tax Payable
Particulars Rs Rs
Gross Total Income (including LTCG of Rs. 2,80,000) 3,50,000
Less: Deduction Under Chapter VI-A
U/s 80CCC – Contribution to Pension Fund 12,000
U/s 80D – Medical Insurance Premium (1,000 + 600 + 1,200) (Note 1) 2,800
U/s 80DD – Medical Expenditure on Dependent (Note 2) 1,25,000
Total (Note 3) 1,39,800 (70,000)
Total Income 2,80,000
Tax on Total Income [2,80,000 – 2,50,000] x 20% (Note 4) 6,000
Less: Rebate u/s 87A (Note 5) (2,500)
Tax Payable + HEC at 4% 3500 + 140 3,640

DT QUESTION BANK BY CA PRANAV CHANDAK Subscribe us on YouTube 148


Note:
1. Mediclaim Premium:
(a) Premium paid on Major Son not dependent on the Assessee is not eligible for deduction.
(b) Premium paid on dependent brother is not eligible for deduction.
(c) It is assumed that the Premium is not paid in cash.
2. Deduction of 1,25,000 is available irrespective of the amount spent for severe disable person. (fully blind.
3. Deduction restricted to Rs. 70,000 [Gross Total Income of Rs. 3,50,000 Less LTCG of Rs. 2,80,000] since (a)
LTCG is not eligible for Chapter VI-A Deduction, & (b) Deductions cannot exceed GTI exclusive of LTCG]
4. Tax Liability: Since Total Income excluding LTCG is less than BEL, benefit of unexhuatsed BEL will be
available from LTCG. Thus tax is determined as [Total Income including LTCG – Basic Exemption] * 20%.
5. Rebate u/s 87A: When Total Income of Resident Individual does not exceed Rs. 3.5 Lacs, Rebate u/s 87A =
100% of Tax Payable or Rs. 2,500 whichever is less.

Q11. Mr. B has taken three education loans on April 1, 2018. Compute deduction u/s 80E for AY 2019-20.
Particulars Loan 1 Loan 2 Loan 3 Loan 4 Loan 5
Education loan was taken for B Son of B Daughter of B Spouse Y, his Friend
Purpose of Loan MBA B. Sc. 10th BA CA
Amount of Loan 5,00,000 2,00,000 4,00,000 10,00,000 6,00,000
Annual repayment of Loan 1,00,000 40,000 80,000 2,00,000 1,20,000
Annual repayment of Interest 20,000 10,000 18,000 40,000 24,000
Solution:
▪ Deduction u/s 80E is available to Individual for any Interest paid by him in PY in respect of loan taken for
pursuing higher education of himself/spouse/children. No deduction is vaialble for loan taken for 10th class.
▪ Therefore, Interest repayment of Loan 1, Loan 2, Loan 4 will qualify for deduction.
▪ Deduction u/s 80E = 20,000 + 10,000 + 40,000 = Rs. 70,000.

Q12. Mr. A purchased a residential house property for self-occupation @ Rs. 45 lacs on 1.4.2017, in respect of
which he took a housing loan of Rs. 35 lacs from Bank of India @ 11% p.a. on same date. Loan was sanctioned on
28th March 2017. Compute deduction u/s 80EE for AY 2019-20 assuming that the entire loan was outstanding as
on 31.3.2019 & he does not own any other house property.
Solution:
▪ Total Interest payable for AY 2019-20 = 35 lacs × 11% = Rs. 3,85,000.
▪ Deduction allowable u/s 24(b) while computing Income u/h “Income from house property” = Rs. 2,00,000.
▪ Deduction u/s 80EE = Rs. 1,85,000 (3,85,000 – 2,00,000).
▪ But Maximum Deduction u/s 80EE = Rs. 50,000.

Q13. Mr. Shiva aged 58 years, has GTI of Rs. 7,75,000 comprising of Income from Salary & House Property. He
has made the following payments & investments:
(i) Premium paid to insure life of major daughter on 1.4.2018 (Sum Assured Rs. Rs. 1,80,000) - Rs. 20,000;
(ii) Medical Insurance Premium for self - Rs. 12,000; Spouse- Rs. 14,000;
(iii) Donation to a public charitable institution registered under 80G - Rs. 50,000 by way of cheque;
(iv) LIC Pension Fund - Rs. 60,000;
(v) Donation to National Children’s Fund - Rs. 25,000 by way of cheque;
(vi) Donation to Jawaharlal Nehru Memorial Fund - Rs. 25,000 by way of cheque;
(vii) Donation to approved institution for promotion of family planning - Rs. 40,000 by cheque.
(viii) Deposit in PPF – Rs. 1,00,000.
Compute the total income of Mr. Shiva for AY 2019-20.
Solution: Computation of Total Income of Mr. Shiva for AY 2019-20
Gross Total Income 7,75,000
Less:
Deduction u/s 80C
Deposit in PPF (1,00,000)
Life insurance premium paid for major daughter (Max 10% of Sum Assured) (18,000)
Deduction u/s 80CCC in respect of LIC pension fund (60,000)
Total Investment eligible for deduction u/s 80C 1,78,000
Deduction u/s 80C [Restricted to Rs. 1,50,000] (1,50,000)
Deduction u/s 80D
Medical Insurance premium in respect of self & spouse (restricted to Rs. 25,000) (25,000)
Deduction u/s 80G (See Working Note below) (87,500)
Total income 5,79,950

DT QUESTION BANK BY CA PRANAV CHANDAK Subscribe us on YouTube 149


Working Note: Computation of Deduction u/s 80G
(i) Adjusted GTI = GTI – All Deductions except 80G = Rs. 7,75,000 – Rs. 1,50,000 – Rs. 25,000 = Rs. 6,00,000.
(ii) Qualifying Limit = 10% of Adjusted GTI = Rs. 60,000.

Particulars of donation Donation % of Deduction Deduction u/s 80G


National Children’s Fund 25,000 100% 25,000
Jawaharlal Nehru Memorial Fund 25,000 50% 12,500
Approved Institution for promotion 40,000 100%, subject to Qualifying Limit 40,000
of family planning
Public Charitable Trust 1,50,00050% subject to qualifying limit 10,000
(See Note below)
Total Deduction u/s 80G Rs. 87,000
Note:
▪ Firstly, donation of Rs. 40,000 to approved institution for family planning qualifying for 100% deduction
subject to qualifying limit, should be adjusted against Qualifying Limit of Rs. 60,000.
▪ Thus, Qualifying Limit left after such adjustment = Rs. 20,000 [Rs. 60,000 – Rs. 40,000]
▪ Now, even though we have donated Rs. 1,50,000 to public charitable trust which qualify for 50% donation, only
the balance qualifying limit of Rs. 20,000 shall be available for taking deduction.
▪ Thus, we will calculate deduction as if we have contributed only Rs. 20,000 to public charitable trust.
▪ Hence contribution of Rs. 1,50,000 to public charitable trust is restricted to Rs. 20,000 (Rs. 60,000 - Rs. 40000)
50% of which would be the deduction u/s 80G.
▪ Therefore, deduction u/s 80G in respect of donation to public charitable trust = Rs. 10,000 (50% of Rs. 20,000).

Q14. Ram has computed his income under various heads for Previous Year 2018-2019 as under -
Particulars Rs. Particulars Rs.
Income under the head Salary 1,25,000 Donations to -
Income under the head House Property (15,000) Prime Minister's Drought Relief Fund 3,000
Profits & Gains of Business or Profession 90,000 National Fund for Communal Harmony 4,000
Short-term Capital Gains 30,000 Jawaharlal Nehru Memorial Fund 4,000
Long-term Capital gains 40,000 Prime Minister's National Relief Fund 4,200
Income from Other Sources - Government for Family Planning 13,000
Winnings of Lotteries 15,000 Approved Charitable Institution 7,000
Interest on Govt Securities 20,000 Deposits in PPF A/c 70,000
Payment by cheque for Mediclaim Policy 5,000 Re-payment of loan including interest of Rs. 35,000
Expenses on Medical treatment of 25,000 15,000 to Canara Bank,which was taken for
dependent handicapped son pursuing approved higher education
Compute (a) Total Income for AY 2019-2020, & (b) Tax Liability.
Solution: Computation of Total Income & Tax Payable thereon
Particulars Rs. Rs.
Salaries 1,25,000
Income from House Property (Loss) (15,000)
Profits & Gains of Business or Profession 90,000
Capital Gains
(i) Short Term Capital Gain 30,000
(ii) Long Term Capital Gain 40,000 70,000
Income from Other Sources
Winnings from Lotteries 15,000
Interest on Government Securities 20,000 35,000
Gross Total Income 3,05,000
Less: Deduction under Chapter VI-A
u/s 80C Contribution to PPF 70,000
u/s 80D Medical Insurance Premium Paid 5,000
u/s 80DD Expenditure on Medical Treatment of Dependent Son 75,000
u/s 80E Interest paid on Repayment of Education Loan 15,000,
u/s 80G Donations (See Note below) 21,700 (1,86,700)
Total Income 1,18,300

DT QUESTION BANK BY CA PRANAV CHANDAK Subscribe us on YouTube 150


Tax on Total Income
1. Income taxable @ slab rate: (Rs. 1,18,300 – Rs. 15,000 – Rs. 40,000 = Rs. 63,300) Nil
2. Income chargeable at Special Rates
(a) Lottery Income = Rs. 15,000 x 30% 4,500
(b) LTCG [Unexhuasted BEL = 186,700 (2,50,000 – 63,300)] Nil 4,500
Less: Rebate u/s 87A (2,500)
Tax Payable 2,000
Add: HEC at 4% 80
Total Tax Payable (Rounded off) 2,080

Working Note: Computation of Deduction u/s 80G


1. Computation of 10% of Adjusted Total income
Particulars Rs. Rs.
Gross Total Income 40,000 2,25,000
Less: Long-Term Capital Gain + Deduction u/s 80C to 80U, excluding Sec. 80G 1,65,000 (2,05,000)
(70,000 + 5,000 + 75,000 + 15,000)
Adjusted Total Income 20,000
10% of Adjusted Total Income (10% x Rs. 20,000) Rs. 2,000
Note: It is assumed that Short Term Capital Gains is not arising from transfer of Listed Securities (i.e. not covered
by Sec. 111A) & hence, not reduced to determine Adjusted Total Income.

2. Computation of Amount of Donation & Deduction u/s 80G


Nature of Donation Amt donated Eligible % of deduction Deduction
National Fund for Communal Harmony 4,000 Unrestricted, 100% 4,000
Prime Minister’s National Relief Fund 4,200 Unrestricted, 100% 4,200
Prime Minister’s Drought Relief Fund 3,000 Unrestricted, 50% 1,500
Jawaharlal Nehru Memorial Fund 4,000 Unrestricted, 50% 2,000
Family Planning (Government) 13,000 See WN 3, 4 10,000
Approved Charitable Institutions 7,000
Total 35,200 21,700

3. Maximum Permissible Deduction for donations qualifying for restricted deduction = Least of 10% of Adjusted
Total Income (WN 1) or Amount under Restricted Donations (13,000 + 7,000) = Least of 10,000 or 20,000 = 10,000.

4. Amount of Restricted Deduction is computed as under, out of the total eligible limit of Rs. 10,000
Particulars Rs.
Eligible for 100%: Family Planning = Rs. 13,000 x 100% – restricted to deduction available 10,000
Eligible for 50%: Approved Charitable Institutions = 7,000 * 50% = 3,500. Nil
restricted to deduction available u/s 80G for this category of Donation
Deduction available u/s 80G under this category of Donation (WN 3) 10,000
5. Assumed that Assesse is Resident. Hence, LTCG is not liable to tax since it is less than unexhausted BEL.

Q15. Mr. X declares gross total income Rs. 4,00,000 for AY 2019-20. Gross total income includes taxable LTCG of
Rs. 65,000 & STCG u/s 111A of Rs. 35,000. The details of fund investment made during the year 2018-19 are:
Medical insurance premium paid by cheque -
(a) in the name of Mr. X 4,000
(b) in name of Mrs. X 5,000
Contribution made to -
(a) Indira Gandhi Memorial Trust by cheque 7,000
(b) Delhi University (declared as an institution of national eminence) by cheque 3,000
(c) Zila Saksharta Samiti by cheque 5,000
(d) An approved charitable institute by cheque 30,000
(e) Government by cheque for the purpose of promoting family planning 10,000
(f) Hanuman Temple in Mohalla by cheque 20,000
Compute the total income of Mr. X chargeable to tax for AY 2019-20 & compute his tax liability. [NOV – 2008]

DT QUESTION BANK BY CA PRANAV CHANDAK Subscribe us on YouTube 151


Answer: Computation of Total Income of Mr. X for the AY 2019-20
Gross Total Income 4,00,000
Less : Deduction u/s 80D
Medical insurance premium paid by cheque
(i) in the name of Mr. Prasad (4,000)
(ii) in name of Mrs. Prasad (5,000)
Deduction u/s 80G
Donation to Indira Gandhi Memorial trust @ 50% of Rs. 7,000 (3,500)
Donation to Delhi University @ 100% (3,000)
Donation to Zila Saksharta Samiti @ 100% (5,000)
Other donations u/s 80G [Refer working Note below] (19,550)

Working Note:
1. Donations subject to Qualifying Limit:
Donation to Government for promoting family planning 10,000
Donation to approved Charitable Institute 30,000
Total 40,000

2. Adjusted GTI = GTI - LTCG - STCG u/s 111A - Deduction u/s 80C to 80U (Except 80G)
= 4,00,000 - 65,000 - 35,000 - 9,000 = 2,91,000
3. Qualifying amount = 10% of Adjusted GTI or Donation whichever is less = 29,100 or 40,000 = 29,100.
* Deduction for family planning = 100 % of Rs. 10,000 = Rs. 10,000;
* Approved charitable Institute = 50% of balance amount (i.e. 19,100) = 9,550
* Total deduction = 10,000 + 9,550 = 19,550
Note: Donation to hanuman temple in Mohalla is not deductible u/s 80G.

Q16. Mr. X is a retired Government officer aged 65 years, who derived the following income in respect of financial
year 2018-19. He resides in Cochin:
Pension 1,95,000
Interest from bank deposits (fixed deposits) 1,52,000
He has paid Rs. 18,000 as premium to effect an insurance on his health & his dependant parents & it was paid by
a cheque. He pays a rent of Rs. 3,000 per month in respect of furnished accommodation. What is his eligibility for
deduction u/s 80GG? Compute his total income & tax liability for AY 2019-20. [MAY – 2000]
Solution:
Income u/h Salary: Pension 1,95,000
Income u/h “Other Sources”: Bank Interest 1,52,000
Gross Total Income 3,47,000
Less: Deduction u/s 80D (18,000)
Less: Deduction u/s 80TTB (50,000)
Less: Deduction u/s 80GG [Note below] (8,100)
Note: Adjusted GTI = Rs. 3,47,000 - Rs. 18,000 - Rs. 50,000 = Rs. 2,79,000.
Least of the following is exempt u/s 80GG:
(a) Rs. 5,000 Per Month = Rs. 60,000
(b) Excess of Rent paid over 10% of Adjusted GTI = Rs. 36,000 - 27,900 = Rs. 8,100.
(c) 25% of Adjusted GTI = 25% x 2,79,000 = Rs. 69,750.

Q17. Mr. A has commenced the business of manufacture of computers on 1.4.2018. He employed 350 new
employees during the PY 2018-19, the details of whom are as follows:
No. of employees Date of employment Regular/ Casual Total monthly emoluments per employee
75 1.4.2018 Regular 24,000
125 1.5.2018 Regular 26,000
50 1.8.2018 Casual 25,500
100 1.9.2018 Regular 24,000
Regular employees participate in RPF while casual employees do not. Compute the deduction available to Mr. A
for AY 2019-20, if the profits & gains derived from manufacture of computers that year is Rs. 75 lacs & his total
turnover is Rs. 2.16 crores. What if Mr. A has commenced business of manufacture of apparel on 1.4.2018?

DT QUESTION BANK BY CA PRANAV CHANDAK Subscribe us on YouTube 152


Solution:
Mr. A is eligible for deduction u/s 80JJAA since he is subject to tax audit u/s 44AB for AY 2019-20, as his total
turnover from business exceeds Rs. 1 crore & he has employed “additional employees” during the PY 2018-19.

(I) If Mr. A is engaged in the business of manufacture of computers


➢ Additional employee cost = Rs. 24,000 × 12 × 75 [See Working Note below] = Rs. 2,16,00,000
➢ Deduction u/s 80JJAA = 30% of Rs. 2,16,00,000 = Rs. 64,80,000.

Working Note: Number of Additional Employees


Particulars No. of workmen
Total number of employees employed during the year 350
Less: Casual employees employed on 1.8.2018 who do not participate in RPF (50)
Regular employees employed on 1.5.2018 [Total monthly emoluments > 25,000] (125)
Regular employees employed on 1.9.2018 [Employed for < 240 days in PY 2018-19 (100) (275)
Number of “Additional Employees” 75
Therefore, only 75 employees employed on 1.4.2018 qualify as additional employees, & the total emoluments
paid or payable to them during the PY 2018 - 19 is deemed to be the additional employee cost.

(II) If Mr. A is engaged in the business of manufacture of apparel


➢ If Mr. A is engaged in the business of manufacture of apparel, then, he would be entitled to deduction u/s
80JJAA in respect of employee cost of regular employees employed on 1.9.2018, since they have been
employed for more than 150 days in PY 2018-19.
➢ Additional employee cost = Rs. 2,16,00,000 + Rs. 24,000 × 7 × 100 = Rs. 3,84,00,000.
➢ Deduction u/s 80JJAA = 30% of Rs. 3,84,00,000 = Rs. 1,15,20,000.

Q18. Mr. Satya is a manufacturer of household goods in a factory located in Navi Mumbai & commenced his
business on 1st April 2018 & he employed 120 new work men during the previous year 2018-19 which included:
(a) 20 employee whose total emoluments paid @ Rs. 30,000 p.m. per employee;
(b) 40 worker employed on 1st April, 2018
(c) 35 worker employed on 1st May, 2018
(d) 25 worker employed on 5th October, 2018
Compute the Deduction u/s 80JJAA, if available to Mr. Satya for AY 2019-20, if wages are paid to each worker @
Rs. 3,000 per month. His profit from the manufacture of goods for AY 2019-20 is Rs. 9.50 Lacs. The Assessee is
liable to Audit his accounts. [NOV-2016]
Solution: Mr. Satya is eligible for deduction u/s 80JJAA since he is liable to Audit of his accounts in PY 2018-19.
Calculation of additional employee
Particulars No. of Employee
Total No. of workmen employed 120
Less: Employee Emoluments paid @ 30,000 p.m. (20)
Less: Employee Employed for less than 240 days (employed on 05.10.2018) (25)
Total No. of Additional Employee 75
➢ Additional employee cost = ( Rs. 3,000 x 12 months x 40) + ( Rs. 3,000 x 11 months x 35) = Rs. 25,95,000.
➢ Deduction u/s 80JJAA = 30% of Rs. 25,95,000 = Rs. 7,78,500.

Q19. Ms. X, employed in a private sector company, furnishes following information for PY 2018-19:
Income from salary (computed) 3,45,000
Bank interest on savings bank account 15,000
Tax on non-monetary perquisite paid by employer 20,000
Amount contributed by her during the year of given below:
Contribution to Recognized Provident Fund 60,000
Health Insurance Premium -on self (paid by crossed cheque) 7,000
Medical expenditure for dependent sister with disability 20,000
Compute the total income of Ms. X for the AY 2019-20. [MAY-2012]
Solution: Computation of Total Income of Ms. X for the AY 2019-20
Income under the head Salary 3,45,000
Income under the head Other Sources 15,000
(Bank Interest)
Gross Total Income 3,60,000

DT QUESTION BANK BY CA PRANAV CHANDAK Subscribe us on YouTube 153


Less: Deduction u/s 80C - Contribution to Recognized Provident Fund (60,000)
Less: Deduction u/s 80D - Health Insurance Premium (7,000)
Less: Deduction u/s 80DD - Medical expenditure for dependent sister with disability (75,000)
Less: Deduction u/s 80TTA (10,000 or 15,000 whichever is less) (10,000)
Total Income 2,08,000
Note: Tax on non-monetary perquisite paid by employer is exempt u/s 10(10CC).

Q20. Mr. Pramod, a Writer & a Professional, furnishes following particulars for the PY ended 31.03.2019-
Royalty on Books Rs. 1,26,000; Expenditure relating to Royalty Income Rs. 24,000
Income from Profession: Rs. 7,20,000 Deposited in Public Provident Fund: Rs. 1,40,000 (15.03.2019)
You are required to compute – (a) Taxable Income, & (b) Tax Payable for Assessment Year 2019-2020. [N01]
Solution: Computation of Total Income & Tax Liability
Particulars Rs. Rs.
1. Profits & Gains of Business or Profession 7,20,000
2. Income from Other Sources – Royalty on Books 1,26,000
Less: Expenditure incurred on realising Royalty Income (24,000) 1,02,000
Gross Total Income 8,22,000
Less: Deduction Under Chapter VI-A
U/s 80C Deposit in PPF (Maximum Rs. 1,50,000) 1,40,000
U/s 80QQB Royalty on Books – Least of Rs. 3,00,000 or the Royalty Income 1,02,000 (2,42,000)
Total Income (Round Off) 5,80,000
Tax on Total Income [ Rs. 12,500 + ( Rs. 5,80,000 – Rs. 5,00,000) x 20%] 28,500
Add: HEC at 4% 1,140
Total Tax Payable (Rounded Off) 29,640

DT QUESTION BANK BY CA PRANAV CHANDAK Subscribe us on YouTube 154


8. TAX DEDUCTED AT SOURCE
Q1. Punjab National Bank pays Rs. 1,00,000 p.m as Rent to the Central Government for a Building in which one of
its Branches is situated. Calculate the amount of tax to be deducted u/s 194I for the AY 2019-20.
Answer:
▪ Sec. 194I requires TDS on payment of Rent exceeding Rs. 1,80,000 p.a by all persons other than Individuals &
HUFs who are not subject to Tax Audit in the immediately preceding Financial Year.
▪ TDS u/s 194I is applicable in respect of Rental Payments made by a Bank.
▪ However as per Section 196, payments made to Government are exempted from TDS.
▪ Hence, Punjab National Bank is not required to deduct TDS.

Q2. Mr. X is employed in ABC Ltd. & salary is Rs. 50,000 p.m. & he has invested Rs. 50,000 in NSC. If employer has
deducted tax at source for the month of April & May & salary was increased to Rs. 70,000 p.m., calculate the tax
deductible at source u/s 192 for the PY 2018-19 ?
Solution: Tax has to be deducted at source at the time of payment of salary.
Computation of TDS for April & May 2018
Gross Salary (50,000 x 12) 6,00,000
Less: Standard Deduction u/s 16(ia) (40,000)
Income under the head Salary 5,60,000
Gross Total Income 5,60,000
Less: Deduction u/s 80C {NSC} (50,000)
Total Income 5,10,000
Tax on Rs. 5,10,000 at slab rate 14,500
Add: HEC @ 4% 580
Tax Liability 15,080
Monthly instalment for April & May = (15,080/12) 1,256.67
Computation of TDS for June 2018 – March 2019
Gross Salary (50,000 x 2) + (70,000 x 10) 8,00,000.00
Less: Standard Deduction u/s 16(ia) (40,000.00)
Income under the head Salary 7,60,000.00
Gross Total Income 7,60,000.00
Less: Deduction u/s 80C {NSC} (50,000.00)
Total Income 7,10,000.00
Tax at slab rate including HEC 56,680.00
TDS in April & May (1,256.67 x 2) (2,513.34)
Balance amount of tax 54,166.66
TDS in subsequent instalments (54,166.66/10) 5,416.67

Q3. Mr. X purchased 20 lottery tickets of Rs. 250 each. He has won the net amount of Rs. 2.80 Lacs. He has also
received interest on securities of Rs. 72,000 & cheque was collected by bank. The bank has deducted collection
charges @ 2%. He has income from subletting of house property of Rs. 9,000 p.m. He has received family pension
of Rs. 4,000 p.m. Compute his tax liability & tax payable/refund for AY 2019-20.
Solution:
(i) Income from lottery (2,80,000/70%) [after grossing up] 4,00,000
(ii) Interest (72,000/90%) [after grossing up] 80,000
Less: Bank charges deductible u/s 57 (2% of Rs. 72,000) (1,440) 78,560
(iii) Sub-letting of house property (9,000 x 12) 1,08,000
(iv) Family pension 48,000
Less: Exempt = Lower of Rs. 15,000 or 1/3rd of Family pension Received (15,000) 33,000
Income under the head Other Sources 6,19,560
Computation of Tax Liability
Tax on casual income Rs. 4,00,000 @ 30% u/s 115BB 1,20,000
Tax on Rs. 2,19,560 at slab rate Nil
Tax before HEC 1,20,000
Add: HEC @ 4% 4,800
Tax Liability 1,24,800
Less: TDS (1,20,000 + 8,000) (1,28,000)
Tax Refund 3,200

DT QUESTION BANK BY CA PRANAV CHANDAK Subscribe us on YouTube 155


Note:
▪ Interest received is an interest other than interest on deposit in saving bank A/c with bank, cooperative
society, post office & thus it is not eligible for deduction u/s 80TTA.
▪ Interest on securites is subject to TDS u/s 193A @ 10%.
▪ Winning from lottery & interest on securities are subject to TDS provisions. Thus the amount received should
be grossed up to find the taxable income.

Q4. When will tax not required to be deducted at source on interest payable to a resident on any bond or security
issued by a company though the aggregate interest exceeds Rs. 5,000 (Exemption limit u/s 193).
Answer.
▪ As per section 193 of the Act, no tax is required to be deducted at source on any interest payable to a resident
on any bond or security issued by a company, where the following conditions are satisfied -
(i) where such security is in dematerialised form & (ii) is listed on a recognised stock exchange in India.

Q5. Examine the implications of tax deduction at source u/s 194A in the following cases:
(a) On 1.10.2018, Mr. Mohit made a six-month fixed deposit of Rs. 12 lacs @ 8% p.a. with Theta Cooperative Bank.
The fixed deposit matures on 31.3.2019.
(b) Mr. Harish made fixed deposits carrying interest @10% p.a. with the following branches of Omega Bank, a
bank which has adopted CBS.
Branch Amount Date of deposit Date of Maturity
Adyar 60,000 1.6.2018 31.3.2019
Anna Nagar 80,000 1.7.2018 31.3.2019
Nungambakkam 75,000 1.8.2018 31.3.2019
(c) What if no CBS is adopted in case (b)?
(d) On 1.4.2018, Ms. Meena started a 1-year recurring deposit of Rs. 20,000 per month @ 10% p.a. with Gamma
Bank. The recurring deposit matures on 31.3.2019. Gamma bank pays interest of Rs. 13,000. [N-16/RTP]
(e) On 1.6.2018, Mr. Ganesh made three 9 months FDs of Rs. 1 lac each carrying interest @ 9% with A , B & C
Branches of XYZ Bank, a bank which has adopted CBS. The fixed deposits mature on 28.2.2019.
Solution:
▪ As per section 194A, every person making payment of interest other than interest on securities to any resident
shall deduct tax at source @ 10% provided the amount being paid or payable during a particular year to a
particular person is exceeding Rs. 5,000.
▪ But if payment is being made by bank or post office, tax shall be deducted only if interest being paid or payable
exceeds Rs. 10,000.
▪ Further TDS shall be only on time deposit including “recurring deposit”.
▪ Limit of Rs. 10,000 shall be per branch of the bank but if the bank has adopted core banking solution, limit shall be
per bank & not per branch.
(a) Interest for 6 months = Rs. 12 Lacs x 8 % x 6/12 = Rs. 48,000. Since interest > Rs. 10,000, Theta Co-operative
Bank has to deduct tax at source @ 10% u/s 194A on interest of Rs. 48,000. Thus TDS = Rs. 4,800.
(b) Since Omega Bank has adopted CBS, aggregate interest credited/paid by all branches has to be considered &
if the same exceeds Rs. 10,000, tax is deductible u/s 194A.
Branch Amount of deposit Rate of Interest Period in months Interest
Adyar 60,000 10% 10 5,000
Anna Nagar 80,000 10% 9 6,000
Nungambakkam 75,000 10% 8 5,000
Total 16,000
Omega Bank has to deduct tax at source @ 10% u/s 194A, since aggregate interest on fixed deposit with 3
branches of the bank is Rs. 16,000, which exceeds Rs. 10,000. Thus TDS = Rs. 1,600.
(c) If there is no core banking services in the bank; then the limit of Rs. 10,000 shall be per branch & amount has
not exceeded Rs. 10,000 per branch. Hence no tax is required to be deducted.
(d) Tax has to be deducted @ 10% u/s 194A by Gamma Bank on the interest of Rs. 13,000 on recurring deposit
on 31.3.2019 to Ms. Meena, since-
(i) recurring deposit has been included in the definition of “time deposit”; &
(ii) such interest exceeds the threshold limit of Rs. 10,000. Thus, TDS = Rs. 1,300.
(e) Total Interest on 3 FDs of 9 months = Rs. 20,250 [Rs. 1,00,000 × 9% × 9/12 × 3 FDs].
Since XYZ Bank has adopted CBS, the aggregate interest credited/paid by all branches has to be considered.
Since aggregate interest on FDs with all the branches of the bank is Rs. 20,250 which exceeds Rs. 10,000, XYZ
Bank has to deduct tax at source @ 10% u/s 194A.Thus, TDS = Rs. 2,025.

DT QUESTION BANK BY CA PRANAV CHANDAK Subscribe us on YouTube 156


Q6. An individual having gross receipts of Rs. 110 lacs during FY 2017-18 is not required to deduct tax at source
u/s 194C of the Income-tax Act, 1961, on payment made to contractors during FY 2018-19.
State with reasons, whether the following statements are true or false for AY 2019-20:
Answer: False.
An individual whose total sales/gross receipts/tumover from the business or profession exceeds the monetary
limits specified in section 44AB during the financial year immediately preceding the financial year in which such
sum is credited or paid to the account of the contractor shall be required to deduct tax at source.

Q7. ABC Ltd. makes the following payments to Mr. X, a contractor, for contract work during the PY 2018-19.
(i) Rs. 15,000 on 01.05.2018; (ii) Rs. 25,000 on 01.08.2018; (iii) Rs. 30,000 on 01.12.2018;
On 1.3.2019, a payment of Rs. 48,000 is due to Mr. X on account of a contract work.
Discuss whether ABC Ltd. is liable to deduct tax at source u/s 194C from payments made to Mr. X.
Solution:
▪ TDS u/s 194C should be made where the amount paid or credited to a Contractor / Sub-contractor exceeds
(a) Rs. 30,000 for a single payment / contract, or
(b) Rs. 1,00,000 in aggregate during the Previous Year.
▪ In this case, single contract payments made to Mr. X does not exceed Rs. 30,000.
▪ However, since aggregate amount paid to Mr. X during the PY 2018-19 exceeds Rs. 1,00,000 (on account of the
last payment of Rs. 48,000, due on 1.3.2019, the TDS provisions u/s 194C would get attracted.
▪ ABC ltd. would be liable to deduct tax @ 1% on entire amount of Rs. 1,18,000 from last payment of Rs. 48,000.
▪ Balance of Rs. 46,820 (i.e. Rs. 48,000 – Rs. 1,180) has to be paid to Mr. X. Thus, TDS = Rs. 1,180.

Q8. ABC Ltd. has given orders to Mr. X to stitch uniform for their employees. Mr. X purchased material from the
market & has stitched uniform for ABC Ltd & has charged Rs. 7,00,000. Calculate the amount of Tax to be deducted
at source. (b) What if Mr. X has used material purchased from ABC Ltd. & charged Rs. 1,10,000 as labour charge.
Answer:
▪ Section 194C deals with the provisions of TDS in case of payment to contractors.
▪ It deals with payment of any sum for carrying out any work in pursuance of contract for Manufacturing or
supplying a product according as per the requirement or Specification of customer using materials purchased
from the customer.
▪ Thus in this case, since material has been purchased from market, section 194C is not applicable. Thus no tax is
required to be deducted if material is supplied by ABC Ltd.
▪ (b) Mr. X has charged Rs. 1,10,000 as labour charges. Thus, TDS shall be @ 1 % i.e. Rs. 1100.

Q9. State the concessions granted to transport operators onwards in the context of cash payments u/s 40A(3) &
deduction of tax at source u/s 194-C.
Answer:
▪ Section 40A(3) provides for disallowance of expenditure incurred in respect of which payment or aggregate of
payments made otherwise than by A/c payee cheque or draft to a person in a day exceeding Rs. 10,000.
▪ However, in case of payment made to transport operators for plying, hiring or leasing goods carriages, the
disallowance will be attracted only if the payments made to a person in a day exceeds Rs. 35,000.
▪ U/s 194C, tax had to be deducted in respect of payments made to contractors @ 1% in case the payee is
Individuals or HUF or @ 2 % in case pof other payee.
▪ However, no tax deduction is required to be made from any sum credited or paid or likely to be credited or paid
during PY to a contractor in the business of plying, hiring or leasing goods carriages in following cases:
(1) He owns ≤ 10 Goods carriages at any time during PY.
(2) He is engaged in the business of plying, hiring or leasing goods carriages;
(3) He has furnished a declaration to this effect along with his PAN.

Q10. A Foreign Enterprise enters into a contract for Fabrication & supply of components for Machinery with X &
Co. (a Firm in India). X & Co sub-contracts the work to Y (an Individual & pays him Rs. 20 Lacs during PY 2018-19.
X & Co. receives payment of Rs. 50 lacs from a foreign enterprise. Discuss TDS implications. [Mod. N-98 (Final)]
Answer:
▪ U/s 194C, Payments to Contractors (including Sub-Contractors) for contracts shall be subject to TDS @ 1% if
the Payee is a Resident Individual/HUF & 2% in case of other Resident Payees.
▪ Foreign Enterprise is liable to deduct TDS on its payment to Main Contractor (X & Co.) @ 2% payee being a firm.
▪ Payment made by foreign enterprise to X & Co. shall be subject to TDS at 2% [i.e. Rs. 50 lacs × 2 % = 1 lac].
▪ Amount payable by foreign enterprise to X & Co. = Rs. 50 lacs – Rs. 1 lac = Rs. 49 lacs.
▪ Payment made by X & Co. to Mr. Y shall be subject to TDS at 1% [i.e. Rs. 20 lacs × 1% = 20,000].
▪ Since the payment is made to an Individual, Rs. 20 Lacs shall be subject to TDS at 1% i.e. Rs. 20,000.
▪ X & Co should deduct Rs. 20,000 from the amount payable to Mr. Y & pay the balance of Rs. 19,80,000.

DT QUESTION BANK BY CA PRANAV CHANDAK Subscribe us on YouTube 157


Q11. By virtue of an agreement with a Nationalised Bank, a Catering Organisation (a Limited Company) receives
a sum of Rs. 50,000 per month towards supply of food, water, snacks, etc. during office hours to the employees of
the Bank. Discuss the liability for tax deduction at source for the AY 2019-2020. [May 03(F)]
Solution:
▪ U/s 194C, “Contract” includes Catering Services.
▪ Thus, Bank should deduct tax at source at 2% (Payee being a Domestic Company) from Rs. 50,000 paid monthly.
▪ Therefore, Rs. 1,000 should be deducted from the monthly payments as TDS.

Q12. Examine the applicability of the provisions for tax deduction at source u/s 194DA in the following cases -
(a) Mr. X, a resident, is due to receive Rs. 4.50 Lacs on 31.3.2019, towards maturity proceeds of LIC policy taken
on 1.4.2016, for which the sum assured is Rs. 4 Lacs & the annual premium is Rs. 1,25,000.
(b) Mr. Y, a resident, is due to receive Rs. 2.75 Lacs on 31.3.2019 on LIC policy taken on 31.3.2012, for which the
sum assured is Rs. 2.50 Lacs & the annual premium is Rs. 35,000.
(c) Mr. Z, a resident, is due to receive Rs. 95,000 on 1.10.2018 towards maturity proceeds of LIC policy taken on
1.10.2013 for which the sum assured is Rs. 90,000 & the annual premium was Rs. 15,000.
Solution:
(a) Since Annual premium > 10% of Sum Assured in respect of a policy taken after 31.3.2012, maturity proceeds
of Rs. 4.50 lacs are not exempt u/s 10(10D) in the hands of Mr. X. Therefore, tax is required to be deducted@ 1%
u/s 194DA on the maturity proceeds of Rs. 4.50 Lacs payable to Mr. X. TDS = Rs. 4500.
(b) Since Annual premium < 20% of Sum Assured in respect of a policy taken before 1.4.2012, Sum of Rs. 2.75 lacs
due to Mr. Y would be exempt u/s 10(10D) in his hands. Hence, NO TDS u/s 194DA to Mr. Y.
(c) Even though the Annual premium > 10% of Sum Assured in respect of a policy taken after 31.3.2012, &
consequently, the maturity proceeds of Rs. 95,000 would not be exempt u/s 10(10D) in the hands of Mr. Z, TDS
provisions u/s 194DA are not attracted since the maturity proceeds are less than 1 lac.

Q13. Shakira, a NR, received Rs. 40 lacs for her stage shows in India from Optimistic Ltd., an event management
company in India on 26.12.2018. State whether any tax has to be deducted at source.
Solution:
▪ Payments made to NR entertainer shall be subject to tax deduction at source @ 20% u/s 194E + HEC @ 4%.
▪ TDS u/s 194E = Rs. 40 lacs × 20.8% = Rs. 8,32,000

Q14. Rent paid for hire of machinery by B Ltd. to Mr. A of Rs. 2,10,000. Calculate the amount of tax to be deducted
at source by B Ltd. (b) What if Mr. A did not furnish his PAN to B Ltd. [Nov – 11]
Solution:
▪ Since the rent paid for hire of machinery by B. Ltd. to Mr. A exceeds Rs. 1,80,000, Sec 194-I gets attracted.
▪ The rate applicable for deduction of tax at source u/s 194-I on rent paid for hire of P&M is 2% assuming that
Mr. A had furnished his PAN to B Ltd. Thus, TDS = 2,10,000 x 2% = Rs. 4200.
(b) If Mr. A does not furnish his PAN to B Ltd., TDS @ 20% on Rs. 2,10,000 as per sec 206AA. TDS = Rs. 42,000.

Q15. Mrs. X, a landlord, derived income from rent from letting a house property to M/s ABC Ltd. of Rs. 1 lac p.m.
She also charged GST @ 15% on lease rent. Calculate TDS to be made by M/s ABC Ltd. on payment made to Mrs. X.
Answer:
▪ As per Circular No. 4/2008 dated 28th April, 2008 issued by the CBDT, GST paid by the tenant does not partake
the nature of income of the landlord. The landlord only acts as a collecting agency for collection of GST.
▪ Thus, tax deducted at source u/s 194-I shall be made on the amount of rent paid or payable excluding GST.
▪ Thus, tax has to be deducted u/s 194-I @ 10% only on rental income i.e Rs. 12 lacs. TDS = 10,000 every month.
▪ Tax deducted will be deposited within one week from the last day of the month in which deduction is made but
if the tax has been deducted in the month of March, tax should be deposited on or before 30th April.
▪ Form No. 16-A has to be issued by M/s. ABC Ltd. to Mrs. X within 15 days from last date of submitting quarterly
return u/s 200.
▪ Assessee should file quarterly statement upto 31st of the month succeeding the relevant quarter but for the last
quarter ending March, return can be submitted upto 31st May.

Q16. XYZ Ltd. raised an invoice of Rs. 3,00,000/- to ABC Limited for renting of commercial building. The above
figure includes 10,000/- of parking charges. The bill is raised on 30th June, 2018 & ABC Limited made the payment
on the same date. Compute the Amount of TDS required to be deducted by ABC Limited.
Solution:
Rent 3,00,000
Less: TDS (3,00,000 x 10%) (30,000)
Amount payable 2,70,000

DT QUESTION BANK BY CA PRANAV CHANDAK Subscribe us on YouTube 158


*Q17. A Ltd. has taken a building on lease. It has sub-leased the building & furniture & fixtures to B Ltd. from
1.4.2018 but separate agreement has been made for both the sub-leases. A Ltd. receives the following amounts on
31.3.2019 as consideration for the sub-lease from B Ltd. during the PY 2018-19:
Rent for the period 1.4.2018 to 31.3.2019 of building 120000
Furniture hire charges for the period 1.4.2018 to 31.3.2019 12000
Non-refundable deposit received during the year 50000
What is the liability of A Ltd. for deduction for tax at source u/s 194-I for PY 2018-19?
Answer:
▪ Section 194-I provides that where any person, is responsible for paying to any person, any income by way of the
rent, amounting in aggregate to more than Rs. 180000 in a financial year, he shall deduct income-tax thereon.
▪ Limit of Rs. 1,80,000 for TDS u/s 194-I applies to the aggregate rent of all the assets i.e. whether such asset is
building or machinery or plant or equipment, etc.
▪ If there are separate agreements, one for sub-lease of building & other for hiring of machinery, rent & hire
charges under two agreements have to be aggregated for the purpose of application of the limit of Rs. 1,80,000.
▪ Rate of TDS u/s 194-I in case of rent from land & building & Furniture & fixtures is 10%.
▪ It is to be noted that Non-refundable deposit received during the year shall also be subject to TDS.
▪ Tax should be deducted at the time of actual payment of rent or at the time of its credit whichever is earlier.
▪ TDS Liability of B Ltd. will be: Total payment = (1,20,000 + 12,000 + 50,000) = 1,82,000. TDS @ 10% = 18,200.

*Q18. Mr. X sold his House Property in Bangalore as well as his Rural Agricultural Land for 60 Lacs & 15 Lacs,
respectively to Mr. Y on 1.8.2018. He has purchased the House Property & land in 2016 for 40 Lacs & 10 Lacs
respectively. SDV on date of transfer is 85 Lacs & 20 Lacs respectively. Determine the tax implications in the hands
of Mr. X & Mr. Y & TDS implications in the hands of Mr. Y assuming that both Mr. X & Mr. Y are resident Indians.
Solution: Tax Implications in the hands of Mr. X
1. As per Sec. 50C, SDV of House Property (85 Lacs) is deemed as Full Value of Consideration arising on transfer of
property. Also the Property is held for less than 36 months & is hence an STCA.
2. Short Term Capital Gain on Sale of Bangalore Property in AY 2019-20 =
Full Value of Consideration [SDV as per Section 50C] Rs. 85 lacs
Less: Cost of Acquisition (Rs. 40 Lacs )
Short term capital gains Rs. 45 lacs
3. Since Rural Agricultural Land is not a Capital Asset, no capital gains arises in such case. Thus it is not taxable.

Tax Implications in the hands of Mr. Y


1. If Immovable Property is received for inadequate consideration, difference b/w SDV & Actual consideration
would be taxable/s 56(2)(x), if such difference exceeds higher of (i) Rs. 50,000 or (ii) 5% of sale consideration.
2. Therefore, in this case Rs. 25 Lacs (85 Lacs – 60 Lacs) would be taxable in the hands if Mr. Y u/s 56(2)(x).
3. However, since Agricultural land is not a Capital Asset, Sec. 56(2)(x) is not attracted for receipt of Agricultural
Land for inadequate consideration since definition of property u/s 56(2)(vii) includes only Capital Assets.

TDS Implications in the hands of Mr. Y


1. Since sale consideration of Bangalore House Property > Rs. 50 Lacs, TDS provisions u/s 194 – IA gets attracted.
2. Tax to be deducted u/s 194-IA = 1% of Rs. 60 Lacs = Rs. 60,000.
3. TDS u/s 194-IA are not attracted in respect of transfer of Rural Agricultural Land.
Note: Person deducting TDS u/s 194- IA shall not be required to obtain Tax deduction A/c number as per sec 203A.

Q19. Mr. X, a salaried individual, pays rent of Rs. 55,000 per month to Mr. Y from June, 2018. [ICAI -SM]
(a) Compute tax to be deducted at source; Time of deduction of Tax at source;
(b) Would your answer change if Mr. X vacated the premises on 31 st December 2018?
(c) What would be your answer if Mr. Y does not provide his PAN to Mr. X?
Solution:
(a) Since Mr. X pays rent exceeding Rs. 50,000 p.m in PY 2018-19, he is liable to deduct tax at source @ 5% on
such rent for PY 2018-19 u/s 194-IB.
Time of TDS shall be earlier of (a) At the time of credit of the Rent for Last month of PY or Last month of
tenancy (if property is vacated during the year)] to the account of the payee OR (b) At the time of Payment.
Thus Rs. 27,500 [Rs. 55,000 x 5% x 10 months] has to be deducted from rent payable for March 2019.
(b) If Mr. X vacated the premises in December 2018, then tax of Rs. 19,250 [Rs. 55,000 x 5% x 7] has to be deducted
from rent payable for December 2018.
(c) If Mr. Y does not provide his PAN to Mr. X, TDS @ 20%, instead of 5%.

DT QUESTION BANK BY CA PRANAV CHANDAK Subscribe us on YouTube 159


(i) TDS = Rs. 1,10,000 [Rs. 55,000 x 20% x 10] but restricted to Rs. 55,000 only being rent for March 2019.
(ii) TDS = 77,000 [Rs. 55,000 x 20% x 7] but restricted to Rs. 55,000 only, being rent for December 2018.

Q20. State, in brief, the Applicability of Rate & Amount of TDS in the following cases: [RTP]
(a) Payment of Royalty of Rs. 25,000 & fee for Professional Services of Rs. 28,000 to Mr. Varun.
(b) Payment of Rs. 1,98,000 to Mr. Karan for Compulsory Acquisition of his Urban Land by the State Government.
(c) A company pays to a doctor a monthly retainership of 1500 for attending outpatient clinic at its factory.
(d) Fee paid to Dr. Srivatsan by Sundar (HUF) Rs. 35000 for surgery performed to a member of the family. [N-11]
(e) A Ltd paid Rs. 19,000 to its Director as Sitting Fees on 1.1.2019. State the provisions relating to TDS. [M 14]
Answer:
(a) Sec. 194J, will be attracted only in case the payment made as fees for Professional Services & Royalty,
individually, exceeds Rs. 30,000 during the Financial year.
In the given case, since, the individual payment for fee of Rs. 28,000 for professional services & royalty of Rs.
25,000 is less than Rs. 30,000 each, there is no TDS liability. It is assumed that no other payment towards fees for
Professional Services & Royalty were made during the year to Mr. Varun.
(b) U/s 194LA TDS has to be deducted on compensation on compulsory acquisition, if the aggregate payment
exceeds Rs. 2,50,000. In the given case, there is no TDS liability as the payment made does not exceed Rs. 2,00,000.
(c) Sec 194J provides that any person, other than Individual/HUF, who is responsible for paying to a resident any
sum by way of fees for professional or technical services, shall deduct tax @ 10% on income comprised therein.
However, no tax is to be deducted if the aggregate amounts of such fees does not exceed Rs. 30,000 in FY.
In present case, total payment is Rs. 18000 (Rs. 1,500 x 12) & thus NO TDS.
(d) As per the provisions of section 194J, Individual/HUF is required to deduct tax at source on fees paid for
professional services only if it is subject to tax audit u/s 44AB(a)/(b) in last PY.
However, if such payment is made for professional services exclusively for personal purpose of any member of
HUF, then liability to deduct tax is not attracted.
Therefore, in the given case, even if Sundar (HUF) is liable to tax audit in last PY, liability to deduct tax at source is
not attracted in this case since, the fees for professional service to Dr. Srivatsan is paid for a personal purpose i.e. the
surgery of a member of the family.
(e) U/s 194J, Tax is deductible for any payment made which is in the nature of any Remuneration or Fees or
Commission other than those on which Tax is deductible u/s 192, to a Director of a Company.
▪ TDS @ 10% should be deducted.
▪ There is no Exemption Limit for Deduction of Tax on payments to a Director.
▪ Hence, ABC & Co Ltd should deduct Tax of Rs. 1,900 on Sitting Fee paid, being 10% of Rs. 19,000.
Note: In case Sitting Fee is paid to a Whole Time Director in Employment with the Company, the same may be
considered as taxable u/s 192, in which case provisions of Sec.194-J may not be applicable.

Q21. Ashwin a resident Individual carrying on business, furnishes you the following information:
Total turnover of PY 2017-18: Rs. 120 lacs Total turnover of PY 2018-19: Rs. 98 lacs
State whether provisions of TDS are attracted for the under - mentioned expenses during the PY 2018-19:
Commission paid to Babloo 18,500
Payment to Vijay for repair of office building 23,000
Payment of fees for Technical Services, to Vivek 35,000
Interest paid to Indian Bank on Term Loan 92,800
Advertisement expenses to Mr. X (two individual expense of Rs. 24,000 & Rs. 34,000) 58,000
Factory rent paid to C 1,85,000
Brokerage paid to B, a sub-broker 16,000
Rent paid by a partnership firm for use of plant & machinery 1,70,000
All payments are made to residents. State the amount of tax to be deducted at source. [M-16 + N-12]
Answer:
As the turnover of Mr. Ashwin for PY 2017-18 (Rs. 120 lacs) has exceeded the limit prescribed u/s 44AB, he has to
comply with the tax deduction provisions during the financial year 2018-19.
(a) Commission paid to Babloo:
Tax has to be deducted u/s 194-H as the commission exceeds Rs. 15,000.
Tax shall be deducted at source u/s 194H & shall be = 18,500 x 5% = Rs. 925
(b) Contract payment of Rs. 23,000 to Mr. Vijav
TDS provisions u/s 194C would not be attracted if the amount paid to a contractor does not exceed Rs. 30,000
in a single payment or Rs. 1,00,000 in the aggregate during the financial year. Therefore, TDS provisions u/s
194C are not attracted in this case.
(c) Payment of fees for Technical Services, to Vivek

DT QUESTION BANK BY CA PRANAV CHANDAK Subscribe us on YouTube 160


Tax shall be deducted at source u/s 194J @ 10% because the total amount payable is exceeding Rs. 30,000 &
amount of TDS shall be = 35,000 x 10% = Rs. 3,500.
(d) Interest paid to Indian Bank on Term Loan
TDS u/s 194A is not attracted in respect of interest paid to a banking company.
(e) Advertisement expenses to Mr. X
Advertisement = TDS on Rs. 34,000 @ 1% = Rs. 340 (Sec 194C is applicable if single payment > Rs. 30,000 or
aggregate payment > Rs. 1,00,000).
(f) Factory rent paid to C
Tax has to be deducted u/s 194-I @ 10% as rent amount exceeds Rs. 1,80,000. TDS = Rs. 18,500.
(g) Brokerage paid to B, a sub-broker
Tax has to be deducted u/s 194-H as the commission exceeds Rs. 15,000. TDS @ 5% on Rs. 16,000 = Rs. 800.
(h) Rent of Rs. 1,70,000 paid by a partnership firm for use of plant & machinery
As per Section 194-1 TDS is not required to be deducted as the Rent amount does not exceeds Rs. 1,80,000.
Thus NO TDS.

Q22. State the applicability of tax deduction at source provisions, the rate & amount of tax deduction in the
following cases for the financial year 2018-19: [NOV – 2014]
(1) Payment of Rs. 27,000 made to a South African cricketer, by Indian newspaper agency on 2.7.2018 for
contribution of articles in relation to the sport of cricket.
(2) Winning from horse race Rs. 1,50,000.
*(3) Rs. 2,00,000 paid to Mr. X, a resident individual on 22.02.2019 by the State of UP on compulsory acquisition
of his urban land.
(4) Mr. PC, an employee of CG receives arrears of salary for 3 earlier years. He is liable for deduction of tax on the
entire amount during CY.
(5) A TV channel pays Rs. 11,00,000 on 1.7.2018 as prize money to the winner of quiz, programme, "A Millionaire".
(6) SBI pays Rs. 50,000 pm as rent to the CG for a building in which one of its branches is situated.
(7) Television company pays Rs.50000 to cameraman for shooting of film.
(8) A turf club awards a jackpot of Rs. 5 lacs to the winner of one of its races.
Answer:
(1) TDS shall be deducted u/s 194E @ 20%+HEC. TDS shall be 20.8% of Rs. 27,000 = Rs. 5,616 .
(2) TDS shall be deducted u/s 194BB @ 30% as the amount exceeds Rs. 10,000. TDS = Rs. 45,000.
(3) As per Section 194LA, No TDS is deductible by State of UP as the amount paid does not exceed Rs. 2,50,000.
(4) Arrears of salary are taxable in PY in which there are paid & thus shall be liable for deduction of tax at source.
However, if an employee receives any salary in arrears, he can claim relief as per sec 89 r/w rule 21A provided the
employee furnishes the details of such arrears in Form No. 10E to the employer. Further, relief u/s 89 shall be
given to concerned employee while deducting tax at source u/s 192.
(5) As per sec 194B, TV channel is required to deduct tax @ 30% at the time of payment of Rs. 11,00,000.
(6) As per sec 196 where payee is Government, there is no requirement to deduct tax at source on income by way
of 'rent' & therefore SBI is not liable to deduct tax while paying rent to the CG.
(7) As per sec 194J, television company shall deduct tax at the time of making payment to the cameraman @ 10%.
(8) As per Sec 194BB, the payer shall deduct tax at source @ 30% at time of making payment. TDS = Rs. 1,50,000.

Q23. Compute amount of tax to be deducted at source on the following payments made by M/s ABC Ltd. during
the financial year 2018-19 as per the provisions of the Income-Tax Act, 1961. [NOV-2011]
(a) Payment of Rs. 2,00,000 to Mr. “X” a transporter who is having PAN & who do not have more than 10 goods
carriages on 1.10.2018.
(b) Payment of fee for technical services of Rs. 45,000 to Mr. X who is having PAN on 1.11.2018
(c) Payment of Rs. 25,000 to M/s X Ltd. for repair of building on 30.12.2018
(d) Payment of Rs. 2,00,000 made to Mr. Y for purchase of diaries made according to specifications of M/s ABC
Ltd. However, no material was supplied for such diaries to Mr. Y by M/s ABC Ltd on 1.1.2019.
(e) Payment of commission of 25,000 to Mr. A on 1.02.2019
Answer:
(a) No tax shall be deducted at source in case of payment to a transporter who has submitted his PAN.
(b) Tax shall be deducted at source u/s 194J @ 10% because the total amount payable is exceeding Rs. 30,000 &
amount of TDS shall be = 45,000 x 10% = Rs. 4,500
(c) It is covered u/s 194C but payment is not exceeding Rs. 30,000 hence no tax shall be deducted at source.
(d) Tax shall not be deducted at source in case of purchase of goods.
(e) Tax shall be deducted at source u/s 194H & shall be = 25,000 x 5% = Rs. 1,250.

DT QUESTION BANK BY CA PRANAV CHANDAK Subscribe us on YouTube 161


Q24. State the applicability of TDS provisions, rate & amount of TDS in following cases for FY 2018-19: [NOV 12]
1. Payment made by a Firm to Sub-Contractor Rs. 3,00,000 with outstanding balance of Rs. 1,20,000 shown in the
books as on 31.03.2019.
2. Rent paid for Plant & Machinery – Rs. 1,50,000 by a Partnership Firm having Sales Turnover of Rs. 20 Lacs & Net
Loss of Rs. 15,000.
Answer:
1. Payment by Firm to Sub-Contractor Rs. 3,00,000 with o/s Balance on 31.03.2019 Rs. 1,20,000 = U/s 194C, TDS
shall be deducted at the time of payment or credit, whichever falls earlier.
For the payment & also for the credit, TDS will have to be deducted, hence the TDS should be deducted for Rs.
4,20,000. TDS = 4,20,000 x 1% = 4,200.
2. As per Section 194-I, All Assessees except Individual & HUF, who are not subject to Tax Audit u/s 44AB during
the preceding FY are liable to deduct tax.
Thus, section 194I is always applicable to the Firm whether or not it is subject to Tax Audit. However, since the
payment is less than Rs. 1,80,000, no TDS needs to be deducted.

*Q25. State whether tax deduction at source provisions are applicable to the following transactions: [M– 10]
(i) X & Co. (Firm) engaged in wholesale business assigned a contract for construction of its godown building to Mr.
X, a contractor. It paid Rs. 25,00,000 to Mr. X as contract payment.
(ii)Y & Co. engaged in real estate business conducted a lucky dip & gave Maruti car to a prize winner.
(iii) An Insurance Company paid Rs. 45,000 as Insurance Commission to its agent Mr. Y.
(iv) AB Ltd. allowed a discount of 50,000 to X & Co. (firm) on prompt payment towards supply of automobile parts.
Answer:
(i) Section 194C provides for deduction of tax at source from the payment made to resident contractors & sub-
contractors. Therefore, tax is deductible at source u/s 194C for the contract payments made for the construction
of godown building. The rate of TDS u/s 194C on payments made to contractors who are individuals or HUF shall
be @ 1%. Flence, X & Co. (firm) must deduct tax at source on the contract payments made to Mr. X.
(ii) In respect of lucky dip conducted by Y & Co., the provisions of Section 194B would apply. As per Section 194B,
winning from lottery or crossword puzzle or card game or other game of any sort exceeding Rs. 10,000 payable
by any person to any other person, subject to tax deduction at the rate of 30%. Since the value of prize i.e. Maruti
car would exceed Rs. 10,000 tax is deductible at source @ 30%. As the winning is in kind, the winner must deposit
30% of the prize value to Y & Co. for remitting the same as tax. Only after such deduction / recovery, the Maruti
car is to be delivered to the prize winner.
(iii) As per section 194D, any person paying insurance commission in excess of Rs. 15,000 to any resident person
is liable to deduct tax at the rate of 5% in case of all assesses. Therefore, the insurance company must deduct tax
at source @ 5% in respect of the insurance commission paid to Mr. Y.
(iv) Discount allowed to a customer for prompt payment is not covered by any of the TDS provisions. Therefore, AB
Ltd. need not deduct any tax at source since no payment was involved in allowing discount to its customer.

Q26. Mrs. X has received following incomes. Compute her tax liability for AY 2019-20:
1. Interest on savings bank account with State Bank - Rs. 50,000 (gross).
2. Interest from Government securities - Rs. 1,00,000 on 1.1.2019 (collection charge paid to the bank @ 1.5%).
3. Interest from ABC Ltd on non-listed debentures - Rs. 3,60,000 on 1.3.2019.
4. Interest credited to post office savings bank account during the year - Rs. 10,000.
5. Interest credited to public provident fund during the year - Rs. 15,000.
6. Interest received from XYZ Ltd on listed debentures - Rs. 1,35,000 (Net). Amount was invested by taking a loan
of Rs. 15,00,000 @ 12% p.a.
Solution: Computation of Income u/h “other sources”
Interest on savings bank account from State Bank of India 50,000
Interest from Government securities [Rs. 1,00,000 - Rs. 1,500] 98,500
Interest from ABC Ltd [(Rs. 3,60,000/90%)] 4,00,000
Interest on post office savings bank [10,000 - 3,500] – Rs. 3500 is exempt 6,500
Interest on PPF (Exempt u/s 10(15)) Nil
Interest from XYZ Ltd. [Gross: 1,35,000/90%] - Interest paid on loan = (1,80,000) (30,000)
Income under the head Other Sources/ Gross Total Income 5,25,000
Less: Deduction u/s 80TTA (10,000)
Total Income 5,15,000
Computation of tax liability
Tax on Rs. 5,25,000 at slab rate + 4 % HEC 18,200
Less: TDS u/s 193 on Non-Listed Debentures (40,000)
Less: TDS u/s 193 on Listed Debentures (15,000)
Tax Refund 36,800

DT QUESTION BANK BY CA PRANAV CHANDAK Subscribe us on YouTube 162


*Q27. R has deposited in fixed deposit with the company Rs.200000 @ 8% p.a. for 3 years. He submits declaration
in Form 15G & claims interest payment without tax deduction. The accountant feels that Form 15G submitted is
incorrect & wants to ignore the same & deduct tax @ 10%. Is he justified?
Answer: Sec 194A provides for TDS from interest. Where depositor furnish declaration in Form 15G, tax is not
deductible u/s 194A. Deductor is not authorized to scrutinize the same. Thus accountant is not justified.

Q28. What are the consequences of failure to deduct or pay the tax u/s 201 of the Income-tax Act, 1961?
Answer:
▪ Any person, including principal officer of a company, responsible for deducting tax at source shall be deemed to
be assessee in default in respect of such tax, if he does not deduct or after deducting fails to pay, the whole or
any part of the tax as required by or under the provisions of the Income-tax Act, 1961.
▪ However, no penalty shall be charged u/s 221 from such person, unless the Assessing Officer is satisfied that
such person, without good & sufficient reasons, has failed to deduct & pay such tax.
▪ As per sec 201(1A), a person who fails to deduct tax or after deduction, fails to pay tax, is liable to pay SI @
1% for every month or part on amount of such tax from the date on which such tax was deductible to the date
on which such tax is actually deducted & SI @ 1.5% for every month or part from the date on which tax was
deducted upto date of actual payment. Such interest should be paid before furnishing TDS statement u/s 200(3).
▪ Where such tax has not been paid after it is deducted, the amount of tax together with the amount of simple
interest thereon shall be a charge upon all the assets of the person, or the company, as the case may be.

Q29. Rs. 40,000 was paid to Mr. X on 1.7.2018 towards professional fees w/o TDS. Subsequently, another payment
of Rs. 50,000 was due to Mr. X on 28.2.2019, from which tax @ 10 % (Rs. 9,000) on entire amount of Rs. 90,000
was deducted. However, this tax of Rs. 9,000 was deposited only on 22.6.2019. Compute Interest u/s 201(1A).
Solution: Computation of Interest u/s 201(1A)
Particulars Rs
SI of 1% on tax deductible but not deducted [1 % on Rs. 4,000 for 8 months (1.7.2018 to 28.2.2019) 320
SI of 1.5% on tax deducted but not deposited [1.5% on Rs. 9,000 for 4 months (28.2.2019 to 22.6.2019) 540
Total Interest u/s 201(1A) 860

Q30. M/s Soft Drinks Limited entered into an agreement for Warehousing of its products with XYZ Warehousing,
& deducted Tax at Source as per provisions of Sec. 194C out of Warehousing Charges paid during PY 2018-19. AO
while completing the Assessment for 2019-2020 of Soft Drinks Limited, asked the Company by treating the
Warehousing Charges as Rent as defined in Sec. 194-1 to make payment of difference amount of TDS with interest.
It was submitted by the Company that the Recipient had already paid tax on the entire amount of Warehousing
Charges & therefore, now the difference amount of TDS be not recovered. However, it will make the payment of
due interest on the difference amount of TDS. Decide whether the Company's contention is correct. [M 09]
Answer:
▪ TDS has to be deducted u/s 194I from payment made towards Warehousing Charges [C.No. 718/22.8.1995].
▪ No Demand u/s 201(1) should be enforced after the tax deductor has satisfied the Office-in-charge of TDS that
the taxes due have been paid by the Deductee-Assessee. However, this will not alter the liability to charge
interest u/s 201(1A) till the date of payment of taxes by the Deductee or liability for Penalty u/s 271C.
[Hindustan Coco Cola Beverage (P) Ltd vs CIT 293 ITR 226 (SC)].
▪ Person shall not be Assessee in Default, if Deductee has furnished his ROI u/s 139 taking into account such sum
for computing income & has paid tax on that income. Person shall furnish a Certificate from a CA in Form 26A.
▪ For a person not regarded as Assessee in Default – Interest should be paid at 1% p.m. or part thereof from the
date on which tax was deductible to the date of furnishing Return of Income by deductee.
In the instant case, M/s Soft Drinks Limited has deducted tax u/s 194C at 1% instead of 10% u/s 194-1. However, it is
given that XYZ Warehousing has already paid the Tax on its Income from M/s Soft Drinks Limited.
If M/s Soft Drinks Ltd proves to the satisfaction of AO that taxes have paid on relevant income by the Deductee, then
the AO cannot recover the TDS amount from it. But M/s Soft Drinks Ltd is liable for interest on the differential amount.

Q31. Write a note on Certificate for Tax Deducted.


Answer: Certificate for Tax Deducted Section 203/Rule 31 TDS Certificate
▪ Every person deducting tax at source shall issue a certificate to the person with regard to whom tax has been
deducted at source. In case of payment of salary, certificate shall be issued in Form No. 16 & in other cases it will
be in Form No. 16A.
▪ The certificate in Form No. 16 should be given upto 31st May of the succeeding year in case of an employee & it
will be an annual certificate.
▪ In other cases certificate in Form No. 16A should be issued on quarterly basis & it should be issued within 15
days from the last date of submitting the quarterly statement u/s 200.

DT QUESTION BANK BY CA PRANAV CHANDAK Subscribe us on YouTube 163


Q32. Differentiate between TDS & TCS? [ICAI Exam]
Answer:
TDS (Tax Deduction at Source) TCS (Tax Collection at Source)
This represents tax to be deducted by the Payer, from This represents tax to be collected by Seller who sells
payments made by him to specified persons. specified goods subject to certain conditions.
This represents Expense of the Payer. TCS is on sale of specified goods (Ex:Alcoholic Liquor
TDS is not on sale of goods. for human consumption,Scrap, Bullion/Jewellery, etc.)
TDS is deductible at the time of – (a) payment in Time of collection of TCS is at the time of debiting the
certain cases, or (b) payment or credit, whichever is amount to the account of the Buyer or at the time of
earlier, in certain other cases. receipt of amount, whichever is earlier.

Q33. Write a note on Tax Deduction Account Number (TAN).


Answer:
▪ Tax Deduction Account Number (TAN) (Sec 203A)
▪ Every person, deducting tax at source shall apply for allotment of tax deduction account number & application
has to be given in Form No.49B within one month from the end of the month in which tax was deducted for the
first time.

Q34. Write a note on Furnishing of Statement of Tax Deducted.


Answer: Furnishing of Statement of Tax Deducted Section 203AA / Rule 31AB
▪ Income Tax Authorities shall prepare annual statement for every person with regard to whom tax has been
deducted at source & such statement shall be prepared in Form No. 26AS on the basis of quarterly return filed
by the person deducting tax at source.
▪ Such statement shall be sent to every person upto 31st July of AY. Tax credit can be claimed on the basis of such
statement.
▪ In case of any discrepancy, such person shall report it to the Income Tax Department.

Q35. Discuss the following issues with specific reference to clarification given by Central Board of Direct Taxes:
(a) Moon TV, a television channel, made payment of Rs. 50 Lacs to a production house for production of
programme for telecasting as per the specifications given by the channel. The copyright of the programme is
also transferred to Moon TV. Would such payment be liable for tax deduction at source u/s 194C? Discuss. Also,
examine whether the provisions of tax deduction at source u/s 194C would be Attracted if the payment was
made by Moon TV for acquisition of telecasting rights of the content already produced by the production house.
(b) Mudra Adco Ltd., an advertising company, has retained 15 Lacs, towards charges for procuring & canvassing
advertisements, from payment of 1 crore due to Cloud TV, a television channel, & remitted 85 Lacs to television
channel. State whether TDS u/s 194H be attracted on 15 Lacs retained by the advertising company?
Answer:
(a) Since programme is produced by production house as per specifications given by Moon TV, television channel,
& copyright is also transferred to television channel, the same falls within the scope of ‘work’ u/s 194C.
Therefore, payment of 50 Lacs made by Moon TV to production house is subject to TDS u/s 194C.
If, however, the payment was made by Moon TV for acquisition of telecasting rights of the content already
produced by the production house, there is no contract for ‘’carrying out any work”, as required in section
194C(1). Therefore, such payment would not be liable for tax deduction at source u/s 194C.
(b) The issue of whether fees/charges taken or retained by advertising companies from media companies for
canvasing/booking advertisements (typically 15% of the billing) is ‘commission’ or ‘discount’ to attract the
provisions of TDS has been clarified by the CBDT vide its Circular No.5/2016 dated 29.2.2016.
Relationship between media company & advertising agency is that of a ‘principal-to-principal’ & thus not liable
for TDS u/s 194H.
In view of the same, CBDT has clarified that no liability to deduct tax is attracted on payments made by television
channels to the advertising agency for booking or procuring of or canvassing for advertisements.
Accordingly, in view of the clarification given by CBDT, no tax is deductible at source on the amount of 15 Lacs
retained by Mudra Adco Ltd., the advertising company, from payment due to Cloud TV, a television channel.

DT QUESTION BANK BY CA PRANAV CHANDAK Subscribe us on YouTube 164


9. ADVANCE TAX & INTEREST U/S 234
Q1. Who is liable to pay Advance Tax? Also state who is not liable to pay advance tax?
Answer:
▪ As per section 208, Any person whose Advance Tax liability is Rs 10,000 or more in the FY on estimated current
income is liable to pay Advance Tax.
▪ Thus Any person whose Advance Tax liability is less than Rs 10,000 in the FY on estimated current income is
not liable to pay Advance Tax.
▪ Senior Resident Individual (Age ≥ 60 yrs during PY) who does not have any Income u/h PGBP is not required to
pay Advance Tax even if his Advance Tax liability ≥ Rs. 10,000.

Q2. How is advance tax liability calculated? OR On what amount advance tax will be payable ?
Answer:
▪ As per section 208, If advance tax liability is Rs 10,000 or more in the FY on estimated current income, advance
tax shall be payable on Estimated current Income.
▪ Advance Tax Liability = Tax on Total Income - Rebate & Relief + SC (if applicable) + 4 % HEC – TDS/TCS
▪ It is to be noted that the amount of TDS/TCS shall not be reduced from Total Income, if the Deductor has paid
the amount without deduction of tax or the Collector has received the amount without the collection of tax.
▪ Estimated Net Agricultural Income of the PY has to be considered for rate purpose while computing advance
tax.
▪ Each installment shall be calculated on estimated income on cumulative basis after revision of estimated income
@ every date of payment (15th of June/Sep/Dec/March).

Q3. What are the due dates of instalments & the quantum of advance tax payable by companies ?
Answer:
Payment date All assessee (other than eligible person u/s 44AD)
15 June 15% of Advance Tax Liability
15 September 45% of Advance Tax Liability - Amount paid in 1st Installment
15 December 75% of Advance Tax Liability - Amount paid in 1st & 2nd Installments
15 March 100% of Advance Tax Liability - Amount paid in 1st, 2nd & 3rd Installments

Q4. Determine Advance Tax Liability with due dates for FY 2018-19. [Nov-16]
▪ Estimated Tax Liability for FY 2018-2019: Rs. 65,000; TDS: Rs. 5,000
Solution: Advance Tax Payable = (65,000 – 5,000) = Rs. 60,000
Installment Date 15th June 2018 15th Sep 2018 15th Dec 2018 15th Mar 2019
Cumulative Amt payable = 15% of 60,000 = 45% of 60,000 = 75% of 60,000 = 100% of 60,000
= Rs. 9,000 = Rs. 27,000 = Rs. 45,000 = Rs. 60,000
Amount payable 9,000 18,000 18,000 15,000

Q5. What are the consequences of failure to deduct or pay the tax u/s 201 of the Income-tax Act, 1961?
Answer: Any person, including principal officer of a company, responsible for deducting tax at source shall be
deemed to be assessee in default in respect of such tax, if he does not deduct or after deducting fails to pay, the
whole or any part of the tax as required by or under the provisions of the Income-tax Act, 1961.
However, no penalty shall be charged u/s 221 from such person, unless the Assessing Officer is satisfied that such
person, without good & sufficient reasons, has failed to deduct & pay such tax.

Q6. Briefly discuss the provisions relating to payment of advance tax in case of capital gains & casual income.
Answer:
▪ Proviso to section 234C contains provisions for payment of advance tax in case of capital gains & casual income.
▪ Since it is not possible for an assessee to estimate certain incomes which are generally unexpected.
▪ Such incomes include:
▪ Capital gains & Winnings from lotteries, crossword puzzles etc;
▪ Dividend referred in Sec 115BBDA [Aggregate Dividend received in PY > Rs. 10 lacs].
▪ Income u/h “PGBP” in cases where income accrues or arises for the first time.
▪ If any such income arises after DD of any instalment, then entire amount of advance tax payable (after TDS) on
such income, shall be paid in remaining instalments of advance tax or by 31st March of relevant PY (if no
instalment is remaining).
▪ If the entire amount of tax payable is paid, then No Interest u/s 234B or 234C shall be payable.
Note: In case of casual income the entire tax liability is fully deductible at source @30% u/s 194B & 194BB.
Therefore, advance tax liability would arise only in respect of the health & education cess element of such tax, if
the same along with tax liability in respect of other income, if any, is Rs. 10,000 or more.

DT QUESTION BANK BY CA PRANAV CHANDAK Subscribe us on YouTube 165


Q7. Mr. Sachal, a Resident Individual aged 54, furnishes income details as under:
1. Wholesale Cloth Business, whose turnover is Rs. 150 Lacs. Income from such Business Rs. 8,10,000.
2. Income from Other Sources: Rs. 2,70,000.
3. Tax Deducted at Source: Rs. 25,000.
4. Advance Tax paid: Rs. 1,03,000 on 14.03.2019.
Return of Income will be filed on 11.12.2018. The Assessee is willing to pay the requisite Self- Assessment Tax.
Calculate the Interest Payable u/s 234B of the Income-Tax Act, 1961. [M-17]
Solution:
1. Total Income = Income from Cloth Business (8,10,000) + Income from Other Sources (2,70,000) 10,80,000
2. Tax thereon = [(10,80,000 – 10,00,000) x 30% + 1,12,500] 1,36,500
3. Add: HEC at 4% 5,460
4. Total Tax & Cess Payable (2+3) 1,41,960
Less: Tax Deducted at Source (25,000)
Advance Tax liability (Assessed Tax) 1,16,960
Advance Tax paid on 14.03.2019 1,03,000
Balance Payable before considering Interest 13,960

Computation of Interest u/s 234B:


▪ 90% of Assessed Tax = 90% of Rs. 1,16,960 = Rs. 1,05,264.
▪ Since Advance Tax paid (Rs. 1,03,000) is less than 90% of Assessed Tax, Interest u/s 234B is applicable.
▪ Interest u/s 234B = Shortfall, i.e. (1,16,960 – 1,03,000) x 1% x 9 months (Apr to Dec 2018) = Rs. 1,257.

Q8. A Firm made the following payments of Advance Tax during PY 2018-2019:
15.06.2018 15.09.2018 15.12.2018 15.03.2019 Total
Rs. 4,00,000 Rs. 5,00,000 Rs. 9,00,000 Rs. 12,00,000 Rs. 30,00,000
Income returned by the Firm is Rs. 100 Lacs u/h “PGBP” & Rs. 10 Lacs by way of LTCG on sale of a property effected
on 1.3.2019. What is the interest payable by the Assessee u/s 234B & 234C ?
ROI is filed on 31.07.2019 & Tax is fully paid upon self-assessment.
Solution: 1. Computation of Actual Tax Payable by the Firm
Tax on LTCG of 10 Lac at 20% = 2 lacs + Tax on Business Income @ 30% = Rs. 30 lacs = Rs. 32 lacs + 4 % = 33,28,000.
2. Computation of Interest u/s 234B
▪ 90% of Assessed Tax = 90% of Rs. 33,28,000 = Rs. 29,95,200.
▪ Since Advance Tax paid (Rs. 30 lacs) is more than 90% of Assessed Tax, Interest u/s 234B is not applicable.

3. Interest u/s 234C


Due date Advance Tax Cumulative Advance Tax Difference Months Interest at 1 % p.m
payable actually paid before DD (2) - (3)
(1) (2) (3) (4) (5) (6) = (4) x (5) x 1%
15.6.2018 15% of 31,20,000 4,00,000 68,000 3 2,040
= 4,68,000
15.9.2018 45% of 31,20,000 9,00,000 5,04,000 3 15,120
= 14,04,000
15.12.2018 75% of 31,20,000 18,00,000 5,40,000 3 16,200
= 23,40,000
15.3.2019 100% of 33,28,000 30,00,000 3,28,000 1 3,280
= 33,28,000
Note:
1. Tax on Business Income alone considered for computation of 1 st , 2nd & 3rd instalments.
2. Tax on LTCG has been considered only for 3rd instalment, as such gain had arisen only on 1.3.2019.

Q9. Compute Interest payable u/s 234A by Mr. Thermal Gattu for AY 2019-20.
(i) Due Date of filing ROI: 30.9.2019; (ii) Actual Date of filing ROI: 20.3.2020;
(iii) TDS = Rs. 5000; (iv) Advance Tax paid = Rs. 15000.
(v) Tax paid on Self-assessment before DD of filing ROI = Rs. 2,000;
(vi) Tax determined on Regular Assessment on the basis of Returned Income = Rs. 25,000.
Solution:
▪ Due Date of filing ROI: 30.9.2019; Date of Filing ROI: 20.3.2020.

DT QUESTION BANK BY CA PRANAV CHANDAK Subscribe us on YouTube 166


▪ Delay in filing ROI: 5 months & 20 days = 6 months (part of the month is treated as full)
▪ Interest u/s 234A = [Tax determined on Regular Assessment – TDS – Advance Tax] × 1% p.m * No. of Months.
= 25,000 – 5,000 – 15,000 – 2,000 (SAT paid before DD) = 3,000 × 1% p.m × 6 Months = Rs. 180.
Note: For computing interest u/s 234A, SAT u/s 140A shall be deducted if paid before DD of filing ROI.

Q10. Mr. Amol Chandak estimates his income for PY 2018-19 at Rs. 5,00,000. Besides this income, he has LTCG of
Rs. 1,00,000 on transfer of gold on 1.12.2018. He has won a lottery on 25th March 2019 of Rs. 2,00,000. Compute
the advance tax payable by Mr. Amol Chandak in various instalments.
Solution:
▪ Amol Chandak has LTCG on 1.12.2018 which falls after payment of 2 installment of advance Tax.
▪ Thus, for calculating first 2 installments, LTCG will not be considered.
Computation of Tax Liability of Mr. Amol Chandak for AY 2019-20 for 1st & 2nd Installment
Tax on Rs. 5,00,000 Rs. 12,500
Add: Health & Education cess @ 4% of Tax Rs. 500
Total Tax Liability Rs. 13,000
(i) First Installment of Advance Tax payable on 15.6.2018 = 15% of Rs. 13,000 = Rs. 1950.
(ii) Second Installment of Advance Tax payable on 15.9.2018 = 45% of Rs. 13,000 – Rs 1950 = Rs. 3900.
Computation of Tax Liability of Mr. Amol Chandak for AY 2019-20 for 3rd & 4th Installment
Tax on Rs. 5,00,000 Rs. 12,500
Tax on LTCG of Rs. 1 Lac @ 20% Rs. 20,000
Add: Health & Education cess @ 4% of Tax Rs. 1,300
Total Tax Liability Rs. 33,800
(iii) Third Installment payable on 15.12.2018 = 75% of 33,800 – 1950 – 3900 = Rs. 19,500.
(iv) Fourth Installment payable on 15.03.2019 = 100% of 33,800 – 1950 – 3900 - 19,500 = Rs. 8450
▪ Now, Mr Amol has won a lottery on 25th March 2019 which falls after the payment of all installments.
▪ Thus, he will have to pay the tax on such amount before 31st march. If paid before 31st march, it will be
considered to have been paid before the due date & thus No interest u/s 234B or 234C will be levied.
▪ Tax on Rs. 2 Lacs @ 30% = Rs. 60,000. Thus Mr. Amol has to pay Rs. 60,000 as tax on lottery before 31st March
2019. Otherwise Interest u/s 234B & 234C will get attracted.
▪ Since Tax on Lottery would have been deducted (TDS) @ 30% [i.e Rs. 60,000]. Mr Amol effectively will not be
required to pay any tax. He will just have to pay HEC element of 4 % & he will only have to give the details about
this transaction (Income & TDS) to the prescribed authority.
Note: If MR. Amol receives any Income after 15th March on which provisions of TDS are not applicable, he will
have to pay the tax on such income before 31st March 2019.

Q11. ABC Ltd. has estimated its tax payable to be Rs. 5,00,000 for previous year 2018-19 & has paid advance tax
accordingly but actual tax liability of the company was found to be Rs. 5,50,000 & difference of tax amount was
paid on 10.12.2019. Compute interest u/s 234A, 234B & 234C.
Solution:
(i) Interest u/s 234A shall be computed from 1.10.2019 to 10.12.2019 = 50,000 x 1% x 3 = 1,500
(ii) Interest u/s 234B: Advance tax paid is more than 90% of actual tax liability, no interest is payable.
(iii) Interest u/s 234C shall be computed in the manner given below:
Tax Payable Tax Paid Default
15.06.2018 82,500 75,000 7,500
Interest u/s 234C = Nil (because advance tax paid is at least 12%)
15.09.2018 2,47,500 2,25,000 22,500
Interest u/s 234C = Nil (because advance tax paid is at least 36%)
15.12.2018 4,12,500 3,75,000 37,500
Interest u/s 234C = 37,500 x 1 % x 3 months = 1,125
15.03.2019 5,50,000 5,00,000 50,000
Interest u/s 234C = 50,000 x 1% x 1 month = 500
Total interest payable u/s 234C = Rs. 1,625

Q12. Tax liability of X (53 years), a resident individual, for the financial year 2018-19 (AY 2019-20) is computed
as Rs. 1,00,000. X has paid advance tax as follows –
(i) Sep 10, 2018: Rs. 20,000; (ii) Dec 21, 2018: Rs. 30,000; (iii) March 11, 2019: Rs. 35,000.

DT QUESTION BANK BY CA PRANAV CHANDAK Subscribe us on YouTube 167


X intends to file his income-tax return with balance tax & interest payable. There was no TDS from any of his
income. Compute the tax & interest payable if -
1. Balance tax & interest are paid on July 21, 2019 & return is filed on same date.
2. Balance tax & interest are paid on January 4, 2020 & he files return on same date.
3. Balance tax & interest are paid on July 21, 2019, but he forgot to file return & return is Interfiled on Jan 4, 2020.
Solution: Computation of interest payable u/s 234C for deferment of advance tax:
DD for payment of Amount which should Amount Difference Interest Interest @ 1 %
Advance tax have been paid Actually paid (moths) p.m
June 15, 2018 15,000 Nil 15,000 3 450
September 15, 45,000 20,000 25,000 3 750
2018
December 15, 2018 75,000 20,000 55,000 3 1,650
March 15, 2019 1,00,000 85,000 15,000 1 150
Interest u/s 234C (in all cases) 3,000

Computation of interest payable u/s 234B for default in payment of advance tax:
▪ Case 1 (on Rs. 15,000 for 4 months @1% per month) = Rs. 600
▪ Case 2 (on Rs. 15,000 for 10 months @1% per month) = Rs. 1,500
▪ Case 3 (on Rs. 15,000 for 4 months @1% per month) = Rs. 600
Computation of interest payable u/s 234A for default in filing return of income:
▪ Case 1 - No interest u/s 234A as ROI for AY 2019-20 is filed before due date of filing ROI (July 31, 2019).
▪ Case 2 - There is delay in filing ROI by 6 months &, consequently, interest payable u/s 234A is Rs. 900 (on Rs.
15,000 for 6 months @1% per month).
▪ Case 3 - Interest u/s 234A is not applicable as balance tax & interest are paid before the due date of filing
return of income though the return is actually filed in January 2020.
Answer:
Particulars Case 1 Case 2 Case 3
Balance tax payable 15,000 15,000 15,000
Interest u/s 234A Nil 900 Nil
Interest u/s 234B 600 1,500 600
Interest u/s 234C 3,000 3,000 3,000
Total tax & interest 18,600 20,400 18,600

Q13. Find out the amount of interest payable u/s 234A in the following cases:
Particulars Mrs. X (59 years) X Ltd. (Industrial co.) X Co. (a firm)
Due date of filing return (a) 31.07.2019 30.9.2019 30.09.2019
Date of filing return (b) 6.11.2019 20.1.2020 10.12.2019
Tax on assessed income (c) 14,000 4,00,000 34,000
Advance tax paid during FY 2018-19 (d) 10,000 67,580 10,000
TDS during FY 2018-19 (e) 210 1,12,000 567
Self-assessment tax paid u/s 140A (f) 4,000 2,25,000 23,500
Date of payment of SAT u/s 140A (g) 2.9.2019 13.1.2020 3011.2019
Solution:
Particulars Mrs. X X Ltd. X Co.
Tax on assessed income (after deducting advance tax & TDS) [c - d - e] 3,790 2,20,420 23,433
Less: Self-assessment tax paid 4,000 2,25,000 23,500
Balance (-) 210 (-) 4,580 (-) 67
Period of default (since the entire outstanding amount is paid by SAT, 2 4 2
interest is payable till the date of payment of SAT) [g – a] months months months
Short fall [i.e., (c) -(d)- (e)] (rounded off) 3,700 2,20,400 23,400
Amount of interest payable u/s 234A 74 8,816 468

Q14. For PY 2018-19, Mr. X has Total Income Rs. 7,00,000 & he files his return on 10th August 2019. What will
be the penalty/fees payable u/s 234F ? (b) What if he files his return on 16th January 2020.
Answer: If he files his return on 10th August 2019, Penalty of Rs. 5000 is payable u/s 234F.
(b) If he files his return on 16th January 2020, Penalty payable u/ 234F is Rs. 10,000.
However if his total income is upto Rs. 5,00,000, Penalty shall be Rs. 1000 in both cases.

DT QUESTION BANK BY CA PRANAV CHANDAK Subscribe us on YouTube 168


10. RETURN OF INCOME
Q1. Mr. X, a non-resident (aged 82 years) having total income of Rs. 1,60,000 after deduction of Rs. 1,20,000 u/c
VI-A. His total income comprises of property & interest income. Whether he is required to file ROI?
Answer: As per section 139(1), every person, whose total income before giving effect to Chapter VI-A deduction
& exemption u/s 10(38)/10AA/B/BA exceeds BEL, is required to furnish ROI for relevant AY on/before due date.
▪ GTI of Mr. X (before deduction under Chapter VI-A) is Rs. 2,80,000 which exceeds BEL of Rs. 2,50,000
▪ Therefore, Mr. X has to furnish his ROI for AY 2019-20.
Note: Even though Mr. X is over 80 years of age, he is not entitled to BEL of Rs. 5 lacs, since he is a NR.

Q2. Paras is resident of India. During FY 2018-19, interest of Rs. 2,88,000 was credited to his Non-resident
(External) Account with SBI. Rs. 30,000, being interest on fixed deposit with SBI, was credited to his saving bank
A/c during this period. He also earned Rs. 3,000 as interest on this saving account. Is Paras required to file ROI ?
(b) What will be your answer, if he owns one shop in Kerala having area of 150 sq. ft.?
(c) What will be your answer, if he owns a house in London?
Solution: Individual is required to furnish a ROI u/s 139(1) if his total income, before giving effect to the
deductions under Chapter VI-A & exemption u/s 10(38), exceeds BEL.
Computation of Total Income of Mr. Paras for AY 2019-20
Income from other sources
Interest earned from Non-resident (External) Account [Exempt u/s 10(4)(ii), if permitted by RBI Nil
Interest on fixed deposit with SBI 30,000
Interest on savings bank account 3,000
Gross Total Income 33,000
Less: Deduction u/s 80TTA (Interest on saving bank account) 3,000
Total Income 30,000
Since total income of Mr. Paras for AY 2019-20, before giving effect to the deductions u/c VI-A, is less than BEL of
Rs. 2,50,000, he is not required to file return of income for AY 2019-20.
(b) Owning a shop having area of 150 sq.ft in Kerala would not make any difference to the answer.
(c) As per the fourth proviso to section 139(1), every resident & ordinarily resident having, inter alia, any asset
located outside India during the previous year is required to file a ROI, whether or not he has taxable income.
If Mr. Rajveer owns a house in London, he has to compulsorily file his ROI for AY 2019-20 on or before 31st July
2019, irrespective of the fact that his total income is less than BEL of Rs. 2.5 lacs.
Note: In the above solution, Interest of Rs. 2,88,000 earned from NR (External) A/c has been taken as exempt
assuming that Mr. Paras, a resident, has been permitted by RBI to maintain aforesaid account.
▪ However, if he is not permitted, interest would be taxable. In such case, his total income, before giving effect to
the deductions under Chapter VIA, would be Rs. 3,21,000 (Rs. 30,000 + Rs. 2,88,000 + Rs. 3,000), which is higher
than BEL of Rs. 2,50,000.
▪ Consequently, he would be required to file ROI for AY 2019-20. Ownership of shop in Kerala is immaterial.

Q3. State whether filing of ROI/ROL is mandatory for AY 2019-20 for LLP with Business Loss of Rs. 1,30,000.
Answer: Yes. As per Sec. 139(1), every Company & Partnership Firm (including LLP) should furnish ROI/ROL in
respect of its Income/Loss for every previous year. Even in case of Loss, they are compulsorily required to file ROL.

Q4. Mr. Yogesh, 80 years of age, carrying on retail trade business with turnover of Rs. 160 lacs for PY 2018-19,
declares his business income from such trade u/s 44AD as Rs. 4,80,000 (which also represents his taxable income)
in his ROI. What will be the Due Date of filing ROI to Mr. Yogesh for the AY 2019-20?
Answer: Although Mr. Yogesh is showing his income lower than 8% of total turnover, his total income is not
exceeding the BEL of Rs. 5,00,000 (since Mr. Yogesh is 80 years of age).
▪ Therefore, he is not required to maintain books of A/c as per section 44AA & get his accounts audited u/s 44AB.
▪ In such case the due date for filing return would be 31st July, 2018.

Q5. X & Co. is a partnership firm of chartered accountants. For PY 2018-19, taxable income is Rs. 7,10,000 & gross
receipt is Rs. 24,50,000. What is the Form No. in which return should be submitted? What is the due date of
submission of return of income? In which mode return should be submitted - paper mode or electronic mode?
Answer:
▪ Since gross receipt is not more than Rs. 50,00,000, books of account of the firm are not required to be audited.
Due date of submission of return of income is July 31,2019.
▪ Return should be submitted electronically with or without digital signature in ITR-5.

DT QUESTION BANK BY CA PRANAV CHANDAK Subscribe us on YouTube 169


Q6. What will be the Time limit for filing return of income u/s 139(1) in the case of a firm having total turnover of
Rs. 80 lacs for PY 2018-19, whether or not opting to offer presumptive income u/s 44AD.
Answer:
▪ In case an assessee opts to offer his income as per the presumptive taxation provisions of section 44AD, then,
the due date u/s 139(1) for filing of ROI for PY 2018-19 shall be 31st July, 2019.
▪ Where an assessee does not opt for presumptive taxation provisions u/s 44AD & offers income to be lower than
8% of total turnover & his total income exceeds the BEL, he has to maintain books of account as per section
44AA & get his accounts audited u/s 44AB.
▪ In such case the due date for filing return would be 30th September 2019.

Q7. X is a working partner in a firm : X & Co. Turnover of the firm for PY 2018-19 is 107 lacs. X gets 5 lacs as salary
& 10 lacs as interest from firm. Payment of interest is authorized by partnership deed. However, there is no
provision in partnership deed to pay any remuneration to working partners. Other incomes of X are:
(i) Capital gains : Rs. 80,000; (ii) House property Income : Rs. 2,70,000; (iii) Interest: Rs. 23,000.
What is the due date of submission of ROI ? In which mode return should be submitted - paper or electronic
Answer:
▪ X is a working partner in a firm whose books of account are required to be audited.
▪ The due date of submission of return of income by X is, therefore, September 30, 2019.
▪ This rule is applicable even if salary paid to working partner is not deductible in the hands of firm.
▪ Return should be submitted electronically with or without digital signature.
▪ Since turnover of the firm is more than Rs. 1 crore, return should be submitted on/before 30.09.2019.
▪ It should be submitted in ITR-5 electronically with digital signature.

Q8. Whether loss can be carried forward if ROI is furnished after DD specified in section 139(1)?
Answer:
▪ Section 80 r/w section 139(3) provides that the loss u/s 72(1)/73(2)/73A(2)/74(1)/74A(3) cannot be carried
forward if ROL is filed after DD u/s 139(1).
▪ However, Loss from House Property & unabsorbed depreciation can be carried forward even if ROI is furnished
after DD of filing ROI.

Q9. Return of loss of Mr. X for AY 2019-20 was filed in Dec 2019, in which he has claimed carry forward of current
year non-speculation Business loss of 1 lac & unabsorbed depreciation of 50,000 (relating to AY 2009-10). Discuss
provisions of carry forward for current year non-speculation business loss & b/f unabsorbed depreciation?
Answer:
▪ As per the provisions of section 139(3), any person who has sustained loss, inter alia, under the head ‘Profit
& gains of business or profession’ is allowed to carry forward such a loss u/s 72(1), only if he has filed the return
of loss within the time allowed u/s 139(1).
▪ Also, the provisions of section 80 specify that a loss which has not been determined as per the return filed u/s
139(3) shall not be allowed to be carried forward & set-off under, inter alia, section 72(1).
▪ However, there is no such condition for carry forward of unabsorbed depreciation u/s 32(2).
▪ In the given case, assessee has filed its return of loss in December 2019, which is a belated return filed u/s
139(4) & therefore, the benefit of carry forward of business loss u/s 72(1) shall not be available.
▪ However, the assessee shall be entitled to c/f unabsorbed depreciation as per the provisions of section 32(2).

Q10. Can loss be set off if ROI is furnished after the due date specified in Section 139(1)?
Answer:
▪ Section 80 r/w section 139(3) prohibits the "carry forward of losses" if ROI is filed after DD u/s 139(1).
▪ It does not prohibit the set off of losses.
▪ Therefore, losses can be set-off even if ROI is furnished after Due Date.

Q11. DD of filing ROI is 30.9.2019 of Mr. A. ROI is filed on 15.10.2019 as follows: Loss from Business: (Rs. 8 Lacs);
IFOS: Rs. 6 Lacs; Total Income: (Rs. 2 lacs). Is set off of loss u/h "PGBP" correct as per Section 80 r/w Sec 139(3) ?
Answer:
▪ Yes. Loss can be set off (Inter- source u/s 70 & Inter-Head u/s 71) even if ROL is filed after the DD u/s 139(1).
▪ However, loss of Rs. 2 Lacs cannot be carried forward.

Q12. Due Date of filing ROI is September 30, 2019 in case of Mr. C. ROI is filed on October 15, 2019 as follows:
Current Year Business Loss: (Rs. 1 Lacs); B/F losses for AY 2018-19 (ROL filed w/I DD): (Rs. 3,00,000). Whether
losses can be carried forward?
Answer:
▪ Losses of AY 2018-19 of 3 lac can be carried forward to AY 2019-20 as ROI for AY 2018-19 was filed before DD.
▪ However, Loss of AY 2019-20 shall not be carried forward to AY 2020-21.

DT QUESTION BANK BY CA PRANAV CHANDAK Subscribe us on YouTube 170


*Q13. X whose Income consists of Salary only, files his ROI for AY 2018-19 on 2.4.2019. Is it a valid return? [N 95]
Answer:
▪ U/s 139(4), where an Assessee who fails to file the Return of Income within the due date u/s 139(1), may file a
Belated Return u/s 139(4) either before the completion of assessment or the end of the relevant AY.
▪ In the given case, Mr. Smarajit, filed ROI for AY 2018-2019 on 2.4.2019, i.e. after time limit u/s 139(4).
▪ So, it shall be treated as an Invalid Return.

Q14. Mr. Sachin filed his ROI on 30.9.2019 related to AY 2019-20. In oct 2018, his tax consultant found that the
interest on fixed deposit was omitted in the tax return. [N-2017]
(i) What is the time limit for filing a belated return. (ii) Can Mr. Sachin file a revised return.
Answer:
(i) As per sec 139(4), any person who has not furnished ROI within the time allowed to him u/s 139(1) may furnish
ROI for any PY at any time before end of relevant AY or before completion of the assessment whichever is earlier.
Therefore, in the given question, Mr. Sachin can file his belated return on or before 31 st March, 2020.
(ii) As per sec 139(5), if any person having furnished a return u/s 139(1) or belated return u/s 139(4), discover
any omission or wrong statement, he may furnish a revised return at any time before the end of the relevant AY or
before completion of assessment, whichever is earlier. Hence Mr. Sachin can revise his return on/before 31.3.2020.

Q15. Mr. Vineet submits his ROI on 12.9.2019 for AY 2019-2020 consisting of Income u/h “HP” & “IFOS”. On
21.1.2020, he realized that he had not claimed deduction u/s 80 TTA in respect of his Interest on Savings Bank
A/c. He wants to revise his ROI, since one year has not elapsed from the end of Relevant AY. Discuss. [M-14, N-16]
Answer:
▪ Since Mr. X has income only u/h “Income from house property” & “IFOS”, he does not fall under the category of
a person whose accounts are required to be audited under the Income-tax Act, 1961 or any other law in force.
▪ Therefore, the due date of filing return for AY 2019-20 u/s 139(1) is 31st July, 2019.
▪ Since Mr. X had submitted his return only on 12.09.2019, the said return is a belated return u/s 139(4).
▪ U/s 139(5), any Return filed within the due date u/s 139(1) can be revised before the end of the relevant AY or
completion of assessment whichever is earlier.
▪ As per section 139(5), a return furnished u/s 139(1) or u/s 139(4) can be revised.
▪ Therefore, Mr. X can revise the return of income filed by him u/s 139(4), to claim deduction u/s 80TTA.

Q16. Mr. X filed a ROI for PY 2018-19 on 31.7.2019. He later files a revised return on 15.12.2019 declaring a loss
of Rs. 1,00,000. Can the loss be allowed to be carried forward?
Answer:
▪ Revised return substitutes the original return.
▪ Since original ROI was filed within DD u/s 139(1), revised ROL shall be deemed to have been filed within DD &
thus loss of Rs. 1,00,000 shall be allowed to be carried forward.

Q17. ROI for PY 2018-19 was submitted by X on 15.6.2019. Return was processed u/s 143(1) on 5.8.2019. X wishes
to file revised return. (a) Upto what time can he do? (b) What if assessment is completed on 31.8.2019.
Answer:
(a) He can file a revised return, processing of return u/s 143(1) is not treated as assessment for this purpose.
Revised return can be filed at any time before the end of AY 2019-20 i.e upto 31.3.2020.
(b) In this case revised return can be filed before completion of Assessment i.e. upto 30.8.2019.

*Q18. Explain whether ROI can be revised u/s 139(5) of the Income- tax Act, 1961 in the following cases:
(i) Defective or incomplete return filed u/s 139(9). (ii) Belated return filed u/s 139(4).
(iii) Return already revised once u/s 139(5). (iv) Return of loss filed u/s 139(3).
Answer:
Any person who has furnished a return u/s 139(1) or u/s 139(4) can file a revised return if he discovers any
omission or any wrong statement in the return filed earlier. Accordingly:-
(i) A defective or incomplete return filed u/s 139(9) cannot be revised. However, the defect can be removed.
(ii) A belated return filed u/s 139(4) can be revised.
(iii) A return revised earlier can be revised again as first revised return replaces original return. Therefore, if
assessee discovers any omission or wrong statement in such a revised return, he can furnish a second revised
return within prescribed time i.e. Earlier of (a) Before end of relevant AY or before completion of assessment.
(iv) A return of loss filed u/s 139(3) is deemed to be return filed u/s 139(1), & therefore, can be revised u/s 139(5).

Q19. Assessing Officer has the power, inter alia, to allot PAN to any person by whom no tax is payable. Discuss.
Answer: True: As per section 139A, Assessing Officer may having regard to the nature of transactions as may be
prescribed also allot a PAN to any other person, whether any tax is payable by him or not, in the manner & in
accordance with the procedure as may be prescribed.

DT QUESTION BANK BY CA PRANAV CHANDAK Subscribe us on YouTube 171


Q20. Mrs. Hetal, an individual engaged in the business of Beauty Parlour, has got her books of account for FY ended
on 31.03.2019 audited u/s 44AB. Her total income for AY 2019-20 is Rs. 3,35,000. She wants to furnish her ROI for
AY 2019-20 through a tax return preparer. Can she do so?
Answer:
▪ Section 139B provides a scheme for submission of ROI for any AY through a TRP.
▪ However A person whose books of account are required to be audited u/s 44AB (or under any other law) or a
company, cannot submit return of income through tax return preparers (TRP).
▪ Therefore, Mrs. Hetal cannot furnish her return of income for AY 2019-20 through a TRP.

Q21. By whom should the return of income be signed in the case of following persons: [M-17]
(i) Political Party; (ii) Company which is being wound up;
(iii) Hindu Undivided Family, when karta is unable to sign, & (iv) Scientific research association.
Answer: As per section 140, Return should be signed by the authorised person, as given below:
(i) Political Party: In the case of a political party, Return can be signed by Chief Executive Officer.
(ii) Company which is being wound up: If company is in liquidation, Return can be signed by Official liquidator.
(iii) HUF when karta is unable to sign: By any other adult member (male or female) of such family.
(iv) Scientific research association: If such Association is -
▪ A Company: MD or Any other Director if MD not able to sign or there is no MD.
▪ An AOP: Any Member or Principal Officer.
▪ Any other Person: That Other Person or some other person who is competent to sign on his behalf.

Q22. Does AO have power to make any adjustment to income disclosed by the assessee in the return of income in
course of processing the return u/s 143(1)?
Answer:
▪ AO has the power to adjust income returned by the assessee in the course of processing the return u/s 143(1).
▪ An arithmetical error in return can be corrected by the Assessing Officer u/s 143(1).
▪ Moreover, if there is any incorrect claim of an item, adjustments can be made by the Assessing Officer.

*Q23. X, an individual, filed his ROI for AY 2018-19 on 15.06.2018. He later discovered that he had not claimed
deduction u/s 80C in the said return. He claimed the said deduction through a letter addressed to AO. AO
completed the assessment without allowing the deduction claimed by X. Is AO justified in doing so?
Answer:
▪ AO does not have the power to entertain a claim for deduction made after filing of return of income in a way
other than by way of a revised return u/s 139(5). Thus, Assessing Officer is justified in his action.
▪ Thus Mr. X should have filed revised ROI u/s 139(5) with the prescribed time limit.

*Q24. ROI for AY 2017-18 was filed in time as per sec 139(1). Assessee during the course of assessment proceeding
u/s 143(2), noticed certain omissions &, therefore, filed a revised return on 18.4.2019. AO ignoring the revised
return so filed framed the order on April 27, 2019. Is the action of Assessing Officer correct?
Answer: Revised return in this case can be filed up to March 31,2019. Since the revised return is filed after the
said date, it is not a valid revised return. The action of the Assessing Officer is legally correct.

Q25. Write a note on Defective Return of Income u/s 139(9).


Answer: Refer Page no. 233 of the “Concept Book”.

Q26. An assessing officer finds a defect in the return of income & intimated the defect vide letter dated 09.10.2018,
which was received by Mr. Ram on 11.10.2018, What is the date by which Mr. Ram has to rectify the defect,
assuming that Mr. Ram has not applied for extension of time. [Nov 2012]
Answer: Date by which Mr. Ram has to rectify the mistake = 09.10.2018 + 15 days = 24.10.2018

Q27. Write a note on “Self-Assessment u/s 140A” & “Best Judgement Assessment u/s 144.
Answer: Refer Page no. 240 of the “Concept Book”.

Q28. State the exception to the provisions of section 139AA for quoting of Aadhar number.
Answer: Provisions of Sec. 139AA would not apply to Individual who does not possess Aadhar number or
Enrolment ID & is:
(a) Residing in States of Assam, Jammu & Kashmir and Meghalaya;
(b) Super Senior Citizen [Age ≥ 80 years at any time during PY;
(c) Non-Resident;
(d) Not a Citizen of India.

DT QUESTION BANK BY CA PRANAV CHANDAK Subscribe us on YouTube 172

You might also like